Você está na página 1de 278

PROVAS DA AFA 1998 - 2010

AFA 1997/1998 – Física volume imerso. Nesse caso pode-se afirmar que o
(a)
1) Um certo calorímetro contém 80 gramas de a) peso do corpo é igual ao peso da água
água à temperatura de 15O C. Adicionando-se à deslocada.
água do calorímetro 40 gramas de água a 50O C, b) empuxo da água é igual à metade do peso do
observa-se que a temperatura do sistema, ao ser corpo.
atingido o equilíbrio térmico, é de 25O C. Pode-se c) massa do corpo é igual à metade da massa da
afirmar que a capacidade térmica do calorímetro, água.
em cal/OC, é igual a d) volume do corpo é igual a duas vezes o volume
(calor específico da água é 1 cal/gO C) da água deslocada.

a) 5 b) 10 c) 15 d) 20 8) Um campo de indução magnética B que possui
linhas de indução paralelas e eqüidistantes é
2) Um satélite artificial foi lançado para entrar em denominado
órbita circular ao redor da Terra, a uma distância d a) uniforme. b)divergente.
do seu centro. Sabendo-se que G é a Constante c)estacionário. d)convergente.
Gravitacional Universal e M a massa da Terra, o
período de revolução do satélite, ao redor da Terra, 9) Um capacitor C1 de 1 F é ligado a uma bateria
será dado por de 12 V para ser carregado. Após a carga, a
GM
a) 2 b) 1d2 GM bateria é desligada, e outro capacitor C2 de 3 F,
d inicialmente descarregado, é ligado em paralelo
3
GM d com C1 . A soma das novas cargas dos
c) 2d d) 2
d GM capacitores, em C, será
a) 3 x 10-6 b)6 x 10-6 c) 9 x 10-6 d)12 x 10-6
3) Misturando-se 1,5 kg de água a 85O C com 3,0
kg de água a 10O C, obtém-se 4,5 kg de água à 10) Na figura abaixo, o ponto P está situado a uma
temperatura, em O C, de distância r de um condutor reto percorrido pela
(calor específico da água: 1 cal/gO C) corrente elétrica i. O campo de indução magnética

a) 35 b) 45 c) 55 d) 65 B nesse ponto é melhor representado por
4) Certa massa de um gás ideal sofre uma a)
transformação na qual a pressão duplica e o P
volume cai a um terço do valor inicial. A r
b) X x
temperatura absoluta final, em relação à inicial, é i
a)a mesma. b)2/3 c)3/2 d)5 c)

5) 10 mols de hélio a 273 K e 2 atm ocupam o d)


mesmo volume que x mols de nêonio a 546 K e 4
atm. Considerando-se os dois gases como ideais, o 11) A intensidade do campo de indução magnética
valor de x é →
a)5 b)10 c)15 d)20 B, medida em mT, no centro de uma espira
circular de raio 0,1 mm e corrente elétrica de 2 A,
6)A densidade do mercúrio a 0O C vale 13,6 g/cm3 é
e tem um coeficiente de dilatação cúbica de 1,82 x (Dado O = 4 x 10-7 Tm/A)
10-4 OC-1. A sua densidade em g/cm3, na a) 4 b)6 c)2 d) 4
temperatura de 40O C, vale
a)13,40 b)13,50 c)13,55 d)13,56 12) Na figura abaixo, temos um gerador com
resistência interna de 1, ligado a um resistor
7) Um corpo de massa m e volume V é colocado externo de 3. Os valores da potência dissipada
em um recipiente contendo água. Verifica-se que no resistor de 3, e a diferença de potencial entre
esse corpo flutua na água com metade de seu os pontos A e B são, respectivamente,
PROVAS DA AFA 1998 – 2010
r =12V
e) 48 W e 12 V A 18) Um corpo de 5kg de massa percorre uma
trajetória circular no interior de uma esfera oca,
f) 48 W e 0 V cujo raio é 2 metros. A figura abaixo ilustra o fato.
R=3
B
Pode-se afirmar que o trabalho, em J, realizado
g) 27 W e 12 V pela força peso entre os pontos
C
h) 27 W e 0 V i) AC é 100
D B
13) Nove fios de cobre, cada um de comprimento l j) ABCDA é nulo.
e diâmetro d, e resistência R, são ligados em série,
formando um único condutor, com resistência total k) ABCDA é 200 A

RT. O diâmetro D de um único fio de cobre, de


comprimento também l, para se obter a mesma l) ABC é igual ao trabalho da força centrípeta.
resistência total deve ser
a) d/2 b)d/3 c)d/4 d)d/5 19) Uma máquina térmica opera entre duas fontes,
uma quente, a 600 K, e outra fria, a 200 K. A
14) Uma força elétrica de intensidade F aparece fonte quente libera 3700 J para a máquina.
quando duas pequenas esferas idênticas, com Supondo que esta funcione no seu rendimento
cargas 3 C e 9 C são colocadas a uma distância d, máximo, o valor do trabalho, em J, por ciclo e o
no vácuo. Quando colocadas em contato e seu rendimento, são, respectivamente,
afastadas a uma distância 3d, a nova intensidade a) 1233 e 33%. b)1233 e 100%.
da força elétrica, em função de F, será c)2464 e 67%. d)3700 e 100%.
a) 2F 27 b) 4F 27 c) 7F 27 d) 8F 27
20) Duas lâmpadas, uma L1 de resistência R1, e
15) Faz-se um experimento com 4 esferas outra L2 de resistência R2, sendo R1 > R2, são
metálicas iguais e isoladas uma da outra. A esfera ligadas em paralelo e em série. Respectivamente,
A possui carga elétrica Q, e as esferas B, C e D para cada ligação, a lâmpada mais brilhante será
estão neutras. Colocando-se a esfera A em contato a) L1 e L2 b) L2 e L1 c) L2 e L2 d) L1 e L1
sucessivo com as esferas B, C e D, a carga final de
A será 21) Uma carga puntual q de 2 C é colocada em
a) Q/3 b)Q/4 c)Q/8 d)Q/9 um ponto P, a uma distância d de uma carga Q de
3 C. Nestas condições a intensidade do campo
16) Uma máquina térmica, ao realizar um ciclo, elétrico criado pela carga Q, no ponto P, depende
retira 20 J de uma fonte quente e libera 18 J para a) somente de q. b)de Q e de q. c)de Q e de d.
uma fonte fria. O rendimento dessa máquina, é d)somente de Q.
a) 0,1% b) 1,0% c) 2,0% d)10 %
22) Aplica-se uma d.d.p. de 10 V em uma
17) I.Um objeto é acelerado não somente quando associação em série de três capacitores. Sabendo-
sua velocidade escalar varia, mas também quando se que C1 = 2 F, C2 = 2 F e C3 = 4F, a energia
seu vetor velocidade muda de direção. armazenada, em J, na associação e a d.d.p., em
II.Para descrever completamente o movimento de volts, no capacitor C2, são, respectivamente,
um objeto basta conhecer como varia sua a) 40 e 4 b)60 e 4 c)40 e 8 d) 60 e 8
velocidade escalar com o tempo.
III.Um corpo pode ter velocidade escalar nula e 23) Uma pequena esfera é abandonada em queda
estar submetido a uma aceleração tangencial nula. livre, de uma altura de 80 m, em relação ao solo.
→ → Dois segundos após, uma segunda esfera é atirada,
IV.Na expressão da 2O Lei de Newton, F = m a , a verticalmente para baixo. Despreze a resistência
massa m é chamada massa gravitacional. do ar e considere g = 10 m/s2. A fim de que as
Das afirmações acima, são verdadeiras esferas atinjam o solo no mesmo instante, a
a) I e II. b)I e III. c)I, II e IV. d)I, III e velocidade de lançamento da segunda esfera, em
IV. m/s, deve ser
2
PROVAS DA AFA 1998 – 2010
a) 15 b)20 c)25 d)30 28) Na figura abaixo, a densidade do líquido A é
dA = 0,4 g/cm3 e a do líquido C é dC = 2,5 g/cm3,
24) Um corpo de massa 3M desloca-se no sentido então, a densidade do líquido B, em g/cm3, é
Oeste-Leste, com velocidade de 10 m/s, e colide a) 1,45
inelasticamente com outro corpo de massa 2M,
deslocando-se no sentido Sul-Norte com b) 2,50 A 9 cm
6 cm C
velocidade de 20 m/s. A velocidade do sistema
4 cm
formado pelos dois corpos, após a colisão, em m/s, c) 2,85
B
será
a) 10 b)20 c)30 d)40 d) 5,20

25) Na figura abaixo , o ângulo  vale 30O, e a 29) No interior de um solenóide, a dependência do

relação entre as massas M2/M1 tem valor 3/2. Para campo de indução magnética B , em relação à
que o sistema permaneça em equilíbrio, qual deve corrente elétrica i, pode ser representada por
ser o valor do coeficiente de atrito entre o bloco 2
e o plano? a) B

a) 3 /3
i

b) 3 /2
B
 b)
c) 3 M2 M1
i
d) 1/2

26) No avião de treinamento T-25 utilizado na c)


B
AFA, a hélice gira 2700 rpm durante a corrida no
solo e, após a decolagem, a rotação é reduzida para i
2450 rpm em apenas 5 segundos. Supondo-se que
a hélice sofre uma desaceleração uniforme, a
aceleração angular da hélice, em valor absoluto, d)
B
vale aproximadamente, em rad/s2,
a) 1,67 b) 3,14 c) 5,23 d) 8,72 i

27) Duas partículas A e B desenvolvem


movimentos sobre uma mesma trajetória, cujos 30) Em uma revista especializada em automóveis,
gráficos horários são dados por: afirma-se que um determinado veículo acelera de
s(m)
zero a 108 km/h em 7,2 segundos. Supondo-se
que tal veículo desenvolveu MRUV, no citado
32 B intervalo de tempo, o espaço percorrido pelo
28
mesmo, em metros, é
14 A a) 72 b)96 c)108 d)120

31) Dois corpos idênticos estão ligados por um fio


O 4 7 8 t(s) ideal, passando por uma roldana, conforme figura
abaixo. Inicialmente, os corpos estão em repouso.
No instante em que A e B se encontram, os Sendo
módulos das velocidades de A e de B valem, mA = mB = 3 kg, a densidade do fluido 0,6 g/cm3, a
respectivamente, densidade dos corpos 1,2 g/cm3 e g = 10 m/s2, a
a) 2 e 12 b) 2 e 16 c) 2,57 e 12 d) 2,57 e 16 aceleração do sistema, após um certo intervalo de
tempo, será, em m/s2,

3
PROVAS DA AFA 1998 – 2010
a) 0 38) Uma bolinha de tênis de 50 gramas de massa é
atingida pela raquete de um tenista e, após a
b) 0,6 colisão, adquire uma velocidade de 180 km/h.
Sabendo-se que esta colisão é elástica e dura um
c) 1,6 B milésimo de segundo, a força média impulsiva
A exercida pela raquete, em newtons, vale
d) 2,5 a) 2500 b) 5000 c) 7000 d) 9000

32) Em relação a um observador parado na 39) Uma partícula de massa m, carga elétrica q e
margem, a velocidade com que um barco sobe o velocidade v descreve uma trajetória circular de
rio vale 8 km/h e a com que o mesmo barco desce raio r1 numa região dotada de campo de indução
o rio vale 20 km/h, sempre com movimento →
magnética B . Após um certo tempo, nota-se que o
uniforme. A velocidade da correnteza, em km/h,
raio da trajetória passa a ser r2 = 2r1 . Pode-se
vale
afirmar que
a) 3 b) 6 c) 8 d) 12
a) essa partícula perdeu massa.
b)a velocidade da partícula aumentou.
33) Dois líquidos X e Y, miscíveis entre si,
c)a carga elétrica da partícula aumentou.
possuem densidades 0,6 g/cm3 e 0,9 g/cm3 ,
d)o campo de indução magnética dobrou de
respectivamente. Ao se misturar 3 litros do líquido
intensidade.
X com 6 litros do líquido Y, a densidade da
mistura, em g/cm3 , será
40) Um fuzil FM-1908 dispara horizontalmente
a) 0,6 b) 0,7 c) 0,8 d) 0,9
uma bala de massa m, com velocidade vb , contra
um bloco de madeira de massa M em repouso,
34) Um termômetro mal graduado assinala, nos
sobre uma superfície horizontal sem atrito.
pontos fixos usuais, respectivamente -1O C e
Sabendo-se que na colisão inelástica a bala fica
101O C. A temperatura na qual o termômetro não
cravada no bloco, então a velocidade final do
precisa de correção é
sistema, após o choque, será
a) 49 b)50 c)51 d)52
m+M m
a) vb b) vb
35) Uma corrente de 5 A percorre uma resistência M m+M
de 10  durante 4 minutos. A carga, em
coulombs, e o número de elétrons que atravessam m+M (m + M )
c) M vb d) m vb
a resistência nesse período são, respectivamente, m M
(Dado: carga do elétron 1,6 x 10-19 C)
a)1200 e 7,5 x 1021 b)7,5 x 1021 e 1200
c)6,28 x 10-18 e 1200 d)1200 e 6,28 x 1021 AFA 1998/1999 – Física
36) Uma prensa hidráulica é utilizada para levantar 1) A estrela mais próxima está a 4,5 anos-luz da
um automóvel de massa 1,8 toneladas até a altura Terra. O ano-luz é a distância que a luz percorre,
de 1 metro. Considerando g = 10 m/s2 e que o no vácuo, em um ano. Sendo assim, ao
êmbolo maior tem diâmetro de 60 cm e o menor observarmos a abóboda celeste, estamos vendo o
diâmetro de 10 cm, então, a força necessária para a) futuro. b)infinito. c)passado. d)presente.
que o automóvel permaneça erguido, em newtons,
é 2) Uma esfera de raio muito pequeno movimenta-se
a) 50 b) 300 c) 500 d) 3000 em uma concha esférica de raio R = 1,8 m, como
na figura abaixo. Desprezando-se o atrito e
37) A aceleração da gravidade na superfície da considerando g = 10 m/s2, pode-se afirmar que a
Terra, de raio R, é g. Calcule a altura, em relação à velocidade, em m/s, da esfera no ponto B e o
superfície, na qual a aceleração da gravidade trabalho, em joules, da força normal valem,
valerá g/9. respectivamente,
a) R b) 2R c) 3R d) 4R

4
PROVAS DA AFA 1998 – 2010
R R
A
a) 3 e 0. a) B
b) 3 e 10.
A V
c) 6 e 0.
d) 6 e 5. B
B
b) A
R
3) Considere um cometa se aproximando do Sol.
Desprezando-se a sua perda de massa, pode-se V
afirmar que, em relação ao Sol, sua energia
c) B
a) cinética diminui. b)cinética aumenta. A V
c) mecânica diminui. d) mecânica aumenta. R

4) Uma esfera maciça e homogênea, de densidade


B
2,0 g/cm3, flutua em um líquido, mantendo 20% de d)
R V
seu volume acima do
A
nível do líquido. A densidade do líquido, em
g/cm3, é
a) 1,5. b)2,0. c) 2,5. d) 3,0.
9) Um objeto é lançado obliquamente ao ar com
5) Qual a carga, em coulomb, de uma partícula de ângulo de lançamento . Sabendo-se que o alcance
2x10-3 kg de massa para que permaneça máximo foi 122,5 m, qual sua velocidade inicial de
estacionária, quando colocada em um campo lançamento, em m/s?
elétrico vertical, de módulo 50 N/C? (considerar g = 10 m/s2)
(considerar g = 10 m/s2) a) 10 b)12,5 c) 35 d) 49,5
a) -2x10 b)-1x10 c) 2x10-4 d) 4x10-4
-4 -4

10) Em uma experiência realizada na Lua, uma


6) O gráfico abaixo representa três isotermas de pedra de 200 g é lançada verticalmente para cima
um mesmo sistema gasoso, nas temperaturas T1, T2 e, no mesmo instante, outra pedra idêntica é
e T3. Pode-se então afirmar que abandonada de uma altura de
40 m. Sabendo-se que as duas pedras colidem a 20
P T3 m de altura e que a aceleração da gravidade na Lua
a) T1 = T2 = T3. T1 é g = 1,6 m/s2, a velocidade com que foi lançada a
b) T2 < T1 < T3. T2 primeira pedra, em m/s, é
c) T2 > T1 > T3. a) 2. b)4. c) 6 . d) 8.
d) T1 < T2 < T3.
V 11) Duas partículas partem da mesma posição, no
mesmo instante, e descrevem a mesma trajetória
7) Em uma região de campo elétrico uniforme, de circular de raio R. Supondo que elas girem no
intensidade 2.103 N/C, a diferença de potencial, em mesmo sentido a 0,25 rps e 0,2 rps, após quantos
volts, entre dois pontos, situados sobre uma linha segundos estarão juntas novamente na posição de
de força do campo elétrico e separados por uma partida?
distância de 50 cm, é a) 5 b)10 c) 15 d) 20
a) 103. b)105. c)4x103. d) 2,5x10-4.
12) O sistema abaixo encontra-se em equilíbrio.
8) Em um laboratório, encontramos uma bateria B, d1 d2
1 2
um amperímetro A, um voltímetro V e
um resistor de resistência R. Qual é o circuito
que permite determinar, experimentalmente, o
valor de R ? Sabe-se que d1 = 5 cm, d2 = 4,0 cm, (1) e (2) são
esferas de raios 1 cm e 1,2 cm, respectivamente, e
que d2 = 2,0 g/cm3 é a densidade da esfera (2).

5
PROVAS DA AFA 1998 – 2010
Nessas condições, a densidade de (1) vale, em 19) A pressão no interior de um líquido
g/cm3, aproximadamente, homogêneo em equilíbrio varia com a
a) 1,0. b) 2,0. c) 3,0. d) 4,0. profundidade, de acordo com o gráfico abaixo. A
densidade do líquido, em kg/m3, é
13) Um automóvel entra em uma curva de 30O de (considerar g = 10 m/s2)
inclinação, com velocidade 30 m/s. O raio da P(105N/m2)
curva, em metros, para que não haja a) 103.
4
escorregamento, é
(considerar g = 10 m/s2) b) 2x103. 3

( )
−1
a) 9 3 . b) 90 3 . c) 90 3 . d) 900 3 . 2
c) 3x103.
14) O gráfico da posição em função do tempo para 1
um objeto, que se move em trajetória retilínea, é d) 4x103.
0 5 10 15 h(m)
dado na figura abaixo. A velocidade inicial, em
m/s, e a aceleração, em m/s2, são, respectivamente,
S(m) 20) Mergulham-se dois termômetros na água: um
9 graduado na escala Celsius e outro na Fahrenheit.
a) 6 e 2. Depois do equilíbrio térmico, nota--se que a
b) 6 e 3. diferença entre as leituras nos dois termômetros é
c) 9 e 3. 172. Então, a temperatura da água em graus
d) 9 e 6. Celsius e Fahrenheit, respectivamente, é
0 3 6 T(s) a) 32 e 204. b) 32 e 236.
c) 175 e 347. d) 175 e 257.
15) Um paralelepípedo de massa 750 kg, com
arestas de medida 0,5 m, 1,0 m e 1,5 m, 21) Um recipiente cuja capacidade volumétrica a
está apoiado em uma superfície horizontal. zero graus Celsius é 3000 cm3, está completamente
A pressão, em N/m2, exercida por ele na superfície, cheio de um líquido. O conjunto foi aquecido de
pode ser igual a 0OC a 100OC, ocorrendo um transbordamento de
a) 7500. b) 10000. c) 12500. d) 16500. 24 cm3. O coeficiente de dilatação aparente desse
líquido, em OC-1, é
16) Uma bola de 0,40 kg, movimentando-se a) 8x10-5. b) 8x10-3. c) 8x10-2. d) 8x10-1.
horizontalmente com velocidade 14 m/s, é rebatida
com uma força média de 1000 N. Supondo-se que 22) Um cubo de gelo com massa 100 g e
a colisão tenha durado temperatura -10OC é colocado em um recipiente
27 ms, a velocidade final da bola, em m/s, é contendo 200 ml de um líquido
a) 14,0. b) 32,5. c) 53,5. d) 81,5. a 100OC. Supondo-se que não há
perda de calor para o meio ambiente, qual o valor
17) Um astronauta, em órbita, a 1600 km da final aproximado da temperatura, em OC, do
superfície terrestre, está sujeito a uma aceleração sistema?
da gravidade igual (Considerar calor específico em cal/gOC
(Considerar o raio da Terraigual a 6400 km) do gelo = 0,5, da água = 1 e do líquido = 2,
a) a zero. calor latente de fusão do gelo = 80 cal/g e
b) ao valor na superfície terrestre. densidade do líquido = 1,5 g/cm3)
c) a 0,04 vezes o valor na superfície terrestre. a) 43 b) 53 c) 3 d) 73
d) a 0,64 vezes o valor na superfície terrestre.
23) No diagrama pressão-volume, indicado na
18) Misturando-se volumes iguais de dois líquidos figura, um sistema sofre uma transformação do
de densidades 0,4 g/cm3 e 0,8 g/cm3, a densidade estado inicial i para o estado final f. Caso tal
da mistura, em g/cm3, supostamente homogênea, é fenômeno ocorra, tendo como estado intermediário
a) 0,4. b) 0,6. c) 0,8. d) 1,2. o ponto A, o calor absorvido e o trabalho
realizado, em joules, serão, respectivamente, 50 e
20. Entretanto, se for utilizado como estado
6
PROVAS DA AFA 1998 – 2010
intermediário o ponto B, o calor absorvido será 36 linear do cobre é 4x10-3 OC-1, a resistência elétrica,
J. A energia interna inicial tem valor 10 J. Então, a em  , de 200 m de fio 2,5, a 80OC, é
energia interna final e o trabalho realizado segundo a) 1,36. b ) 1,69. c) 2,08. d) 3,65.
a trajetória i B f, valem, respectivamente, em
joules, 30) A figura mostra como três líquidos imiscíveis,
P de densidades diferentes, se dispõem em um tubo
A f
em U. Sendo dadas as densidades do líquido 1
a) 10 e 6. igual a 0,4 g/cm3 e do líquido 3 igual a 2,5 g/cm3, a
b) 20 e 36. densidade do líquido 2, em g/cm3, será igual a
c) 30 e 20. i B
d) 40 e 6. a) 4,85.
V b) 6,50. 1 7 cm
c) 7,25.
24) Duas cargas de valor q estão separadas de um d) 8,32. 5 cm
3
ponto A pela distância d. A que distância do ponto 2 cm
A deve ser colocada uma carga -q para que o
potencial em A seja nulo? 2
a) d/2 b) d c) 2d d) 4d

25) Sejam dois condutores isolados A e B 31) Em um alto-falante, desses utilizados em


cujas capacitâncias e tensões são CA= 6 F, sonorização de autos, temos uma bobina imersa em
CB = 4 F, VA= 80 V e VB= 30 V. Quando um campo magnético intenso produzido por um
colocados em contato, o potencial comum, em ímã permanente, conforme o esquema abaixo.
volts, é Nessas condições, podemos afirmar que
a) 30. b) 40. c) 60. d) 90.
Cone de papelão
26) Um chuveiro elétrico, quando ligado, tem um Bobina
rendimento de 70%. Se a potência elétrica recebida móvel

é de 2000 W, a potência dissipada, em W, será i


Ímã permanente
a) 300. b) 600. c) 1000. d) 1400.

27) Doze lâmpadas idênticas, usadas na decoração a) os pólos do ímã repelirão a bobina móvel,
natalina, associadas em série, são ligadas a uma mantendo-a parada sobre o seu eixo.
fonte de tensão igual a 120 V. Sabendo-se que a b) a bobina poderá mover-se para frente ou para
corrente elétrica que passa pelas lâmpadas é 50 trás, dependendo do sentido da corrente i.
mA, o valor da resistência de cada lâmpada, em  , c) a bobina móvel ficará paralela às linhas do
é campo magnético do ímã, se a corrente i for igual a
a) 100. b) 200. c) 300. d) 400. zero.
d) o campo magnético produzido pela bobina será
28) Uma bola de beisebol de 0,15 kg se aproxima anulado pelo campo do ímã permanente.
de um bastão com uma velocidade de 20 m/s e,
após o choque, retorna, na mesma direção, sem 32) O circuito abaixo é constituído de três
alterar o módulo de sua velocidade. O impulso lâmpadas com resistência R, uma lâmpada com
recebido pela bola, na interação com o bastão, é, resistência desconhecida RX, uma fonte de 24 V,
em N.s, um amperímetro A e uma chave S. Com a chave S
a) 0. b) 6. c) 10. d) 20. aberta, o amperímetro indica 4 ampéres e, com a
chave S fechada,
29) Atualmente, um fio muito utilizado em indica 6. O valor da resistência da lâmpada, em  ,
instalações elétricas residenciais é o 2,5, ou seja, é
área de secção transversal igual a
2
2,5 mm . Sabendo-se que a resistividade do cobre a
20OC é 1,7x10-8  m e o coeficiente de dilatação
7
PROVAS DA AFA 1998 – 2010
sobre o bloco é N. Quando desce com a mesma
S R aceleração, a reação é
RX N1. Considerando-se g = 10,0 m/s2, a razão N1 /N é
A
a) 1/5. b) 2/3. c) 3/2. d) 5.
24 V
38) A figura abaixo mostra um corpo em um plano
R R → →
inclinado, submetido à força F e ao peso P . O
a) 2. b) 4. c) 7. e) 9.

trabalho, em joules, realizado por F para deslocar
33) Certa máquina térmica opera entre as o corpo por um metro, com velocidade constante,
temperaturas de 27OC e 477OC, em um ciclo que ao longo do plano, é, aproximadamente,
absorve 5 kJ da fonte quente e libera 2 kJ para a (Considerar g = 10 m/s2; massa do corpo m = 1,0
fonte fria. Isso posto, é possível afirmar que kg; e coeficiente de atrito  = 0,3)
a) o rendimento da máquina é 40%. →
b) o rendimento da máquina é 94%. F
a) 3,8. 30O
c) a máquina obedece a um ciclo reversível. b) 6,8.
d)a máquina obedece a um ciclo irreversível. c) 7,8.
d) 9,8. →
34) Um barco vazio tem metade de seu volume P
imerso. Colocando-se 1000 kg de carga em seu 45O
interior, ele imerge mais, ficando apenas 1/3 de seu
volume fora d’água. Então, o peso do barco, em N,
é 39) Um menino de 30 kg, carregando duas pedras
a) 5000. b) 10000. c) 20000. d) 30000. de 1 kg cada, está em um carrinho de 10 kg,
inicialmente em repouso numa superfície
35) Uma esfera de aço de massa 0,5 kg, amarrada a horizontal. O menino arremessa as pedras
uma corda de 70 cm de comprimento, é solta horizontalmente, ao mesmo tempo, na direção
quando a corda está na horizontal, conforme figura possível do movimento do carrinho,
abaixo. Na parte mais baixa de sua trajetória, com a mesma velocidade de 6 m/s, em relação ao
colide elasticamente com um bloco de aço de mesmo. O módulo da velocidade do carrinho, em
massa 2,5 kg, inicialmente em repouso sobre uma m/s, após o arremesso é, aproximadamente,
superfície sem atrito. A velocidade do bloco, após a) 0,05. b) 0,07. c) 0,28. d) 0,40.
a colisão, em m/s, é, aproximadamente
40) Em uma impressora de jato de tinta, uma
a) 0,86. gotícula de massa m = 2x10-10 kg carregada com q
b) 1,23. = -1,1x10-13 C, passa entre duas placas paralelas de
c) 2,50. comprimento L = 2,0 cm, entre as quais existe um
d) 3,20. campo elétrico de módulo
6
EY = 1,6x10 N/C, conforme figura abaixo. Se vX =
36) Um projétil é disparado com velocidade de 250 20 m/s é a velocidade com que a gotícula penetra
m/s em uma direção que faz um ângulo  com a na região entre as placas, desprezando-se a força
horizontal. Após um intervalo de tempo, o projétil gravitacional, o módulo da deflexão y, em
choca-se com um obstáculo a 5250 m do ponto de metros, que esta sofre é
disparo. Desprezando-se a resistência do ar e
+++++++++++++++++
considerando-se a) 2x10-5. +

g = 10 m/s2, sen  = 0,7, a velocidade do projétil,
em m/s, no instante do choque, é
b) 3,3x10-4.
c) 4,4x10-4. q
E
 Y
vX
a) 125. b) 175. c) 215. d) 250. d) 1,6x10-3. -------------------------

37) Um bloco de massa m repousa sobre o


piso de um elevador. Quando o elevador
sobe com aceleração a = 2,0 m/s2, a reação do piso
8
PROVAS DA AFA 1998 – 2010
AFA 1999/2000 – Física Desprezando-se a resistência do ar, a que distância,
em metros, do ponto diretamente abaixo do avião,
no momento da queda, o objeto atingirá o solo?
1) Uma série de n projéteis, de 10 gramas cada um,
a) 200 b) 720 c) 2000 d) 4000
é disparada com velocidade v = 503 m/s sobre um
bloco amortecedor, de massa M = 15 kg, que os
7) Uma pessoa, partindo do repouso no ponto A,
absorve integralmente. Imediatamente após, o
desliza sobre um tobogã, representado na figura
bloco desliza sobre um plano horizontal com
abaixo. Ao atingir o final do tobogã, no ponto C,
velocidade V = 3 m/s. Qual o valor de n?
projeta-se no espaço, atingindo o ponto D, na
a) 4 b) 6 c) 7 d) 9
superfície de uma piscina. Sabe-se que a altura do
ponto A é 4 metros acima do ponto C e que o
2) Um pósitron é uma micropartícula que possui
ponto C está a uma altura igual a 1 metro acima do
massa de repouso igual à do elétron e carga
ponto D e, ainda, que o trecho BC é horizontal.
idêntica, mas positiva. Suponha, por hipótese, que
Desprezando-se todas as forças de resistência,
durante a atração entre um pósitron e um elétron a
pode-se afirmar que a distância horizontal, em
velocidade das partículas é da ordem de 36000
metros, entre os pontos C e D é
km/h e obedece às leis clássicas. Considerando a
massa do elétron e do pósitron igual a 9 X 10-31 kg,
podemos afirmar que a quantidade de movimento
do sistema imediatamente antes da recombinação,
é
a) zero b) 9x10-27 kgm/s c) 18x10-27 kgm/s d)
35x10-27 kgm/s

3) A altitude típica de um satélite de comunicação


é da ordem de 36000 km e o raio da Terra é a)1 b) 2 c) 4 d) 8
aproximadamente 6000 km. Designa-se por gO, a
aceleração da gravidade nas vizinhanças da 8) A figura abaixo representa um vagão se
superfície terrestre e por gS, a aceleração movendo sobre trilhos, retilíneos e horizontais,
gravitacional da Terra, na órbita do satélite. A com aceleração constante igual a 3,0 m/s2. No
partir dessas considerações, o valor da razão gO/gS interior do vagão, existe uma mesa de tampo
é horizontal e sobre ela está colocado um corpo
a) 6 b) 7 c) 36 d) 49 preso à parede dianteira do vagão por meio de uma
mola de constante elástica desconhecida. Sabe-se
4) Um terço de um percurso retilínio é percorrido que a massa do corpo é 2,0 kg e que está em
por um móvel com velocidade escalar média de 60 repouso, em relação ao vagão, e que a mola está
km/h e o restante do percurso, com velocidade distendida 4,0 cm, em relação ao seu comprimento
escalar média de 80 km/h. Então a velocidade normal. Pode-se afirmar que a constante elástica da
média do móvel, em km/h, em todo percurso, é mola, em N/cm, é
a) 70,0 b) 72,0 c) 73,3 d) 75,0

5. Um corpo movimenta-se sobre uma reta, e sua


posição, em metros, é dada em função do tempo,
em segundos, pela equação s = 7 + 6t
– t2. O instante em que o corpo inverte o sentido do
movimento e a sua velocidade no instante t = 4
a) 1,5 b) 3,0 c) 4,5 d) 6,0
segundos são, respectivamente,
a) 0 e 7 b) –4 e 10
9) Desde que a cronometragem eletrônica começou
c) 1,5 e –10 d) 0,67 e –20
a ser utilizada, os tempos dos recordes na prova de
100 metros rasos baixaram de 9,95 segundos
6) Um avião, sobrevoando em linha reta uma
(1968) para 9,79 segundos (obtido por Maurice
planície com velocidade 720 km/h e a uma altura
Greene em 1999), ou seja, apenas 16 centésimos
de 2000 metros, deixa cair um objeto.
9
PROVAS DA AFA 1998 – 2010
de segundo em 31 anos. As velocidades médias,
assoalho horizontal por uma força que faz um
em km/h, dos recordes citados foram, o
ângulo de 30 com a horizontal, conforme mostra a
aproximadamente,
figura abaixo. O coeficiente de atrito entre o bloco
a) 2,8 e 2,9 b) 10,0 e 10,2
c) 36,2 e 36,8 d) 41,2 e 41,6 e o assoalho é 0,25. O valor da força , em
newtons, necessária para colocar o bloco na
10) Um corpo tem seu movimento representado iminência de deslizar é, aproximadamente,
pelo gráfico abaixo, onde s é sua posição e t o
tempo. A equação horária que representa esse
movimento é

a) 35,1 b) 46,2 c) 54,0 d) 68,0

15) Uma esfera metálica de peso P está presa a


uma das extremidades de um fio de massa
desprezível, cuja extremidade oposta está ligada a
um suporte fixo. Sabendo-se que o sistema está em
equilíbrio, em uma posição na qual o fio forma
com a vertical um ângulo ?, equilíbrio este
a) s = 12 – 3t b) s = 15 + 3t conseguido pela ação de uma força horizontal F
c) s = 15 – 3t d) s = 15 – 5t aplicada à esfera, pode-se afirmar que o módulo de
tal força é
11) Um carro parte do repouso e em 10 segundos
atinge a velocidade de 108 km/h, ao percorrer uma
estrada retilínea e plana. Nesse intervalo, a
aceleração média, em m/s2, e a velocidade média,
em m/s, desse carro, são, respectivamente,
a) 3,0 e 15,0 b) 3,0 e 18,0
c) 3,6 e 30,0 d) 3,6 e 10,0
a) P tg  b) P/ tg  c) P cos  d) P/ cos 
12. Um bloco de 250 gramas cai sobre uma mola
cuja constante elástica é 250 N/m. O bloco prende- 16) Partindo do repouso, um automóvel gasta 1
se à mola, que sofre uma compressão de 12 cm minuto para cobrir uma distância de 1,44 km,
antes de ficar momentaneamente parada. A desenvolvendo movimento uniformemente
velocidade do bloco imediatamente antes de acelerado. Ao final desse intervalo de tempo, a
chocar-se com a mola é, em m/s, velocidade do mesmo será, em km/h,
a) 24,0 b) 48,0 c) 86,4 d) 172,8

17) Pode-se afirmar que, quando a distância entre


duas massas m1 e m2 é reduzida pela metade, a
força de atração gravitacional entre elas é
a) 2,00 b) 2,51 c) 3,46 d) 4,23 a) duas vezes maior.
b) duas vezes menor.
13) Um bloco é colocado na borda exterior de um c) quatro vezes maior.
carrossel de raio 5,0 metros e que dá uma volta a d) quatro vezes menor.
cada 30 segundos. Para que o bloco permaneça
sobre o carrossel, o coeficiente de atrito deve ser 18) Considere uma chapa quadrada, metálica, de
a) 0,02 b) 0,03 c) 0,20 d) 0,30 material homogêneo, contendo um orifício circular
em seu centro. Se a chapa for aquecida de modo
14) Um bloco de 20 kg é empurrado sobre um
10
PROVAS DA AFA 1998 – 2010
uniforme e o seu lado aumentar em 2%, então a 24) A variação aproximada do volume, em cm3, de
área do orifício uma esfera de alumínio de raio 10 cm, quando
a) diminuirá em 2%. aquecida de 20OF a 110OF, é
b) diminuirá em 4%. (dado: coeficiente de dilatação linear do alumínio
c) aumentará em 2%.  = 23x10-6/OC)
d) aumentará em 4%. a) 1,45 b) 14,50 c) 18,50 d) 29,00

19) Quando você bebe refrigerante, utilizando um 25) No interior de um cilindro, encontram-se 30
canudinho, o líquido chega até sua boca, porque a cm3 de um gás perfeito, sob pressão de 3 atm e
a) pressão atmosférica aumenta com a altura. temperatura de 50OC. Inicialmente, o gás sofre
b) densidade do refrigerante é maior que a expansão isotérmica e seu volume passa a ser 70
densidade do ar. cm3. A seguir, sofre transformação isocórica e a
c) pressão no interior de sua boca é menor que a pressão torna-se 2,5 atm. No final, a temperatura
pressão atmosférica. do gás, em OC, vale
d) pressão do líquido se transmite ao canudinho até a) 323 b) 355 c) 430 d) 628
chegar à boca.
26) Colocam-se 10 gramas de gelo a 0OC em um
20) Certa escala termométrica adota os valores -30 calorímetro de cobre com massa 150 gramas e
O
G e 370 OG, respectivamente, para os pontos de calor específico 0,093 cal/gOC. No interior do
fusão do gelo e ebulição da água, sob pressão de 1 calorímetro, há 200 gramas de água, cujo calor
atm. A fórmula de conversão entre essa escala e a específico é 1,0 cal/gOC. A temperatura do
escala Celsius é calorímetro e da água, antes de receber o gelo, era
a) tG = tC – 30 b) tG = tC + 370 de 20OC. Após o equilíbrio, colocam-se 55 gramas
c) tG = 4tC – 30 d) tG = 3,4tC + 30 de um metal a 90OC no interior do calorímetro.
Restabelecido o equilíbrio térmico, a temperatura
21) Um navio, flutuando em água doce, está atingiu 25 OC. O calor específico do metal, em
sujeito a um empuxo E1 e desloca um volume de cal/gOC, é
água V1. Flutuando em água salgada, o empuxo (dado: calor latente de fusão do gelo = 80 cal/g)
sobre ele será E2 e o volume de líquido deslocado a) 0,21 b) 0,40 c) 0,60 d) 0,80
será V2. Pode-se concluir então que
a) E2 > E1 e V2 < V1 27) O princípio fundamental em que se baseia o
b) E2 = E1 e V2 = V1 termômetro é a (o)
c) E2 = E1 e V2 < V1 a) lei zero da termodinâmica.
d) E2 > E1 e V2 > V1 b) primeira lei da termodinâmica.
c) segunda lei da termodinâmica.
22) Uma caixa com 2 metros de comprimento, 1 d) das trocas de calor sensível e latente.
metro de largura e 3 metros de altura, contém 5000
litros de água. A pressão exercida pela água no 28) A massa específica de um sólido é 6,19 g/cm3
fundo da caixa, em pascal, é a 6 OC e 6,12 g/cm3 a 56OC. O coeficiente de
a) 2,5 x 103 b) 5,0 x 103 c) 2,5 x 104 d) 5,0 x 104 dilatação linear do sólido, em oC-1, é
a) 6,52 x 10-5
23) De acordo com Johannes Kepler (1571-1630), b) 7,62 x 10-6
“o quadrado do período de qualquer planeta é c) 6,52 x 10-6
proporcional ao cubo do semi-eixo maior de sua d) 7,62 x 10-5
órbita”. Com respeito à órbita da Terra em relação
ao Sol, sabe-se que o período é de um ano e o 29) Dois líquidos A e B, com a massa de A
semi-eixo maior é 15 x 1010 metros. A partir valendo 5/4 da massa de B, são misturados no
dessas informações, pode-se afirmar que a ordem interior de um calorímetro. Verifica-se que não há
de grandeza da constante de proporcionalidade, em mudanças de estado e que a temperatura inicial de
s2/m3, é B e a temperatura de equilíbrio correspondem ao
a) 10-12 b) 10-15 c) 10-19 d) 10-23 quádruplo e ao triplo, respectivamente, da
temperatura inicial de A. Desprezando-se as trocas
11
PROVAS DA AFA 1998 – 2010
de calor com o calorímetro e com o ambiente, a c) 1,53 x 107 d) 3,82 x 107
relação entre os calores específicos dos líquidos A
eBé 34) Sabe-se que um condutor percorrido por uma
a) 1,25 b) 0,80 c) 0,75 d) 0,40 corrente elétrica pode sofrer o efeito de uma força
magnética devido ao campo magnético uniforme
30) Uma máquina térmica trabalha entre as em que o condutor estiver inserido. Nessas
temperaturas de 300 K e 600 K. Em cada ciclo, a condições, pode-se afirmar que a força magnética
máquina retira 221 J de calor da fonte quente e a) atuará sempre de modo a atrair o condutor para
rejeita 170 J de calor para a fonte fria. O a fonte do campo magnético.
rendimento da máquina e o rendimento máximo, b) atuará sempre de modo a afastar o condutor da
em porcentagem, que ela poderia ter com as fonte do campo magnético.
temperaturas entre as quais opera são, c) será máxima quando o ângulo entre a direção do
respectivamente, condutor e o vetor for 90O.
a) 44 e 56 b) 23 e 50 c) 50 e 77 d) 23 e 77 d) será sempre paralela à direção do condutor e o
seu sentido será o da movimentação das cargas
31) Oito mols de um gás ideal monoatômico negativas.
sofrem o processo termodinâmico AB indicado no
gráfico. A quantidade de calor, em kJ, trocada pelo 35) Na figura abaixo, temos dois capacitores
gás na transformação de A para B, é ligados em série, sendo C1 = 10 mF e C2 = 20 mF,
(dado: R = 8,31 J/molK) com uma fonte de 30 V. Sendo V1 e V2 dois
voltímetros, pode-se dizer em relação às leituras de
V1 e V2 que

a) 6 b) 12 c) 15 d) 48
a) V2 = 0 b) V1 = V2 c) V1 < V2 d) V1 > V2
32) Certa massa de metano, cuja molécula-grama é
16 gramas, ocupa volume de 120 litros sob pressão 36) Ligando-se um resistor de 0,10  a uma
de 2,5 atm e à temperatura de 427 oC. A massa do bateria com f.e.m. de 1,5 V, tem-se uma potência,
metano, em gramas, é dissipada no resistor, de 10 W. A diferença de
(dado: R = 0,082 atm?l/molK) potencial, em Volts, e a resistência interna da
a) 3,06 b) 5,22 c) 19,06 d) 83,60 bateria, em , são, respectivamente,
a) 1 e 0,05 b) 1 e 0,005 c) 10 e 0,05 d) 10 e
33) Quatro cargas são colocadas nos vértices de 0,005
um quadrado, de lado a = 10 cm, conforme a figura
abaixo. Sendo q1 = q2 = 3 uC e q3 = q4 = - 3 uC, 37) Um gerador de f.e.m. 12 V alimenta um
a intensidade do campo elétrico no centro do receptor cuja resistência é 10 . Sabendo-se que o
quadrado, em N/C, é rendimento do gerador é 60%, sua resistência
interna, em , é
a) 2,1 b) 4,3 c) 6,7 d) 8,9

38) Uma resistência é alimentada por uma linha de


220 V (CA) e possui como proteção um fusível
que interrompe a alimentação quando ocorre uma
sobrecarga no valor máximo de sua corrente, que é
25 A. Estando ligado um chuveiro que consome
4400 W, o número máximo de lâmpadas de 100 W
a) 7,64 x 106 b) 5,40 x 106
12
PROVAS DA AFA 1998 – 2010
que podem ser ligadas, sem interromper a 2) Durante um jogo de basquetebol, um jogador
alimentação, é arremessa a bola com velocidade inicial de
a) 1 b) 5 c) 10 d) 11 10 m/s formando um ângulo de 30º acima da
horizontal. Sabendo-se que a altura do cesto é 3,05
39) Pode-se afirmar que o campo magnético m e que o lançamento foi feito de uma altura de 2
existente na região em torno de um ímã natural é m, a distância horizontal, em metros, do jogador ao
devido cesto, para que ele consiga fazer os pontos sem o
a) à vibração das moléculas no interior do material auxílio da tabela, deverá ser aproximadamente
do imã. a) 2,02 b) 4,00 c) 6,09 d) 7,05
b) aos movimentos específicos dos elétrons
existentes nos átomos do material do imã. 3) Ao ultrapassar uma viga de madeira, uma bala
c) à repulsão, causada pelo núcleo dos átomos do tem sua velocidade escalar variada de
material, que atua gerando uma corrente elétrica. 850 m/s para 650 m/s. A espessura da viga é 10
d) a pequenos ímãs de magnetita existentes no cm. Admitindo o movimento como sendo
interior de cada átomo de óxido de ferro. uniformemente variado, o intervalo de tempo, em
segundos, em que a bala permaneceu no interior da
40) Assinale a alternativa incorreta. viga foi aproximadamente
a) A agulha magnética de uma bússola é um ímã a) 5,0 x 10-4 b) 1,3 x 10-4
que se orienta na direção do campo magnético c) 5,0 x 10-2 d) 1,3 x 10-2
terrestre.
b) O pólo sul geográfico atrai o pólo sul de uma 4) Um corpo é abandonado do topo de um
agulha magnetizada. precipício. O ruído produzido pela queda do corpo
c) Uma carga elétrica submetida à ação de um ao atingir o chão é ouvido 10 s após o seu
campo magnético sempre sofrerá a ação de uma abandono. Considerando a velocidade do som no
força magnética. ar igual a 340 m/s, pode-se afirmar que a altura do
d) Se um fio for percorrido por uma corrente precipício, em metros, é aproximadamente
elétrica, será produzido um campo magnético, que a) 200 b) 288 c) 391 d) 423
poderá atuar sobre cargas em movimento,
exercendo sobre elas uma força magnética 5) Considere um corpo em movimento uniforme
numa trajetória circular de raio 8 m. Sabe-se que,
entre os instantes 5 s e 8 s, ele descreveu um arco
AFA 2000/2001 – Física de comprimento 6 m. O período do movimento do
corpo, em segundos, é
a) 2  b) 3  c) 6  d) 8 
1) Os dois condutores retilíneos e compridos da
figura produzem um campo magnético resultante
6) Quando um corpo é elevado verticalmente por
no ponto A de intensidade 10-5T, saindo
uma força constante maior que seu peso, há
perpendicularmente do plano do papel. Se
variação
substituirmos os dois condutores por um único
a) apenas da energia cinética.
condutor, colocado exatamente onde se encontra o
b) apenas da energia potencial.
condutor 2, a intensidade de corrente e o sentido,
c) tanto da energia cinética como da potencial.
para que o campo em A continue inalterado, serão
d) da energia cinética, da energia potencial e do
trabalho. 1

a) 2i, para a direita. 10 . 7) Um automóvel A com o motorista e um passageiro


2i 10
move-se em movimento retilíneo uniforme.
b) 4i, para a direita.
2 Repentinamente, o motorista faz uma curva para a
esquerda, e o passageiro é deslocado para a direita.
c) 2i, para a esquerda.
4i O fato relatado pode ser explicado pelo princípio
da
d) 4i, para a esquerda.
a) inércia.
b) ação e reação.
13
PROVAS DA AFA 1998 – 2010
c) conservação da energia. Dado: G = 6,6 x 10-11 N m2/kg2
d) conservação do momento angular. a) 1,28 x 1032 b) 3,20 x 1032 c) 6,48 x 1031 d)
8,00 x 1031
8) Assinale a alternativa correta.
a) As forças de ação e reação são duas forças 13) A partir da superfície da Terra, um foguete,
sempre iguais. sem propulsão, de massa m, é lançado

b) O peso de um corpo é uma grandeza física que é verticalmente, com velocidade v 0 e atinge uma
igual a intensidade da força de reação do apoio. altitude máxima igual ao raio R da Terra. Sendo M
c) A condição necessária e suficiente para um a massa da Terra e G a constante de gravitação

corpo permanecer em repouso é que a somatória de universal, o módulo de v 0 é dado por
forças sobre ele seja zero. GM GM 3GM 3GM
d) Um canhão dispara um projétil para a direita e a) b) c) d)
R 2R 4R 2R
sofre um recuo para a esquerda. A variação da
quantidade de movimento do sistema é nula. 14) Dois carrinhos A e B de massas mA = 8 kg e
mB = 12 kg movem-se com velocidade
9) Um veículo faz uma curva de raio R, sem v0 = 9 m/s, ligados por um fio ideal, conforme a
derrapar, apesar de não haver atrito. Nesse caso, o figura. Entre eles existe uma mola comprimida, de
ângulo de inclinação da pista é tal que sua tangente massa desprezível. Num dado instante, o fio se
é igual a 1/2. Isso posto, podemos afirmar que a rompe e o carrinho A é impulsionado para a frente
força (sentido positivo do eixo x), ficando com
a) normal é metade do peso do veículo. velocidade de 30 m/s. A energia potencial
b) centrípeta máxima é metade da força normal. inicialmente armazenada na mola, em joules, era
c) centrípeta máxima é metade do peso do veículo. de
d) normal é metade da soma do peso e da y
centrípeta. 
v0
10) Uma bomba necessita enviar 200  de óleo a
um reservatório colocado a 6 metros de altura, em B A
25 minutos. A potência média da bomba, em watts, x
para que isso ocorra, é aproximadamente
Dado: densidade do óleo = 0,8 a) 2570 b) 2640 c) 2940 d) 3750
a) 5,15 b) 6,40 c) 7,46 d) 8,58
15) Duas esferas A e B, de massas respectivamente
11) Uma bola de borracha é lançada verticalmente iguais a 4 kg e 2 kg, percorrem a mesma trajetória
para baixo com energia cinética K1, a partir de uma retilínea, apoiadas num plano horizontal, com
altura h. Após colidir elasticamente com o solo, a velocida-des de 10 m/s e 8 m/s, respectivamente,
bola desloca-se para cima atingindo um ponto cuja con-forme a figura. Após a ocorrência de um
altura é 25% maior que a da posição inicial. choque frontal entre elas, as esferas movem-se
Considere K2 a energia cinética da bola separadamente e a energia dissipada na colisão
imediatamente antes de chocar-se com o solo e vale 162 J. Os módulos das velocida-des de A e de
calcule a razão K 1 K 2 . Despreze a resistência do B, após a colisão, em m/s, valem, respectivamente,
ar.
a) 0,25 b) 0,20 c) 0,75 d) 1,25 → 
VA VB
12) Um determinado sistema planetário é
A B
composto por uma estrela e 5 planetas orbitando
em torno dela. A massa da estrela é igual a 3,2 x
1033 kg e a do 3o planeta é de
1,6 x 1026 kg. Sabendo-se que a distância do a) 8 e 6 b) 2 e 7 c) 1 e 8 d) 1 e 10
planeta à estrela vale 3,3 x 108 km e que sua órbita
é aproximadamente circular, a sua quantidade de 16) Uma estrada de ferro retilínea liga duas
movimento, em kg m/s, vale, aproximadamente cidades A e B separadas por uma distância de 440
14
PROVAS DA AFA 1998 – 2010
km. Um trem percorre esta distância com encontram a 30 ºC. Se a temperatura do sistema
movimento uniforme em 8h. Após 6h de viagem, líquido-recipiente sobe para
por problemas técnicos, o trem fica parado 30 90 ºC, qual é o volume de mercúrio, em m  , que
minutos. Para que a viagem transcorresse sem transborda do recipiente?
atraso, a velocidade constante, em km/h, que o Dados: Hg = 1,8 x 10-4 ºC-1
trem deveria percorrer o restante do percurso seria vidro = 3 x 10-5 ºC-1
de aproximadamente a) 1,8 b) 2,6 c) 5,0 d)0 9,0
a) 55,0 b) 61,2 c) 73,3 d) 100,0
23) Um rapaz deseja tomar banho de banheira
17) Uma esteira rolante com velocidade Ve, misturando 80  de água fria a 18 ºC, com uma
transporta uma pessoa de A para B em 15 s. Essa certa quantidade de água quente a 60 ºC. Para o
mesma distância é percorrida em 30 s se a esteira rapaz tomar banho a 35 ºC, o tempo, em segundos,
estiver parada e a velocidade da pessoa for que a torneira de água quente deverá ficar aberta
constante e igual a vp. Se a pessoa caminhar de A será aproximadamente
para B, com a velocidade Vp, sobre a esteira em Dados: A vazão da torneira de água quente é de
movimento, cuja velocidade é Ve, o tempo gasto 0,25  /s. Desprezar a capacidade térmica da
no percurso, em segundos, será banheira e a perda de calor da água.
a) 5 b)10 c) 15 d) 30 a) 79 b) 152 c) 218 d) 303

18) O empuxo, em newtons, que a atmosfera 24) O volume de um mol de gás ideal varia
exerce sobre uma pessoa de massa 60 kg é linearmente em função da temperatura, conforme
aproximadamente gráfico abaixo. O trabalho realizado pelo gás ao
Dados: densidade média do corpo passar do estado A para o estado B, em joules, é
humano = 1,08 g/cm3 Dado: R = 8,3 J/mol K = 0,082 atm  /mol K
densidade do ar = 1,22 kg/m3
a) 4,22 x 10-1 b) 5,34 x 10-3 c) 6,77 x 10-1 d)7,28 V(  )
x 10-3
B
a) 25 A.
19) Misturando-se massas iguais de duas b) 51 20
substâncias, obtém-se densidade igual a 2,4 g/  , c) 2573
misturando-se volumes iguais dessas substâncias, a d) 5146 310 620
densidade é 2,5 g/  . As densidades das T(K)
substâncias, em g/  , são
a) 2 e 3 b) 3 e 5 c) 5 e 7 d) 7 e 9 25) Um gás sofre a transformação cíclica ABCA
indicada no gráfico abaixo. A quantidade de calor,
20) Uma escala termométrica, que mede a em joules, trocada no ciclo é
temperatura em graus L, indica 30 ºL e 50 ºL,
respectivamente, para as temperaturas de 10 ºC e a) 125 (
p N/m 2 )
90 ºC. Determine quantos graus L a escala indica
para o ponto de vapor da água (100 ºC). b) 175 600 B
a) 52,5 b) 75,0 c) 100,0 d) 105,0
c) 300 100 A
C
21) Uma chapa metálica feita de um material cujo
coeficiente de dilatação superficial vale  = 2 x 10- d) 600 0,2 0,7 V (m 3 )
5 A
ºC-1 apresenta um orifício circular de área igual a
1000 cm2. Quando a chapa é aquecida e sua 26) Uma máquina térmica funcionando segundo o
temperatura varia 50 ºC, a área do orifício, em ciclo de Carnot entre as temperaturas T1 = 700 K e
cm2, passa a ser T2 = 300 K recebe da fonte quente 1250 J de calor.
a) 999 b) 1000 c) 1001 d) 1010 O calor rejeitado, em joules, para a fonte fria é
aproximadamente
22) Um recipiente de vidro de 200 m  de volume, a) 423 b) 536 c) 641 d) 712
está completamente cheio de mercúrio, e ambos se
15
PROVAS DA AFA 1998 – 2010
27) Dois corpos, de massas e volumes diferentes, 33) Uma pequena esfera condutora, fixa e isolada é
estão em equilíbrio térmico quando apresentam os carregada com uma carga Q = 10-6 C. A uma
mesmos valores de distância de 2 mm, é colocada uma partícula
a) entropia. b) temperatura carregada com carga q = 1,6 x 10-9 C e de massa
c) capacidade térmica. d) quantidade de calor. m = 9 x 10-2 kg. Essa partícula é liberada, de
maneira que se move em relação a Q. A aceleração
28) Um pequeno recipiente de gás tem 5  de da carga q, no instante de sua liberação, em m/s2,
volume e, à temperatura de 27 ºC, apresenta vale
pressão interna de 12 atm. Resfriando-se o Dado: K = 9 x 109 N m2/C2
recipiente até a temperatura de –23 ºC e a) 0,04 b) 0,40 c) 4,00 d) 40,00
desprezando-se a variação externa de seu volume,
qual será a pressão final, em atm, do gás? 34) Uma pequena esfera condutora, isolada
Considere o gás ideal. eletricamente, é carregada com uma quantidade de
a) 3,2 b) 6,4 c) 10,0 d) 12,0 carga Q. Em seguida essa esfera é aterrada através
de um resistor de 0,25  . A carga da esfera é
29) Considere as afirmações abaixo, com relação descarregada em 0,5 s através da resistência, que
às transformações físicas de um gás. dissipa uma potência de 0,5 W. A carga Q, em
“A energia cinética média das moléculas do gás se coulombs, vale
mantém constante”. a) 2 b) 4 c) 2 d) 2 2
“A pressão do gás é diretamente proporcional à sua
temperatura”. 35) A queda de tensão através de uma associação
Estas afirmações se referem, respectiva-mente, às em série de resistências é de 5 V. Quando uma
transformações nova resistência de 2  é colocada na associação
a) isobárica e adiabática. inicial, mantendo-se a mesma diferença de
b) isotérmica e isotrópica. potencial, a queda de tensão na associação inicial
c) isobárica e isovolumétrica. cai para 4 V. O valor, em ohms, dessa associação
d) isotérmica e isovolumétrica. de resistências do conjunto inicial é de
a) 2 b) 4 c) 6 d) 8
30) Um fio de cobre com resistividade 1,69 x 10-8
 m é enrolado em um suporte cilíndrico, com 36) A figura abaixo mostra três cargas pontuais.
raio 10 cm, com 500 voltas. Sendo o raio do fio 2 Em relação aos potenciais dos pontos 1 e 2, V1 e
mm, sua resistência elétrica, em ohms, é V2, respectivamente, podemos dizer que
a) 0,42 b) 4,20 c) 42,00 d) 420,00
Q• 1 . -Q • 2 . Q •
31) Um tostador elétrico de 590 W, um relógio de
4 W, um rádio de 6 W e uma dúzia de lâmpadas de 2x x x x
60 W cada uma são alimentados simultaneamente
por uma rede elétrica com tensão 220 V. A a) V1 = V2 b) V1  V2 c)V2 = V12 d) V2  V1
potência total dissipada em watts e a corrente, em
ampéres, que circula na rede, são, respectivamente, 37) Dois capacitores planos, de placas paralelas, de
a) 1230 e 7 b) 1230 e 6 c) 1320 e 7 d) 1320 e 6 mesma capacitância, 1 mF, são ligados em paralelo
e conectados a uma fonte de tensão de 20 V. Após
32) Baseando-se na Lei de Coulomb e na definição ambos estarem completamente carregados, são
de campo elétrico de uma carga puntiforme, desconectados da fonte, e uma resistência é
podemos estimar, qualitativa-mente, que o campo colocada no lugar da fonte, de maneira que, em um
elétrico produzido por uma linha de transmissão de intervalo de tempo de 0,5 s, ambos se descarregam
energia, que tem uma densidade linear de cargas completamente. A corrente média, em ampéres, na
 (C/m), a uma distância r, perpendicular à linha, é resistência vale
proporcional a a) 2 x 10-1 b) 4 x 10-1 c) 5 x 10-2 d) 8 x 10-2
a) r  b) r/  c) r2  d)  /r
38) Um capacitor de placas planas e paralelas é
ligado a uma fonte de tensão de 10 V até ficar
16
PROVAS DA AFA 1998 – 2010
totalmente carregado. A seguir é desligado da
fonte e conectado a uma resistência R, de maneira 2) A maior aceleração (ou retardamento) tolerada
que se descarrega completamente em 0,1 s, pelos passageiros de um trem urbano é 1,5 m/s2. A
dissipando 1 W de potência. A capacitância, em F, maior velocidade que pode ser atingida pelo trem,
e a carga acumulada no capacitor, em C, são, que parte de uma estação em direção a outra,
respectivamente, distante 600 m da primeira, em m/s, é
a) 2 x 10-2 e 2 x 10-3 b) 2 x 10-3 e 2 x 10-2 a) 42. b) 30. c) 68. d) 54.
c) 2 x 10-3 e 2 x 10-1 d) 2 x 10-1 e 2 x 10-3
3) As figuras abaixo apresentam pontos que
39) Um resistor de10  é ligado a uma bateria de indicam as posições de um móvel, obtidas em
10 V por meio de um fio. Se o raio do fio é de intervalos de tempos iguais.
3 mm, a quantidade de carga elétrica que atravessa
uma secção do fio por unidade de tempo e por
unidade de área em (C s cm 2 ) é aproximadamente
I II

a) 3,54 b) 35,40 c) 354,00 d) 3540,00

40) Uma carga lançada perpendicularmente a um III IV IV


campo magnético uniforme realiza um movimento
circular uniforme (MCU) em função de a força
magnética atuar como força centrípeta.
Nesse contexto, pode-se afirmar que, se a
velocidade de lançamento da carga dobrar, o Em quais figuras o móvel apresenta aceleração
a) período do MCU dobrará. NÃO nula?
b) raio da trajetória dobrará de valor. a) Apenas em I, III e IV.
c) período do MCU cairá para a metade. b) Apenas em II e IV.
d) raio da trajetória será reduzido à metade. c) Apenas I, II e III.
d) Em I, II, III e IV.

AFA 2001/2002 – Física 4) Uma bola abandonada de uma altura H, no


vácuo, chega ao solo e atinge, agora, altura
máxima h. A razão entre a velocidade com que a
1) A posição x de um corpo que se move ao longo
bola chega ao solo e aquela com que ela deixa o
de uma reta, em função do tempo t, é mostrada no
solo é
gráfico. Analise as afirmações abaixo e marque a
1/2 3/2 2
alternativa correta. H H H H
a)   b) c)   d)  
x h h h h

5) Dois corpos A e B giram em movimento


circular uniforme presos aos extremos de cordas de
comprimentos, respectivamente, r e 2r. Sabendo
que eles giram com a mesma velocidade
I II III IV tangencial, pode-se dizer que
t → →
v v
a) A velocidade do corpo é positiva nos quatro r 2r

trechos. A B
b) A aceleração do corpo é nula apenas no trecho
IV.
c) A trajetória descrita pelo corpo no trecho I é a) ambos desenvolverão mesma velocidade
parabólica. angular.
d) O movimento descrito pelo corpo no trecho III é b) ambos estarão submetidos à mesma força
progressivo e retardado. centrípeta.
17
PROVAS DA AFA 1998 – 2010
c) num mesmo intervalo de tempo o corpo A dará
maior número de voltas que o B.
d) o corpo A desenvolve menor aceleração
centrípeta que o B. a) T + ma. b) T + 2ma
c) 2T + 2ma. d) 2T + ma.
6) Duas armas são disparadas simultaneamente, na
horizontal, de uma mesma altura. Sabendo-se que 10) Dois corpos de massas iguais, unidos por um
os projéteis possuem diferentes massas e fio inextensível, descem ao longo de um plano
desprezando a resistência do ar, pode-se afirmar inclinado. NÃO há atrito entre o corpo I e o plano.
que
a) a bala mais pesada atinge o solo em um tempo
menor.
b) o tempo de queda das balas é o mesmo.
c) a bala que foi disparada com maior velocidade
atinge o solo em um tempo maior.
De acordo com o enunciado, analise as afirmativas
d) nada se pode dizer a respeito do tempo de
abaixo.
queda, porque não se sabe qual das armas é mais
I- Se não houver atrito entre o corpo II e o
possante.
plano, a tensão no fio é nula.
II - Se houver atrito entre o corpo II e o plano,
7) Um audacioso motociclista deseja saltar de uma
a aceleração do corpo II é menor que a do corpo I.
rampa de 4 m de altura e inclinação 300 e passar
III - Se houver atrito entre o corpo II e o plano,
sobre um muro (altura igual a 34 m) que está
o movimento do corpo I será retardado.
localizado a 50 3 m do final da rampa.
Assinale a alternativa que contém apenas
afirmativa(s) INCORRETA(S)
a) II. b) I e III. c) II e III. d) I, II, e III.

11) Para levantar um pequeno motor até


determinada altura, um mecânico dispõe de três
50 3 m associações de polias:
4m I II III
0
30

obs.: o desenho está fora de escala.

Para conseguir o desejado, a velocidade mínima da


moto no final da rampa deverá ser igual a
a) 144 km/h. b) 72 km/h.
c) 180 km/h. d) 50 km/h.

8) Sob a chuva que cai verticalmente a 10 3 m/s,


um carro se desloca horizontalmente com
velocidade de 30 m/s. Qual deve ser a inclinação
do vidro traseiro (em relação à horizontal) para que
o mesmo não se molhe?
a) 300. b) 450. c) 600. d) 900. Aquela(s) que exigirá(ão) MENOR esforço do
mecânico é (são) somente
9) Um avião reboca dois planadores idênticos de a) I. b) II. c) I e III. d) II e III.
massa m, com velocidade constante. A tensão no
cabo (II) é T. De repente o avião desenvolve uma 12) Um piloto de 80 kg executa um loop perfeito
aceleração a. Considerando a força de resistência de raio 90 m. Se no ponto P do loop, conforme
do ar invariável, a tensão no cabo (I) passa a ser figura, a velocidade do avião é de 216 km/h, o

18
PROVAS DA AFA 1998 – 2010
módulo da força com a qual o piloto comprimirá a
poltrona, em newtons, é igual a b) d)

a) 1800.
b) 2400. P
c) 2700.
d) 3200.

13) Sobre uma partícula situada num plano 15) Uma partícula de massa 1 kg se move ao longo
→ do eixo Ox. O módulo da força, em newtons, que
horizontal aplica-se uma força F variável,
atua sobre a partícula é dado por F(x) = 2x – 2. Se
somente em módulo, cujo valor cresce desde zero.
a partícula estava em repouso na posição x = 0, a
Assinale, dentre os gráficos abaixo, aquele que
sua velocidade na posição x = 4 m é
MELHOR representa a intensidade da força de
a) 3,5 m/s. b) 4,0 m/s. c) 4,5 m/s. d) 5,0 m/s.
atrito (fat) em função da força (F) aplicada.

V0 = 0 → 16) A energia cinética EC de um corpo de massa m
f at F que se desloca sobre uma superfície horizontal e
retilínea é mostrada no gráfico em função do
a) fat c) fat deslocamento x.
EC

F F
fat fat

b) d)

F F 0 1 2 3 4 x

14) A figura representa uma curva plana de um O gráfico da força resultante FR que atua sobre o
circuito de fórmula 1. corpo em função do deslocamento x é
FR FR

a) 0 1 2 3 4 c) 0 1 2 3
x 4 x

FR FR

Se, durante uma corrida, um piloto necessitar fazer 0 1 2 3 4 0 1 2 3


tal curva com velocidade elevada, evitando o risco b) x 4 x
de derrapar, deverá optar pela trajetória
representada em qual alternativa?
17) Uma partícula de massa m e velocidade v,
a) c) colide com outra de massa 3m inicialmente em
repouso. Após a colisão elas permanecem juntas,
movendo-se com velocidade V. Então, pode-se
afirmar que
a) V = v. b) 2V = v. c) 3V = v. d) 4V = v.

19
PROVAS DA AFA 1998 – 2010
18) O motor de um avião a jato que se desloca a a) P b) T c) P + T d) P – T
900 km/h, expele por segundo 200 kg de gases
provenientes da combustão. Sabendo-se que estes 23) Um mergulhador encontra-se em repouso no
produtos da combustão são expelidos pela fundo do mar a uma profundidade de 10 m. A
retarguarda, com velocidade de 1800 km/h massa total do mergulhador, incluindo
em relação ao avião, pode-se afirmar que a equipamentos e acessórios é de 100 kg. Num
potência liberada pelo motor vale determinado instante, percebendo a presença de
a) 1,00 . 105 W. b) 2,50 . 107 W. um tubarão, ele resolve subir rapidamente. Para
c) 3,70 . 107 W. d) 3,24 . 108 W. obter uma aceleração inicial, o mergulhador enche
um balão dos seus acessórios com todo o ar
19) A relação entre o peso aparente PA e o real P comprimido existente em um de seus tubos de
de um astronauta no interior de uma nave espacial oxigênio. Considere o volume do tubo eqüivalente
que gira em torno da Terra, em órbita circular, é a 20% do volume total (mergulhador -
PA PA PA PA equipamentos - acessórios) e que o ar comprimido
a) = 0. b) = 1. c)  1. d)  1.
P P P P se comporte como um gás ideal, estando dentro do
tubo a uma pressão de 5 x 105 N/m2. Ao passar
20) Um corpo é sustentado por duas cordas instantaneamente do tubo para o balão, sem sofrer
inextensíveis, conforme a figura. alteração na sua temperatura, o ar fará com que o
mergulhador sofra uma aceleração, em m/s2, de
A a) 2 b) 3 c) 4 d) 5

24) O gráfico abaixo, representa a relação entre a


30o temperatura medida numa escala arbitrária E e a
C
temperatura na escala Celsius.
B
 (°E)
Y

47
Sabendo-se que a intensidade da tração na corda
AB é de 80 N, a intensidade da tração na corda
BC será X
a) 60 N. b) 40 N c) 40 3 N. d) 60 3 N.  (°C)
0 60 100

21) Considere a Terra um planeta de raio R A equação que representa corretamente a relação
estacionário no espaço. A razão entre os períodos entre Y e X é
de dois satélites, de mesma massa, em órbitas a) Y = 235 − 2X c) Y = 235 + 8X
3 5
circulares de altura R e 3R, respectivamente, é b) 141 + 2X d) 141 − 8X
Y= Y=
3 5
1 3 2 3
a) b) c) d)
2 4 4 2
25) A figura abaixo mostra um recipiente que está
22) Uma bola de peso P é mantida totalmente com 95% de volume ocupado por um líquido,
submersa em uma piscina por meio de um fio inicialmente a 10 ºC.
inextensível submetido a uma tensão T, como
mostra a figura.

Sendo os coeficientes de dilatação linear do


A intensidade do empuxo sobre a bola pode ser recipiente e volumétrico do líquido,
calculada por
20
PROVAS DA AFA 1998 – 2010
–5 –1 Analisando o gráfico pode-se afirmar que, na
respectivamente, iguais a 1,7 . 10 ºC e
5,8 . 10–4 ºC–1, pode-se afirmar que o transformação
a) recipiente estará completamente cheio a 110 ºC. a) AB, o gás recebe calor do meio externo.
b) volume da parte vazia não se altera. b) BC, a energia interna do gás aumenta.
c) recipiente estará com 98% de seu volume c) AB, o gás perde calor para o meio externo.
ocupado a 110 ºC. d) BC, a energia interna do gás diminui.
d) recipiente só estará completamente cheio a 220
ºC. 29) Um gás ideal sofre a transformação cíclica
ABCA indicada no seguinte gráfico:
26) Um corpo A foi colocado em contato com p
outro corpo B, e suas temperaturas variam de
acordo com o gráfico abaixo.
 (°C) p2
C
80
B p T2
A B
1
T1
30
V
A V1 V2
10
t (min)
0 10
Sendo a massa de B o dobro da massa de A, e Dos diagramas abaixo, o que MELHOR
considerando que as trocas de calor tenham representa a transformação anterior é
ocorrido apenas entre os dois, a razão entre o calor V
p

específico de A e o calor específico de B (cA/cB)


B C C B
vale a) c)
a) 2,5. b) 5,0. c) 0,4. d) 0,2.
A A
T T
27) Deseja-se resfriar um barril de vinho,
dispondo-se de uma única pedra de gelo. O V p
resfriamento se dará com MAIOR eficiência na
alternativa b) C d) C

A B A B
T T

30) Um motor térmico que funciona segundo o


Ciclo de Carnot, absorve 400 cal de uma fonte
quente a 267 oC e devolve 220 cal para uma
28) Um gás ideal monoatômico sofre as
fonte fria. A temperatura da fonte fria, em oC, é
transformações AB e BC representadas no gráfico
a) 12. b) 24 c) 147. d) 297.
p x V abaixo.
p 31) Uma máquina térmica funciona de acordo com
2p B o ciclo dado pela figura abaixo. Essa máquina foi
construída usando dois mols de um gás ideal
C monoatômico, e no decorrer de cada ciclo não há
A
p entrada nem saída de gás no reservatório que o
contém.
V
V 2V 4V

21
PROVAS DA AFA 1998 – 2010
5
p (10 N/m ) 3 Mesmo sob o efeito da gravidade a gota move-se
para cima com aceleração g. O módulo do campo
elétrico é
4 B C a) E = 2mg b) E = 2mq c) E = 2qg d) E = 2m
q g m qg

2 35) Uma partícula de carga q e massa m é lançada


A D
-2
V (10 m )3 com velocidade v, perpendicularmente ao campo
elétrico uniforme produzido por placas paralelas de
2,4 3,2 4,8
comprimento a, distanciadas de b entre si. A
O máximo rendimento e o trabalho realizado por partícula penetra no campo num ponto eqüidistante
essa máquina valem, respectivamente, das placas e sai tangenciando a borda da placa
a) 13% e 8 x 102 J. b) 75% e 8 x 102 J. superior, conforme representado na figura a seguir.
c) 13% e 4 x 103 J. d) 75% e 4 x 103 J.

32) Duas cargas pontuais positivas, q1 e q2 = 4q1,


são fixadas a uma distância d uma da outra. Uma
terceira carga negativa q3 é colocada no ponto P
entre q1 e q2, a uma distância x da carga q1,
conforme mostra a figura.
Desprezando a ação gravitacional, a intensidade do
campo elétrico é
b 2 mv bmv b 2 mv 2 bmv 2
a) b) c) d)
qa 2qa 2 qa qa 2

36 - Considere o circuito abaixo.


Para que as forças sobre a carga q3 sejam nulas, o P
valor de x é
2
a) d b) d c) d d) d 2
6V
2 3 4 6 10 V

4 1
33) Três esferas condutoras de raio R, 3R e 5R e
2
eletrizadas, respectivamente, com quantidade de
cargas iguais a –10 C, –30 C e +13 C estão A
1 2
muito afastadas entre si. As esferas são, então,
interligadas por fios metálicos de capacitância Afirma-se que:
desprezível até que o sistema atinja completo I - O amperímetro ideal A registra 2 A.
equilíbrio. Nessa situação, o valor da quantidade II - O potencial no ponto P é 10 V.
de carga, em microcoulombs, da esfera de raio 3R III - A potência dissipada no resistor de 4  é 4 W.
é São verdadeiras
a) – 9. b) – 3. c) 3. d) 9. a) apenas I e II. b) apenas I e III.
c) apenas II e III. e) I, II e III.
34) Uma gota de óleo de massa m e carga q é solta
em uma região de campo elétrico uniforme E, 37) Um forno de microondas opera na voltagem de
conforme mostra a figura. 120 V e corrente de 5,0 A. Colocaram nesse forno
200 mililitros de água à temperatura de 25 °C.
Admite-se que toda energia do forno é utilizada
E para aquecer a água. O tempo para elevar a
temperatura da água a 100 °C é
a) 60 s. b) 100 s. c) 120 s. e) 150 s.

22
PROVAS DA AFA 1998 – 2010
38) O gráfico abaixo mostra a potência elétrica 40 - Dois fios metálicos retos, paralelos e
consumida, ao longo do dia, em uma certa longos são percorridos por correntes 3i e i de
residência alimentada com a voltagem de 120 V. sentidos iguais (entrando no plano do papel).
Potência em kW 3i P i
10
8
a b
6
4
O campo magnético resultante produzido por essas
2 correntes é nulo num ponto P, tal que
a 1 a a 1 a
0 a) = b) =3 c) = d) =9
0 2 4 6 8 10 12 14 16 18 20 22 24 b 3 b b 9 b
Se o kWh custa R$ 0,10, Hora
o valor pago por 30 dias
de consumo é
a) R$ 88,00. b) R$ 112,00.
AFA 2002/2003 – Física
c) R$ 144,00. d) R$ 162,00.
01 - Um automóvel faz uma viagem em que, na
primeira metade do percurso, é obtida uma
39) Uma partícula de carga positiva, com
velocidade média de 100 km/h. Na segunda
velocidade dirigida ao longo do eixo x, penetra,
metade a velocidade média desenvolvida é de 150
através de um orifício em O, de coordenadas (0,0),
km/h. Pode-se afirmar que a velocidade média, ao
numa caixa onde há um campo magnético
longo de todo o percurso, é, em km/h,
uniforme de módulo B, perpendicular ao plano do
a) 130. c) 120.
papel e dirigido "para dentro" da folha.
b) 125. d) 110.
Sua trajetória é alterada pelo campo, e a partícula
sai da caixa passando por outro orifício, P, de
02 - O gráfico mostra a variação, com o tempo,
coordenadas (a,a), com velocidade paralela ao eixo
da altura de um objeto lançado verticalmente para
y. Percorre, depois de sair da caixa, o trecho PQ,
cima a partir do solo.
paralelo ao eixo y, livre de qualquer força. Em Q h(m)
sofre uma colisão perfeitamente elástica, na qual hmax

sua velocidade é simplesmente invertida, e volta


pelo mesmo caminho, entrando de novo na caixa,
pelo orifício P. A ação da gravidade nesse
problema é desprezível.
y 0 6 12
Q Desprezando a resistência
t(s) do ar, a altura máxima
a atingida pelo objeto vale, em m,
a) 300.c) 180.
P b) 240.d) 60.
x x

a
x x B
03 - Um móvel desloca-se ao longo de uma
v x linha reta, sendo sua posição em função do tempo
O dada pelo gráfico abaixo.
x x
x
D
2a
As coordenadas do ponto, em que a partícula deixa
a região que delimita o campo magnético, são B C
a) (0,0). c) (2a,0).
b) (a,-a). d) (2a,-a).
E
A
t

23
PROVAS DA AFA 1998 – 2010
Pode-se afirmar que a) c)
a) nos trechos CD e DE, o movimento foi
acelerado.
b) no trecho BC, a velocidade foi constante e não
nula.
c) no trecho AB, a velocidade é decrescente.
d) no trecho DE, a velocidade é negativa.
b) d)
04 - Um corpo desenvolve movimento circular
em um plano horizontal. Se no ponto A a
velocidade escalar tem intensidade menor que no
ponto B, então a opção em que o vetor aceleração
em C está MELHOR representado é

A A
C C 07 - Um corpo é lançado com uma velocidade
inicial de baixo para cima num plano inclinado
a) c)
perfeitamente liso. Se o corpo gasta um tempo tS
O O
para subir, qual dos gráficos abaixo representa a
B B
velocidade do corpo em função do tempo?
v v
a) c)
A A
C C

tS tS
b) d) t t

O O
B B
v v

05 - Dois aeroportos, A e B, estão no mesmo


meridiano, com B 600 km ao sul de A. Um avião P b) d)
decola de A para B ao mesmo tempo que um avião tS t
tS t
Q, idêntico a P, decola de B para A. Um vento de
30 km/h sopra na direção sul-norte. O avião Q
chega ao aeroporto A 1 hora antes do avião P
chegar ao aeroporto B. A velocidade dos dois 08 - Dois projéteis A e B são lançados
aviões em relação ao ar (admitindo que sejam obliquamente em relação à horizontal. Sabendo
iguais) é, aproximadamente, em km/h, que ambos permanecem no ar durante o mesmo
a) 690. c) 190. intervalo de tempo e que o alcance de B é maior
b) 390. d) 90. que o alcance de A, afirma-se que:
I- Ambos atingem a mesma altura máxima.
06 - Um garoto está em repouso sobre o vagão II - A velocidade inicial de B é maior que a de
de um trem que se move com velocidade constante A.
igual a 10 m/s em relação à Terra. Num certo III - A maior altura é atingida por A que foi
instante o garoto chuta uma bola com uma lançado com maior velocidade.
velocidade de módulo 20 m/s, em relação ao
vagão, formando um ângulo de 120o com o sentido É(são) verdadeira(s) apenas
do movimento do trem. Para uma pessoa que está a) I. c) III.
em repouso na Terra, a trajetória da bola é b) II. d) I e II.
MELHOR representada pela alternativa
09 - Na figura, os fios são ideais, o corpo tem
massa M e a aceleração da gravidade no local tem
24
PROVAS DA AFA 1998 – 2010
módulo g. A intensidade da tração no fio AB e a Em Em
intensidade da força F que mantém o sistema em
equilíbrio, valem, respectivamente, a) c)

A t1 t3
a) Mg cos  ; Mg sen    0 t2 t4 t5 t
Mg
b) cos 
; Mg sen  F 0 t1 t2 t3 t4 t5 t
B
c) Mg sen  ; Mg cos 
Mg
d) cos 
; Mg tg  Em
C Em

M b)
d)

0 t1 t2 t3 t4 t5 t
10 - Um automóvel desloca-se numa estrada
0 t1 t2 t3 t4 t5 t
horizontal com velocidade constante de 30 m/s.
Num dado instante o carro é freado e, até parar,
desliza sobre a estrada numa distância de 75 m. O
coeficiente de atrito entre os pneus e a estrada vale 13 - Uma partícula está sob efeito de uma força
a) 0,3. c) 0,5. conforme o gráfico abaixo:
b) 0,4. d) 0,6. F(N)

11 - A figura abaixo representa uma pista 3


pertencente ao plano vertical. O raio R da parte x(m)
circular vale 4 m. Um corpo parte do repouso no 0 4 8 10
ponto A. Desprezando o atrito e a resistência do ar -2
e considerando que, em B, a força que comprime o O trabalho, em joules, realizado pela força no
móvel contra a pista vale 1/4 do seu peso, pode-se intervalo x = 0 a x = 10 é de
afirmar que, a sua velocidade em B vale, em m/s, a) 23. c) 7.
aproximadamente, b) 10. d) 4.
3,2.
A
a) 5,5.
b) 6,3. B 14 -O motor da figura imprime ao corpo de massa
m uma aceleração para cima de módulo igual a g.
c) 7,1. R Calcule a potência fornecida pelo motor em função
do tempo, sabendo-se que o corpo partiu do
repouso no instante t = 0.
12 - Um corpo de massa m se movimenta num
campo de forças conservativas e sua energia
potencial (EP) varia com o tempo de acordo com o
gráfico abaixo. Ep

mg 2
a) P = 2mg 2t c) P=
2t
t2 2mg 2
b) P= d) P = mgt 2
0 t1 t3 t4 t5 t t

15 - Dois carrinhos A e B, de massa 2 kg cada,


O gráfico que MELHOR representa a variação da movem-se sobre trilhos retilíneos horizontais e
energia mecânica (Em) do corpo com o tempo (t) é sem atrito. Eles se chocam e passam a se mover
grudados. O gráfico representa a posição de cada
carrinho em função do tempo, até o instante da
colisão.
25
PROVAS DA AFA 1998 – 2010
s (m) mostrado na figura 1. Por fim derramou sobre o
óleo, conforme figura 2, uma coluna de 5 cm do
40 A líquido estranho, alcançando novamente o
30 equilíbrio.
B líquido
óleo estranho

0 10 t (s) óleo
10 cm 5 cm

A energia dissipada com o choque, em joules, é


água 10 cm
igual a 20 cm água
20,5 cm
15 cm
a) 40. c) 8.
b) 32. d) 0.
Figura 1 Figura 2
16 - Em telecomunicações são utilizados
satélites geoestacionários que se mantêm imóveis Depois de fazer seus cálculos descobriu que a
em relação a um observador na Terra. Um destes densidade do líquido estranho valia, em g/cm3,
satélites é colocado em órbita circular, a uma a) 0,20. c) 0,40.
altura 5R, onde R é o raio da Terra, acima da linha b) 0,30. d) 0,50.
do Equador. A velocidade linear do satélite é
a) R c) R/2 20 - Um barril flutua na superfície de um lago,
b) R d) R/2 deslocando 30 litros de água. Colocando-se esse
mesmo barril para flutuar sobre um líquido 1,5
17 - Um homem de dois metros de altura, com vezes mais denso que a água, quantos litros desse
peso igual a 900 N, preso por um dos pés a uma líquido ele irá deslocar?
corda elástica, pula de uma ponte de 100 m de a) 45. c) 20.
altura sobre um rio. Sendo a constante elástica da b) 30. d) 15.
corda equivalente a 300 N/m e seu comprimento
igual a 72 m, pode-se afirmar que a menor a 21 - Um médico durante uma consulta percebe
distância entre a cabeça do homem e a superfície que seu termômetro está com a escala apagada,
da água foi, em metros, então pede a sua secretária que enquanto ele
a) 0. c) 4. examina o paciente, coloque o termômetro em
b) 2. d) 6. contato com gelo fundente e logo depois com
vapor d’água (pressão normal). Para cada medida,
18 - Um garoto segura uma bexiga de 10 g, a secretária anota a altura atingida pela coluna de
cheia de gás, exercendo sobre o barbante uma mercúrio como sendo 10 cm e 30 cm,
força para baixo de intensidade 0,1 N. Nessas respectivamente. Nesse meio tempo, o médico
condições, pode-se afirmar que acha um outro termômetro e mede a temperatura
a) a pressão no interior da bexiga é menor que a do paciente: 36° C.
pressão atmosférica local. A secretária conseguiu calibrar corretamente o
b) o empuxo que a bexiga sofre vale 0,1 N. termômetro de escala apagada e verificou que a
c) a densidade média da bexiga é menor que a do altura atingida pela coluna de mercúrio ao medir a
ar que a envolve. temperatura do paciente era, em cm,
d) o empuxo que a bexiga sofre tem a mesma a) 6,7. c) 18,0.
intensidade que seu peso. b) 17,2. d) 20,7.

19 - Um estudante tendo encontrado um líquido 22 - A figura abaixo mostra um disco metálico


estranho em sua casa, tentou descobrir o que era. de raio R1 com um orifício circular concêntrico, de
Inicialmente observou que esse era miscível em raio R2. À temperatura to, a relação entre esses
água, cuja densidade ele conhecia (dágua = 1 raios é R1 = 2R2.
g/cm3), mas imiscível em óleo. Logo depois,
colocou em vasos comunicantes, uma coluna de 10
cm de óleo sobre água, obtendo o equilíbrio
26
PROVAS DA AFA 1998 – 2010
À temperatura t > to, a relação entre os raios do
disco R’1 e do orifício R’2 será V (litros)

B
a) R’1 = R’2
b) R’1 = 2R’2 R1
R2 A
c) R’1 = 4R’2
T (K)
d) R’1 = 21 R’2

São feitas três afirmações a respeito desse gás ao


23 - n mols de um gás perfeito estão confinados evoluir de A para B.
em um recipiente como ilustra a figura. A I- A sua pressão aumentou.
temperatura inicial do conjunto, em oC, vale t1. II - Ele realizou trabalho.
Após o aquecimento, a pressão do gás no interior III - Ele recebeu calor.
do recipiente torna-se três vezes maior. É(são) verdadeiro(s) apenas o(s) item(ns)
a) I. c) I e III.
b) II. d) II e III.
h

27 - Para o cultivo de flores em lugares frios, é


necessário a construção de estufas com cobertura
t1 t2 de plástico transparente. Com isso, a temperatura
Nas condições apresentadas, a temperatura final do no interior fica bem mais elevada que a do
conjunto (t2), em kelvin, será exterior. Considere as afirmações:
a) 3t1 c) t1 + 819 I- O calor entra por condução e sai muito
b) 3t1 + 819 d) t1 pouco por convecção.
II - O calor entra por radiação e sai muito
24 - Duas substâncias, A e B, se encontram à pouco por convecção.
mesma temperatura de 20 oC e cada qual III - O calor entra por radiação e sai muito
termicamente isolada. Fornecendo a mesma pouco por condução.
quantidade de calor a cada uma delas, verifica-se IV - O calor entra por condução e convecção e
que a temperatura de A passa a ser de 60 oC e que a só pode sair por radiação.
temperatura de B passa a ser de 80 oC. A partir A(s) alternativa(s) que pode(m) justificar a elevada
dessa situação, as substâncias são colocadas em temperatura do interior da estufa é(são)
contato térmico. A temperatura final de equilíbrio a) I e III. c) IV.
é, em oC, b) I e II. d) II e III.
a) 64. c) 70.
b) 68. d) 72. 28 - Uma máquina térmica, que opera segundo
o ciclo de Carnot e cujo reservatório a baixa
25 - Um corredor despende 60.000 J durante 10 temperatura encontra-se a 27 ºC, apresenta um
s, numa competição de 100 metros rasos. Três rendimento de 40%. A variação da temperatura em
quartos dessa energia são liberados, diretamente, kelvin, da fonte quente, a fim de aumentarmos seu
sob a forma de calor, e o restante é dissipado pelo rendimento em 10%, será
seu corpo em trabalho mecânico. A força média a) 100. c) 500.
que esse atleta desenvolve, em N, é b) 300. d) 600.
a) 150.c) 450.
b) 300.d) 600. 29 - Considere o diagrama abaixo que mostra a
curva de aquecimento de m gramas de uma
26 - Um gás ideal evolui de um estado A para substância pura ao receber calor.
um estado B, de acordo com o gráfico a seguir:

27
PROVAS DA AFA 1998 – 2010
t (o C)
a) 4,0.106 c) 1,6.106
b) 2,4.106 d) zero
t2

32 - Duas esferas eletrizadas com carga Q são


t1
mantidas fixas, em pontos eqüidistantes de um
ponto O onde é colocada uma terceira esfera de
0 Q1 Q2 Q3 Q4 Q (cal)
carga q.
t0 y

É correto afirmar que Q Q


q
a) Após o fornecimento da quantidade de calor (Q2
O x
– Q1)/2 tem-se m/2 gramas da substância no estado
sólido.
b) O calor específico da substância no estado
sólido é Q1/(m.t1). Considere as afirmativas:
c) O calor latente de fusão é Q2/m. I- Se Q.q > 0 haverá equilíbrio estável de q
d) O calor específico da substância no estado em relação a Ox.
líquido é Q1/[m(t2 – t1)]. II - Se Q.q < 0 haverá equilíbrio instável de q
em relação a Oy.
30 - Um projétil de chumbo (c = 120 J/kg.ºC) se III - Tanto para Q.q > 0 ou Q.q < 0 o equilíbrio
movimenta horizontalmente com velocidade de de q será indiferente.
100 m/s e colide com uma parede ficando nela É (são) correta(s)
alojado. Durante o choque, 60% da energia a) apenas I. c) apenas II e III.
cinética se transforma em calor e 80% desse calor b) apenas I e II. d) I, II e III.
é absorvido pelo projétil. A temperatura
correspondente ao ponto de fusão do chumbo é 33 - Um elétron desloca-se na direção x, com

327 ºC e o projétil se encontra inicialmente à velocidade inicial v 0 . Entre os pontos x1 e x2,
temperatura de 25 ºC. Nessas condições, pode-se existe um campo elétrico uniforme, conforme
afirmar que o projétil mostra a figura abaixo.

a) se funde, pois o calor que ele absorve é mais E
que o necessário para ele atingir 327 ºC.
b) não se funde, pois sua temperatura aumenta 
v0
apenas 20 ºC.
e
c) não se funde, pois sua temperatura não varia.
d) não se funde, mas sua temperatura atinge 327
ºC.
O x1 x2 x
31 - Um gás perfeito sofre as transformações
conforme o gráfico a seguir. Desprezando o peso do elétron, assinale a
alternativa que MELHOR descreve o módulo da
p (N/m2)
velocidade v do elétron em função de sua posição
10 .105 C x.
v v
a) c)
4 .105
v0
A B
v0

0 4 10 V (m3)
0 x1 x2 x 0 x1 x2 x
O trabalho, em joules, realizado na transformação
AB é

28
PROVAS DA AFA 1998 – 2010
chave S é aberta, a indicação no amperímetro e no
v v
b) d) voltímetro será, respectivamente,
a) 0; U  2 c) i 2 ; U  2
v0
b) 0; E’ d) i 2 ; E’
v0
37 - Um fabricante de chuveiros deve escolher
0 x1 x2 x 0 x1 x2 x um dos circuitos abaixo.

34 - Considere a associação da figura abaixo: U R R


C2 = 2 F I
C1 = 3 F

R/2

C3 = 4 F

300 V
I U R/2

I
As cargas, em C, de cada capacitor C1, C2 e C3
são, respectivamente Ambos devem funcionar na posição “inverno” ou
a) 200, 400 e 600. c) 600, 400 e 200. “verão”. O responsável pelos projetos afirma que:
b) 200, 300 e 400. d) 600, 200 e 400. I- a potência dissipada por I na posição
inverno e na posição verão é a mesma dissipada
35 - Um fio condutor homogêneo de secção por II nas respectivas posições.
transversal constante de área A e comprimento , II - se queimar um dos resistores em I o
tem resistência elétrica R. Esse fio é dividido em chuveiro ainda pode funcionar.
10 pedaços iguais que são ligados em paralelo, III - o chuveiro II só não funcionará se
formando um cabo, cuja resistência vale R’. Assim queimarem os dois resistores.
sendo, pode-se afirmar que a relação entre R’ e R O técnico está INCORRETO apenas na(s)
vale afirmativa(s)
a) 1 c) 10 a) I . c) III.
b) 1/10 d) 1/100 b) II. d) I e III.
36 - A figura abaixo representa o esquema de
um motor elétrico M, de força contra- 38 - A figura abaixo mostra uma região onde
eletromotriz E’ e resistência interna r’, ligado à existe um campo elétrico de módulo E, vertical e
rede elétrica. apontando para baixo. Uma partícula de massa m e
S carga q, positiva, penetra no interior dessa região
A através do orifício O, com velocidade horizontal,
de módulo v. Despreze os efeitos da gravidade.
M 
V E

v
O

Com a chave S fechada, o amperímetro A indica a


intensidade i da corrente elétrica que circula pelo
circuito e o voltímetro V mede a ddp U’ nos
terminais do motor. Considera-se os fios de ligação Introduz-se na região considerada um campo
com resistência desprezível e os aparelhos de magnético de módulo B com direção perpendicular
medida como sendo ideais. No instante em que a à folha de papel. Para que a partícula se mova, com

29
PROVAS DA AFA 1998 – 2010
velocidade v e em linha reta nessa região, o valor pista horizontal. A razão entre a velocidade do
de B será avião e a velocidade de sua sombra sobre a pista é
a) Ev c) mv
Eq
Ev mq
b) q
d) Ev

39 - Uma carga elétrica q de massa m penetra


num campo de indução magnética B, conforme a
figura abaixo: 
B 1 3 2 3
a) b) 2 c) d)
2 2 2
 02) Afigura abaixo representa o gráfico posição x
v tempo para um móvel em movimento retilíneo.

anteparo

q, m

Sabendo-se que, ao penetrar no campo com


velocidade v, descreve uma trajetória circular, é
INCORRETO afirmar que o tempo gasto para
atingir o anteparo é
a) independente de v. É correto afirmar que
b) proporcional a m. a) a velocidade no instante tA é menor que a
c) proporcional a B. velocidade no instante tB.
d) inversamente proporcional a q. b) em tC, a aceleração do móvel é nula.
c) para tA < t < tC, o movimento é acelerado.
40 - Um feixe de elétrons com velocidade v d) para tB < t < tC, a velocidade do móvel decresce
penetra num capacitor plano a vácuo. A separação de maneira uniforme.
entre as armaduras é d. No interior do capacitor
existe um campo de indução magnética B, 03) Um avião necessita percorrer 750 m de pista
perpendicular ao plano da figura. para decolar. O gráfico a seguir representa a
velocidade desse avião em função do tempo desde
 o instante da partida até a decolagem. Então, a
v
d velocidade atingida no instante da decolagem é

A tensão em que se deve eletrizar o capacitor, para


que o feixe não sofra deflexão, pode ser calculada
por
a) vdB c) B
vd
vd vB
b) d)
B d a) 50 km/h b) 120 km/h
c) 90 km/h d) 180 km/h

04) Um avião, em vôo horizontal a 500m de altura,


AFA 2003/2004 – Física deve lançar uma bomba sobre um alvo móvel. A
velocidade do avião é de 360 km/h e a do alvo é de
01) Durante uma decolagem, ao perder o contato 72 km/h, ambas constantes e de mesmo sentido. Se
com a pista, um avião tem velocidade constante o projétil é lançado com velocidade horizontal
em direção que forma um ângulo de 30º com a constante em relação ao avião de 432 km/h, para

30
PROVAS DA AFA 1998 – 2010
que o alvo seja atingido, a distância d entr o avião
e o alvo, no instante do lançamento, é

a) 350 b) 665 c) 700 d) 735


a) 1500 m b) 2000 m c) 2500 m d) 3000 m
09) Um bloco de massa m é arrastado, à velocidade
05) Um canhão dispara projéteis com velocidade constante, sobre uma superfície horizontal aplicada
 a uma corda, conforme o esquema da figura
v 0 . Desprezando-se os efeitos do ar e adotando-se
abaixo. Sendo  o coeficiente de atrito entre as
g como módulo do vetor aceleração da gravidade,
superfícies, o módulo da força abaixo é
pode-se afirmar que a altura máxima atingida pelo
projétil, quando o alcance horizontal for o
máximo, é
v 02 v 02 v0 v0
a) b) c) d)
4g 2g g 2g

06) Um odômetro de um automóvel é um aparelho


que mede a distância percorrida. Na realidade, esse a)  (T – mg) b)  (mg + Tsen)
aparelho é ajustado para fornecer a distância c) Tcos d) Tsen
percorrida através do número de voltas e do
diâmetro do pneu. Considere um automóvel cujos 10) A massa de uma bola de pingue-pongue é de
pneus, quando novos, têm diâmetro D. Suponha 2,43 g e a sua velocidade terminal, no ar, é de 9
que os pneus tenham se desgastado e apresentem m/s. A força retardadora que atua sobre a bola é
98% do diâmetro original. Quando o velocímetro diretamente proporcional ao quadrado da
assinalar 100 km/h, a velocidade real do automóvel velocidade. Nestas condições, a constante de
será proporcionalidade vale
a) 104 km/h b) 102 km/h a) 3.10– 4 kg/m b) 4.10– 3 kg/m
c) 98 km/h d) 96 km/h –4
c) 6.10 kg/m d) 8.10– 3 kg/m
07) A figura apresenta um plano inclinado no qual 11) Um carro de 1500 kg faz uma curva sem
está fixa uma polia ideal. O fio também é ideal e superelevação, com um raio de 75 m, à velocidade
não há atrito. Sabendo-se que os blocos A e B têm de 54 km/h. O coeficiente de atrito mínimo que
massas iguais, o módulo da aceleração de B é: deve haver entre o pavimento da estrada e os
pneus, a fim de impedir a derrapagem do carro, é
a) 0,1 b) 0,3 c) 0,5 d) 0,6

12) Para manter uma lancha a uma velocidade


constante de 36 km/h, é necessário que o motor
a) 2,5 m/s2 b) 4,0 m/s2 forneça às hélices propulsoras uma potência de 40
c) 5,0 m/s2 d) 7,5 m/s2 cv (29400 W). se a lancha estivesse sendo
rebocada a esta velocidade, qual seria a tensão no
08) Um homem de massa 70 kg está subindo por cabo de reboque?
um fio ideal com aceleração igual a 0,50 m/s2. a) 294 N b) 2940 N c) 8160 N d) 816 N
Nessas condições, a intensidade da tração, em
newtons, no fio, vale 13) Duas crianças estão brincando de atirar bolas
de gude dentro de u, caixa no chão. Elas usam um
brinquedo que lança as bolas pela descomprenssão
de uma mola que é colocada horizontalmente sobre

31
PROVAS DA AFA 1998 – 2010
uma mesa onde o atrito é desprezível. A primeira vão cair numa rede de segurança, na mesma altura
criança comprime a mola 2 cm e a bola cai a 1,0 m que o canhão. Veja a figura abaixo.
antes do alvo, que está a 3,0 m horizontalmente da
borda da mesa. A deformação da mola imposta
pela segunda criança, de modo que a bola atinja o
alvo é

Desprezando a resistência do ar e considerando sen


37º = 0,6 e cos 37º = 0,8, pode-se afirmar que o
alcance A atingido pelo homem é
a) 60 m b) 48 m c) 36 m d) 24 m

17) Quanto a um satélite artificial geoestacionário,


a) 1,7 cm b) 2,0 cm c) 3,0 cm d) 9,0 cm e órbita circular em torno da Terra, afirma-se que

14) Durante uma manobra, ao atingir velocidade


nula, um avião desliga o motor e após queda livre
realiza um looping, conforme indica a figura.

I. A força que o mantém em órbita é de natureza


gravitacional
II. seu período é de 24 horas
III. sua aceleração é nula

É(são) correta(s), apenas a(s) afirmativa(s)


a) III b) I e II c) I e III d) II e III

18) O Centro de Lançamento de Alcântara (CLA)


está preparado para lançar foguetes de sondagem e
Desprezando-se a resistência com o ar e veículos lançadores de satélites. Localizado na
considerando-se a trajetória do looping circular de costa do nordeste brasileiro, próximo ao Equador,
raio R, a menor altura h para que o avião consiga a posição geográfica do CLA aumenta as
efetuar esse looping é condições de segurança e permite menores custos
a) 1,5 R b) 2,0 R c) 2,5 R d) 3,0 R de lançamento. Afirma-se que são fatores
determinantes do menor custo de lançamento no
15) Um foguete cuja massa vale 6 toneladas é CLA (latitude 0º) em relação a outros centros de
colocado em posição vertical para lançamento. Se lançamento situados em regiões de maiores
a velocidade do escape dos gases vale 1 km/s, a latitudes:
quantidade de gases expelida por segundo, a fim I. maior velocidade tangencial, devido à inércia do
de proporcionar o empuxo necessário para dar ao movimento de rotação da Terra
foguete uma aceleração inicial para cima igual a 20 II. menor aceleração da gravidade, devido ao
m/s2 é movimento de rotação da Terra.
a) 180 kg b) 120 kg c) 100 kg d) 80 kg III. menor distância das órbitas próprias para
satélites geoestacionários.
16) Num circo, um homem-bala, de massa 60 kg, é
disparado por um canhão com a velocidade de m/s, São verdadeiras as assertivas
sob um ângulo de 37º com a horizontal. Sua a) apenas I e II
parceira, cuja massa é 40 kg, está numa plataforma b) apenas II e III
localizada no topo da trajetória. Ao passar pela c) apenas I e III
plataforma, o homem-bala e a parceira se reúnem e d) I, II e III
32
PROVAS DA AFA 1998 – 2010
19) Um aquário, com um peixe, está equilibrado 23) Considere um recipiente fechado contendo um
no prato de uma balança. Num certo instante, o líquido que ocupa somente 3/4 do volume desse
peixe nada em direção à superfície. É correto recipiente. Quando esse líquido está em equilíbrio
afirmar que dinâmico com o seu vapor, pode-se afirmar que
a) a leitura da balança aumenta I. o vapor, nas condições descritas, é denominado
b) a leitura da balança diminui vapor saturante
c) não há alteração na leitura da balança II. o vapor está exercendo pressão máxima e essa
d) o enunciado é inconclusivo cresce com o aumento da temperatura
III. não há transferência de moléculas entre o
20) Ao se colocar água muito quente num copo de líquido e o vapor
vidro comum geralmente ele trinca, enquanto que IV. essa situação de equilíbrio líquido-vapor
um copo de vidro pirex dificilmente trinca. Isso ocorre nos botijões de “gás” liquefeito de petróleo,
ocorre devido ao fato de que usados na cozinha
a) o calor específico do vidro pirex é maior que o
do vidro comum São verdadeiras
b) para aquecimentos iguais o vidro comum sofre a) apenas I, II e III c) apenas II, III e IV
maior variação de temperatura b) apenas I, II e IV d) I, II III e IV
c) o coeficiente de dilatação do vidro comum é
maior que o do vidro pirex 24) A figura apresenta o esquema simplificado da
d) são ambos materiais anisótropos experiência de Joule. O bloco tem massa 10 kg e
está a uma altura H = 4,20 m. Quando ele cai,
21) No início do curso de compreensão, o cilindro produz o movimento das pás, mergulhadas em 1
de um motor diesel contém 800 cm3 de ar, à kg de água. Supondo que toda variação de energia
pressão atmosférica (1 atm) e à temperatura de 27º potencial gravitacional do sistema foi transformada
C. No fim desse curso, o volume de ar foi reduzido em calor, considerando Cágua = 1 cal/gºC e 1 cal =
para 50 cm3 e a pressão manométrica aumentada 4,2 J, a variação de temperatura da água é
para 40 atm. A variação de temperatura da massa
de ar no cilindro foi de
a) 450º b) 477º C c) 177º C d) 750º

22) Suponha que uma determinada quantidade de


calor Q fula, em regime estacionário, através de
uma barra de uma superfície mantida à
temperatura 1, para a superfície oposta mantida à
temperatura 2, nas situações 1 e 2, abaixo
ilustradas. a) 0,1º C b) 0,4º C c) 0,8º C d) 1,0º C

25) Durante tempestade, um raio atinge um avião


durante o vôo. Pode-se afirmar que a tripulação

A mesma quantidade de calor Q gasta tempos t1


e t2 para atravessar a barra nas situações 1 e 2, a) não será atingida, pois aviões são obrigados a
Δt 2 portar um pára-raios em sua fuselagem
respectivamente. A razão vale
Δt1 b) será atingida em virtude da fuselagem metálica
a) 1/4 b) 1/2 c) 2 d) 4 ser boa condutora de eletricidade

33
PROVAS DA AFA 1998 – 2010
c) não sofrerá dano físico pois a fuselagem
metálica atua como blindagem
d) será parcialmente atingida, pois a carga será 28) Os gráficos a seguir representam a tensão (U) e
homogeneamente distribuída na superfície interna a intensidade de corrente (i) num aquecedor, em
do avião função do tempo (t)

26) Considere o campo elétrico uniforme criado


por duas placas plenas e paralelas.Um próton e
uma partícula  são lançados com a mesma

velocidade v 0 paralelas às placas, como mostra a
figura.
O consumo de energia elétrica,em kWh, nos trinta
minutos de funcionamento, é
a) 0,6 b) 1,2 c) 1,8 d) 3,6

29) Três lâmpadas iguais de tensão nominal 12 V


cada uma, estão ligadas a uma associação de duas
Sabendo-se que a partícula  é o núcleo do átomo baterias, também de 12 V, como mostra a figura.
de hélio (He), constituída, portanto, por 2 prótons e Os fios de ligação são de resistência elétrica
desprezível.
2 nêutosn, a razão dp da entre as distâncias
horizontais percorridas pelo próton (dp) e pela
partícula  (d) até colidirem com a placa negativa
é
1 1 2 2
a) b) c) d)
4 2 2 4

27) O eixo Ox é mediatriz do segmento em cujas


extremidades se encontram duas cargas iguais.
Com base nos dados acima, pode-se afirmar que
I. com a chave K aberta, as lâmpadas brilharão
com igual intensidade.
II. com a chave K fechada, a lâmpada A apaga e as
lâmpadas B e C brilharão com a intensidade para
qual foram fabricadas.
III. estando a chave K aberta ou fechada, nenhuma
lâmpada queimará.
A opção que representa o potencial elétrico V, São verdadeiras as assertivas
devido a essas cargas, ao longo do eixo Ox, é a) apenas I e II c) apenas II e III
b) apenas I e III d) I, II e III
a) b)
30) Uma partícula eletrizada com carga negativa é

lançada com velocidade v numa região onde há

dois campos

uniformes: um magnético B e um
elétrico E , conforme figura.

c) d)

34
PROVAS DA AFA 1998 – 2010
5 –3 a) iniciais. c) instantâneas.
Sabendo que v = 2,0.10 m/s e B = 1,0.10 T,
calcule a intensidade de vetor campo elétrico, em b) finais. d) médias.
volts por metro, de modo que a partícula descreva
um movimento retilíneo uniforme. 4) Observe os pontos A e B marcados nas pás de
a) 1,0.108 b) 2,0.102 c) 5,0.101 d) 5,0.100 um ventilador que gira com freqüência constante,
conforme a figura abaixo.
AFA 2004/2005 – Física
1) Uma equipe de resgate se encontra num
helicóptero, parado em relação ao solo, a 305 m de
altura. Um pára-quedista abandona o helicóptero e
cai livremente durante 1,0 s, quando abre o pára-
quedas. A partir desse instante, mantendo-se
constante sua velocidade, o pára-quedista atingirá
o solo em
a) 15 s b) 28 s c) 30 s d) 60 s
É INCORRETO afirmar que em A
a) a velocidade escalar é maior que em B.
2) Certa mãe, ao administrar um medicamento para
b) a velocidade angular é a mesma que em B.
o seu filho, utiliza um conta-gotas pingando em
c) o período é o mesmo que em B.
intervalos de tempo iguais. A figura a seguir
d) a aceleração é menor que em B.
mostra a situação no instante em que uma das
gotas está se soltando.
5) Considere uma partícula M lançada
verticalmente para cima com uma velocidade de
30 m/s. No mesmo instante, uma outra partícula N
é lançada horizontalmente de um ponto situado a
120 m do solo. Sabe-se que elas irão se chocar em
Y um ponto Q, conforme a figura. Desprezando os
X efeitos do ar, a altura do ponto Q é

Considerando que cada pingo abandone o conta


gotas com velocidade nula e desprezando a
resistência do ar, pode-se afirmar que a razão X ,
Y
entre as distâncias X e Y, mostradas na figura, vale
a) 2 b) 1/2 c) 1/4 d) 4

3) Os gráficos a seguir referem-se a movimentos


unidimensionais de um corpo em três situações
diversas, representando a posição como função do a) 80 m b) 60 m c) 40 m d) 15 m
tempo.
6) Um bloco encontra-se em repouso sobre um
plano inclinado que se move com aceleração
horizontal de intensidade a, como indica a figura.
Desprezando-se o atrito entre quaisquer
superfícies, o valor de a é proporcional a

Nas três situações, são iguais as velocidades


35
PROVAS DA AFA 1998 – 2010

a) cos  c) cotg 
b) cossec  d) tg 

7) O conjunto abaixo, constituído de fio e polia


ideais, é abandonado do repouso no instante t = 0 e
a velocidade do corpo A varia em função do tempo I - a força centrípeta é a força resultante.
segundo o gráfico dado. II - variando a velocidade o período permanece
inalterado.
III - a tensão no fio diminui com o aumento de .
Estão corretas as afirmativas
a) I e II apenas. c) II e III apenas.
b) I e III apenas. d) I, II e III.

10) Um corpo é abandonado em queda livre, a


partir do repouso, sob ação da gravidade. Se sua
velocidade, depois de perder uma quantidade E de
Desprezando o atrito, a razão entre a massa de A e energia potencial gravitacional, é v, pode-se
a massa de B é concluir que a massa do corpo é dada por
a) 1/2 b) 1/3 c) 3/2 d) 2 2E 2v 2
a) 2Ev b) c) 2Ev2 d)
v2 E
8) Uma partícula descreve trajetória circular com
movimento uniforme, no sentido horário, como 11) Um atirador utiliza alvos móveis. Em um
mostra a figura. treinamento, deixa cair um bloco de massa M, a
partir de uma altura h. Ao final do primeiro
segundo de queda, o bloco é atingido
horizontalmente por uma bala de massa m e
velocidade v. A bala se aloja no bloco e observa-se
um desvio horizontal x na sua trajetória em relação
ao ponto que tocaria o chão, caso não houvesse
acontecido a colisão. O valor de x é dado por
1 1
O conjunto de vetores

que melhor representa a  2h  2  2h  2 m
força resultante F , a velocidade v e a aceleração a)   (M + m )v b)   [
+ m)
]v
  g   g  ( M
a da partícula, no ponto P indicado na figura é 1 1
a) c)  2 (M + m )  2
c) [  2h  − 1] [ ]v d) [  2h  − 1] [
m
]v
 g  m  g  (M + m )

12) Um lavador de carros segura uma mangueira


do modo que aparece na figura abaixo:

b) d)

Qual a força necessária para manter o bico da


mangueira estacionário na horizontal, sabendo que
9) O pêndulo da figura abaixo gira apresentando a vazão da água é de 0,60 kg/s, com a velocidade
um ângulo  de abertura em relação à vertical. de saída na mangueira de 25 m/s?
Afirma-se que a) 5,0 N b) 10,0 N c) 15,0 N d) 20,0 N

36
PROVAS DA AFA 1998 – 2010
13) A figura abaixo mostra uma vista superior de Um objeto é observado através de uma lupa. Para
dois espelhos planos E1 e E2 que formam entre si que se consiga observar seus mínimos detalhes, é
um ângulo . Sobre o espelho E1 incide um raio de necessário que o objeto esteja localizado
luz horizontal e que forma com este espelho um ................... e, neste caso, a imagem conjugada é
ângulo . Após reflexão nos dois espelhos, o raio ...................... .
emerge formando um ângulo  com a normal ao a) entre a lente e seu foco / virtual e direita em
espelho E2. O ângulo  vale relação ao objeto.
b) entre a lente e seu foco / real e invertida em
relação ao objeto.
c) além do foco / virtual e invertida em relação ao
objeto.
d) além do foco / real e direita em relação ao
objeto.

a)  +  c)  +  + 90 17) Uma mola, de massa desprezível, se distende


de b quando equilibra um bloco de massa m. Sabe-
b)  +  − 90 d)  – 
se que no instante t = 0, o bloco foi
14) Uma fonte pontual de luz monocromática está abandonado do repouso a uma distância  abaixo
imersa numa piscina de profundidade h. Para que a de sua posição de equilíbrio. Considerando g a
luz emitida por essa fonte não atravesse a aceleração da gravidade e desprezando os atritos, a
superfície da água para o ar, coloca-se na equação do movimento resultante em função do
superfície um anteparo opaco circular cujo centro tempo t é
encontra-se na mesma vertical da fonte. O raio
mínimo desse anteparo é a) x =  cos ( gb t ) c)
 b 
x =  sen  t
 g 
 
Considere
nAR – índice de refração do ar b)
 g 
x =  cos  t
 b 
d) (
x =  tg gb t )
nÁGUA – índice de refração da água  

a) h.tg[arc sen(nAR/ nÁGUA)] c) h.sen(nAR/ nÁGUA) 18) Um bloco ligado a uma mola presa a uma
tg(nAR / nÁGUA ) parede oscila em torno de 0, sobre uma superfície
b) d) h.arc tg[sen(nAR/ nÁGUA)]
h sem atrito, como mostra a figura.

15) Para que os raios luminosos sempre convirjam


na retina, os músculos ciliares, que garantem
também sustentação mecânica ao globo ocular,
podem contrair-se variando a curvatura das faces
do cristalino. Quando um objeto se aproxima do
olho, o cristalino
a) atua como lente convergente e os músculos
ciliares vão se contraindo, diminuindo a distância
focal do cristalino. O gráfico que MELHOR representa a
b) atua como lente convergente e os músculos energia cinética EC em função de x é:
ciliares ficam relaxados. a) c)
c) atua como lente divergente e os músculos
ciliares vão se contraindo, diminuindo a distância
focal do cristalino.
d) atua como lente divergente e os músculos
ciliares ficam relaxados.
b) d)
16) Assinale a alternativa que preenche correta e
respectivamente as lacunas abaixo.

37
PROVAS DA AFA 1998 – 2010
D (figura abaixo), a capacitância do capacitor
2

19) Uma onda transversal é aplicada sobre um fio


preso pelas extremidades, usando-se um vibrador
de freqüência f = 60Hz. A distância média entre os
pontos que praticamente não se movem é 40 cm. A
velocidade das ondas nesse fio é, em m/s, igual a a) triplica c) reduz à terça parte
a) 80 b) 60 c) 48 d) 20 b) dobra d) reduz à metade
20) Uma esfera A, com carga positiva, é mantida 23) Na figura, temos o gráfico da intensidade em
em uma região plana e horizontal através de um função do tempo para uma corrente elétrica que
anteparo semicircular, com extremidades B e C, percorre um fio. A intensidade média da corrente
como mostra a figura. que passa por uma secção reta do fio entre os
instantes 0 e 6,0 s é, em ampères,

A esfera pode se deslocar sem atrito. Nos pontos B


e C são fixadas pequenas esferas com cargas de 8
C e 64 C, respectivamente. A tangente do a) 1,0 b) 1,5 c) 2,0 d) 2,5
ângulo , para o qual a esfera A permanece em
equilíbrio é 24) Três lâmpadas 1, 2 e 3 são conectadas a uma
a) 0,30 b) 0,40 c) 0,50 d) 0,60 bateria, com tensão constante U, conforme a
figura. Se a lâmpada 2 queimar, então
21) Uma partícula de carga q e massa m é lançada
com velocidade v, perpendicularmente ao campo
elétrico uniforme produzido por placas paralelas de
comprimento L e separadas por uma distância D.
A partícula penetra no campo num ponto
eqüidistante das placas e sai tangenciando a borda
da placa superior, conforme representado na
figura.

a) a potência lançada pela bateria diminui.


b) as potências dissipadas pelas lâmpadas 1 e 3
aumentam.
c) a resistência equivalente do circuito diminui.
d) a corrente total do circuito permanece
Desprezando ações gravitacionais, a intensidade do constante.
campo elétrico pode ser calculada por
mLv 2 mv 2 2mDv mDv 2 25) No circuito abaixo, F é uma fonte de
a) 2
b) c) d) 2
qD qLD qL qL resistência interna desprezível, L uma lâmpada de
resistência elétrica constante e R um reostato cuja
22) As placas de um capacitor a ar estão separadas resistência varia de r1 até r2. Dentre os gráficos
entre si por uma distância igual a D. Ao se apresentados abaixo, o que MELHOR representa
introduzir entre as placas, simetricamente em a potência P lançada pela fonte em função da
relação a elas, uma chapa metálica de espessura resistência (R) do reostato é o da alternativa

38
PROVAS DA AFA 1998 – 2010
28) Espectrômetros de massa são aparelhos
utilizados para determinar a quantidade relativa de
isótopos dos elementos químicos. A figura mostra
o esquema de um espectrômetro e a trajetória
descrita por um íon de massa m e carga 2e.
a) c)

b) d)

Esse íon é acelerado a partir do repouso, na região


I, por um campo elétrico uniforme de intensidade
E.
Ao penetrar na região II, descreve uma trajetória
circular sob efeito de um campo magnético de
26) A figura seguinte representa duas espiras intensidade B.
circulares, concêntricas e coplanares percorridas Desprezando-se as ações gravitacionais, a massa m
por correntes elétricas contínuas cujo sentido está do íon pode ser calculada por
indicado. RB 2e REe RB2 eB 2
a) b) c) d)
2E B2 Ee 2RE

29) A figura a seguir mostra uma espira retangular



abcd imersa num campo magnético uniforme B ,
que a atravessa perpendicularmente.

O campo magnético gerado por estas duas espiras


poderá ser nulo
a) apenas em C. c) em C ou D.
b) apenas em D. d) em nenhum deles.

27) Um campo magnético uniforme B é aplicado
na direção e sentido do eixo y onde um elétron é Se metade da espira for girada no sentido anti-
lançado no sentido positivo do eixo z. horário, como mostra a ilustração acima, pode-se
afirmar que, durante este processo, a corrente
elétrica induzida
a) é constante.
b) varia linearmente com o tempo.
c) independe da velocidade de giro.
d) tem o sentido de b para a.

A trajetória descrita pelo elétron é 30) Uma espira metálica é abandonada, a partir do
a) retilínea, na direção do eixo Ox. repouso, de uma altura h acima do solo. Em
b) circular, situada no plano xz. determinado trecho, ela passa por uma região

onde
c) parabólica, situada no plano yz. existe um campo magnético uniforme B , conforme
d) hélice cilíndrica, com eixo Oz. mostra a figura.
39
PROVAS DA AFA 1998 – 2010
3) Analise as afirmativas abaixo sobre movimento
circular uniforme:
I - A velocidade vetorial tem direção variável.
II - A resultante das forças que atuam num corpo
que
descreve esse tipo de movimento não é nula.
III - O módulo da aceleração tangencial é nulo.
Está(ão) correta(s)
a) I apenas b) I e III apenas
c) II e III apenas. d) I, II e III

Pode-se afirmar que 4) O movimento da coroa dentada (A) de uma


a) há conservação da energia mecânica durante bicicleta é transmitido a uma catraca (B) localizada
toda a queda. no eixo da roda traseira (C) por meio de uma
b) sua velocidade final é menor que 2gh . corrente. A opção que representa a bicicleta mais
c) sua velocidade é constante quando totalmente veloz para o mesmo número de pedaladas do
 ciclista é
imersa em B .

d) sua aceleração diminui ao penetrar em B e

aumenta ao abandonar a região de B .

AFA 2005/2006 – Física


1) O gráfico abaixo mostra como variou a
velocidade de um atleta durante uma disputa de
100 m rasos.

5) Um avião a jato, cuja massa é de 40 toneladas,


ejeta, durante 5 segundos, 100 kg de gás e esse gás
sofre uma variação de velocidade de 500 m/s. Com
base nessas informações, analise as seguintes.
Sendo de 8,0 m/s a velocidade média deste atleta, afirmativas:
pode-se afirmar que a velocidade v no instante em I - A variação da velocidade do avião é de 1,25
que ele cruzou a linha de chegada era, em m/s, m/s.
a) 5,0 b) 3,5 c) 8,5 b) 3,5 d) 10 II - A força aplicada no avião é de 104 N.
III - O impulso sofrido pelo avião vale 5.104
2) Um operário puxa a extremidade de um cabo kg.m/s.
que está enrolado num cilindro. À medida que o Está(ão) correta(s)
operário puxa o cabo o cilindro vai rolando sem a) apenas I. c) apenas I e III .
escorregar. Quando a distância entre o operário e o b) apenas I e II. d) I, II e III.
cilindro for igual a 2 m (ver figura abaixo), o
deslocamento do operário em relação ao solo será 6) Os satélites de comunicação são operados
de. normalmente em órbitas cuja velocidade angular w
é igual à da Terra, de modo a permanecerem
imóveis em relação às antenas receptoras. Na
figura abaixo, estão representados dois destes
satélites, A e B, em órbitas geoestacionárias e em
diferentes alturas. Sendo a massa de A maior que a
a) 1 m b) 2 m c) 4 m d) 6 m de B, pode-se afirmar que as relações entre os
módulos das velocidades vA e vB e os períodos de

40
PROVAS DA AFA 1998 – 2010
rotação TA e TB dos satélites A e B estão respectivamente, ligados por um fio ideal, formam
representados corretamente na alternativa um sistema que submetido a ação de uma força
constante F de intensidade 15 N, desloca-se com
aceleração de 1 m/s2, conforme a figura abaixo. Se
a tração no fio que liga os blocos durante o
deslocamento é de 9 N, pode-se afirmar que a
razão entre os coeficientes de atrito dos blocos A e
a) vA = vB e TA = TB c) vA > vB e TA = TB B com a superfície vale
b) vA < vB e TA < TB d) vA > vB e TA > TB

7) Duas partículas são lançadas nos pontos A e B


com a mesma velocidade v0, conforme indica a 1 2
figura abaixo: a) c)
3 3

2
b) d) 1
3

10) Uma barra rígida homogênea de comprimento


2 L e massa m está apoiada em dois suportes A e B,
como mostra a figura abaixo.

Enquanto a partícula de massa m passa por um


trecho em elevação, a outra, de massa M, passa por
uma depressão com a mesma forma e
“profundidade” h. Desprezando-se quaisquer
forças dissipativas, pode-se afirmar que a razão t A O gráfico que melhor indica a intensidade NA da
tB
reação que o apoio A exerce sobre a barra, em
entre os tempos gastos pelas partículas para função da intensidade da força F aplicada na
atingirem os pontos D e C é. extremidade é.
a) menor que 1, se m > M.
b) maior que 1, independentemente da razão m/M.
c) igual a 1, independentemente da razão m/M.
d) pode ser igual a 1, se m < M.

8) Uma partícula de massa m é lançada


bliquamente com velocidade v0 próxima à
superfície terrestre, conforme indica a figura
abaixo. A quantidade de movimento adquirida pela
partícula no ponto Q, de altura máxima, é

11) Uma pessoa deita-se sobre uma prancha de


madeira que flutua mantendo sua face superior no
mesmo nível da superfície da água.

A prancha tem 2 m de comprimento, 50 cm de


largura e 15 cm de espessura. As densidades da
água e da madeira são, respectivamente, 1000
9) Os blocos A e B , de massas iguais a 2 kg e 3 kg,
41
PROVAS DA AFA 1998 – 2010
3 3 2 15) A figura mostra uma barra metálica de secção
kg/m e 600 kg/m . Considerando g = 10 m/s ,
pode-se afirmar que o peso da pessoa é reta constante sendo aquecida por uma chama de
a) 600 N c) 400 N um fogareiro.
b) 700 N d) 500 N

12) Um líquido é colocado em um recipiente


ocupando 75% de seu volume. Ao aquecer o
conjunto (líquido + recipiente) verifica-se que o
volume da parte vazia não se altera. A razão entre
os coeficientes de dilatação volumétrica do

material do recipiente e do líquido M é Quando se estabelece o regime estacionário de
L
condução do calor, os termômetros A e C registram
200 ºC e 80 ºC, respectivamente. Assim, a leitura
no termômetro B será de

13) Dispõe-se de uma balança de braços iguais e


recipientes idênticos contendo água cuja 16) Uma das aplicações do fenômeno da condução
temperatura está indicada na figura de cada térmica é o uso de telas metálicas. Sabe-se que,
alternativa. Aquela que mostra corretamente a colocando um recipiente de vidro comum
situação de equilíbrio é diretamente numa chama, ele se rompe. No
entanto, interpondo uma tela metálica entre a
chama e o recipiente, a ruptura não acontece
porque.

a) os gases não queimam na região logo acima da


tela, pois ali a temperatura não alcança valores
suficientemente elevados.
b) há uma diferença entre os coeficientes de
dilatação linear da tela e do recipiente.
c) a tela, por ser boa condutora, transmite
rapidamente o calor para todos os pontos de sua
própria extensão.
d) como são dois corpos, o aumento da
temperatura não é suficiente para que seja
verificada uma dilatação aparente.

17) Um sistema é formado por dois reservatórios,


14) Para intervalos de temperaturas entre 5 ºC e 50 A e B, de mesmo volume, ligados por um tubo
ºC, o calor específico (c) de uma determinada longo, com área de secção transversal constante e
substância varia com a temperatura ( t ) de acordo igual a S, conforme indica o esquema abaixo:
com a equação c = 1 t + 2 , onde c é dado em
60 15
cal/gºC e t em ºC. A quantidade de calor necessária
para aquecer 60 g desta substância de 10 ºC até 22
ºC é
Enche-se os reservatórios com dois tipos de gases
ideais, à mesma temperatura absoluta T0 e mesmo
volume V0, que ficam separados por um êmbolo
que pode deslizar sem atrito. O êmbolo permanece
no interior do tubo durante uma transformação em

42
PROVAS DA AFA 1998 – 2010
que a temperatura do gás do reservatório B é transparentes com índices de refração n1 = 2 e n2 =
mantido sob temperatura constante T0. Assim, o 4, como mostra a figura abaixo:
deslocamento do êmbolo foi de

A imagem que se obterá com essa lente será

18) Com recursos naturais cada vez mais escassos,


urge-se pensar em novas fontes alternativas de
energia. Uma das idéias sugeridas consiste em se
aproveitar a energia térmica dos oceanos, cuja
água pode apresentar em uma superfície uma
temperatura de 20 ºC e no fundo temperatura em
torno de 5,0 ºC. Um motor térmico operando neste 22)Considere o sistema apresentado na figura
intervalo de temperatura poderia ter um abaixo formado por um conjunto de três molas
rendimento de ideais e de constantes elásticas iguais acopladas
a) 3,0% c) 9,0% em paralelo e ligadas por meio de uma haste de
b) 7,5% d) 27% massa desprezível a um segundo conjunto,
formado por duas massas M e m, tal que M = 2m.
19) A figura mostra um cilindro que contém um Considere, ainda, que o sistema oscila
gás ideal, com um êmbolo livre para se mover sem verticalmente em MHS (movimento harmônico
atrito. À temperatura de 27 ºC, a altura h na qual o simples) com freqüência f1.
êmbolo se encontra em equilíbrio vale 20 cm.

Aquecendo-se o cilindro à temperatura de 39 ºC e


mantendo-se inalteradas as demais características
da mistura, a nova altura h será, em cm,
Se o fio ideal que une a massa m ao sistema for
cortado simultaneamente com a mola central da
associação de molas, o sistema passará a oscilar
20 - Considere uma superfície de separação plana e
com uma nova freqüência f2, tal que a razão f2/f1
horizontal entre o ar e a água. Se uma onda
seja
luminosa (L) e uma onda sonora (S) incidem sobre
essa superfície, com um ângulo de incidência , a
opção que MELHOR ilustra a configuração física
das ondas luminosa e sonora, que se refratam é
23) Duas pequenas esferas eletrizadas com cargas
positivas iguais estão fixas nos pontos A e B, como
mostra a figura abaixo:

21) Considere um objeto AB colocado sobre o eixo


óptico de uma lente delgada biconvexa de raio de
curvatura R, composta por dois meios
43
PROVAS DA AFA 1998 – 2010
Considerando apenas a influência de forças
elétricas sobre uma carga q de prova em equilíbrio
no ponto P, afirma-se que
I - se q é positiva, então está em equilíbrio estável
em relação ao segmento AB.
II - se q é negativa, então está em equilíbrio
instável em relação à mediatriz do segmento AB. Variando a posição do cursor C, a potência
III - se q é negativa, então está em equilíbrio dissipada pelo fio AB será
instável em relação ao segmento AB.
IV - se q é positiva, então está em equilíbrio
estável em relação à mediatriz do segmento AB.
Estão corretas apenas
a) I e II c) III e IV.
b) II e III d) I e III. 27) No circuito abaixo, para que a bateria de f.e.m.
ɛ1 e resistência interna r1 funcione como receptor,
24)Uma partícula de carga q e massa m penetra o valor da resistência R poderá ser igual a
perpendicularmente às linhas de força de um
campo elétrico uniforme E com a menor
velocidade suficiente para sair sem tocar as placas,
como mostra a figura abaixo:

a) 15 c) 25
b) 20 d) 30

28 - Considere o circuito da figura abaixo:


A velocidade que ela deixa o campo elétrico é

25) Uma casca metálica esférica e não eletrizada A leitura do voltímetro ideal V é
envolve uma partícula eletrizada. Afirma -se que
I - a casca esférica não interfere no campo elétrico
gerado pela partícula.
II - em pontos exteriores à casca o campo elétrico é
nulo III - qualquer ponto interior à casca apresenta
o mesmo potencial elétrico.
Está(ão) correta(s) apenas 29) A figura abaixo mostra uma espira condutora
a) I. c) III. quadrada, de lado ℓ= 0,1 m, que gira com
b) II e III. d) I e II. velocidade angular w constante em torno do eixo z
num campo magnético uniforme de intensidade B
26) Uma bateria fornece tensão constante U e está = 1T, na direção do eixo x. A velocidade angular
ligada a um fio homogêneo AB de seção da espira para que seja induzida uma f.e.m. de, no
transversal constante e comprimento L, conforme máximo, 10 V é
mostra o circuito esquematizado abaixo:

44
PROVAS DA AFA 1998 – 2010
o som do disparo chegar ao ouvido do espectador,
é
a) 0,5 m c) 0,7 m
b) 0,6 m d) 0,8 m

2) Um pára-quedista, ao saltar na vertical de um


avião que se desloca na horizontal em relação ao
solo, sofre uma redução crescente da aceleração
até atingir a velocidade limite. O gráfico que
MELHOR representa o módulo da componente
a) 100 rad/s c) 1000 rad/s vertical da velocidade do pára-quedista em função
b) 200 rad/s d) 2000 rad/s do tempo, a partir do instante em que começa a
cair, é
30) O esquema a seguir é de um aparelho utilizado
para medir a massa dos íons.

3) O gráfico abaixo representa o movimento de


subida de um protótipo de foguete em dois estágios
lançado a partir do solo.
O íon de carga +q é produzido, praticamente em
repouso, por meio da descarga de um gás,
realizada na fonte F. O íon é, então, acelerado por
uma d.d.p. U, penetrando, depois, num campo
magnético B . No interior do campo, o íon
descreve uma órbita semicircular de raio r,
terminando por atingir uma placa fotográfica, na
qual deixa uma imagem. A massa do íon pode ser
calculada por
Após ter atingido a altura máxima, pode-se afirmar
que o tempo de queda livre desse protótipo será de
a) 1 s c) 3 s
b) 2 s d) 4 s

4) Um avião voa na direção leste a 120 km/h para


ir da cidade A à cidade B. Havendo vento para o
sul com velocidade de 50 km/h, para que o tempo
AFA 2006/2007 – Física de viagem seja o mesmo, a velocidade do avião
deverá ser
1) Uma pessoa está observando uma corrida a 170 a) 130 km/h c) 170 km/h
m do ponto de largada. Em dado instante, dispara- b) 145 km/h d) 185 km/h
se a pistola que dá início à competição. Sabe-se
que o tempo de reação de um determinado 5) Uma partícula descreve movimento circular
corredor é 0,2 s, sua velocidade é 7,2 km/h e a
passando pelos pontos A e B com velocidades V A e
velocidade do som no ar é 340 m/s. A distância
desse atleta em relação à linha de largada, quando
45
PROVAS DA AFA 1998 – 2010
VB conforme a figura abaixo. A opção que
representa o vetor aceleração média entre A e B é

8) Durante um show de patinação, o patinador,


representado na figura abaixo, descreve uma
evolução circular, com velocidade escalar
constante, de raio igual a 10,8 m. Considerando
6) A figura abaixo representa as trajetórias de dois desprezíveis quaisquer resistências, a velocidade
projéteis A e B lançados no mesmo instante num do patinador, ao fazer a referida evolução, é igual
local onde o campo gravitacional é constante e a a.
resistência do ar é desprezível.

Dados: sen 53° = 0,80


cos 53° = 0,60
Ao passar pelo ponto P, ponto comum de suas
trajetórias, os projéteis possuíam a mesma a) 12 m/s c) 8 m/s
a) velocidade tangencial. b) 7 m/s d) 9 m/s
c) aceleração centrípeta.
b) velocidade horizontal. 9) Um projétil de massa m incide orizontalmente
d) aceleração resultante. sobre uma tábua com velocidade v1 e a bandona
com velocidade, ainda horizontal, v2.
7) Duas esteiras mantêm movimentos uniformes e Considerando-se constante a força exercida pela
sincronizados de forma que bolinhas tábua de espessura d, pode-se afirmar que o tempo
sucessivamente abandonadas em uma delas de perfuração é dado por
atingem ordenadamente recipientes conduzidos
pela outra. Cada bolinha atinge o recipiente no
instante em que a seguinte é abandonada. Sabe-se
que a velocidade da esteira superior é v e que o
espaçamento das bolinhas é a metade da distância
d, entre os recipientes. Sendo g a aceleração da
gravidade local, a altura h, entre as esteiras, pode
ser calculada por

10) Três blocos, cujas massas mA = mB = m e mC =


2m, são ligados através de fios e polias ideais,
conforme a figura. Sabendo-se que C desce com
uma aceleração de 1 m/s2 e que 0,2 é o coeficiente

46
PROVAS DA AFA 1998 – 2010
de atrito entre B e a superfície S, pode-se afirmar
que o coeficiente de atrito entre A e S vale

13) Uma prancha de comprimento 4 m e de massa


2 kg está apoiada nos pontos A e B, conforme a
figura. Um bloco de massa igual a 10 kg é
colocado sobre a prancha à distância x = 1 m da
extremidade da direita e o sistema permanece em
repouso. Nessas condições, o módulo da força que
a prancha exerce sobre o apoio no ponto B é, em
newtons,
a) 0,10 c) 0,30
b) 0,20 d) 0,40

11)

a) 340 c) 85
b) 100 d) 35

14) Na figura abaixo, as polias e os fios são ideais.


Se o sistema está em equilíbrio, pode-se afirmar
que a razão m1 é
m2

a) 1 e 2 apenas. c) 1 e 3 apenas.
b) 2 e 3 apenas. d) 1, 2 e 3.

12) Uma vela acesa, flutuando em água, mantém-


se sempre em equilíbrio, ocupando a posição 15) O recipiente mostrado na figura apresenta 80%
vertical. Sabendo-se que as densidades da vela e da de sua capacidade ocupada por um líquido.
água são, respectivamente, 0,8 g/cm3 e 1,0 g/cm3, Verifica-se que, para qualquer variação de
qual a fração da vela que permanecerá sem temperatura, o volume da parte vazia permanece
queimar, quando a chama se apagar ao entrar em constante. Pode-se afirmar que a razão entre os
contato com a água? coeficientes de dilatação volumétrica do recipiente
e do líquido vale

47
PROVAS DA AFA 1998 – 2010
apresentará pelo menos a fase líquida.
d) com a pressão mantida constante em P1 e
variando a temperatura de T1 a T2, a substância
sofrerá duas mudanças de estado.

18) N mols de um gás ideal possui volume v e


pressão p, quando sofre as seguintes
a) 0,72 c) 0,92 transformações sucessivas:
b) 1,00 d) 0,80 I - expansão isobárica até atingir o volume 2v;
II - aquecimento isométrico até a pressão tornar-se
igual a 3p;
16) Três barras cilíndricas idênticas em III - compressão isobárica até retornar ao volume
comprimento e secção são ligadas formando uma v; e
única barra, cujas extremidades são mantidas a 0 IV - resfriamento isométrico até retornar ao estado
°C e 100 °C. A partir da extremidade mais quente, inicial.
as condutividades térmicas dos materiais das
barras valem k, k/2 e k/5. Supondo-se que, em Assim, o trabalho trocado pelo gás, ao percorrer o
volta das barras, exista um isolamento de lã de ciclo descrito pelas transformações acima, vale
vidro e desprezando quaisquer perdas de calor, a a) zero c) 3pv
razão 2/ 1 entre as temperaturas nas junções b) –2pv d) –Npv
onde uma barra é ligada à outra, conforme mostra
a figura, é 19 - A variação volumétrica de um gás, em função
da temperatura, à pressão constante de 6 N/m2 está
indicada no gráfico.

a) 1,5 c) 1,2
b) 1,4 d) 1,6

17 - O gráfico abaixo representa o diagrama de


fases de uma determinada substância. Se, durante a transformação de A para B, o gás
receber uma quantidade de calor igual a 20 joules,
a variação da energia interna do gás será igual, em
joules, a
a) 32 c) 12
b) 24 d) 8

20) Pela manhã, um motorista calibra os pneus de


seu carro sob uma pressão de 28,0 lb/pol2 quando a
temperatura era de 7 °C. À tarde, após rodar
bastante, a temperatura dos pneus passou a ser 37
Da análise do gráfico, conclui-se que °C. Considerando que o volume dos pneus se
a) aumentando a pressão e mantendo a temperatura mantém constante e que o comportamento do ar
constante em T1, ocorrerá a vaporização da seja de um gás ideal, a pressão nos pneus
substância. aquecidos, em lb/pol2, passou a ser.
b) à temperatura T3 é possível liquefazer a a) 30,0 c) 33,0
substância. b) 31,0 d) 35,0
c) sob pressão PT e temperatura T0 a substância
48
PROVAS DA AFA 1998 – 2010
21) Considere uma bola de diâmetro d caindo a
partir de uma altura y sobre um espelho plano e
horizontal como mostra a figura abaixo.

24) Considere duas cordas, A e B, presas pelas


extremidades e submetidas à força de tração T,
com densidades lineares A e B, tal que A = B,
conforme mostra a figura abaixo.

O gráfico que MELHOR representa a variação do


diâmetro d’ da imagem da bola em função da
distância vertical y é

Ao se provocar ondas na corda A, essas originam


ondas sonoras de freqüência fA, que fazem com
que a corda B passe a vibrar por ressonância. As
ondas que percorrem a corda B, por sua vez,
produzem som de freqüência fB que é o segundo
harmônico do som fundamental de B. Nessas
f
condições, o valor da razão A , onde f 0A é o
f 0A
22) Uma lente convergente L de distância focal som fundamental da corda A, será
igual a f e um espelho esférico E com raio de a) 2 c) 4
curvatura igual a 2f estão dispostos coaxialmente, b) 3 d) 5
conforme mostra a figura abaixo.
25) Na figura abaixo, a esfera A suspensa por um
fio flexível e isolante, e a esfera B, fixa por um
pino também isolante, estão em equilíbrio.

Uma lâmpada de dimensões desprezíveis é


colocada no ponto P. A imagem da lâmpada
produzida por essa associação é
a) imprópria. É correto afirmar que
b) real e estará localizada à direita da lente. a) é possível que somente a esfera A esteja
c) virtual e estará localizada à direita da lente. eletrizada.
d) virtual e estará localizada entre o espelho e a b) as esferas A e B devem estar eletrizadas com
lente. cargas de mesma natureza.
c) a esfera A pode estar neutra, mas a esfera B
23) Considere uma figura de interferência obtida certamente estará eletrizada.
na superfície de um líquido por fontes que emitem d) as esferas devem estar eletrizadas com cargas de
em fase e na freqüência f. Considere ainda que mesmo módulo.
essas ondas se propagam com velocidade v. A
soma das diferenças de caminhos entre as ondas 26) Uma partícula eletrizada positivamente com
que se superpõem para os pontos pertencentes às 3 carga q é lançada em um campo elétrico uniforme
primeiras linhas nodais é. de intensidade E, descrevendo o movimento
representado na figura abaixo.

49
PROVAS DA AFA 1998 – 2010

A variação da energia potencial elétrica da


partícula entre os pontos A e B é

a) qEy c) qE x 2 + y 2 29) Na figura, L representa uma lâmpada de


b) qEx d) qE(x2 + y2) potência igual a 12 W ligada a uma bateria de
tensão igual a 24 V.
27) Uma partícula com carga -q e massa m gira em
torno de uma esfera de raio R uniformemente
eletrizada com uma carga +Q. Se o potencial no
centro da esfera é Vc , a energia cinética da
partícula para que ela se mantenha em movimento
circular uniforme a uma distância 2R do centro da
esfera é
Para que a intensidade da corrente elétrica do
circuito seja reduzida à metade, é necessário
associar em
a) série com a lâmpada L, um resistor de
resistência elétrica 24 .
b) paralelo com a lâmpada L, dois resistores
idênticos, também associados em paralelo, de
resistência elétrica 48  cada.
c) paralelo com a lâmpada L, um resistor de
resistência elétrica de 48 .
d) série com a lâmpada L, um resistor de
28) No circuito esquematizado abaixo, o reostato
resistência elétrica de 48 .
tem resistência R (R1 < R < R2) e o gerador tem
resistência interna desprezível.
30) A figura abaixo representa uma espira
retangular em repouso num campo magnético de
um imã. Ao ser percorrida por uma corrente no
sentido indicado na figura, a espira passará a

Qual dos gráficos propostos MELHOR representa


a potência P dissipada pela lâmpada L em função
de R? a) girar no sentido horário.
b) girar no sentido anti-horário.
c) oscilar em torno do eixo y.
d) oscilar em torno do eixo x.

50
PROVAS DA AFA 1998 – 2010
AFA 2007/2008 – Física 5) A figura mostra uma bola de isopor caindo, a
partir do repouso, sob efeito da resistência do ar, e
outra bola idêntica, abandonada no vácuo no
1) Uma partícula move-se com velocidade de 50
instante t1 em que a primeira atinge a velocidade
m/s. Sob a ação de uma aceleração de módulo 0,2
limite.
m/s2 , ela chega a atingir a mesma velocidade em
sentido contrário. O tempo gasto, em segundos,
para ocorrer essa mudança no sentido da
velocidade é
a) 500 b) 250 c) 100 d) 50

2) Um corpo é abandonado do repouso de uma


altura h acima do solo. No mesmo instante, um
outro é lançado para cima, a partir do solo,
segundo a mesma vertical, com velocidade v. A opção que pode representar os gráficos da altura
Sabendo que os corpos se encontram na metade da h em função do tempo t para as situações descritas
altura da descida do primeiro, pode-se afirmar que é
h vale.
1 2
v v2 v 2 v
a) b) c)   d)  
g g g g

3) Considere um pequeno avião voando em


trajetória retilínea com velocidade constante nas
situações a seguir.
(1) A favor do vento.
(2) Perpendicularmente ao vento.
Sabe-se que a velocidade do vento é 75% da
velocidade do avião. Para uma mesma distância
t 1
percorrida, a razão , entre os intervalos de
t 2
tempo nas situações (1) e (2), vale.
1 3 5 7
a) b) c) d)
3 5 7 9
6) Na questão anterior, considere que a bola da
4)Um corpo de massa m, preso à extremidade de situação 2 atinge o solo com uma velocidade duas
um fio, constituindo um pêndulo cônico, gira num vezes maior que a velocidade limite alcançada pela
círculo horizontal de raio R, como mostra a figura. bola na situação 1. Nestas condições, pode-se
afirmar que o percentual de energia dissipada na
situação 1 foi de
a) 10% b) 25% c) 50% d) 75%

7) A figura abaixo representa dois corpos idênticos


girando horizontalmente em MCU com
T1
velocidades lineares v1 e v2. A razão entre as
T2

Sendo g a aceleração da gravidade local e θ o intensidades das trações nos fios ideais 1 e 2 é
ângulo do fio com a vertical, a velocidade do corpo
pode ser calculada por
a) Rg b) 2Rg c) Rgsen  d) Rgtg 

51
PROVAS DA AFA 1998 – 2010
Para um referencial externo, fixo na terra, as forças
que atuam sobre uma pessoa estão representadas
pela opção

2 v12 + v 22 v12 + v 22
a) b)
v 22 v 22
v12 − v 22 v 22
c) d)
v 22 v12

8) O volume de água necessário para acionar cada


turbina de uma determinada central hidrelétrica é
cerca de 700 m3 Por segundo, "guiado" através de
um conduto forçado de queda nominal igual a 112
m. Considere a densidade da água igual a 1 kg/L.
Se cada turbina geradora assegura uma potência de
700 MW, a perda de energia nesse processo de 10) Em uma apresentação da Esquadrilha da
transformação mecânica em elétrica é, Fumaça, uma das acrobacias é o "loop",
aproximadamente, igual a representado pela trajetória circular da figura. Ao
a) 5% b) 10% c) 15% d) 20% passar pelo ponto mais baixo da trajetória, a força
que o assento do avião exerce sobre o piloto é
9) A figura representa um brinquedo de parque de
diversão em que as pessoas, apenas em contato
com a parede vertical, giram juntamente com uma
espécie de cilindro gigante em movimento de
rotação.

a) maior que o peso do piloto.


b) igual ao peso do piloto.
c) menor que o peso do piloto.
d) nula.
Considere as forças envolvidas abaixo 11) Três esferas idênticas estão suspensas por fios
relacionadas. ideais conforme a figura. Se a esfera A for
deslocada da posição inicial e solta, ela atingirá
Pé a força peso uma velocidade v e colidirá frontalmente com as
outras duas esferas estacionadas.
Fat é a força de atrito estático Considerando o choque entre as esferas
perfeitamente elástico, pode-se afirmar que as
Fcp é a força centrípeta velocidades vA, vB e vC de A, B e C,
imediatamente após as colisões, serão
Fcf é a força centrífuga

52
PROVAS DA AFA 1998 – 2010

a) vA = vB = vC = v
A razão entre as trações nos fios A e B vale
b) vA = vB = 0 e vC = v 1 2 3 5
v a) b) c) d)
c) vA = 0 e vB = vC = 2 3 4 6
2
v
d) vA = vB = vC = 15) Uma balança está em equilíbrio, no ar, tendo
3
bolinhas de ferro num prato e rolhas de cortiça no
12) A respeito de um satélite artificial estacionário outro. Se esta balança for levada para o vácuo,
em órbita sobre um ponto do equador terrestre, pode-se afirmar que ela
afirma-se que I - a força que a Terra exerce sobre a) penderia para o lado das bolinhas de ferro, pois
ele é a resultante centrípeta necessária para mantê- a densidade do mesmo é maior que a densidade da
lo em órbita. cortiça.
II - o seu período de translação é 24 horas. b) não penderia para nenhum lado, porque o peso
III - os objetos soltos em seu interior ficam das bolinhas de ferro é igual ao peso das rolhas de
flutuando devido à ausência da gravidade. cortiça.
Está(ão) correta(s) c) penderia para o lado das rolhas de cortiça, pois
a) apenas I. b) apenas I e II. enquanto estava no ar o empuxo sobre a cortiça é
c) apenas II e III. d) I, II e III. maior que o empuxo sobre o ferro.
d) não penderia para nenhum lado, porque no
13) A figura abaixo apresenta dois corpos de vácuo não tem empuxo.
massa m suspensos por fios ideais que passam por
roldanas também ideais. Um terceiro corpo, 16) Duas esferas A e B de mesmo volume, de
também de massa m, é suspenso no ponto médio materiais diferentes e presas por fios ideais,
M do fio e baixado até a posição de equilíbrio. encontram-se em equilíbrio no interior de um vaso
com água conforme a figura.

O afastamento do ponto M em relação à sua


posição inicial é Considerando-se as forças peso (PA e PB),
d 3 d 3 d 3 d 3 empuxo (EA e EB) e tensão no fio (TA e TB)
a) b) c) d) relacionadas a cada esfera, é INCORRETO afirmar
2 3 4 6
que
14)Uma viga homogênea é suspensa a) PA > PB b) EA = EB
horizontalmente por dois fios verticais como c) TA + TB = PA – PB d) TA < TB
mostra a figura abaixo.

53
PROVAS DA AFA 1998 – 2010
17) O diagrama de fases apresentado a seguir externo.
pertence a uma substância hipotética. Com relação
a essa substância, pode-se afirmar que, 19) Um cilindro de volume constante contém
determinado gás ideal à temperatura T0 e pressão
p0. Mantém-se constante a temperatura do cilindro
e introduz-se, lentamente, a partir do instante t = 0,
certa massa do mesmo gás.
O gráfico abaixo representa a massa m de gás
existente no interior do cilindro em função do
tempo t.

a) nas condições normais de temperatura e pressão,


a referida substância se encontra no estado sólido.
b) se certa massa de vapor da substância à
temperatura de 300 °C for comprimida lentamente,
não poderá sofrer condensação pois está abaixo da
temperatura crítica.
Nessas condições, a pressão do gás existente no
c) para a temperatura de 0 °C e pressão de 0,5 atm,
recipiente, para o instante t = a, é igual a
a substância se encontra no estado de vapor.
a) 1,5 p0 b) 2,0 p0 c) 2,5 p0 d) 4,0 p0
d) se aumentarmos gradativamente a temperatura
da substância, quando ela se encontra a 70 °C e
20 ) A figura mostra um objeto A, colocado a 8 m
sob pressão de 3 atm, ocorrerá sublimação da
de um espelho plano, e um observador O, colocado
mesma.
a 4 m desse mesmo espelho.
18) A figura a seguir representa o Ciclo de Carnot
realizado por um gás ideal que sofre
transformações numa máquina térmica.
Considerando-se que o trabalho útil fornecido pela
máquina, em cada ciclo, é igual a 1500 J e, ainda
que, T1 = 600 K e T2 = 300 K, é INCORRETO
afirmar que

Um raio de luz que parte de A e atinge o


observador O por reflexão no espelho percorrerá,
nesse trajeto de A para O,
a) 10 m b) 12 m c) 15 m d) 18 m

21) Um espelho esférico E de distância focal f e


uma lente convergente L estão dispostos
coaxialmente, com seus eixos ópticos coincidentes.
Uma fonte pontual de grande potência, capaz de
emitir luz exclusivamente para direita, é colocada
a) a quantidade de calor retirada da fonte quente é em P. Os raios luminosos do ponto acendem um
de 3000 J. palito de fósforo
b) de A até B o gás se expande isotermicamente. com a cabeça em Q, conforme mostra a figura.
c) de D até A o gás é comprimido sem trocar calor
com o meio externo.
d) de B até C o gás expande devido ao calor
recebido do meio
54
PROVAS DA AFA 1998 – 2010
o oscilador harmônico simples é dada pela
expressão x = a cos(ωt + 0), onde a e ω valem,
respectivamente,

mv M+m k
a) e
M+m k M+m

b)
(M + m)v e k
k M+m

k M+m
Considerando-se as medidas do esquema, pode-se c) e
M+m k
afirmar que a distância focal da lente vale
f f 2
a) f b) c) d) f
M+m k M+m
2 3 3 d) e
mv M+m k
22) Considere uma película transparente de faces
paralelas com índice de refração n iluminada por 24) Considere um sistema formado por duas
luz monocromática de comprimento de onda no ar cordas diferentes, com densidades μ1 e μ2 tal que
igual a λ, como mostra a figura abaixo. μ1 > μ2, em que se propagam dois pulsos idênticos,
conforme mostra a figura abaixo.

A opção que melhor representa a configuração


resultante no sistema após os pulsos passarem pela
junção das cordas é
Sendo a incidência de luz pouco inclinada, a
mínima espessura de película para que um
observador a veja brilhante por luz refletida é
   
a) b) c) d)
n 2n 4n 5n

23) Um projétil de massa m e velocidade v atinge


horizontalmente um bloco de massa M que se
encontra acoplado a uma mola de constante
elástica K, como mostra a figura abaixo.

Após o impacto, o projétil se aloja no bloco e o 25) Um corpo B, de massa igual a 4 kg e carga
sistema massa-mola-projétil passa a oscilar em elétrica +6 μC, dista 30 mm do corpo A, fixo e com
MHS com amplitude a. Não há atrito entre o bloco carga elétrica –1 μC. O corpo B é suspenso por um
e o plano horizontal nem resistência do ar. Nessas fio isolante, de massa desprezível ligado a uma
condições, a posição em função do tempo para mola presa ao solo, como mostra a figura. O
comprimento natural da mola é L0 = 1,2 m e ao
sustentar estaticamente o corpo B ela se distende,
55
PROVAS DA AFA 1998 – 2010
atingindo o comprimento L = 1,6 m.
Considerando-se a constante eletrostática do meio
k = 9 . 109 N.m2/C2 , que as cargas originais dos
corpos pontuais A e B são mantidas e
desprezando-se os possíveis atritos, o valor da
constante elástica da mola, em N/m, é
O gráfico que melhor representa a carga
acumulada Q no capacitor em função da
resistência R do reostato é

a) 200 b) 320 c) 600 d) 800

26) Aqueceu-se certa quantidade de um líquido


utilizando um gerador de f.e.m. ε = 50 V e
resistência interna r = 3  e um resistor de
resistência R. Se a quantidade de calor fornecida
pelo resistor ao líquido foi de 2.105 J, pode-se
afirmar que o tempo de aquecimento foi 29) A figura mostra uma região na qual atua um
a) inferior a 5 minutos. campo magnético uniforme de módulo B. Uma
b) entre 6 e 10 minutos. partícula de massa m, carregada positivamente
c) entre 12 e 15 minutos. com carga q, e lançada no ponto A com uma
d) superior a 15 minutos. velocidade de módulo v e direção perpendicular às
linhas do campo. O tempo que a partícula levará
27) No circuito representado abaixo, os geradores para atingir o ponto B é
G1 e G2 são ideais e os resistores têm a mesma
resistência R.

Bq m 2m Bq


a) b) c) d)
m Bq Bq 2m

Se a potência dissipada por R2 é nula, então a


razão entre as f.e.m. de G1 e G2 é 30) Uma espira condutora é colocada no mesmo
plano e ao lado de um circuito constituído de uma
1 1 pilha, de uma lâmpada e de um interruptor.
a) b) c) 2 d) 4
4 2

28)No circuito esquematizado abaixo, C é um


capacitor, G um gerador de f.e.m. ε e resistência
interna r e AB um reostato.

56
PROVAS DA AFA 1998 – 2010
a) acelerado no trecho 1 e retardado no trecho 2,
sendo V1 > V > V2
b) acelerado nos dois trechos, sendo v1 = v2>
v
c) uniforme nos dois trechos, sendo v1 = v2 > v
d) uniforme nos dois trechos, sendo v1 = v2 = v

3) Uma bola de basquete descreve a trajetória


mostrada na figura após ser arremessada por um
jovem atleta que tenta bater um recorde de
As alternativas a seguir apresentam situações em arremesso.
que, após o interruptor ser ligado, o condutor AB
gera uma corrente elétrica induzida na espira,
EXCETO
a) desligar o interruptor.
b) "queimar" a lâmpada.
c) mover a espira na direção x.
d) mover a espira na direção y.

AFA 2008/2009 – Física


1) O diagrama abaixo representa as posições de
dois corpos A e B em função do tempo. A bola é lançada com uma velocidade de 10 m/s e,
ao cair na cesta, sua componente horizontal vale
6,0 m/s. Despreze a resistência do ar e considere g
= 10 m/s2. Pode-se afirmar que a distância
horizontal (x) percorrida pela bola desde o
lançamento até cair na cesta, em metros, vale
a) 3,0 b) 3,6 c) 4,8 d) 6,0
Por este diagrama, afirma-se que o corpo A iniciou
o seu movimento, em relação ao corpo B, depois 4) Uma pessoa, brincando em uma roda-gigante,
de ao passar pelo ponto mais alto, arremessa uma
a) 2,5 s b) 5,0 s c) 7,5 s d) 10 s pequena bola (Figura 1), de forma que esta
descreve, em relação ao solo, a trajetória de um
2) Uma bola rola com velocidade v , constante, lançamento vertical para cima.
sobre uma superfície de vidro plana e horizontal,
descrevendo uma trajetória retilínea. Enquanto a
bola se desloca, a sua sombra percorre os planos
representados pelos trechos 1 e 2 da figura abaixo,
com velocidades escalares médias v1 e v2 ,
respectivamente.

A velocidade de lançamento da bola na direção


Considerando que a sombra está sendo gerada por vertical tem o mesmo módulo da velocidade
uma projeção ortogonal à superfície de vidro, escalar (v) da roda-gigante, que executa um
pode-se afirmar que o seu movimento é movimento circular uniforme. Despreze a

57
PROVAS DA AFA 1998 – 2010
resistência do ar, considere a aceleração da
gravidade igual a g e π = 3. Se a pessoa consegue
pegar a bola no ponto mais próximo do solo
(Figura 2), o período de rotação da roda-gigante
pode ser igual a
v 10v 20v v
a) b) c) d) 12
g 7g 3g g

5) Uma partícula é abandonada de uma


determinada altura e percorre o trilho
esquematizado na figura abaixo, sem perder
contato com ele.

Considere que não há atrito entre a partícula e o


trilho, que a resistência do ar seja desprezível e que 7) Um planeta Alpha descreve uma trajetória
a aceleração da gravidade seja g. Nessas elíptica em torno do seu sol como mostra a figura
condições, a menor velocidade possível da abaixo.
partícula ao terminar de executar o terceiro looping
é
a) 3Rg b) 7Rg c) 11Rg d) 15 Rg

6)Dispõe-se de quatro polias ideais de raios RA=R,


R R Considere que as áreas A1, A2 e A3 são varridas
RB = 3R. RC e RD que podem ser
2 10 pelo raio vetor que une o centro do planeta ao
combinadas e acopladas a um motor cuja centro do sol quando Alpha se move
freqüência de funcionamento tem valor f. As polias respectivamente das posições de 1 a 2, de 2 a 3 e
podem ser ligadas por correias ideais ou unidas por de 4 a 5. Os trajetos de 1 a 2 e de 2 a 3 são
eixos rígidos e, nos acoplamentos, não ocorre realizados no mesmo intervalo de tempo Δt e o
escorregamento. Considere que a combinação trajeto de 4 a 5 num intervalo Δt’ < Δt. Nessas
dessas polias com o motor deve acionar uma serra condições é correto afirmar que
circular (S) para que ela tenha uma freqüência de a) A1 < A3 b) A2 < A3 c) A1 > A2 d) A3 < A2
rotação igual a 5/3 da freqüência do motor. Sendo
assim, marque a alternativa que representa essa 8) Dois corpos A e B, esféricos, inicialmente
combinação de polias. estacionários no espaço, com massas
respectivamente iguais a mA e mB, encontram-se
separados, centro a centro, de uma distância x
muito maior que os seus raios, conforme figura
abaixo.

Na ausência de outras forças de interação, existe


um ponto P do espaço que se localiza a uma
58
PROVAS DA AFA 1998 – 2010
distância d do centro do corpo A. Nesse ponto P é
nula a intensidade da força gravitacional
resultante, devido à ação dos corpos A e B sobre
um corpo de prova de massa m, ali colocado.
Considere que os corpos A e B passem a se afastar
com uma velocidade constante ao longo de uma
trajetória retilínea que une os seus centros e que
mA = 16mB.Nessas condições, o gráfico que melhor
representa d em função de x é.
11) Um paciente, após ser medicado às 10 h,
apresentou o seguinte quadro de temperatura:

9) Na situação de equilíbrio abaixo, os fios e as


polias são ideais e a aceleração da gravidade é g. A temperatura desse paciente às 11 h 30 min, em
Considere μe o coeficiente de atrito estático entre o °F, é.
bloco A, de massa mA, e o plano horizontal em que a) 104 b) 98,6 c) 54,0 d) 42,8
se apóia.
12) Um frasco de vidro, cujo volume é 2000 cm3 a
0 ºC, está completamente cheio de mercúrio a esta
temperatura. Sabe-se que coeficiente de dilatação
volumétrica do mercúrio é 1,8 x 10-4 ºC-1
coeficiente de dilatação linear do vidro de que é
feito o frasco 1,0 x 10-5 ºC-1. O volume de
A maior massa que o bloco B pode ter, de modo mercúrio que irá entornar, em cm3 quando o
que o equilíbrio mantenha, é conjunto for aquecido até 100 ºC, será
a) μe mA b) 3μe mA c) 2μe mA d) 4μe mA a) 6,0 b) 18 c) 30 d) 36

10) A figura abaixo representa um vagão em 13)Um estudante, querendo determinar o


repouso, no interior do qual se encontram um equivalente em água de um calorímetro, colocou
pêndulo simples e um recipiente fixo no piso, em seu interior 250 g de água fria e, aguardando
cheio de água. O pêndulo simples é composto de um certo tempo, verificou que o conjunto alcançou
uma bolinha de ferro presa ao teto do vagão por o equilíbrio térmico a uma temperatura de 20 °C.
um fio ideal e, dentro do recipiente, existe uma Em seguida, acrescentou ao mesmo 300 g de água
bolinha de isopor, totalmente imersa na água e morna, a 45 °C. Fechando rapidamente o aparelho,
presa no seu fundo também por um fio ideal. esperou até que o equilíbrio térmico fosse refeito;
verificando, então, que a temperatura final era de
30 °C. Baseando-se nesses dados, o equivalente
em água do calorímetro vale, em gramas,
a) 400 b) 300 c) 200 d) 100

14) O diagrama a seguir representa o ciclo


Assinale a alternativa que melhor representa a
percorrido por 3 mols de um gás perfeito.
situação física no interior do vagão, se este
começar a se mover com aceleração constante para
a direita.

59
PROVAS DA AFA 1998 – 2010

Sabendo-se que no estado A a temperatura é –23 Nessas condições, a imagem fornecida pela lente e
ºC e considerando R 8 J/mol K = . , o trabalho, em projetada no anteparo poderá ser
joules, realizado pelo gás no ciclo
a) 12000 b) – 6000 c) 1104 d) – 552

15) O gás contido no balão A de volume V e


pressão p é suavemente escoado através de dutos
17)A imagem de um ponto P, posicionado a uma
rígidos e de volumes desprezíveis, para os balões
distância d de um espelho plano E, pode ser
B, C, D e E, idênticos e inicialmente vazios, após a
visualizada por dois observadores A e B, como
abertura simultânea das válvulas 1, 2, 3 e 4, como
mostra a figura abaixo.
mostra a figura abaixo.

A respeito da imagem P’ do ponto P vista pelos


observadores, é correto afirmar que
a) o observador A visualiza P’ a uma distância d/2
do espelho.
Após atingido o equilíbrio, a pressão no sistema de b) o observador B visualiza P’ a uma distância d/4
p do espelho.
balões assume valor .Considerando que não
3 c) o observador A visualiza P’ a uma distância
ocorre variação de temperatura, volume de dois 3d/2 do espelho e o observador B, à distância 5d/4
dos balões menores é do espelho.
a) 0,5 V b) 1,0 V c) 1,5 V d) 2,0 V d) ambos os observadores visualizam P’ a uma
distância 2d do ponto P.
16) A figura I representa uma lente delgada
convergente com uma de suas faces escurecida por 18) Considere dois pássaros A e B em repouso
tinta opaca, de forma que a luz só passa pela letra sobre um fio homogêneo de densidade linear μ,
F impressa. que se encontra tensionado, como mostra a figura
abaixo. Suponha que a extremidade do fio que não
aparece esteja muito distante da situação
apresentada.

Um objeto, considerado muito distante da lente, é


disposto ao longo do eixo óptico dessa lente, como
mostra a figura II.
Subitamente o pássaro A faz um movimento para
alçar vôo, emitindo um pulso que percorre o fio e
atinge o pássaro B Δt segundos depois.

60
PROVAS DA AFA 1998 – 2010
Despreze os efeitos que o peso dos pássaros possa velocidade inicial de módulo v0, no vácuo,
exercer sobre o fio. O valor da força tensora para inclinada de um ângulo θ em relação à horizontal.
que o pulso retorne à posição onde se encontrava o
pássaro A, em um tempo igual a 3Δt, é
9d 2 4d 2 d 2 d 2
a) b) c) d)
(t )2 (t )2 (t )2 9(t )2
Considere que, além do campo gravitacional de
19) Um par de blocos A e B, de massas mA = 2 kg intensidade g, atua também um campo elétrico
e mB = 10 kg, apoiados em um plano sem atrito, é uniforme de módulo E. Pode-se afirmar que a
acoplado a duas molas ideais de mesma constante partícula voltará à altura inicial de lançamento
elástica K = 50 N/m, como mostra a figura abaixo. após percorrer, horizontalmente, uma distância
igual a.
V2  qE 
a) sen21 + tg 
g  mg 
V2  qE 
b) sen con + sen 
2g  m 
Afastando-se horizontalmente o par de blocos de
V  qE 
sua posição de equilíbrio, o sistema passa a oscilar c)  sen2 + 
g  mg 
em movimento harmônico simples com energia
V  qE 
mecânica igual a 50 J. Considerando g = 10 m/s2, d) 1 + sen2 
o mínimo coeficiente de atrito estático que deve 2g  m 
existir entre os dois blocos para que o bloco A não
escorregue sobre o bloco B é 22) O elemento de aquecimento de uma torneira
a) 1/10 b) 5/12 c) 5/6 d) 1 elétrica é constituído de dois resistores e de uma
chave C, conforme ilustra a figura abaixo.
20) Os valores do potencial elétrico V em cada
vértice de um quadrado estão indicados na figura
abaixo.

Com a chave C aberta, a temperatura da água na


saída da torneira aumenta em 10 ºC. Mantendo-se
a mesma vazão d’água e fechando C, pode-se
afirmar que a elevação de temperatura da água, em
graus Celsius, será de
a) 2,5 b) 5,0 c) 15 d) 20

Os valores desses potenciais condizem com o fato 23) Parte de um circuito elétrico é constituída por
de o quadrado estar situado num campo seis resistores ôhmicos cujas resistências elétricas
eletrostático estão indicadas ao lado de cada resistor, na figura
a) uniforme, na direção do eixo x. abaixo.
b) uniforme, na direção da bissetriz do 1º
quadrante.
c) criado por duas cargas puntiformes situadas no
eixo y.
d) criado por duas cargas puntiformes situadas nas
bissetrizes dos quadrantes ímpares.

21) Na figura abaixo, uma partícula com carga Se a d.d.p. entre os pontos A e B é igual a U, pode-
elétrica positiva q e massa m é lançada se afirmar que a potência dissipada pelo resistor
obliquamente de uma superfície plana, com R3 é igual a

61
PROVAS DA AFA 1998 – 2010
2 2 2 2
1 U 2 U 2U 1 U
a)   b)   c)   d)  
2R  3  R 3 3R  2R  6  AFA 2009/2010 – Física
24) Uma bateria de f.e.m. igual a ε e resistência 1) O gráfico da posição (S) em função do tempo (t)
interna de valor igual a r (constante) alimenta o a seguir representa o movimento retilíneo de um
circuito formado por uma lâmpada L e um reostato móvel.
R, conforme ilustra a figura abaixo.

Considerando constante a resistência da lâmpada,


o gráfico que melhor representa a potência por ela
dissipada quando o cursor do reostato move-se de
A para B é
A partir do gráfico é correto afirmar que,
a) no primeiro segundo, o seu movimento é
progressivo.
b) entre 1 s e 3 s, a aceleração é negativa.
c) no instante 2 s, a velocidade do móvel é nula.
d) nos instantes 1 s e 3 s, os vetores velocidades
são iguais.

2)Um carro percorre uma curva circular com


25) O trecho AB, de comprimento 30 cm, do velocidade linear constante de 15 m/s
circuito elétrico abaixo, está imerso num campo completando-a em 5 2 s, conforme figura abaixo.
magnético uniforme de intensidade 4 T e direção
perpendicular ao plano da folha. Quando a chave
CH é fechada e o capacitor completamente
carregado, atua sobre o trecho AB uma força
magnética de intensidade 3 N, deformando-o,
conforme a figura.

Sabe-se que os fios são ideais. A intensidade da É correto afirmar que o módulo da aceleração
corrente elétrica, em ampères, e a diferença de média experimentada pelo carro nesse trecho, em
potencial elétrico entre os pontos C e D, em volts, m/s², é
são, respectivamente a) 0 b) 1,8 c) 3,0 d) 5,3
a) 25 e 50 b) 5 e 10 c) 2,5 e 5 d) 1,25 e 2,5
3) No instante t = 0, uma partícula A é lançada
obliquamente, a partir do solo, com velocidade de
80 m/s sob um ângulo de 30° com a horizontal. No

62
PROVAS DA AFA 1998 – 2010
instante t = 2 s, outra partícula B é lançada cinética, E1 , E2 e E3 , adquirida em cada situação,
verticalmente para cima, também a partir do solo, é tal que
com velocidade de 70 m/s, de um ponto situado a a) E1 =E2 =E3 c) E1 <E2 <E3
200 3 m da posição de lançamento da primeira. b) E1 >E2 =E3 d) E1 =E2 >E3
Sabendo-se que essas duas partículas colidem no
ar, pode-se afirmar que no momento do encontro 7) O bloco da Figura 1 entra em movimento sob
a) ambas estão subindo. ação de uma força resultante de módulo F que
b) A está subindo e B descendo. pode atuar de três formas diferentes, conforme os
c) B está subindo e A descendo. diagramas da Figura 2.
d) ambas estão descendo.

4) Um satélite cujo raio da órbita vale R gira ao


redor da Terra com velocidade angular constante
ω . Por necessidade técnica será feito um ajuste na
trajetória que dobrará o raio orbital desse satélite,
fazendo-o girar com uma nova velocidade angular
constante ω' .A razão ω/ω' vale
a) 2 2 b) 2 / 2 c) 2 d) 1/2

5) Um vagão movimenta-se sobre trilhos retos e


horizontais obedecendo à equação horária S = 20t
– 5t² (SI). Um fio ideal tem uma de suas Com relação aos módulos das velocidades v1, v2 e
extremidades presa ao teto do vagão e, na outra, v3 atingidas pelo bloco no instante t = 2 s, nas três
existe uma esfera formando um pêndulo. As situações descritas, pode-se afirmar que
figuras que melhor representam as configurações a) v1 > v2 > v3 c) v3 < v1 < v2
do sistema vagão-pêndulo de velocidade V e b) v2 > v3 > v1 d) v2 < v3 < v1
aceleração a , nos instantes 1 s, 2 s e 3 s, são
respectivamente.
8) Uma esfera de massa m, pendurada na
a) extremidade livre de um dinamômetro ideal, é
imersa totalmente em um líquido A e a seguir em
um outro líquido B, conforme figura abaixo.
b)

c)

d)

6) A figura abaixo representa três formas distintas


para um bloco entrar em movimento.

As leituras do dinamômetro nos líquidos A e B, na


condição de equilíbrio, são, respectivamente, F1 e
Sabe-se que as forças F1 , F 2 e F3 são constantes
F2 . Sendo g a aceleração da gravidade local, a
e de mesma intensidade. Desprezando-se qualquer razão entre as massas específicas de A e B.
resistência, pode-se afirmar que, depois de mg + F1 F1 − mg
percorrida uma mesma distância, a energia a) b)
mg + F2 mg + F2

63
PROVAS DA AFA 1998 – 2010
mg + F1 mg − F1
c) d)
F2 − mg mg − F2
12) No diagrama a seguir, do volume (V) em
9) A água, em condições normais, solidifica-se a função da temperatura absoluta (T), estão
0°C. Entretanto, em condições especiais, a curva indicadas as transformações AB e BC sofridas por
de resfriamento de 160 g de água pode ter o uma determinada massa de gás ideal.
aspecto a seguir.

Num diagrama da pressão (P) em função do


Sabendo-se que o calor latente de fusão do gelo e o
volume (V), essas transformações deveriam ser
calor específico da água valem, respectivamente,
indicadas por
80 cal/g e 1,0 cal/g°C, a massa de água, em
a)
gramas, que se solidifica no trecho MN é
a) 8 b) 10 c) 16 d) 32

10) Um recipiente tem capacidade de 3.000 cm³ a


20 °C e está completamente cheio de um
determinado líquido. Ao aquecer o conjunto até
120 °C transbordam 27 cm³. O coeficiente de
dilatação aparente desse líquido, em relação ao
material de que é feito o recipiente é, em °C–1,
igual a
a) 3,0 . 10-5 c) 2,7 .10-4
5
b) 9,0 .10- d) 8,1 .10-4
b)
11) Considere um objeto AB, perpendicular ao eixo
óptico de um espelho esférico gaussiano, e sua
imagem A’B’ conjugada pelo espelho, como
mostra a figura abaixo.

Movendo-se o objeto AB para outra posição p em


c)
relação ao espelho, uma nova imagem é conjugada
de tal forma que o aumento linear transversal
proporcionado é igual a 2. Nessas condições, essa
nova posição p do objeto, em cm, é
a) 1 b) 2 c) 3 d) 4
64
PROVAS DA AFA 1998 – 2010

d) A razão entre as distâncias AB e BC é.


a) 2 b) 1 c) 1 d) 3
3 2

15) Dois anéis idênticos de centros O e O‘,


uniformemente eletrizados com cargas de
naturezas opostas e mesmo módulo, são mantidos
em planos paralelos conforme indica a figura.

13) Considere a palavra ACADEMIA


parcialmente vista de cima por um observador
através de uma lente esférica gaussiana, como
mostra a figura abaixo.

Os pontos O, O‘ e B são colineares e A pertence à


mediatriz do segmento OO‘. O trabalho realizado
pela força aplicada por um agente externo para
deslocar uma carga de prova negativa do ponto A
até o ponto B, com velocidade constante,
a) dependerá da posição do ponto A. b) será
Estando todo o conjunto imerso em ar, a lente que nulo.
pode representar a situação é c) será positivo. d) será
a) plano-côncava. c) bicôncava. negativo.
b) côncavo-convexa. d) convexo-côncava.
16)Uma esfera de massa m, eletrizada
14) A figura abaixo representa a variação da positivamente com carga q, está fixada na
intensidade luminosa I das franjas de interferência, extremidade de um fio ideal e isolante de
em função da posição x, resultado da montagem comprimento ℓ . O pêndulo, assim constituído, está
experimental, conhecida como Experiência de imerso em uma região onde além do campo
Young. gravitacional g atua um campo elétrico horizontal
e uniforme E . Este pêndulo é abandonado do
ponto A e faz um ângulo θ com a vertical
conforme mostra a figura.
65
PROVAS DA AFA 1998 – 2010

a) 12 b) 8 c) 4 d) 0

Desprezando-se quaisquer resistências, ao passar 19)Uma partícula de massa m carregada


pelo ponto B, simétrico de A em relação à vertical, eletricamente com carga q, é solta em queda livre
sua energia cinética vale de uma altura h acima do plano horizontal xy,
a) 2q E ℓ senθ conforme ilustra a figura abaixo.
b) ℓ (mg + q E senθ)
c) 2ℓ (mg cos θ + q E senθ)
d) q E ℓ cos θ

17) No circuito abaixo, alimentado por três pilhas


ideais de 1,5 V cada, o amperímetro A e os
voltímetros V1 e V2 são
considerados ideais.

Se nesta região, além do campo gravitacional g ,


atua também um campo magnético uniforme B na
direção Oy, a energia cinética da partícula ao
passar pelo plano xy valerá.
a) mgh c) mqh(g + B)
2
b) mh g + B 2
d) mqh(g2 - B2 )

20) Considere um campo magnético uniforme de


Sabe-se que o voltímetro V2 indica 2,0 V e que as intensidade B e um condutor metálico retilíneo
resistências elétricas dos resistores R1 e R3 são, deslocando-se com velocidade vetorial constante
respectivamente, 2,5 Ω e V , perpendicularmente às linhas desse campo,
3,0 Ω. Nestas condições, as indicações de V1 , em conforme a figura abaixo.
volts, de A, em ampères, e o valor da resistência
elétrica do resistor R2 , em ohms, são,
respectivamente
a) 1 , 2 ,6 b) 1 , 1 ,3 c) 5 , 1 ,6 d) 5 , 2 ,3
2 3 2 3 2 3 2 3

18) No circuito elétrico abaixo, a carga elétrica do


capacitor, em μC, é.

66
PROVAS DA AFA 1998 – 2010
AFA 1997/1998 – Português
DIREITA ou ESQUERDA ?
(NÃO SE PREOCUPE, NÃO SE TRATA DE DISCUSSÃO
POLÍTICA...)

Todos sabem que a principal diferença


entre os helicópteros e os aviões está nas asas:
naqueles, elas são rotativas e nestes, fixas. Mas,
dentre todas as diferenças entre os dois tipos de
aparelhos mais-pesados-que-o-ar há uma bem
interessante: nos aviões, o piloto solo ou
Sobre a situação descrita acima, são feitas as comandante ocupa o assento da esquerda e, nos
seguintes afirmações. helicópteros, ele ocupa o da direita. Você já parou
I) A separação de cargas nas extremidades do para pensar no porquê disso?
Vamos analisar os dois casos
condutor dá origem a um campo elétrico E que
separadamente. Primeiro os aviões. Geralmente
exerce sobre os portadores de carga uma força
tenta-se justificar que o piloto, nos aviões lado a
elétrica F e . lado, senta-se à esquerda devido aos circuitos de
II) A força elétrica F e , que surge devido a tráfego nas aproximações, que geralmente são
separação de cargas no condutor, tende a equilibrar feitos para a esquerda, o que lhes facilitaria a visão
a ação da força magnética F m exercida pelo campo da pista para o pouso. Mas, o que surgiu primeiro,
magnético uniforme. o avião, a pista ou o circuito de tráfego?
III) O campo elétrico E , que surge devido a Considerando que o avião foi realmente o
separação de cargas no condutor, dá origem a uma primeiro, se o piloto ocupasse o lado direito da
força eletromotriz ε, que é a diferença de potencial cabine, a maioria das aproximações seriam
nas extremidades do condutor. executadas para a direita.
São corretas A hipótese que melhor justifica a posição
a) somente I e II. c) somente II e III. do piloto é, curiosamente, a mesma que explica o
b) somente I e III. d) I, II e III. tráfego de automóveis pela esquerda em países
como a Inglaterra, por exemplo. E tudo começou
quando não existiam automóveis e muito menos
aviões.
Na Idade Média, os cavaleiros transitavam
pelo lado esquerdo das estradas porque mantinham
a espada na mão direita. E, no sentido contrário,
poderia se aproximar um inimigo. Portanto, a mão
esquerda segurava as rédeas e a direita permanecia
livre para o caso de luta.
Os primeiros pilotos, principalmente os que
lutaram na Primeira Guerra Mundial, eram
cavaleiros e, a partir da colocação de assentos lado
a lado nos aviões (o primeiro com essa disposição
teria sido o Farman Goliath, de 1916), os
cavaleiros-pilotos teriam preferido manter-se à
esquerda e com a mão direita mais livre que a
outra.
Depois disso vieram as pistas com circuitos
de tráfego e, inclusive, diferentemente dos
automóveis, nos países onde o tráfego é pela
direita, as ultrapassagens no ar são feitas por este
mesmo lado, não pela esquerda como os carros.
67
PROVAS DA AFA 1998 – 2010
Agora os helicópteros: os primeiros com III. A melhor hipótese para a posição, na
mais de um lugar seguiram o arranjo interno dos aeronave, do assento do piloto é a mesma que
aviões, e o piloto sentava-se à esquerda. justifica o tráfego de automóveis pela esquerda.
Entretanto, logo foi percebido que não era uma boa Está (ão) correta (s) a (s) afirmativa (s)
idéia, pois os primeiros aparelhos de asas rotativas a) I e II.
eram instáveis, e nem sempre o piloto poderia tirar b) I apenas.
a mão direita do cíclico, principal comando dos c) II apenas.
helicópteros, para acionar outros instrumentos ou d) III apenas.
comandos. Era preferível soltar a mão esquerda do
comando coletivo, o que oferecia menos 4. Segundo o texto, os primeiros aparelhos de asas
problemas ao vôo. rotativas possuíam assento do lado esquerdo para o
Portanto, nos aviões, a mão mais livre do piloto solo ou comandante, entretanto,
piloto solo ou comandante é a direita, ou a que está a) os pilotos estavam habituados com o assento do
do lado interno da cabine, pronta para cuidar da lado direito.
potência, instrumentos, etc. Já no caso dos b) era preciso soltar a mão esquerda do comando
helicópteros, a mão direita cuidará do comando principal para acionar os demais.
principal (o cíclico) e a esquerda estará mais livre c) a melhor posição era o lado esquerdo, pois
(igualmente será do lado interno da cabine). assim os pilotos ficavam com a mão direita livre.
(Solange Galante, in Revista AIR & SPORT BY d) não foi boa idéia, pois os pilotos, muitas vezes,
SKYDIVE) não tinham a mão direita disponível para acionar
outros instrumentos.
1. De acordo com o texto, na Idade Média, os
cavaleiros transitavam pelo lado esquerdo, porque 5. Observe este excerto:
a) mantinham a espada na mão direita. “Um dia Azevedo Gondim trouxe
b) seguravam as rédeas com a mão esquerda. boatos de revolução. O sul revoltado, o
c) permitiam, assim, a passagem do inimigo pela centro revoltado, o nordeste revoltado.”
direita. Faz-se referência a movimento militar de marcante
d) poderiam defender-se facilmente, pois influência na trama de São Bernardo, pela crise
carregavam a espada na mão esquerda. que provoca nas oligarquias rurais do país.
Trata-se da
2. O texto acima afirma, no primeiro parágrafo, a) Coluna Prestes.
que b) Revolução de 30.
a) o piloto solo de avião ocupa o assento do lado c) Campanha do Contestado.
oposto ao do piloto solo de helicóptero. d) Revolta do General Isidoro D. Lopes.
b) apenas as asas são as responsáveis pela
diferença entre os aviões e os helicópteros. 6. Atente para este excerto de “Sarapalha” :
c) a principal diferença entre os aviões e os “Estremecem, amarelas, as flores da
helicópteros é que nestes as asas são fixas e aroeira. Há um frêmito nos caules rosados da
naqueles, rotativas. erva-de-sapo. A erva-de-anum crispa as
d) os pilotos de ambas as aeronaves sentam-se em folhas, longas, como folhas de mangueira.
lugares diferentes para diferenciar os tipos de Trepidam, sacudindo as suas estrelinhas
aeronave. alaranjadas, os ramos da vassourinha. Tirita a
mamona, de folhas peludas, como o corselete
3. Leia com atenção: de um cassununga, brilhando em verde-e-azul.
I. Para explicar a diferença entre aviões e A pitangueira se abala, do jarrete à grimpa. E
helicópteros, a autora inicia analisando primeiro o açoita-cavalos derruba frutinhas fendilhadas,
um (helicóptero) e depois outro (avião). entrando em convulsões.”
II. Chega-se à conclusão de que a visualização
da pista de pouso determinou a posição do assento Observando-se nele várias palavras da área
do piloto. semântica de “tremer”, pode-se dizer que, com
esse recurso estilístico, o narrador busca sugerir o
reflexo na natureza de relevante fato da narrativa :
68
PROVAS DA AFA 1998 – 2010
a) estouro de boiada.
b) morbidez de malária. 11. “A nossa casa, aqui, vai ser uma casa de rico, e
c) prenúncio de vendaval. você vai morar com a gente, vai ter o seu quarto, o
d) aproximação do caipora. seu e do Pagão, por toda a sua vida. Vou trazer
roupa nova pra você e o Pagão, vou trazer comida
7. Assinale a alternativa que apresenta apenas pra gente, vou trazer semente pra se plantar. Vou
palavras paroxítonas (os acentos foram trazer sal pra temperar a panela.”
propositalmente omitidos): As formas verbais acima grifadas correspondem,
a) pudico, transfuga, ibero, erudito. respectivamente, a
b) transfuga, sabido, avaro, erudito. a) era, morava, tinha e tinha.
c) rubrica, pudico, filantropo, ciclope. b) será, morará, terá e trarei.
d) amalgama, quadrumano, rubrica, avaro. c) seria, moraria, teria e traria.
d) fora, morara, tivera e trouxera.
8. O emprego da vírgula está incorreto na
alternativa 12. Assinale a alternativa em que a flexão (de
a) “O governador do Estado disse que será número) dos adjetivos compostos segue as mesmas
organizada uma recepção quando o tenista voltar regras observadas em vermelho-claro, verde-alface
no final do mês, à cidade.” e anti-social, respectivamente.
b) “Nestes três anos de Real, o que mais me a) nipo-brasileira, surdo-mudo e furta-cor.
gratifica é saber que 13 milhões de brasileiros b) anglo-saxão, azul-piscina e recém-florido.
ultrapassaram a fronteira da pobreza.” c) médico-hospitalar, amarelo-limão e cor-de-rosa.
c) “A negativa anterior nos colocou em uma d) infanto-juvenil, cinzento-amarelado e sobre-
posição não confortável, e os alunos estão cientes humano.
disso.”
d) “Foi montado no sábado, um ambulatório 13. Seja
a frase :
médico para emergências ao lado do plenário da - Além de ter excelente pontaria com armas
Assembléia Legislativa.” automáticas, o tenente é habilíssimo no manejo do
revólver.
9. “Entrei nesse ano com o pé esquerdo. Vários Marque a alternativa em que o adjetivo está no
fregueses que sempre tinham procedido bem mesmo grau do grifado acima.
quebraram de repente. Houve fugas, suicídios, o a) Anacleto usa o uniforme limpo, limpo !
Diário Oficial se emprenhou com falências e b) Juvenal é o soldado mais experiente da equipe.
concordatas. Tive de aceitar liquidações c) Nosso esquadrão é o mais competitivo do
péssimas.” campeonato.
d) Este tanque tem o canhão melhor do que o
As palavras grifadas no período acima apresentam, daquele importado.
respectivamente,
a) ditongo crescente, dígrafo, hiato, ditongo 14. Aponte a opção em que cada palavra é exemplo
crescente e encontro consonantal. de um diferente processo de formação.
b) ditongo crescente, encontro consonantal, hiato, a) covil, dantesco, engrossava, opor, achego.
ditongo decrescente e dígrafo. b) passatempo, pedreiro, abuso, cacarejar, Anita.
c) ditongo decrescente, hiato, dígrafo, ditongo c) retorcendo, entristecer, traque, prosaico, atraso.
crescente e encontro consonantal. d) caiporismo, plenilúnio, cordel, veranico,
d) ditongo decrescente, encontro consonantal, desonra.
hiato, ditongo crescente e dígrafo.
15. “E daquele dia em diante, Valentim começou o
10. Aponte a alternativa cuja significação de todos aprendizado. O próprio atirador veio lhe ensinar
os substantivos varia com a mudança de gênero. todos os macetes: o importante era o pulso firme, o
a) dentista, hélice, lente. golpe de vista seguro, o balanço da faca certeiro;
b) cabeça, capital, caixa. quase tudo isso, o homem dizia, ele já trazia do
c) estudante, moral, sentinela. ofício de trapezista.”
d) personagem, rádio, patriarca. Pode-se dizer que o excerto acima contém
69
PROVAS DA AFA 1998 – 2010
a) um período apenas, com várias orações IV - Sempre foi um funcionário imbuído de boas
coordenadas e subordinadas. intenções.
b) vários períodos, formando um parágrafo, com V - Era uma pessoa misericordiosa aos mais
orações curtas e coordenadas. humildes.
c) um parágrafo com dois períodos, em que um Quanto à regência, estão corretas as frases
deles é composto por oração absoluta. a) I, III e IV.
d) um parágrafo com dois períodos, que, por sua b) II, IV e V.
vez, são compostos por orações coordenadas e c) I, II, e IV.
subordinadas. d) II, III e V.

16. Das frases abaixo apenas uma não foi alterada e 19. Das frases abaixo, uma foi propositalmente
apresenta correta concordância verbal. Assinale-a. alterada, passando a apresentar erro de colocação
a) Quinze anos, não havendo vocação, pede antes pronominal. Assinale-a.
o seminário do mundo que o de São José. a) Ora, como tudo cansa, esta monotonia acabou
b) Compreendo o seu gesto; a senhora não crê em por exaurir-me também. Quis variar, e lembrou-me
tais cálculos, parece-lhe que todos tem a alma escrever um livro.
cândida... b) Sim, Nero, Augusto, Massinissa, e tu, grande
c) Mas nem as sombras do céu, nem as danças César, que me incitas a fazer meus comentários,
fantásticas dos pássaros me desviava o espírito do agradeço-vos o conselho, e vou deitar ao papel as
meu interlocutor. reminiscências que me vierem vindo.
d) Outrossim, ria largo, se era preciso, de um c) Tudo era-me agora apresentado pela boca de
grande riso sem vontade, mas comunicativo, a tal José Dias, que me denunciara, e a quem eu
ponto as bochechas, os dentes, os olhos, toda a perdoara tudo, o mal que dissera, o mal que fizera,
cara, toda a pessoa, todo o mundo pareciam rir e o que pudesse vir de um e de outro.
nele. d) Foi então que os bustos pintados nas paredes
entraram a falar-me e a dizer-me que, uma vez que
17. Leia com atenção as frases abaixo. eles não alcançavam reconstituir-me os tempos
I - Com opinião e propostas claras, desfez as idos, pegasse da pena e contasse alguns.
dúvidas que pairavam sobre a questão.
II - Os empresários solicitaram tecnologia e 20. Aponte a alternativa cujo período, extraído da
financiamento estrangeiro. revista VEJA e propositalmente alterado, apresenta
III - O soldado era dotado de talento e coragem falha na sintaxe.
extraordinárias. a) Num dos casos mais angustiantes entre os 23
IV - Todos os presentes manifestaram profundo acidentes fatais debaixo da água, um mergulhador
pesar e dor. morreu quando o sistema de respiração falhou e ele
V - Permaneciam silenciosos o juiz, a advogada e não conseguiu subir, porque estava amarrado a
o réu. uma estrutura metálica, lá no fundo.
Com relação à concordância nominal, estão b) Assim como tantos outros administradores
corretas as frases públicos, Júlio Lacerda, prefeito de Moema, em
a) II e V. Minas Gerais, insistia numa velha fórmula para
b) III e IV. resolver os problemas de caixa de sua cidade:
c) I, II, IV e V. passar o chapéu nos gabinetes burocráticos da
d) I, II, III e IV. capital do Estado.
c) O Massachusetts Institute of Technology, MIT,
18. Observe as frases: acaba de acertar, por intermédio do Ministério da
I - Era uma pessoa alheia de tudo o que se passava Ciência e Tecnologia, sua colaboração com o
na sua família. governo para implementar a infovia - jargão que
II - Sinto-me ansioso por saber o resultado do designa os meios digitais de circulação de dados e
vestibular. informações - no Brasil.
III - Embora fosse pobre e carente de recursos, ela d) Carlos de Almeida Valente, que mora na cidade
o queria muito bem. de Prateados, no extremo norte do país, apontado
pela Polícia Federal como um dos reis do
70
PROVAS DA AFA 1998 – 2010
contrabando, transportando em um de seus aviões desmembrados em outros mais curtos e mais
bimotores e turbinados mais de 70% das claros.
mercadorias contrabandeadas dos Estados Unidos
e Paraguai para o Brasil. 23. Há uma alternativa cujo emprego da crase é
obrigatório. Assinale-a.
21. Assinale a alternativa em que o verbo em a) Ao ouvir a música, Clarissa chegou-se até à
destaque se apresenta com a mesma regência do janela.
grifado no período abaixo. b) Eleonora entregou o presente de aniversário à
“Apenas lhe informaram que os bens de Márcia.
Domingos Leite haviam sido confiscados.” c) Os guardas ficaram à distância de cem metros
a) “Com que então eu amava Capitu, e Capitu a do local da explosão.
mim?” d) Joana, quando voltou à sua residência, na terça-
b) “Também não me esqueceu o que me fez uma feira, encontrou-a assaltada.
tarde.”
c) “Então, Capitu abanava a cabeça (...): mas eu 24. I- Fique com nós, que somos seus verdadeiros
retorquia chamando-lhe maluca.” amigos!
d) “José Dias (...) a quem eu perdoava tudo, o mal II - O piloto convergia e divergia da pista de
que dissera, o mal que fizera (...).” pouso.
III - Os soldados encontravam-se no pátio aonde
22. Observe este excerto: seria hasteada a bandeira.
“Quando, ao contrário, os legisladores ou os IV - Todos os candidatos dissertaram sobre o
governos procuram monopolizar em suas mãos artigo nono, exceto eu e tu.
todo o poder, resumir em si todas as faculdades V - Enviaram-se-me os documentos.
que constituem a soberania popular, VI - Falou tão baixo que não se o pode ouvir.
entorpecendo a livre manifestação do VII - As crianças são mais bem comportadas que
pensamento, bem como a livre expansão da ti.
força e do poder individual, aí não só a Quanto ao emprego das classes de palavras, a
imaginação, como todas as faculdades alternativa que contém todas as construções
intelectuais do povo reprimidas, opressas, corretas dentre as acima é a
cerceadas, abafadas, desde que acha cerrada a a) II e VI.
esfera da livre discussão das opiniões e das b) I, IV e V.
causas que diretamente influem sobre o seu c) II, V e VI.
destino, lançam-se a procurar na ficção e nos d) I, III, IV e VII.
artifícios literários, não só o desenvolvimento
de que carece todo o espírito progressista, mas 25. Observe as formas verbais existentes no texto
ainda a representação colorida e disfarçada das abaixo e assinale a alternativa que contém uma
verdades e das opiniões, que não pode produzir afirmação correta sobre o emprego e a correlação
com liberdade.” de tempos.
Quanto a formação do período, podemos afirmar “Segurava ainda o cálice erguido ao céu. Uma
que detonação estrondosa. O arcebispo cai
a) se trata de um parágrafo “jumbo” constituído pesadamente para trás. Uma bala explosiva lhe
apenas por subordinação, o que torna difícil, ou arrebentara o peito. O sangue da vítima divina se
quase impossível, o seu entendimento. mistura com o sangue da vítima humana. Sela-se
b) é uma frase composta por coordenação e a aliança da verdadeira libertação, que é, a um
subordinação, usadas de forma a privilegiar a tempo, histórica e transcendente.”
clareza e a harmonia do texto. a) O presente “sela-se” situa a ação em um
c) é um período composto por subordinação momento anterior ao do presente “se mistura”.
unicamente, o que beneficia a composição do b) Os quatro verbos que estão no presente
texto, conferindo-lhe concisão e objetividade. constituem exemplo de uso do presente universal.
d) se trata da chamada frase “centopéica”, ou seja, c) O pretérito imperfeito exprime simultaneidade
composta de períodos caudalosos que devem ser em relação a um passado durativo explícito em
frase verbal.
71
PROVAS DA AFA 1998 – 2010
d)O pretérito mais-que-perfeito indica um b) é claro e conciso, porém apresenta falhas
momento anterior a um passado formalmente gramaticais.
expresso pelo presente. c) não é conciso, pois os pronomes “ele” e “seu”
causam ambigüidade.
26. Sejaa frase: d) a clareza está comprometida pelo emprego do
“Na reunião com as lideranças do movimento, o pronome pessoal e do possessivo.
governador anunciou que, a partir daquele
momento, quem apoiasse os grevistas teria as 29. Assinale a alternativa que apresenta narração
verbas estaduais reduzidas.” em primeira pessoa.
Marque a opção correta quanto à classificação da a) “Era magro, chupado, com um princípio de
oração “quem apoiasse os grevistas” e quanto à calva; teria seus cinqüenta e cinco anos. Levantou-
análise morfológica de “quem”, respectivamente. se com o passo vagaroso do costume, não aquele
a) oração subordinada adjetiva restritiva; pronome vagar arrastado dos preguiçosos, mas um vagar
substantivo relativo. calculado e deduzido, um silogismo completo, a
b) oração subordinada substantiva subjetiva; premissa antes da conseqüência, a conseqüência
pronome substantivo indefinido. antes da conclusão.”
c) oração subordinada substantiva completiva b) “Ao cabo, era amigo, não direi ótimo, mas nem
nominal; pronome substantivo indefinido. tudo é ótimo neste mundo. E não lhe suponhas
d) oração subordinada substantiva objetiva direta; alma subalterna; as cortesias que fizesse vinham
conjunção subordinativa integrante. antes do cálculo que da índole. A roupa durava-lhe
muito; (...). Era lido, posto que de atropelo, o
27. Assinale a alternativa que aponta discurso bastante para divertir ao serão e à sobremesa ou
indireto. explicar algum fenômeno (...).”
a) “A máxima é que a gente esquece devagar as c) “Não havendo remédio senão ficar com ele, fiz-
boas ações que pratica, e verdadeiramente não as me pai deveras. A idéia de que pudesse ter visto
esquece nunca. Pobre barbeiro! perdeu duas barbas alguma fotografia de Escobar, que Capitu por
naquela noite (...).” descuido levasse consigo, não me acudiu, nem, se
b) “Prima Justina exortava: ‘Prima Glória! Prima acudisse, persistiria. Ezequiel cria em mim como
Glória!’.José Dias desculpava-se: ‘Se soubesse não na mãe. Se fosse vivo, José Dias acharia nele a
teria falado, mas falei pela veneração, pela estima, minha própria pessoa.”
pelo afeto, (...).” d) “Com efeito há lugares em que o verso vai para
c) “Enfim, acabei as duas tranças. Onde estava a a direita e a música para a esquerda. Não falta
fita para atar-lhe as pontas? Em cima da mesa um quem diga que nisso mesmo está a beleza da
triste pedaço de fita enxovalhado. Juntei as pontas composição, fugindo à monotonia, e assim
das tranças, uni-as por um laço, retoquei a obra.” explicam o terceto do Éden, a ária de Abel, os
d) “Quando me perguntava se sonhara com ela na coros da guilhotina e da escravidão. Não é raro que
véspera, eu dizia que não, ouvia-lhe contar que os mesmos lances se reproduzam, sem razão
sonhara comigo, e eram aventuras extraordinárias, suficiente.”
que subíamos ao Corcovado pelo ar, (...).”
30. Assinale a alternativa em que há quebra do
28. A distância entre o motorista de vidros lacrados paralelismo com efeito estilístico.
e o mendigo que pede esmola no sinal vermelho é a) “Uma noite destas, vindo da cidade para o
maior que a distância entre ele e as trilhas agrestes Engenho Novo, encontrei no trem da Central um
das novelas e dos comerciais. Nas ruas rapaz aqui do bairro, que eu conheço de vista e de
esburacadas das metrópoles, ele talvez se sinta chapéu.”
escalando falésias. No seu coração a cidade b) “Só então senti que os olhos de prima Justina,
embrutecida é a pior de todas as selvas. quando eu falava, pareciam apalpar-me, ouvir-me,
Em relação ao excerto acima, extraído da revista cheirar-me, gostar-me, fazer o ofício de todos os
VEJA e propositalmente alterado, é correto afirmar sentidos.”
que c) “A porta não tinha chave nem taramela, abria-se
a) redundâncias e tautologias interferem na empurrando de um lado ou puxando de outro, e
clareza.
72
PROVAS DA AFA 1998 – 2010
fechava-se ao peso de uma pedra pendente de uma - Os ________ da ilha ainda praticam rituais
corda.” pagãos.
d) “Viúva, sentiu o terror ao separa-se de mim; - O rapaz vai ________ toda a fortuna da família.
mas era tão devota, tão temente a Deus, que - _________ em curvas fechadas é perigoso.
buscou testemunhas da obrigação, confiando a
promessa a parentes e familiares.” beneficente (1) / beneficiente (2)
seção (3) / sessão (4)
31. Leiaos excertos abaixo e assinale a alternativa aborígenes (5) / aborígines (6)
que aponta a relação estabelecida entre os termos despender (7) / dispender (8)
grifados. freiar (9) / frear (10)
I “Mandei-lhe coser as mais finas cambraias, (...) As lacunas serão preenchidas corretamente, na
uma linda touca de renda...” ordem, com os vocábulos de número
II “Cosi-me muito à parede, e vi-o passar com suas a) 2, 4, 5, 7 e 9.
calças brancas engomadas(...).” b) 2, 3, 5, 8 e 9.
a) Sinonímia c) 1, 3, 6, 8 e 10.
b) Paronímia d) 1, 4, 6, 7 e 10.
c) Polissemia
d) Homonímia 35. Leia atentamente o exemplo:
“- Padilha e padre Silvestre incorporaram-se às
32. Observe os períodos abaixo: tropas revolucionárias e conseguiram galões.”
I - “Deu um mal neles, morria de um em um, Assinale a alternativa em que ocorre a mesma
secando, devia ser de tísica. Quando os negros se figura de linguagem utilizada acima.
acabaram, ficou a terra vaga anos e anos.” a) “O sol ia subindo, por cima do vôo verde das
II - “O bodegueiro seguiu viagem, Zé soldado aves itinerantes.”
esperou que ele saísse das vistas; deu meia volta e b) “De repente, na mata resseca do sonho, crepitou
tratou de vir correndo, marche-marche, me contar e chamejou o barulho...”
a notícia no Socorro.” c) “Estava olhando, assim esquecido, para os
III - “Afinal, só debaixo da terra é que se pode olhos... olhos grandes, escuros e meio de-quina,
mesmo esconder a riqueza. Eu, de mim, vou como os de uma suassuapara...”
sumindo com o que eu tenho, no meu bom d) “Durante o mesmo prazo, a amiga não mudou
esconderijo.” de bigode, trocou muitas cartas com ele,
Há conotação em relacionou-se na casa, e ninguém ignorava que
a) I e II. entre ambos existia um laço íntimo.”
b) I, II e III.
c) I apenas. 36. Sejaeste excerto:
d) III apenas. “Ao invés dos salões aristocráticos e dos
ambientes sofisticados, o romance se passa nas
33. “Não consultes dicionários. Casmurro não está ruas e casebres do Rio de Janeiro do ‘tempo do
aqui no sentido que lhes dão, mas no que lhe pôs o rei’ (D. João VI), com seu povo alegre e seus
vulgo de homem calado e metido consigo. Dom tipos pitorescos (as comadres, os moleques, os
veio por ironia para atribuir-me fumos de fidalgo.” soldados, as mulatas dengosas).
Predominam, no excerto acima, as funções A linguagem usada pelo autor reflete bem
a) fática e referencial. tanto o seu descompromisso com relação à moda
b) poética e referencial. da época como ainda sua intenção de reproduzir
c) fática e metalingüística. certas características da fala da gente do povo, de
d) conativa e metalingüística. baixa cultura e condição social.”
(Douglas Tufano)
34. Observe as frases com lacunas e, abaixo, os Fala-se da obra
pares de vocábulos numerados. a) Senhora.
- O bingo _________ esteve bastante concorrido. b) A moreninha.
- Os vereadores votarão o projeto na próxima c) Memórias Póstumas de Brás Cubas.
_________. d) Memórias de um sargento de milícias.

73
PROVAS DA AFA 1998 – 2010
amar e malamar,
37. Assinale a alternativa correta quanto ao amar, desamar, amar ?
Modernismo no Brasil. sempre, e até de olhos vidrados, amar ?”
a) Em sua segunda fase como escritor, Érico
Veríssimo realiza uma investigação das origens e 40. Em Grande sertão: veredas, Riobaldo é ao
da formação social de sua região. São dessa fase mesmo tempo herói e narrador da história,
obras como Clarissa e Olhai os Lírios do Campo. configurando uma situação em que ele
b) A obra São Bernardo, de Graciliano Ramos, a) lê a sua história para um grupo de ouvintes.
pertence à fase do Modernismo que vai de 1922 a b) conta por escrito a sua história para um leitor
1930 e consiste num “relato” que o narrador determinado.
personagem, homem rústico e insensível, faz de c) conta oralmente a sua história para um ouvinte
sua vida fracassada. determinado.
c) A obra Gabriela, Cravo e Canela, de Jorge d) narra a sua história oralmente perante os
Amado, pertence à fase em que autor aborda membros de um júri popular.
“temas sociais”, marcada por heróis e líderes
populares que se recusam a aceitar o latifúndio e as
regalias da burguesia. AFA 1998/1999 – Português
d) Em Macunaíma, Mário de Andrade realiza a
junção de lendas amazônicas com histórias do O excerto abaixo é extraído de uma das obras
folclore brasileiro, recorrendo a várias fontes para indicadas para este concurso. Leia-o atentamente e,
compor o perfil de Macunaíma, um personagem considerando o conhecimento que você deve ter do
cheio de incoerências e contradições. conto na íntegra bem como da obra em questão,
responda às questões a ele vinculadas, de 01 a 11.
38. O darwinismo, o positivismo, o determinismo Informa-se que o título “Família” é criação nossa.
constituem algumas das teorias relevantes para o
estudo do embasamento filosófico do FAMÍLIA
a) Simbolismo.
b) Romantismo. 0 Na cabeceira da mesa, a toalha
c) Pré-Modernismo. 1 manchada de coca-cola, o bolo
d) Realismo-Naturalismo. desabado, ela era a mãe. A
aniversariante piscou.
39. “Ó Formas alvas, brancas, Formas claras Eles se mexiam agitados, rindo, a
de luares, de neves, de neblinas! ... 0 sua família. E ela era a mãe de todos.
Ó Formas vagas, fluídas, cristalinas ... 5 E se de repente não se ergueu, como
Incensos dos turíbulos das aras ...” um morto se levanta devagar e obriga
Assinale a alternativa cujo texto pertence ao mudez e terror aos vivos, a
mesmo período literário do texto acima. aniversariante ficou mais dura na
a) “Debruçada nas águas dum regato cadeira, e mais alta. Ela era a mãe de
A flor dizia em vão 1 todos. E como a presilha a sufocasse,
À corrente, onde bela se mirava ... 0 ela era a mãe de todos e, impotente à
‘Ai, não me deixes, não !’ ” cadeira, desprezava-
b) “Enquanto revolver os meus consultos, -os. E olhava-os piscando. Todos
tu me farás gostosa companhia, aqueles seus filhos e netos e bisnetos
lendo os fastos da sábia, mestra História, que não passavam de carne de seu
e os cantos da poesia.” 1 joelho, pensou de repente como se
c) “Mãos que os lírios invejam, mãos eleitas 5 cuspisse. Rodrigo, o neto de sete
Para aliviar de Cristo os sofrimentos, anos, era o único a ser a carne de seu
Cujas veias azuis parecem feitas coração, Rodrigo, com aquela carinha
Da mesma essência astral dos olhos bentos.” dura, viril e despenteada. Cadê
d) “Que pode uma criatura senão, Rodrigo? Rodrigo com olhar sonolento
entre criaturas, amar ? 2 e intumescido naquela cabecinha
amar e esquecer, 0 ardente, confusa. Aquele seria um
74
PROVAS DA AFA 1998 – 2010
homem. Mas, piscando, ela olhava os 1. Assinale a alternativa que NÃO condiz com
outros, a aniversariante. Oh o esse texto.
desprezo pela vida que falhava. a) Algumas reflexões da protagonista denunciam
Como?! como tendo sido tão forte uma visão machista quanto ao comportamento da
2 pudera dar à luz aqueles seres mulher na família.
5 opacos, com braços moles e rostos b) Embora haja uma livre associação de idéias
ansiosos? Ela, a forte, que casara em aparentemente desconexas, a protagonista
hora e tempo devidos com um bom consegue estabelecer com propriedade a dicotomia
homem a quem, obediente e “aparência X essência”.
independente, ela respeitara; a quem c) “Abandonada” pelo narrador, a personagem
3 respeitara e que lhe fizera filhos e lhe central entrega-se ao monólogo interior, resultando
0 pagara os partos e lhe honrara os disso um feitio frasal caótico tanto no plano da
resguardos. O tronco fora bom. Mas expressão quanto no plano do conteúdo.
dera aqueles azedos e infelizes frutos, d) Percebe-se uma reflexão sobre o papel
sem capacidade sequer para uma boa do idoso em nossa sociedade, na medida em que se
alegria. Como pudera ela dar à luz apresenta a matriarca reacionária, inconformada
3 aqueles seres risonhos, fracos, sem com a mediocridade da sua família.
5 austeridade? O rancor roncava no seu
peito vazio. Uns comunistas, era o 2. É correto dizer que em “Família”
que eram; uns comunistas. Olhou-os a) aparecem em igual proporção denotação e
com sua cólera de velha. Pareciam conotação; a primeira, nas reflexões da
ratos se acotovelando, a sua família. protagonista; e a segunda, na fala do narrador.
4 Incoercível, virou a cabeça e com b) há predomínio da linguagem denotativa, pois a
0 força insuspeita cuspiu no chão. interpretação do texto depende de cada leitor e das
Mamãe! gritou mortificada a dona experiências pessoais de cada um.
da casa. - Que é isso, mamãe! gritou c) predomina a linguagem objetiva, estabelecendo
ela, passada de vergonha, e não uma comunicação eficaz e informativa, que
queria sequer olhar os outros, sabia privilegia a forma em detrimento do conteúdo.
4 que os desgraçados se entreolhavam d) prevalece a linguagem conotativa, utilizada a
5 vitoriosos como se coubesse a ela dar partir de uma visão pessoal da protagonista, que
educação à velha, e não faltaria muito faz uso de comparações para caracterizar alguns
para dizerem que ela já não dava membros de sua família.
mais banho na mãe, jamais
compreenderiam o sacrifício que ela 3. Descartando-se qualquer consideração relativa
5 fazia. - Mamãe, que é isso! disse à liberdade artística, pode-se dizer que falta uma
0 baixo, angustiada. - A senhora nunca vírgula preconizada pela norma culta da língua em
fez isso! Acrescentou alto para que a) “(...) quando o galo cantar pela terceira vez
todos ouvissem, queria se agregar ao renegarás tua mãe.”(53)
espanto dos outros, quando o galo b) “(...) não faltaria muito para dizerem que ela já
cantar pela terceira vez renegarás tua não dava mais banho na mãe (...)”(46)
5 mãe. Mas seu enorme vexame c) “Mas seu enorme vexame suavizou-se quando
5 suavizou-se quando ela percebeu que ela percebeu que eles abanavam a cabeça (...)”(54)
eles abanavam a cabeça como se d) “(...) sabia que os desgraçados se entreolhavam
estivessem de acordo que a velha não vitoriosos como se coubesse a ela dar educação à
passava agora de uma criança. velha (...)”(44)
- Ultimamente ela deu pra cuspir,
6 terminou então confessando contrita 4. As formas verbais “respeitara”(29),
0 para todos. “fizera”(29), “pagara”(29) e “honrara”(30) têm
como equivalentes, respectivamente,
(Clarice Lispector) a) respeitou, fez, pagou e honrou.
b) respeitava, fazia, pagava e honrava.

75
PROVAS DA AFA 1998 – 2010
c) tinha respeitado, havia feito, havia pago e tinha
honrado. 8. Em relação a encontros vocálicos
d) tivesse respeitado, houvesse feito, houvesse e consonantais, é correto afirmar
pago e tivesse honrado. que as palavras “aniversariante”(03),
“austeridade”(35) e “compreenderiam”(48)
5. Aponte o par de frases do texto em que as contêm, respectivamente,
palavras grifadas têm a mesma função sintática. a) ditongo; hiato e encontro consonantal; hiato e
a) “E olhava-os piscando.”(12) hiato.
“(...) a aniversariante ficou mais dura na cadeira b) hiato; ditongo e encontro consonantal; ditongo,
(...)”(08) hiato e ditongo.
b) “Oh o desprezo pela vida que falhava.”(22) c) encontro consonantal e hiato; hiato; encontro
“(...) pudera (ela) dar à luz aqueles seres consonantal, hiato e ditongo.
opacos (...)”(24) d) encontro consonantal e hiato; ditongo e
c) “Olhou-os com sua cólera de velha.”(37) encontro consonantal; encontro consonantal, hiato
“- Que é isso, mamãe! gritou ela passada de e ditongo.
vergonha (...)”(42)
d) “(...) terminou então confessando contrita para 9. “E como a presilha a sufocasse, ela era a mãe
todos.”(59) de todos e, impotente à cadeira, desprezava-
“Eles se mexiam agitados, rindo, a sua -os.”(10)
família.”(04) Assinale a alternativa cujas palavras apresentam,
respectivamente, o mesmo número de letras e de
6. “O rancor roncava no seu peito vazio.”(35) fonemas daquelas grifadas no período acima.
“(...) quando o galo cantar pela terceira vez a) cuspisse, infelizes, roncava
renegarás tua mãe.”(53) b) carinha, manchada, agregar
Ocorrem nessas frases, respectivamente, as figuras c) piscando, confessou, alegria
de linguagem: d) agitados, obedientes, pareciam
a) ironia e perífrase.
b) aliteração e alusão. 10. Marque a alternativa que ordena as frases
c) onomatopéia e pleonasmo. abaixo, formando um parágrafo coerente e coeso,
d) personificação e preterição. respeitando as relações expressas entre parênteses.
I. ainda existisse Rodrigo, com aquela carinha
7. Observe estes excertos: dura e viril (oposição, por subordinação, ao
I. “ - Que é isso, mamãe! gritou ela, passada de pensamento principal)
vergonha, e não queria sequer olhar os outros, II. fora forte, impetuosa e austera (atributo do
sabia que os desgraçados se entreolhavam sujeito de III)
vitoriosos como se coubesse a ela dar educação à III.ela desprezava todos aqueles seus azedos e
velha, e não faltaria muito para dizerem que ela já infelizes frutos (idéia principal)
não dava mais banho na mãe (...)”(42) IV.seria a única carne de seu coração (atributo do
II. “O tronco fora bom. Mas dera aqueles azedos e sujeito de I)
infelizes frutos, sem capacidade sequer para uma a) Ela que fora forte, impetuosa e austera
boa alegria. Como pudera ela dar à luz aqueles desprezava todos aqueles seus azedos e infelizes
seres risonhos, fracos, sem austeridade? O rancor frutos, contudo ainda existisse Rodrigo, que seria a
roncava no seu peito vazio.”(30) única carne de seu coração, com aquela carinha
Ocorrem neles sucessivamente dura e viril.
a) discurso direto, discurso indireto e discurso b) Ela fora forte impetuosa e austera e desprezava
direto. todos aqueles seus azedos e infelizes frutos, ainda
b) discurso indireto, discurso direto e discurso que existisse Rodrigo que seria a única carne de
indireto livre. seu coração, com aquela carinha dura e viril.
c) discurso direto, discurso indireto e discurso c) Ela, que fora forte, impetuosa e austera,
indireto livre. desprezava todos aqueles seus azedos e infelizes
d) discurso indireto, discurso indireto livre e frutos, embora ainda existisse Rodrigo, que seria a
discurso indireto livre.
76
PROVAS DA AFA 1998 – 2010
única carne de seu coração, com aquela carinha acentuação, embora o acento gráfico de ambas
dura e viril. incida na mesma sílaba tônica.
d) Se bem que ainda existisse Rodrigo, que seria a a) intrínseca e século
única carne de seu coração, com aquela carinha b) superfície e indivíduo
dura e viril, ela desprezava todos aqueles seus c) intrínseca e desprezível
azedos e infelizes frutos, embora tivesse sido forte, d) desprezível e superfície
impetuosa e austera.
15. As palavras exterior e intrínseca, presentes no
11. Qual função da linguagem predomina no texto, apresentam, em sua estrutura mórfica,
excerto abaixo? prefixos latinos que indicam circunstâncias
“ - Mamãe! gritou mortificada a dona da casa. - opostas. De posse desse dado, assinale o par de
Que é isso, mamãe!”(41) palavras que contêm, respectivamente, prefixos
a) poética gregos correspondentes aos latinos mencionados.
b) emotiva a) exonerar e incrustar
c) conativa b) êxodo e endoscópio
d) referencial c) exógeno e hipertrofia
d) emigrante e imigrante
Texto para as questões 12 a 15.
“A ilusão de uma verdade puramente exterior, Atente para este texto, ao qual se vinculam as
existindo ‘a priori’ e sem a participação do questões 16, 17 e 18.
indivíduo na sua realidade intrínseca, entrou a
dominar a literatura européia (...). A imaginação Casa no Campo
perdeu seu poder temporal e suas prerrogativas
absolutas; as ‘construções’ que não tinham por Eu quero uma casa no campo
base documento principiavam a vacilar, e a onde eu possa compor muitos rocks rurais
observação limitou o terreno da fantasia, nivelou- e tenha somente a certeza dos amigos do peito
lhe a superfície irregular, marcou-lhe as dimensões e nada mais
e determinou-lhe os confins, reduzindo-o a uma Eu quero uma casa no campo
porção insignificante e quase desprezível.” onde eu possa ficar do tamanho da paz
(Ronald de Carvalho) e tenha somente a certeza dos limites do corpo
12. Essa nova postura literária vai determinar, no e nada mais
Brasil, o surgimento do Eu quero carneiros e cabras pastando solenes
a) Realismo. no meu jardim
b) Simbolismo. Eu quero o silêncio das línguas cansadas
c) Romantismo. Eu quero a esperança de óculos
d) Parnasianismo. um filho de cuca legal
Eu quero plantar e colher com a mão
13. Observando-se o emprego dos pronomes a pimenta e o sal
gritados, 1, 2 e 3, no processo coesivo do texto, Eu quero uma casa no campo
dir-se-á que do tamanho ideal
a) 1, 2 e 3 retomam a expressão “terreno da pau-a-pique e sapê
fantasia”. Onde eu possa plantar meus amigos
b) 1 e 2 retomam “superfície”; 3 retoma meus discos
“dimensões”. meus livros
c) 1 retoma “superfície”; 2 retoma “dimensões”; 3 e nada mais
retoma “confins”.
d) 1 retoma “terreno da fantasia”; 2 retoma (TAVITO & RODRIX, Zé. In: Elis Regina. Elis.
“dimensões”; 3 retoma “confins”. LP Philips no 6349032, 1972, I. 2, f. 5.)

14. Marque a opção em que as duas palavras do 16. Sobre “Casa no campo”, pode-se afirmar que,
texto NÃO podem ser incluídas na mesma regra de

77
PROVAS DA AFA 1998 – 2010
a) quando o eu-lírico diz “e nada mais”, revela sua gente para assistir as cerimônias, que duraram sete
desesperança em relação à civilização e ao dias.”
progresso. d) “Simão Bacamarte explicou-lhe que
b) ao refugiar-se no campo, buscando o “silêncio D. Evarista reunia condições fisiológicas e
das línguas cansadas”, o eu-lírico afirma sua anatômicas de primeira ordem, digeria com
descrença nos homens. facilidade, dormia regularmente, tinha bom pulso,
c) ao enaltecer o campo como espaço ideal, e excelente vista; estava assim apta para dar-lhe
ambiente não corrompido, o eu-lírico revela sua filhos robustos, sãos e inteligentes.”
negação ao enfrentamento da problemática urbana.
d) ao pretender ser o próprio produtor de seus 20. Aponte a opção cuja palavra em destaque está
alimentos e ao “plantar” amigos, discos e livros, o corretamente grafada, livre da interferência do
eu-lírico nega o progresso urbano-industrial, a arte falar inculto.
e fundamentalmente a máquina que, numa a) Bêbados, não percebiam a hilariedade da
inversão de papéis, passou a produzir tudo por ele. situação.
b) Por não conhecerem seus direitos, muitos
17. O bucolismo presente nesses versos é revelador deixam de revindicá-los.
de uma tendência atual de valorização da natureza c) Narizinho, sentada na sala, próxima de D.
- veja-se a preocupação com a Ecologia - Benta, ouvia prazeirosamente as histórias da avó.
demonstrando que ainda estão vivos e circulando d) Para certificar-se da causa da disenteria
entre nós os ideais renitente, o médico indicou alguns exames de raios
a) árcades. X.
b) barrocos.
c) românticos. 21. Marque a alternativa em que ambas as frases
d) simbolistas. estão corretas quanto ao emprego do verbo haver.
a) Há de haver carros mais baratos.
18. Os termos grifados exercem, respectivamente, Haviam alguns meses que o navegante
estas funções sintáticas: partira.
a) adjunto adverbial, objeto indireto e adjunto b) Podem haver alguns casos de dengue na
adverbial. periferia.
b) adjunto adnominal, adjunto adnominal e Há de haver pessoas de bom senso entre os
adjunto adverbial. rebelados.
c) adjunto adnominal, adjunto adnominal e c) É possível que hajam alguns obstáculos para a
predicativo do objeto. sua inscrição no congresso. Durante a cerimônia
d) adjunto adverbial, complemento nominal e religiosa, as crianças se houveram com
predicativo do sujeito. surpreendente discrição.
d) O policial perguntou aos suspeitos onde
19. Das frases abaixo, extraídas de O Alienista, houveram eles tantos dólares. Ainda que houvesse
uma foi propositalmente alterada, passando a alguns candidatos inconformados, o resultado do
apresentar INCORREÇÃO de regência. Assinale- concurso foi mantido.
a.
a) “Não havia na colônia, e ainda no reino, uma só 22. Assinale a alternativa que contém todas as
autoridade em semelhante matéria, mal explorada, frases corretas da relação abaixo.
ou quase inexplorada.” I. Proíbem-se vender tais remédios.
b) “Os lugares de presidente e secretários eram de II. Fui eu que lhe pediu que não viesse.
nomeação régia, por especial graça do finado rei III. As pacientes mesmas fizeram a faxina da
D. João V, e implicavam o tratamento de enfermaria.
Excelência e o uso de uma placa de ouro no IV. A maior parte dos conferencistas faltaram ao
chapéu.” debate.
c) “Inaugurou-se com imensa pompa; de todas as V. A minha nota da prova poderá subir se o
vilas e povoações próximas, e até remotas, e da professor se dispor a rever a questão duvidosa.
própria cidade do Rio de Janeiro, correu muita

78
PROVAS DA AFA 1998 – 2010
VI. A sociedade não pode continuar a assistir e ser ambos os matrimônios com o furor das sedas,
vítima de tão brutais crimes que desfilam pelo veludos, rendas e pedras preciosas que manifestou.
noticiário.
VII.Quando ofereceram-lhe o cargo, por que você 25. Transcrevem-se abaixo quatro excertos de
não o aceitou? ensaio modernista, dos quais três foram alterados
VIII.Fique com nós, que somos os seus propositalmente, restando apenas um
verdadeiros amigos! gramaticalmente correto. Assinale-o.
a) II, V e VI a) Numa terra radiosa vive um povo triste.
b) I, V e VII Legaram-no essa melancolia os descobridores que
c) III, IV e VIII lhe revelaram ao mundo e a povoaram. O
d) I, II, III e VII esplêndido dinamismo dessa gente rude obedecia a
dois grandes impulsos que dominam toda a
23. Aponte o par de frases correto quanto ao psicologia e nunca foram geradores de alegria:
emprego (uso ou omissão) do sinal de crase. ambição do ouro e a sensualidade livre e infrene
a) O eclipse ocorrerá à uma hora da madrugada. que, como culto, a Renascença fizera ressuscitar.
O policial postou-se à distância de uns vinte b) Dessa Renascença surgira um homem novo com
metros. um novo modo de pensar e sentir. A sua história
b) O documento será entregue à V. Exa ainda neste será a própria história da conquista da liberdade
mês. consciente do espírito humano. É assim que a volta
Assim que o barco aportou, levaram o ferido à ao paganismo - se teve um efeito desastroso para a
terra. evolução artística da humanidade que viu
c) A velha não está disposta à fazer as pazes com a estancada a fonte viva da imaginação criadora da
nora. Idade Média - teve como melhor resultado o
Ontem encontramos no aeroporto a cantora à qual alargamento, para assim dizer, das ambições
os jornais aludiram. humanas de poderio, de saber e de gozo.
d) Devemos dar assistência a esta mulher e a que c) A era dos descobrimentos foram o resultado
tem mais filhos. desse movimento de libertação. Dilatava o mundo
Quando excursionar à Europa, não deixe de ir até a em que dois terços ainda não eram conhecidos e
Madri das touradas. exaltava a vida física, como mais tarde a
Revolução Francesa foi a exaltação da vida
24.Observe este período: intelectual, arrogante e independente. As
“Com o furor das sedas, veludos, rendas e pedras navegações comerciais dos venezianos, genoveses
preciosas que manifestou, não podia conciliar-se a e catalães seguia-se outras mais audaciosas,
modéstia com que ela vivera em ambos os abrindo novos céus e terras.
matrimônios, logo que voltou do Rio de Janeiro.” d) Recomeçava na história do mundo o misterioso
Ao se passá-lo para ordem direta, tem-se: impulso que de séculos em séculos põem em
a) A modéstia com que ela vivera em ambos os movimento as massas humanas, após os longos
matrimônios não podia conciliar-se com o furor repousos com que as civilizações nascem, se
das sedas, veludos, rendas e pedras preciosas que desenvolvem e morrem. Mais uma vez, nesse
manifestou logo que voltou do Rio de Janeiro. movimento de fluxo e refluxo, a inquietação
b) A modéstia com que ela vivera em ambos os migratória tomaria o aspecto de imperialismo
matrimônios, logo que voltou do Rio de Janeiro, econômico e comercial. Em procura de ouro, que
com o furor das sedas, veludos, rendas e pedras já escasseava, italianos, portugueses, espanhóis,
preciosas que manifestou, não podia conciliar-se. holandeses, ingleses, franceses, lançavam-
c) Não podia conciliar-se com o furor que -se à porfia pelos novos caminhos marítimos.
manifestou das sedas, veludos, rendas e pedras
preciosas, logo que voltou do Rio de Janeiro, a 26. “É da história do mundo que (1) as elites nunca
modéstia com que ela vivera em ambos os introduziram mudanças que (2) favorecessem a
matrimônios. sociedade como um todo. Estaríamos nos
d) Logo que voltou do Rio de Janeiro, não podia enganando se achássemos que (3) estas lideranças
conciliar-se a modéstia com que ela vivera em empresariais teriam motivação para fazer a

79
PROVAS DA AFA 1998 – 2010
distribuição que (4) uma nação equilibrada precisa em sua cadeira de balanço, admirando o pôr--do-
ter.” sol.
Marque a opção que registra a classificação do Dir-se-á que
vocábulo que em cada uma das quatro ocorrências, a) I é coeso, mas não coerente; II é coeso e
pela ordem. coerente; III é coeso e coerente.
a) 1. conjunção integrante, 2. pronome relativo, 3. b) I é coeso e coerente; II é coeso e coerente; III é
pronome relativo, 4. conjunção integrante coeso, mas não coerente.
b) 1. pronome relativo, 2. pronome relativo, 3. c) I é coeso e coerente; II é coerente, mas não
conjunção integrante, 4. conjunção integrante coeso; III é coeso, mas não coerente.
c) 1. conjunção integrante, 2. pronome relativo, 3. d) I é coerente, mas não coeso; II é coeso, mas não
conjunção integrante, 4. pronome relativo coerente; III é coerente, mas não coeso.
d) 1. pronome relativo, 2. conjunção integrante, 3.
pronome relativo, 4. conjunção integrante 29. Observe as frases abaixo e marque a opção que
contém aquelas em que o rompimento do
27. Observe estes excertos: paralelismo resulta um efeito de bom estilo.
“Eu faço versos como {quem chora I. Não fui à missa por estar gripado e porque não
De desalento...de desencanto.}” (Manuel aprecio os sermões do padre Onofre.
Bandeira) II. Palmeiras perde o jogo e a cabeça na Argentina.
“Mas o hóspede inconvidado III.O que ele mais admira na esposa é a beleza, a
Que mora no meu destino, cultura e não fumar.
Que não sei {como é chegado,} IV.Granofruit - cereais e frutas - Use puro, com
nem {de que honras é digno,} leite, iogurte, ou com sua imaginação.
Constrange meu ser de casa V. Na Europa ele teve a oportunidade de visitar
A adaptações de disfarce.” (Fernando Pessoa) Roma, Madri e o Museu do Louvre.
As orações destacadas classificam-se, a) I e III
respectivamente, como b) II e IV
a) oração subordinada adverbial conformativa, c) I, II e V
oração subordinada adverbial comparativa e d) III, IV e V
oração coordenada sindética alternativa.
b) oração subordinada adverbial comparativa, 30. Em carta de 1944 a um amigo, discorrendo
oração subordinada adverbial conformativa e sobre a gênese de uma de suas obras, Graciliano
oração subordinada substantiva objetiva indireta. Ramos declara:
c) oração subordinada substantiva subjetiva, “Fiz o livrinho sem paisagens, nem diálogos.
oração subordinada substantiva objetiva direta e Nisso, pelo menos, ele deve ter alguma
oração subordinada substantiva objetiva direta. originalidade. Ausência de tabaréus bem falantes,
d) oração subordinada adverbial comparativa, queimadas, cheias, poentes vermelhos, namoros de
oração subordinada substantiva objetiva direta e caboclos. A minha gente, quase muda, vive numa
oração subordinada substantiva objetiva indireta. casa velha da fazenda; as personagens adultas,
preocupadas com o estômago, não têm tempo de
28. Observe os excertos I, II e III. abraçar-
I. Os morcegos entraram pela janela e voejaram -se. Até a cachorra é uma criatura decente, porque
sobre a classe. De repente um dos mamíferos na vizinhança não existem galãs caninos.”
enroscou-se nos cabelos de Pat, a aluna mais chata. Assinale a alternativa cujas assertivas se
Foi uma gozação geral! depreendem desse trecho de carta.
II. Muita gente derrotista apregoa que o plano a) Faz-se velada e sutil crítica ao regionalismo
econômico do atual governo tende ao fracasso, pitoresco; o “livrinho” é Vidas Secas.
mas eles estão completamente enganados. b) Critica-se a falta de originalidade dos
III.A casa de Anacleto no bairro de Santana, em ficcionistas da primeira fase modernista; a
São Paulo, é bastante confortável. Além das “cachorra” é Baleia.
dependências convencionais, possui escritório, c) Elogia-se o regionalismo dos romancistas do
biblioteca, sauna e uma espaçosa varanda voltada ciclo da seca; “as personagens adultas” são Paulo
para o leste, onde todas as tardes se vê Anacleto, Honório e Sinhá Vitória.
80
PROVAS DA AFA 1998 – 2010
d) Hostilizam-se os regionalistas românticos por a) Porfírio.
abusarem do pictórico; a preocupação “com o b) Crispim.
estômago” se deve a problemas gástricos c) João Pina.
decorrentes da ingestão de água salobra. d) padre Lopes.

31. Como o conto cujo fragmento encabeça esta 35. A obra Angústia, de Graciliano Ramos,
obra, a maioria das narrativas de Laços de família segundo as tendências do romance moderno, de 30
tratam da condição feminina em diferentes para cá, que consideram o grau de tensão entre o
contextos. São exemplos disso todos os contos da “herói” e o “mundo”, pode ser caracterizada como
alternativa romance de tensão
a) “O búfalo”, “Amor”, “Uma galinha”. a) crítica.
b) “Devaneio e embriaguez duma rapariga”, “A b) mínima.
imitação da rosa”, “O búfalo”. c) interiorizada.
c) “Mistério em São Cristóvão”, “Começos de d) transfigurada.
uma fortuna”, “Os laços de família”.
d) “A menor mulher do mundo”, “Preciosidade”, 36. “Nas Primeiras estórias é patente o fascínio do
“Começos de uma fortuna”. alógico: são contos povoados de crianças, loucos e
seres rústicos que cedem ao encanto de uma
32. Elemento coletivo e contagiante, o canto iluminação junto à qual os conflitos perdem todo
funciona, num dos contos de Primeiras estórias, relevo e todo sentido.” (Alfredo Bosi)
como fator de solidariedade, ao permitir que as Comprovando essa assertiva do crítico Bosi,
pessoas dividam com a personagem a mágoa da vamos encontrar, na obra em questão,
partida dos parentes dela, preenchendo o vazio que respectivamente, centrados nesses seres, os contos
fica. a) “A menina de lá”, “Um moço muito branco”,
De que conto se trata? “Substância”.
a) “Fatalidade” b) “Nenhum, nenhuma”, “Darandina”,
b) “A terceira margem do rio” “Famigerado”.
c) “Soroco, sua mãe, sua filha” c) “ A terceira margem do rio”, “O cavalo que
d) “Partida do audaz navegante” bebia cerveja”, “Fatalidade”.
d) “Pirlimpsiquice”, “Soroco, sua mãe, sua filha”,
33. Em relação à obra Angústia, é INCORRETO “As margens da alegria”.
dizer que
a) o recurso do fluxo de consciência se 37. “ A única objeção contra a palavra do Quincas
compatibiliza com o estado de desarticulação Borba é que não me sentia doido, mas não tendo
espiritual do narrador. geralmente os doidos outro conceito de si mesmos,
b) o narrador faz uma análise impotente da miséria tal objeção ficava sem valor. E vede se há algum
moral de seu mundo, que só se resolve pelo crime fundamento na crença popular de que o filósofos
e pela autodestruição. são homens alheios às coisas mínimas. No dia
c) a narrativa avança, numa atmosfera de pesadelo seguinte, mandou-
e mau humor, para um desenlace em que não se -me o Quincas Borba um alienista.”
resolvem os conflitos do herói problemático. Esse excerto, cujo conteúdo é recorrente em
d) o narrador, pausada e refletidamente, registra os Machado de Assis, é extraído de narrativa
fatos de que participou, a despeito de ainda viver machadiana que aborda a insânia como um grande
as conseqüências das infaustas ocorrências de seu motivo aberto, permeando a vida e atravessando-a
passado recente. como irônico mistério.
A narrativa em questão é
34. “Preso por ter cão, preso por não ter cão!” a) O alienista.
Assim reage um dos moradores de Itaguaí, b) Quincas Borba.
internado duas vezes na Casa Verde: num primeiro c) Dom Casmurro.
momento por ser desequilibrado, mais tarde por ser d) Memórias póstumas de Brás Cubas.
equilibrado demais. (O alienista)
A personagem em questão é
81
PROVAS DA AFA 1998 – 2010
38. Em relação à obra Lavoura arcaica, NÃO é b) ateísmo.
correta esta assertiva: c) erotismo.
a) exalta-se a ideologia do movimento dos sem- d) esoterismo.
terra.
b) faz-se uma versão transgressiva da parábola REDAÇÃO .
bíblica do filho pródigo. Leia a primeira frase da alternativa a do
c) o tema pouco convencional de amor incestuoso item 25 desta prova e, explorando o tema nela
está presente na narrativa. contido, escreva uma dissertação, em prosa, a
d) a narrativa em prosa poética mescla o lírico e o caneta, de 25 a 30 linhas. Leve em
trágico num drama de contestação familiar. consideração que o autor, ao produzi-la,
referia-se ao Brasil.
39. Observe o excerto:
“Pelo quarto parecia-lhe estarem a se cruzar os
elétricos, a estremecerem-lhe a imagem refletida. AFA 1999/2000 – Português
Estava a se pentear vagarosamente diante da
penteadeira de três espelhos, os braços brancos e Para responder às questões de 1 a 5, considere o
fortes arrepiavam-se à frescurazita da tarde. Os entendimento deste fragmento da obra indicada:
olhos não se abandonavam, os espelhos vibravam “Algum tempo hesitei se devia abrir estas
ora escuros, ora luminosos. Cá fora, duma janela memórias pelo princípio ou pelo fim, isto é, se
mais alta, caiu à rua uma coisa pesada e fofa. Se os poria em primeiro lugar o meu nascimento ou a
miúdos e o marido estivessem à casa, já lhe viria à minha morte. Suposto o uso vulgar seja começar
idéia que seria descuido deles. Os olhos não se pelo nascimento, duas coisas me levaram a adotar
despregavam da imagem, o pente trabalhava diferente método: a primeira é que eu não sou
meditativo, o roupão aberto deixava aparecerem propriamente um autor defunto, mas um defunto
nos espelhos os seios entrecortados de várias autor, para quem a campa foi outro berço; a
raparigas.” segunda é que o escrito ficaria assim mais galante
e mais novo. Moisés, que também contou a sua
Em relação à linguagem do conto “Devaneio e morte, não a pôs no intróito, mas no cabo: a
embriaguês duma rapariga” (Laços de família), do diferença radical entre este livro e o Pentateuco.”
qual esse excerto é uma amostra, é correto dizer (obs.: os grifos são nossos)
que o narrador( , ) (Machado de Assis, Memórias póstumas de Brás
a) se acumplicia com a personagem, utilizando
Cubas)
inclusive linguagem que se compatibiliza com a
nacionalidade dela. 1. Uma palavra pode resumir a atitude de Brás
b) para ressaltar o despojamento do ambiente e o
Cubas ao fazer analogia entre a gênese de seu livro
vazio psicológico da personagem, emprega uma de memórias e a do Pentateuco:
linguagem intencionalmente desprovida de a) imitação.
elementos subjetivos. b) presunção.
c) na ânsia de traduzir o torvelinho de sensações
c) emulação.
que acometem a mente embriagada, instrumenta-se d) superação.
de uma linguagem prenhe de imagens surrealistas.
d) por meio de um estilo repassado de ironia,
2. Segundo Brás Cubas, “a diferença radical” entre
formalizado em linguagem que revaloriza a força o Pentateuco e Memórias póstumas de Brás Cubas
dos arcaísmos, busca reproduzir o drama de uma está neste fato:
alma dilacerada pela banalidade do cotidiano. a) Moisés contou sua morte na narrativa bíblica,
mas Cubas não o fez no seu livro de memórias.
40. Envolvendo as personagens mãe e filha de b) a morte do patriarca de Israel está no final do
Soroco (“Soroco, sua mãe, sua filha”), Coronel Pentateuco, ao passo que a de Brás Cubas consta
Lula (Fogo morto) e Iô João-de-Barros-Diniz- no início das Memórias póstumas.
-Robertes (“Tarantão, meu patrão...”) há um c) o amante de Virgília pôs sua morte no epílogo
estigma de das Memórias, enquanto o condutor do povo judeu
a) insânia.
a omite na Sagrada Escritura.
82
PROVAS DA AFA 1998 – 2010
d) Moisés relatou sua morte no final da Bíblia, d) oração subordinada adverbial temporal, oração
contrariamente ao criador do famoso emplastro, subordinada substantiva predicativa, oração
que o fez no intróito da sua narrativa. subordinada adjetiva restritiva.

3. Ao fazer irônico jogo de palavras com as 6. Questionando os modelos ideológicos e


expressões “defunto autor” e “autor defunto”, Brás literários importados, Machado de Assis ironiza as
Cubas, o personagem-narrador, teorias filosóficas em voga, na época, fazendo o
a) declara sua disposição de manter-se fiel ao “humanitismo” permear algumas de suas obras da
ideário realista, propondo não deformar os fatos e chamada fase realista.
garantir a verossimilhança da história. Assim, é correto afirmar que
b) propõe-se a ferir no cerne as idealizações a) o grande mentor do “humanitismo” é Quincas
românticas, julgando e ironizando as contradições Borba; esse sistema filosófico aparece teorizado
humanas, do pedestal de narrador onisciente. pela primeira vez em Memórias póstumas de Brás
c) insinua, no início das memórias, seu propósito Cubas e é retomado em Quincas Borba.
de fazer uma narração desinteressada e imparcial, b) Brás Cubas é o introdutor desse sistema, na
mas ao longo delas é traído pelas fraquezas do medida em que sugere vagamente a teoria em
indivíduo comum. Memórias póstumas de Brás Cubas, a qual será
d) explora inusitada possibilidade narrativa, mais bem desenvolvida em D. Casmurro.
manejando com mestria a técnica do c) Rubião é o pai do “humanitismo”; em Quincas
distanciamento, para relatar a vida de um herói (ele Borba, esse personagem ilustra a teoria em foco,
próprio) empenhado na afirmação da própria apresentando a demência de Quincas Borba como
vontade. decorrência de conturbado triângulo moroso.
d) Bentinho é o principal articulador dessa teoria,
4. Observe os elementos grifados destas mas é preciso considerar que quase todos os
expressões do texto: personagens maiores de Machado revelam sutil
I – “... autor defunto ...” postura de crítica à pieguice e ao idealismo dos
II – “... defunto autor ...” valores burgueses.
III – “... duas coisas ...”
IV – “... a primeira é que ...” 7. “Fui aos alforjes, tirei um colete velho, em cujo
V – “... levaram a adotar ...” bolso trazia as cinco moedas de ouro, e durante
Dir-se-á que, no plano morfológico, as palavras esse tempo cogitei se não era excessiva a
destacadas são, respectivamente, gratificação, se não bastavam duas moedas.”
a) adjetivo, substantivo, numeral cardinal, numeral (M.A., Memórias Póstumas de Brás Cubas)
ordinal, preposição. Marque a opção que transpõe para o discurso
b) substantivo, adjetivo, numeral ordinal, numeral direto o trecho que, no excerto acima, aparece em
cardinal, pronome pessoal. discurso indireto.
c) adjetivo, adjetivo, numeral cardinal, numeral a) - A gratificação é excessiva! Não bastariam duas
ordinal, pronome demonstrativo. moedas?
d) substantivo, substantivo, numeral ordinal, b) - Se duas moedas são bastantes, a gratificação é
numeral cardinal, artigo definido. excessiva.
c) - Bastam duas moedas; portanto a gratificação é
5. As orações destacadas no texto classificam-se, excessiva.
respectivamente, como d) - Não é excessiva a gratificação? Não bastam
a) oração principal, oração subordinada adverbial duas moedas?
condicional, oração subordinada substantiva
predicativa. Considere este excerto de Memórias póstumas de
b) oração principal, oração subordinada Brás Cubas para responder à questão 8.
substantiva objetiva direta, oração subordinada “ Talvez espante ao leitor a franqueza com que lhe
substantiva predicativa. exponho e realço a minha mediocridade; advirta
c) oração coordenada assindética, oração que a franqueza é a primeira virtude de um
subordinada substantiva objetiva direta, oração defunto. Na vida, o olhar da opinião, o contraste
subordinada adjetiva restritiva. dos interesses, a luta das cobiças obrigam a gente a
83
PROVAS DA AFA 1998 – 2010
calar os trapos velhos, a disfarçar os rasgões e os embaixo na cidade, e ele deixou a quitanda para ir
remendos, a não estender ao mundo as revelações na venda beber.”
que faz à consciência; e o melhor da obrigação é a) O texto está inatacável quanto à correção
quando, à força de embaçar os outros, embaça-se gramatical.
um homem a si mesmo, porque em tal caso poupa- b) Trata-se de uma das numerosas ousadias
se o vexame, que é uma sensação penosa, e a gramaticais da obra em pauta, próprias do
hipocrisia, que é um vício hediondo. Mas, na informalismo do estilo modernista.
morte, que diferença! que desabafo! que liberdade! c) É um excerto insólito da obra, vazada esta em
Como a gente pode sacudir fora a capa, deitar ao padrão lingüístico culto, e só se justifica pela
fosso as lentejoulas, despregar-se, despintar-se, preocupação do autor em reproduzir o falar
desafeitar-se, confessar lisamente o que foi e o que inculto.
deixou de ser! Porque, em suma, já não há d) O fragmento é ocorrência comum ao longo da
vizinhos, nem amigos, nem inimigos, nem narrativa e se explica pelo propósito do autor de
conhecidos, nem estranhos; não há platéia. O olhar (re)criar uma linguagem que o instrumente na
da opinião, esse olhar agudo e judicial, perde a busca de fundir o mais regional ao mais universal.
virtude, logo que pisamos o território da morte;
não digo que ele se não estenda para cá, e nos não 10. Leia os excertos, atentando para a palavra
examine e julgue; mas a nós é que não se nos dá do ressaca.
exame nem do julgamento. Senhores vivos, não há I – “A ressaca que há cinco dias castiga o Rio de
nada tão incomensurável como o desdém dos Janeiro já dá sinais de melhora.” (O Estado de São
finados.” Paulo, 25.5.99)
II – “Olhos de ressaca? Vá, de ressaca. (...)
8. Dando de barato a viabilidade da tese do Traziam não sei que fluido misterioso e enérgico,
narrador, assinale a opção cuja assertiva uma força que arrastava para dentro...” (Machado
corresponde à afirmação (I, II e III) que a segue, de Assis, D. Casmurro)
considerando que III – “Pois a grande questão é saber se hoje, diante
I – expressa afirmação(ões) compatível(eis) com o do Corinthians, estará o Palmeiras de Paulo Nunes
texto; ou o Palmeiras de Alex, que deve ser também o de
II – contém afirmação(ões) incompatível(eis) com Luís Felipe. Pelo que conheço do treinador, ele não
o texto; vai admitir máscara, salto-alto ou ressaca da
III – registra afirmação(ões) contradizente(s) ao Libertadores.” (O Estado de São Paulo, 20.6.99)
texto. IV – “Não era fácil ao juvenil imperador manter-se
a) O parecer (exterioridade) rege a vida; o ser firme e popular na procela dos partidos e na
(essência) só é encontrável na ressaca das opiniões.” (Latino Coelho)
morte. (I) Sabe-se que muitas vezes, na polissemia, o sentido
b) A atitude dos mortos é governada pela das palavras tende a se afastar da base semântica,
autenticidade, enquanto a dos vivos prima pela tornando difícil estabelecer a relação polissêmica.
dissimulação. (II) Assim, dentre os casos acima, pode-se constatar
c) A oposição entre a visão de mundo dos vivos e a que o menor afastamento de sentido ocorre entre
dos mortos pode ser resumida, respectivamente, a) I e II.
pelo par dicotômico “desafetação/afetação”. (I) b) III e IV.
d) “Vida” assume conotação negativa, vinculada a c) I e IV.
termos como “embaçar”, “hipocrisia”, “cobiça”; d) II e III.
por seu turno, “morte” é nome positivo, ligado a
“liberdade”, “desabafo”, “despintar-se”. (III) 11. Assinale a frase em que a crase se justifica pela
mesma regra daquela assinalada no texto abaixo.
9. O texto abaixo consta de uma das narrativas “De repente, a criada, que estava na outra sala,
indicadas para este concurso. Analise-o quanto à ouvindo rumor de alguma cousa que se quebrava,
correção gramatical e marque a assertiva correta. correu à de visitas, e viu a ama, sozinha, de pé.”
“- É um vadio e um bêbado muito grande. Ainda (M. Assis, Quincas Borba)
hoje deixei ele na quitanda, enquanto eu ia lá a) A decisão do bando foi favorável à libertação de
Zé Bebelo.
84
PROVAS DA AFA 1998 – 2010
b) Das filhas do velho fazendeiro, Riobaldo peta, eu querer saldar: que isso não é falável. As
dirigiu-se à mais tímida. coisas assim a gente mesmo não pega nem abarca.
c) A passos lentos todos se dirigiram à sala Cabem é no brilho da noite. Aragem do sagrado.
principal para assistir ao interrogatório. Absolutas estrelas!”
d) Acoitados no velho casarão, os jagunços
ficaram à espera da arremetida dos hermógenes. 13. A idéia subjacente a esse excerto, assim como
em toda a obra, revive um tema recorrente na arte
12. Leia estas frases, atentando para a colocação literária, desde o Renascimento. Trata-se do mito
do sujeito e do verbo. de
I – Como se enganam os médicos! a) Fausto.
II – Como os médicos se enganam! b) Sísifo.
Dir-se-á, de acordo com o gênio da língua, que c) Narciso.
a) I e II são passivas. d) Prometeu.
b) I e II são reflexivas.
c) I é passiva e II é reflexiva. 14. No primeiro parágrafo do texto, percebe-se
d) I é reflexiva e II é passiva. uma sucessão de estados, sentimentos e sensações
assoberbando a mente da personagem. Sem
Tem-se a seguir um fragmento de Grande sertão: pretender esgotar os elementos da sucessão, pode-
veredas, obra indicada para este concurso. Leia-o se alinhar os seguintes, na ordem:
atentamente e, considerando o conhecimento da a) histeria, susto, angústia, agonia, dúvida,
narrativa como um todo, responda às devaneio.
questões de 13 a 16. b) nervosismo, perplexão, impaciência, dúvida,
“Sapateei, então me assustando de que nem gota provocação, reflexão.
de nada sucedia, e a hora em vão passava. Então, c) ansiedade, nervosismo, desespero, histeria,
ele não queria existir? Existisse. Viesse! Chegasse, espanto, impaciência.
para o desenlace desse passo. Digo direi, de d) impaciência, espanto, incerteza, perplexão,
verdade: eu estava bêbado de meu. Ah, esta vida, angústia, coragem.
às não-vezes, é terrível bonita, horrorosamente,
esta vida é grande. Remordi o ar: 15. Os acontecimentos relatados nesse excerto, ao
- ‘Lúcifer! Lúcifer!...’ - aí eu bramei, lado de outros que surgem na trama, justificam o
desengulindo. pensamento que atormenta o narrador ao longo da
Não. Nada. O que a noite tem é o vozeio dum ser- narrativa:
só - que principia feito grilos e estalinhos, e o a) a dúvida sobre ter-se tornado “pactário”.
sapo-cachorro, tão arranhão. E que termina num b) a certeza a respeito da existência de Deus e do
queixume borbulhado tremido, de passarinho diabo.
ninhante mal-acordado dum totalzinho sono. c) a convicção da necessidade de aliar-se ao mal
- ‘Lúcifer! Satanaz!...’ para combater o mal.
Só outro silêncio. O senhor sabe o que o silêncio d) a consciência de que se pode vender algo
é? É a gente mesmo, demais. mesmo não havendo comprador.
- ‘Ei, Lúcifer! Satanaz, dos meus Infernos!’
Voz minha se estragasse, em mim tudo era cordas 16. Observe as palavras em destaque nos excertos
e cobras. E foi aí. Foi. Ele não existe, e não abaixo.
apareceu nem respondeu - que é um falso I – “O que a noite tem é o vozeio dum ser-só - que
imaginado. Mas eu supri que ele tinha me ouvido. principia feito grilos e estalinhos...”
Me ouviu, a conforme a ciência da noite e o envir II – “Me ouviu, a conforme a ciência da noite e o
de espaços, que medeia. Como que adquirisse envir de espaços, que medeia.”
minhas palavras todas; fechou o arrocho do Trata-se de um verbo regular (I) e de outro
assunto. Ao que eu recebi de volta um adejo, um irregular (II) que, por isso, identificam-se,
gozo de agarro, daí umas tranqüilidades - de respectivamente, com o par:
pancada. Lembrei dum rio que viesse adentro a a) arriar e odiar.
casa de meu pai. Vi as asas, arquei o puxo do b) ansiar e variar.
poder meu, naquele átimo. Aí podia ser mais? A c) odiar e remediar.
85
PROVAS DA AFA 1998 – 2010
d) incendiar e alumiar. Os vocábulos em grifo se justificam na medida em
que o autor tenciona
17. Em Grande sertão: veredas, como em Os a) reproduzir o falar canhestro da personagem.
sertões, observa-se a presença dominadora de três b) reconstruir na escrita a variante lingüística
elementos estruturais que apóiam a composição: a regional.
terra bruta; o homem endurecido pela rude lida no c) mesclar neologismos e barbarismos para
sertão; a luta épica. imprimir mais verosimilhança ao processo
A partir dessa assertiva, pode-se dizer que narrativo.
a) pára aí essa identidade, porque, enquanto em Os d) usar a obra como mais uma bandeira da sua tese
sertões a contínua trança desses três elementos de renovação lingüística, propugnada desde a
afasta qualquer hipótese cientificista, em Grande publicação de Sagarana.
sertão: veredas, o enredo evolui linearmente,
pondo a nu a preocupação rosiana em comprovar a 19. Observe o período:
tese de Taine: o meio, a raça e o momento Diadorim e Riobaldo eram jagunços asseados, mas
histórico determinam o comportamento humano. ninguém os via tomar banho juntos, no riacho.
b) a intertextualidade dessas obras está garantida A respeito de sua estrutura morfossintática, é
não apenas por essa semelhança básica, mas correto dizer que
também pelas intenções deterministas de ambos os a) ambos os pronomes exercem função de sujeito.
autores ? particularmente de Euclides da Cunha ? b) a oração reduzida tem papel de objeto indireto.
os quais, imprimindo um lirismo épico à narrativa, c) se trata de período composto de duas orações.
buscam particularizar o espaço, que lhes vai servir d) a não-flexão do infinitivo constitui violação da
de laboratório para dissecar a proposta de norma culta.
sociologia naturalista.
c) essa semelhança projeta-se além disso, visto que 20. Marque a opção em que o adjunto adnominal
em Euclides tudo assume significado universal, do objeto direto assume valor estilístico de adjunto
consoante com os preceitos deterministas, adverbial.
subtraindo o ensaio à matriz local e sugerindo que a) O catrumano fez reluzir uma faca traiçoeira no
o Sertão é o Mundo, ao passo que em Rosa calor da refrega.
sobressai o enfoque do pitoresco, do regional, b) Na quietude orvalhada da manhã do Alto
mercê da fidedigna abordagem da flora, fauna e Urucuia, a nota dissonante é o coaxar do sapo–
topografia dos confins das Gerais. cururu.
d) não vai além disso essa analogia, pois, enquanto c) Fumando um cigarro lânguido, a mulher-dama
a trama rosiana apresenta um constante do Verde-Alecrim tenta seduzir o jagunço
baralhamento desses três elementos numa atoleimado.
atmosfera de sugestão instigadora da sensibilidade d) Braços desconsolados daquela gente do
e da imaginação, o ensaio euclidiano, engendrado Chapadão erguem-se ao céu quase sem nuvens, em
sob a égide determinista, evolui de forma racional dramático apelo.
e sucessiva, marcado por constatações científicas
reforçadoras da argumentação em prol da tese de 21. Observe este excerto de A rosa do povo, de
Taine. Carlos Drummond de Andrade:
“Penetra surdamente no reino das palavras.
18. Observe estes fragmentos de Grande sertão: Lá estão os poemas que esperam ser escritos.”
veredas, considerando as palavras grifadas. Trata-se de uma evidência - e não é a única no
I – “Pois essezinho, essezim, desde que algum livro - de que o Drummond combativo e
entendimento alumiou nele, feito mostrou o que socializante de A rosa do povo também está
é.” preocupado com o (a)
II – “...Cristo mesmo lá só conseguiu aprofundar a) ludismo.
por um relance a graça de sua sustância alumiável, b) erotismo.
em as trevas de véspera para o Terceiro Dia.” c) metafísica.
III – “Vivi puxando difícil de difícel, peixe vivo no d) metapoesia.
moquém: quem mói no asp’ro, não fantaseia.

86
PROVAS DA AFA 1998 – 2010
22. Considere estes excertos: jogando algumas temporadas no futebol europeu.
I – “Peço passagem aos lentos. Não olho os Estão atuando no Japão numerosos craques
cafés brasileiros de primeira categoria.
que retinem xícaras e anedotas, Vários outros jogadores revelados na atual
como não olho o muro do velho hospital em temporada brasileira estão sendo sondados por
sombra.” empresários do exterior.
(Carlos Drummond de Andrade, A rosa do povo) O Brasil é um respeitável exportador de talentos
II – “Agora as cercas de Bom-Sucesso iam futebolísticos.”
comendo São Bernardo.” Dir-se-á que esse escrito
(Graciliano Ramos, São Bernardo) a) não tem textualidade.
Há em I e II, respectivamente, estas figuras: b) tem textualidade garantida pela coesão.
a) sinestesia e metáfora. c) tem textualidade garantida pela coerência.
b) hipérbole e hipérbole. d) tem textualidade garantida pela coesão e
c) metonímia e prosopopéia. coerência
d) personificação e comparação.
25. Leia atentamente:
23. Observe este poema de Drummond: “(...)
“A morte emendou a gramática Foi bom que te calasses.
Morreram Cacilda Becker Meditavas na sombra das chaves,
Não era uma só. Era tantas. das correntes, das roupas riscadas, das
Professorinha pobre de Pirassununga [cercas de arame,
Cleópatra e Antígone juntavas palavras duras, cimento, bombas,
Maria Stuart [invectivas,
Mary Tyrone (...)
Marta de Albee ó Carlito, meu e nosso amigo, teus sapatos e teu
Margarida Gauthier e Alma Winemiller bigode caminham numa
(...)” estrada de pó e esperança.”
Sobre ele, pode-se afirmar que (C.D.A., A rosa do povo)
a) Drummond faz uma reflexão a respeito da Diremos que, nesse fragmento,
polemizante tese de que o verdadeiro artista a) observam-se os paralelismos sintático e
ludibria a morte, eternizando-se em suas semântico e obtém-se apreciável efeito estilístico.
personagens. b) observam-se os paralelismos sintático, rítmico e
b) o poeta vê, no desaparecimento da atriz, grande semântico, privilegiando a clareza e a lógica do
perda para o vernáculo, já que suas performances raciocínio.
cênicas sempre primaram pelo padrão culto da c) rompem-se os paralelismos sintático e rítmico,
língua, especialmente quando personificou a gerando idéias desconexas, mas coerentes do ponto
“Professorinha pobre de Pirassununga”. de vista literário.
c) o passamento de Cacilda é pretexto para d) rompe-se o paralelismo semântico, em bom
Drummond expressar sua concepção do poético, estilo, ao se criarem imagens pela associação de
por analogia com a arte cênica, como um texto termos inconciliáveis à luz da lógica.
aberto, com multiplicidade de sentidos, um
metapoema, portanto. Observe este excerto de São Bernardo, para
d) o verbo “emendar”, no primeiro verso, pode ser responder às questões 26 e 27.
substituído por “corrigir”: a morte corrige a “D. Glória retificou a espinha, ergueu a voz e
gramática na medida em que, com a construção desfez o ar apoucado:
“morreram Cacilda”, - Não me dou. Nasci na cidade, criei-me na cidade.
o autor expressa a simbiose da atriz com as Saindo daí, sou como peixe fora da água. Tanto
personagens vividas por ela ao longo de sua que estive cavando transferência para um grupo da
carreira. capital. Mas é preciso muito pistolão.
Promessas...”
24. Observe este excerto:
“Os craques Edmundo e Romário estiveram
87
PROVAS DA AFA 1998 – 2010
26. Nesse trecho, os dois-pontos e as reticências a) acompanhamento do processo de construção da
são empregados, respectivamente, para narrativa, pelo leitor.
a) introduzir uma transcrição literária; indicar b) jogo de reflexos entre a história contada e as
ironia. intervenções críticas do verdadeiro autor.
b) iniciar uma explicação ou esclarecimento do c) personagem com função de autor, fato que
interlocutor; sugerir uma brusca interrupção da produz para o leitor o efeito de que se tem um livro
frase. dentro do livro.
c) anunciar a entrada do interlocutor; indicar o d) linguagem narrativa voltada para si mesma,
término da frase que deve ser imaginado pelo procedimento de auto-referência que caracteriza o
leitor. exercício da metalinguagem.
d) isolar do restante do texto os pensamentos ou 30. Atente para a predicação destas frases:
falas da personagem; expressar a interrupção da I – A capela da fazenda S. Bernardo está
fala nervosa e desconexa. reformada.
II – A capela da fazenda S. Bernardo foi
27. As palavras do texto desfez, apoucado e reformada.
pistolão são formadas, respectivamente, por Dir-se-á que se tem predicado
a) derivação regressiva, derivação prefixal, a) verbal em I e II.
composição por justaposição. b) nominal em I e II.
b) derivação prefixal, derivação parassintética, c) verbal em I, nominal em II.
derivação sufixal. d) nominal em I, verbal em II.
c) derivação parassintética, derivação regressiva,
derivação imprópria. 31. Assinale a frase de São Bernardo cuja
d) derivação parassintética, composição por pontuação foi intencionalmente alterada, tornando-
aglutinação, derivação imprópria. se, por isso, incorreta.
a) É um corujão da peste, seu Paulo.
28. Leia: b) O algodoal galgava colinas, descia, tornava a
“São Bernardo faz interagir, como num jogo, dois mostrar-se mais longe, desbotado.
modos de focalizar a narrativa: de um lado o modo c) O capim-gordura tinha virado grama e os bois
próprio, auto-suficiente e pragmático do que pastavam nele eram como brinquedos de
fazendeiro; de outro, o modo de narrar de Paulo celulóide.
Honório-escritor, feito de hesitações, dúvidas e d) Li a folha pela terceira vez, atordoado, detendo-
interrogações, as quais ele partilha com o leitor. A me nas expressões claras e procurando adivinhar a
dupla focalização se explica pelos tempos significação de termos obscuros.
diferentes em que um e outro se encontram.”
(Lúcia H. Vianna, Roteiro de leitura: São 32. Leia este fragmento de São Bernardo,
Bernardo de Graciliano Ramos) atentando para o pronome grifado.
Os termos grifados, as quais, ele e um e outro, “Azevedo Gondim reclamava liberdade, aos gritos.
resgatam, respectivamente, estes elementos do Contenta-se hoje com a renda mofina do jornal e
excerto: deve os cabelos da cabeça. Conforma-se com
a) “interrogações”; “escritor”; “modos de focalizar isso.”
a narrativa”. O pronome em questão faz parte do grupo de
b) “modos de focalizar a narrativa”; “Paulo instrumentos lingüísticos que retomam partes do
Honório”; “escritor” e “leitor”. discurso e promovem a coesão textual. Nesse
c) “dúvidas e interrogações”; “fazendeiro”; trecho, refere-se à(s)
“fazendeiro” e “escritor”. a) liberdade outrora reclamada ao brados.
d) “hesitações, dúvidas e interrogações”; “Paulo b) acomodação do jornalista no presente, advinda
Honório-escritor”; “escritor” e “fazendeiro”. de percalços e frustrações do passado.
c) dívidas e à renda ordinária provinda do jornal,
29. Quanto ao modo de narrar, São Bernardo é com a qual Gondim se contenta atualmente.
exemplo de “construção em abismo”. Com base d) dívidas geradas pelo salário mofino que Gondim
nessa assertiva, pode-se afirmar que não é recebe do jornal, com o qual se conforma nesse
característica desse tipo de construção: momento da vida, apesar de ter protestado no
88
PROVAS DA AFA 1998 – 2010
passado. b) I – Era necessário novas máquinas para o
descaroçador e para a serraria.
33. Aponte a alternativa em que há falha(s) na II – Vão fazer dois anos que Madalena morreu,
acentuação da(s) forma(s) verbal(ais). dois anos difíceis.
a) Os capangas vêem o estouro da boiada pelos c) I – Mais de um jagunço manifestaram-se a favor
buracos da parede de taipa e não contêm um riso de Riobaldo e contra Zé Bebelo.
de satisfação pelo prejuízo do chefe. II – Mais de um oficial, mais de um soldado
b) Marciano não pôde trabalhar porque estava recebeu ferimentos na batalha.
doente, e isso mais me aborreceu porque todos os d) I – A ofensiva da atual primavera na Caxemira
de casa sempre apóiam os achaques desse velho veio com força inusitada, e o resultado foram os
malandro. bombardeios iniciados na quarta-feira passada.
c) D. Glória, atarefada com os preparativos, gritou II – Verificam-se bastantes erros metodológicos no
para as negrinhas: - Enquanto eu pélo o leitão para trabalho.
o banquete, é bom que vocês ágüem os gerânios da
varanda. 37. Assinale a alternativa que completa
d) Com os olhos brilhando de maldosa satisfação, corretamente as lacunas das frases:
o patriarca redargüiu: - Se ganharmos a eleição, I – Bentinho é um tipo de narrador ______ isenção
esse maldito vizinho pára de por minhocas na de ânimo não se pode confiar.
cabeça dos nossos empregados. II – Paulo Honório, ______ Casimiro Lopes
obedece, atribui-lhe mais uma perigosa missão.
34. Marque a opção em que está correta a flexão III – Carlos Drummond de Andrade é o escritor
verbal. ______ poemas de caráter social a crítica sempre
a) Diante do corpo sem vida de Diadorim, se ocupará.
Tatarana não cria no que seus olhos viam: era uma IV – Virgília, ______ Brás Cubas amava,
moça! comprovou que as mulheres têm uma certa queda
b) Quando você rever seus conceitos de criação pelos tolos.
literária, aceitará melhor a obra de Guimarães a) cuja / a que / de cujos os / que
Rosa. b) da qual / a que / nos quais / a que
c) Brás Cubas interviu prontamente, e o escravo se c) em cuja a / ao qual / cujos / à qual
viu livre do açoite que lhe impunha o outro negro. d) em cuja / a quem / de cujos / a quem
d) Se o Congresso propor alguma emenda ao
projeto presidencial, certamente a matéria não será Observe este excerto e responda à questão 38.
mais votada neste ano. Sociólogo garante não existir segregação racial nos
blocos do carnaval de Salvador, destacando que a
35. Marque a alternativa em que há adjetivo no folia reflete a tolerância racial reinante na
grau comparativo de superioridade sintético. sociedade baiana. Afirma, também, que lá se tem
a) Uns poucos imaginavam a filosofia de Quincas não apenas uma intensa convivência entre as raças,
Borba avançadíssima para a época. mas também mecanismos que permitem colocar
b) A rosa do povo pode ser considerada como a cada um no seu devido lugar.
obra mais socializante de Drummond.
c) O sentimento amoroso de Diadorim talvez fosse 38. A respeito desse escrito, pode se afirmar:
menor que seu desejo de matar Hermógenes. a) está gramaticalmente correto, mas peca pela
d) O negligente Luis Padilha cuidava cada vez incoerência.
menos de suas posses, e mais sinais de abandono b) é coeso e coerente, mas apresenta ambigüidade
apareciam em São Bernardo. gerada pelo mau uso do pronome possessivo.
c) apresenta falhas de gramática e de coesão, mas é
36. Atente para a concordância das frases abaixo, inatacável quanto à clareza e à coerência.
assinalando o par inatacável. d) revela incompatibilidade do ponto de vista da
a) I – Provavelmente haveriam crimes no passado variante lingüística escolhida e falhas de
de Casimiro Lopes. estruturação do discurso indireto.
II – Em criança, Brás Cubas era as alegrias da
casa.
89
PROVAS DA AFA 1998 – 2010
39. Observe as frases: doce em calda, e o cesto estava vazio, que para
I – Naquela noite Tatarana adormeceu com o buscar framboesas.
pensamento em Otacília e teve os sonhos mais Daí, que, indo, no atravessar o bosque, viu
ternos que jamais sonhara. só os lenhadores, que por lá lenhavam; mas o lobo
II – Paulo Honório manteve uma aventura amorosa nenhum, desconhecido nem peludo. Pois os
com Germana, a quem conhecera num velório. lenhadores tinham exterminado o lobo. Então, ela,
Em I e II os termos em destaque exercem, mesma, era quem se dizia: ⎯ “Vou à vovó, com
respectivamente, a função sintática de cesto e pote, e a fita verde no cabelo, o tanto que a
a) objeto direto e objeto indireto. mamãe me mandou”. A aldeia e a casa esperando-
b) objeto indireto e objeto indireto. a acolá, depois daquele moinho, que a gente pensa
c) objeto indireto e objeto direto pleonástico. que vê, e das horas, que a gente não vê que não
d) objeto direto interno e objeto direto são.
preposicionado. E ela mesma resolveu escolher tomar este
caminho de cá, louco e longo, e não o outro,
40. Observe as frases quanto à regência: encurtoso. Saiu, atrás de suas asas ligeiras, sua
I – O tablóide londrino Gossips, com duas edições sombra também vindo-lhe correndo, em pós.
diárias, entende melhor do que ninguém de Divertia-se com ver as avelãs do chão não voarem,
fofocas. com inalcançar essas borboletas nunca em buquê
II – No horário previsto, a cerimônia foi nem em botão, e com ignorar se cada uma em seu
iniciada com a entrada no salão lugar as plebeiínhas flores, princesinhas e
nobre da universidade dos professores incomuns, quando a gente tanto por elas passa.
aposentados. Vinha sobejadamente.
III – Oferece-se oportunidade a moça de boa Demorou, para dar com a avó em casa, que
aparência, inteligente e fluente em francês, de assim lhe respondeu, quando ela, toque, toque,
acompanhar idoso em temporada de convalescença bateu:
nos Alpes. ⎯ “Quem é?”
As frases I, II e III estariam mais bem estruturadas ⎯ “Sou eu...” ⎯ e Fita-Verde descansou a
se houvesse nelas voz. ⎯ “Sou sua linda netinha, com cesto e pote,
a) uso adequado das preposições. com a fita verde no cabelo, que a mamãe me
b) observância da correta pontuação. mandou.”
c) contigüidade entre regente e regido. Vai, a avó, difícil, disse: ⎯ “Puxa o
d) compatibilidade semântica entre subordinante e ferrolho de pau da porta, entra e abre. Deus te
subordinado. abençoe.”
Fita-Verde assim fez, e entrou e olhou.
A avó estava na cama, rebuçada e só.
AFA 2000/2001 – Português Devia, para falar agagado e fraco e rouco, assim,
de ter apanhado um ruim defluxo. Dizendo: ⎯
FITA VERDE NO CABELO “Depõe o pote e o cesto na arca, e vem para perto
de mim, enquanto é tempo.”
(Nova velha estória) Mas agora Fita-Verde se espantava, além
de entristecer-se de ver que perdera em caminho
Havia uma aldeia em algum lugar, nem sua grande fita verde no cabelo atada; e estava
maior nem menor, com velhos e velhas que suada, com enorme fome de almoço. Ela
velhavam, homens e mulheres que esperavam, e perguntou:
meninos e meninas que nasciam e cresciam. Todos ⎯ “Vovozinha, que braços tão magros, os
com juízo, suficientemente, menos uma seus, e que mãos tão trementes!”
meninazinha, a que por enquanto. Aquela, um dia,
saiu de lá, com uma fita verde inventada no cabelo. ⎯ “É porque não vou poder nunca mais te
Sua mãe mandara-a, com um cesto e um abraçar, minha neta...” ⎯ a avó murmurou.
pote, à avó, que a amava, a uma outra e quase ⎯ “Vovozinha, mas que lábios, aí, tão
igualzinha aldeia. Fita-Verde partiu, sobre logo, arroxeados!”
ela a linda, tudo era uma vez. O pote continha um
90
PROVAS DA AFA 1998 – 2010
⎯ “É porque não vou nunca mais poder te b) a quebra da relação do indivíduo com o seu
beijar, minha neta...” ⎯ a avó suspirou. lugar de origem e suas raízes.
⎯ “Vovozinha, e que olhos tão fundos e c) a perda da segurança, o sentimento de
parados, nesse rosto encovado, pálido?” fragilidade ante um mundo novo e incógnito.
⎯ “É porque já não te estou vendo, nunca d) o rompimento com a sociedade na qual, por
mais, minha netinha...” ⎯ a avó ainda gemeu. meio dela e nela, o indivíduo se expõe e vive seus
Fita-Verde mais se assustou, como se fosse dramas coletivos.
ter juízo pela primeira vez.
4. Observe.
Gritou: ⎯ “Vovozinha, eu tenho medo do
“Mas agora Fita-Verde se espantava, além de
Lobo!”
entristecer-se de ver que perdera em caminho sua
Mas a avó não estava mais lá, sendo que
grande fita verde no cabelo atada; e estava suada,
demasiado ausente, a não ser pelo frio, triste e tão
com enorme fome de almoço.”
repentino corpo.
Nesse momento do conto, constata-se
Guimarães Rosa
a) a realidade se impondo à imaginação.
b) a completa tomada de consciência de Fita-
1. Há identidade entre “Fita Verde no cabelo” e o
Verde.
romance Grande sertão: veredas, do mesmo autor,
c) a perda definitiva do juízo da protagonista.
na medida em que (, )
d) o desencantamento provocado pelo trajeto em
a) como a personagem que atravessa o bosque,
tudo diverso à expectativa da menina.
tomando um caminho louco e longo, Riobaldo,
praticamente, se funde com sua “travessia” pelo
5. Ao lermos o texto, vamos percebendo um
sertão.
intrincado jogo de oposições, fundamentais para o
b) assim como Diadorim em seu diálogo final com
levantamento temático do conto. Assinale a
Riobaldo, a avó de Fita-Verde pressente a morte e
alternativa que melhor formula essas oposições.
demonstra temê-la numa última conversa recheada
a) ilusão/desilusão; medo/enfrentamento;
de pavor.
aconchego/desamparo; vida/morte
c) o tema é tratado com invenção libérrima numa
b) juízo/ausência de juízo; bem/mal;
linguagem recheada de diálogos que estilizam o
esperança/desesperança; alegria/ tristeza
falar regional, mas misturada à linguagem culta,
c) presença/ausência; vermelho/verde;
em síntese criadora e original que atinge a mais
próximo/distante; ignorância/conheci-mento
alta poesia.
d) juízo/ausência de juízo; imaginação/ realidade;
d) além da preocupação com os aspectos materiais
vida/morte; conhecido/ desconhecido
do mundo, por meio da seleção lexical indicativa e
descritiva do espaço/ambiente que recupera com
6. Além do diálogo estabelecido com o conto
fidelidade aspectos da paisagem brasileira, há,
“Chapeuzinho Vermelho”, há uma passagem no
também, a preocupação com o universo humano.
texto que alude a um dos episódios, assimilados
pela imagística popular,
2. No texto, as expressões juízo, fita verde e Lobo
que celebrizaram o clássico Dom Quixote.
têm, respectivamente, a seguinte conotação:
Assinale-a.
a) maturidade, insânia e mal.
a) “Saiu, atrás de suas asas ligeiras, sua sombra
b) responsabilidade, inocência e morte.
também vindo-lhe correndo, em pós.”
c) discernimento, fantasia e desconhecido.
b) “E ela mesma resolveu escolher tomar este
d) capacidade de julgar, liberdade e perigo.
caminho de cá, louco e longo, e não o outro,
encurtoso.”
3. Observe.
c) “A aldeia e a casa esperando-a acolá, depois
“Mas a avó não estava mais lá, sendo que
daquele moinho, que a gente pensa que vê, e das
demasiado ausente, a não ser pelo frio, triste e tão
horas, que a gente não vê que não são.”
repentino corpo.”
d) “Divertia-se com ver as avelãs do chão não
A morte da avó representa para Fita-Verde
voarem, com inalcançar essas borboletas nunca em
a) a ruptura com o universo humano, com toda a
buquê nem em botão, e com ignorar se cada uma
sua complexidade.
em seu lugar as plebeiínhas flores...”
91
PROVAS DA AFA 1998 – 2010
d) Em Grande sertão: veredas, o grande chefe,
7. Observando o texto, é correto afirmar que Joca Ramiro, destaca-se como protetor de
apresenta discurso Riobaldo e que lhe serve de guia.
a) direto somente.
b) direto e indireto livre. 11. Leia o período abaixo.
c) indireto livre somente. Fita-Verde resolveu tomar este caminho de cá,
d) indireto e indireto livre. louco e longo, e não o outro, encurtoso.
Fita-Verde encontrou borboletas, avelãs e
8. Assinale a alternativa em que a palavra em plebeiínhas flores.
destaque resulta de derivação imprópria. Assinale a alternativa em que as idéias acima estão
a) “Fita-Verde partiu, sobre logo, ela a linda, tudo organizadas num só período coerente e coeso e
era uma vez.” expressam relação de causa e conseqüência.
b) “Daí, que, indo, no atravessar o bosque, viu só a) Assim que Fita-Verde resolveu tomar este
os lenhadores, que por lá lenhavam...” caminho de cá, louco e longo, e não o outro,
c) “Divertia-se com ver as avelãs do chão não encurtoso, encontrou borboletas, avelãs e
voarem, com inalcançar essas borboletas...” plebeiínhas flores.
d) “E ela mesma resolveu escolher tomar este b) Fita-Verde encontrou borboletas, avelãs e
caminho de cá, louco e longo, e não o outro, plebeiínhas flores, posto que resolvesse tomar este
encurtoso.” caminho de cá, louco e longo, e não o outro,
encurtoso.
9. Está corretamente empregado o verbo da c) Ainda que Fita-Verde resolvesse tomar este
caminho de cá, louco e longo, e não o outro
alternativa encurtoso, encontraria borboletas, avelãs e
a) Se o caçador se dispor a caçar o lobo, a avó da plebeiínhas flores.
menina escapará de suas garras. d) Na medida em que resolveu tomar este caminho
b) A avó de Fita-Verde disse: ⎯ remedio essa de cá, louco e longo, e não o outro, encurtoso, Fita-
situação, mesmo tendo apanhado um ruim defluxo. Verde encontrou borboletas, avelãs e plebeiínhas
c) Fita-Verde responde: ⎯ não compito com o flores
lobo que conhece muito bem o longo e tenebroso
caminho da floresta. 12. Assinale a passagem em que a coerência
d) Os lenhadores não se entendiam com o feroz temporal encontra-se comprometida.
animal e lhe diziam: ⎯ não intermedie você nos a) “Puxa o ferrolho de pau da porta, entra e abre.
avisos que a mãe dá a Fita-Verde. Deus te abençoe.”

10. Leia. b) “Depõe o pote e o cesto na arca, e vem para


perto de mim, enquanto é tempo.”
“A aldeia e a casa esperando-a acolá, depois
daquele moinho, que a gente pensa que vê, e das c) “Saiu atrás de suas asas ligeiras, sua sombra
horas, que a gente não vê que não são.” também vindo-lhe correndo, em pós.”
No excerto acima, a quebra do paralelismo
semântico provoca um efeito estilístico. Assinale a d) “Vou à vovó, com cesto e pote, e a fita verde no
alternativa em que essa mesma ruptura cabelo, o tanto que a mamãe me mandou”.
compromete o estilo da frase.
a) Os livros de Guimarães Rosa são anti- 13. Assinale a alternativa em que há
intelectuais, isto é, sobrepondo a intuição e a incoerência.
inspiração sobre a razão. a) Em todas as gerações, os jovens criaram gírias
b) Nascido em Cordisburgo, em 1908, o grande para que os mais velhos não pudessem entendê-los.
escritor brasileiro, Guimarães Rosa, além de A gíria é também uma maneira de sentir-se parte
mineiro era diplomata. de um grupo, algo muito importante para os
c) Em Guimarães Rosa, a palavra é valorizada não adolescentes.
tanto pelo significado como também pelos seus b) Nos últimos 500 anos, o português usado no
sons e formas. Brasil desenvolveu-se de forma distinta do idioma
92
PROVAS DA AFA 1998 – 2010
falado em Portugal. Isso não quer dizer que os travessão, marcando a fala de um personagem ⎯
brasileiros falem errado. Falam de acordo com Riobaldo ⎯ fala essa que só é interrompida
uma gramática brasileira. quando ele acaba de contar a história.
c) À medida que as pessoas começaram a usar o e- IV – As histórias contadas por Riobaldo
mail em vez de falar pessoalmente ou pegar o desenrolam-se no sertão, o espaço síntese onde as
telefone, os mal-entendidos foram se ações humanas são refletidas. Nele, cada rio, cada
multiplicando. Isso aconteceu porque muita gente vereda, cada árvore ou pássaro, sem deixarem de
que usa a internet não estava habituada a escrever pertencer ao mundo natural, mantêm profunda
antes do surgimento dela. correspondência com a esfera humana. Daí a
d) O governo brasileiro deveria tomar medidas preocupação do autor com uma delimitação
para proteger os idiomas dos índios da Amazônia. geográfica precisa, que o mantém fiel aos nomes
Pois, se não há mais resquícios da sociedade de rios e cidades existentes na região.
indígena, se eles estão numa favela bebendo Com relação ao romance de Guimarães Rosa, estão
cachaça o dia inteiro, seria mais útil ensinar a eles corretas as assertivas
o português, para ajudá-los a conseguir um a) I e IV.
emprego. b) I, II e III.
c) I, III e IV.
14. Assinale a alternativa que apresenta d) II, III e IV.
concordância verbal correta.
a) Falam-se entre 4000 e 6800 idiomas na Terra. 16. “E Maria Mutema, sozinha em pé, torta
Poderão haver menos de 1000 em 100 anos. Em magra de preto, deu um gemido de lágrimas e
300 anos não mais do que 24. exclamação, berro de corpo que faca estraçalha.
b) É possível que se faça implantes de células Pediu perdão! Perdão forte, perdão de fogo, que da
humanas no cérebro de animais para que a dura bondade de Deus baixasse nela, em dores de
comunicação entre estes e os seres humanos urgência, antes de qualquer hora de nossa morte. E
melhore. rompeu fala, por entre prantos, ali mesmo, a fim de
c) No português existe sons anasalados, e o final perdão de todos também, se confessava.”
das palavras não é pronunciado por completo. Nesse episódio de Grande sertão: veredas, Maria
Quem fala espanhol fica completamente perdido Mutema confessa ter
com essas peculiaridades. a) assassinado o marido e provocado a morte do
d) A influência do inglês em nosso idioma está vigário.
cada vez maior, haja vista os programas de b) despejado chumbo derretido no ouvido do
televisão e os milhares de filmes e músicas que vigário, enquanto este dormia.
invadem nossas fronteiras desde o final da II c) matado o marido de desgosto ao confessar seu
Guerra. amor pelo vigário.
d) mantido um relacionamento pecaminoso com o
15. Leia as observações abaixo a respeito de finado vigário, com o qual teve três filhos.
Grande sertão:veredas.
I – A história é narrada, durante três dias, a alguém Texto para as questões 17 e 18.
culto, que toma notas, mas que não aparece
explicitamente no corpo da narrativa. As falas “Mas, na ocasião, me lembrei dum conselho que
desse homem da cidade não são reproduzidas no Zé Bebelo, na Nhanva, um dia me tinha dado. Que
livro. Sabemos de suas intervenções somente por era: que a gente carece de fingir às vezes que raiva
meio das respostas de Riobaldo. tem, mas raiva mesma nunca se deve de tolerar de
II – Como se trata da longa fala de um fazendeiro ter. Porque, quando se curte raiva de alguém, é a
do noroeste de Minas Gerais, que foi jagunço e não mesma coisa que se autorizar que essa própria
teve muito estudo, a linguagem do livro é marcada pessoa passe durante o tempo governando a idéia
por expressões típicas do lugar em que vive o e o sentir da gente; o que isso era falta de
narrador-perso-nagem, por provérbios e exemplos soberania, e farta bobice, e fato é.”
tirados do seu cotidiano rural.
III – Quanto à estruturação do romance, não há Grande sertão: veredas, Guimarães Rosa
divisão em capítulos. O início se dá com um
93
PROVAS DA AFA 1998 – 2010
17. Sobre o excerto são feitas as seguintes II – “...penetrar no universo do grande sertão é
considerações: trilhar as veredas da poesia e, com Riobaldo,
I – A palavra “raiva” aparece três vezes com a propor-se grandes questionamentos.”
mesma função sintática. III – “Após a batalha, os jagunços pararam para
II – A palavra “raiva”, na oração subordinada descansar num curso d’água orlado de buritis.”
adverbial temporal, e a palavra “idéia” são objetos Observando as relações entre as expressões
diretos. grifadas, é correto afirmar que ocorre
III – “de alguém” e “da gente” são, a) homonímia entre I e II e antonímia entre I e III.
respectivamente, complemento nominal e adjunto b) polissemia entre I e II e sinonímia entre I e III.
adnominal. c) paronímia entre I e II e homonímia entre I e III.
Dessas considerações é(são) verdadeira(s) d) homonímia entre I e II e sinonímia entre II e III.
a) I e II.
b) I, II e III. 21. Leia o excerto abaixo extraído de uma
c) somente a II. suposta entrevista com Riobaldo, personagem de
d) somente a III. Grande sertão: veredas.
“Mire e veja o leitor e a leitora: se não houvesse
18. Considerando a norma culta da língua, ao Brasil, não haveria ‘Grande sertão: veredas’, não
substituirmos o verbo “lembrar” por “esquecer” no haveria Riobaldo. Deviam ter pensado que pelo
excerto “me lembrei dum conselho”, não seria menos para isso serviu. E o resto é silêncio. Ou
aceitável o seguinte: melhor, mais uma pergunta senhor Riobaldo. O
a) esqueci um conselho. que é silêncio?
b) esqueci-me um conselho. R ⎯ O senhor sabe o que o silêncio é? É a
c) esqueceu-me um conselho. gente mesmo, demais.”
d) esqueci-me de um conselho. Alberto Pompeu de Toledo, Veja
No trecho acima, predominam as seguintes
19. Assinale a alternativa que apresenta funções da linguagem:
incorreção quanto ao emprego do pronome a) poética e fática.
relativo. b) fática e conativa.
a) Situado no norte de Minas Gerais, mas podendo c) expressiva e poética.
estar em toda parte, o sertão é o reino onde formas d) conativa e metalingüística.
de vida rústicas e uma paisagem selvagem e bela
se espelham e por vezes se transfiguram. Texto para as questões 22 a 24.
b) No texto, a mistura de romance e narrativa oral “Mas eu ainda espero angariar as simpatias da
toma forma de um monólogo na fala de um velho opinião, e o primeiro remédio é fugir a um prólogo
sertanejo, Riobaldo, que narra sua vida de explícito e longo. O melhor prólogo é o que
aventuras a um interlocutor da cidade. contém menos coisas, ou o que as diz de um jeito
c) O sertão é o vasto campo da guerra jagunça, obscuro e truncado. Conseguintemente, evito
mas, ao mesmo tempo também, o espaço da contar o processo extraordinário que empreguei na
travessia solitária de um herói de romance que se composição destas Memórias, trabalhadas cá no
interroga sobre o sentido da existência. outro mundo. Seria curioso, mas nimiamente
d) Ao abrir-se o livro, o ex-jagunço surge como extenso, e aliás desnecessário ao entendimento da
um contador de casos, especulando sobre a obra. A obra em si mesma é tudo: se te agradar,
existência do demônio, que pode estar misturado fino leitor, pago-me da tarefa; se te não agradar,
em tudo e cuja a sombra se intromete no interior de pago-te com um piparote, e adeus.”
sua própria consciência. Memórias póstumas de Brás Cubas, Machado de
Assis
20. Leia.
I – “A parança que foi ⎯ conforme estou vivo 22. Sobre o fragmento em questão, pode-se
lembrado ⎯ numa vereda sem nome nem fama, afirmar que
corguinho deitado demais, de água muito a) explicita o modo de composição da obra em
simplificada.” questão, feita em estilo “obscuro e truncado”, além

94
PROVAS DA AFA 1998 – 2010
de reticencioso, de tal modo ambíguo, que quase (...) disse-lhe que ela era um monstro (...)
prejudica a compreensão do sentido.
b) revela a visão artística do autor para quem a que me deixara descer a tudo...”
literatura, por estar restrita a um público letrado na 4
Europa e composto, principalmente, pela ala Memórias póstumas de Brás Cubas, Machado de
feminina, deveria atender ao gosto desse leitor. Assis
c) expressa, por meio da ironia e desdém, a visão Assinale a alternativa em que a função sintática do
que Machado tinha da sociedade e do próprio leitor termo sublinhado está incorreta.
de seu tempo, cuja frivolidade se espelhava no a) (1) objeto indireto
gosto pelas narrativas esvaziadas de complexidade b) (2) adjunto adnominal
ou de apelos à reflexão. c) (3) sujeito
d) a excessiva preocupação com as aparências e d) (4) sujeito
com “angariar as simpatias da opinião” conduzirá
a narrativa, seus avanços e recuos, e será essa a Texto para as questões 26 e 27.
lente que ditará a melhor conduta e, por
conseguinte, o destino de Brás Cubas. “E dizendo isso abraçou-me com tal
1
23. O termo “nimiamente” pode ser ímpeto, que não pude evitá-lo. Separamo-nos
substituído, sem alterar o sentido do texto, por 2
a) mormente. finalmente, eu a passo largo, com a camisa
b) sobejamente.
c) sequiosamente. amarrotada do abraço; enfadado e triste. Já
d) paulatinamente.
não dominava em mim a parte simpática da
24. Assinale a alternativa que apresenta,
respectivamente, a correta classificação sensação, mas a outra. Quisera ver-lhe a
morfossintática dos termos em destaque.
a) pronome demonstrativo e predicativo; pronome miséria digna, contudo, não pude deixar de
relativo e sujeito; pronome pessoal e objeto direto;
pronome relativo e objeto direto comparar outra vez o homem de agora com o
b) pronome pessoal e predicativo; pronome
relativo e objeto direto; pronome demonstrativo e de outrora, entristecer-me e encarar o abismo
objeto direto; pronome relativo e sujeito
c) pronome pessoal e adjunto adnominal; pronome que separa as esperanças de um tempo da
relativo e predicativo; pronome demonstrativo e 3
sujeito; pronome relativo e sujeito realidade de outro tempo...”
d) pronome demonstrativo e adjunto adnominal;
conjunção e objeto direto; pronome pessoal e 26. Assinale, entre os fechos seguintes, o que
sujeito; pronome relativo e objeto direto sintetiza o capítulo “O abraço”, de Memórias
25. Observe. póstumas de Brás Cubas, vindo a completar com
coerência e adequação o episódio acima
“Marcela ofereceu-me polidamente o envolvendo Brás Cubas e Quincas Borba.
1 a) “⎯ Ora adeus! Vamos jantar, disse comigo.
refresco; minha resposta foi dar com a mão Meto a mão no colete e não acho o relógio. Última
desilusão! O Borba
no copo e na salva; entornou-se-lhe o líquido furtara-mo...”
2 b) “Tirei a carteira, escolhi uma nota de cinco mil-
no regaço, a preta deu um grito, eu bradei- réis, ⎯ a menos limpa, ⎯ e
dei-lha. Ele recebeu-ma com os olhos cintilantes
-lhe que se fosse embora. Ficando a sós, de cobiça. Levantou a nota ao ar, e agitou-a
3 entusiasmado.
⎯ In hoc signo vinces! bradou.”
95
PROVAS DA AFA 1998 – 2010
c) “⎯ Procure-me, disse eu, poderei arranjar-lhe I - Meu pai era homem de imaginação; escapou à
alguma coisa. tanoaria nas asas de um calembour. Era um bom
Um sorriso magnífico lhe abriu os lábios. ⎯ Não é caráter, meu pai, varão digno e leal como poucos.
o primeiro que me promete alguma coisa, replicou, II - Ela tinha agora a beleza da velhice, um ar
e não sei se será o último que não me fará nada.” austero e maternal; estava menos magra do que
d) “Cuidei que o pobre diabo estivesse doido, e ia quando a vi, na vez passada, numa festa de São
afastar-me, quando ele me pegou no pulso, e olhou João, na Tijuca.
alguns instantes para o brilhante que eu trazia no III - Creio que prefere mais a anedota do que a
dedo. Senti-lhe na mão uns estremeções de cobiça, reflexão, como os outros leitores, seus confrades, e
uns pruridos de posse. acho que faz muito bem.
⎯ Magnífico! disse ele.” Dir-se-á que apresentam, respectivamente,
a) cacófato, eco e pleonasmo.
27. Sobre as orações subordinadas sublinhadas, b) solecismo, cacófato e hiato.
é correto afirmar que c) obscuridade, eco e barbarismo.
a) 1, 2 e 3 são adverbiais. d) galicismo, cacófato e solecismo.
b) 2 é adverbial consecutiva.
c) 2 e 3 são adjetivas restritivas. Texto para as questões 31 a 33.
d) 3 é adjetiva restritiva com pronome relativo em
função de objeto direto. Comunhão
Péricles Eugênio da Silva Ramos
Texto para as questões 28 a 29.
“Custou-lhe muito a aceitar a casa; farejara a O homem que pensa é uma dádiva,
intenção, e doía-lhe o ofício; mas afinal cedeu. é como o pão,
Creio que chorava, a princípio, tinha nojo de si é como os rios.
mesma.”
Memórias póstumas de Brás Cubas, Machado de O homem que pensa é franco e generoso,
Assis é pura chuva,
tem o coração voltado para os outros.
28. Transpondo o verbo “custar” para a
primeira pessoa do singular, considerando a norma O homem que pensa é fonte e hóstia,
culta da língua, ter-se-á é musgo e noite,
a) Custei muito aceitar a casa ... é cor de sangue, cor de Sol a pino.
b) Custou-me muito aceitar a casa ...
c) Custei-me muito a aceitar a casa ... o homem que pensa é justo e solidário:
d) Me custou muito a aceitar a casa ... o pensamento é trigo
a partilhar na mesa dos convivas;
29. A forma “farejara” exprime um processo o pensamento não é fruto, é todo o horto das
a) em curso ou prolongado, equivalendo a nogueiras.
tendo/havendo farejado.
b) que ocorreu antes de outro processo e o pensamento é comunhão: bebei do vinho,
corresponde a tinha/havia farejado. que esse é o vinho do Homem que não morre;
c) concluído e localizado num momento ou o pensamento é comunhão
período definido do passado, equivalendo a tem/há e se oferece para que o homem seja mais humano
farejado. e viva mais humanamente:
d) que estava em desenvolvimento quando da
ocorrência de outro, equivalendo a teria/haveria a Lua não é Lua quando não é vista,
farejado. porém é Lua, e Lua mais terrena e mais perfeita
quando fulgura, cheia, em pleno céu,
30. Os trechos a seguir, de Memórias póstumas a dar-se toda no ato de brilhar,
de Brás Cubas, foram, quando necessário, a desfazer-se em luz por sobre todos.
intencionalmente adulterados. Observe-os.

96
PROVAS DA AFA 1998 – 2010
31. Em “esse é o vinho do Homem que não 35. Leia o excerto abaixo tendo como
morre”, a expressão grifada é exemplo de referência a leitura de São Bernado.
a) perífrase. “⎯ Por que é que sua sobrinha não procura
b) hipérbole. marido?
c) eufemismo. (D. Glória) Melindrou-se:
d) paronomásia. ⎯ Minha sobrinha não é feijão bichado para se
andar oferecendo.
32. Analisando a oração “que pensa” no texto, ⎯ Nem eu digo isso, minha senhora. Deus me
observamos que é adjetiva livre. É um conselho de amigo. Garantir o
a) restritiva, pois a capacidade de pensar é tida futuro...”
como algo inerente a todos os homens. Sobre esse excerto são feitas as seguintes
b) restritiva, pois a palavra homem nesse caso tem afirmações:
seu sentido individualizado, delimitado. I - Os travessões marcam a mudança de
c) explicativa, pois o texto refere-se apenas interlocutor no diálogo, e o ponto de interrogação
àqueles homens que pensam, e não a todos os expressa o tom irônico e provocador típico de
homens. Paulo Honório.
d) explicativa, pois apenas explicita uma idéia que II - Os dois-pontos anunciam a entrada do
já sabemos estar contida no conceito de homem. interlocutor, e a vírgula isola o aposto.
III - As reticências expressam a interrupção da fala
33. Leia. nervosa e desconexa do interlocutor.
“O homem que pensa é fonte e hóstia” IV - A vírgula após “isso” isola o vocativo, e os
“e se oferece para que o homem seja mais travessões podem ser substituídos por aspas.
humano.” Está(ão) correta(s) a(s) afirmativa(s)
Analisando os termos em destaque no texto, a) I, II e III.
assinale a alternativa que aponta, b) I, III e IV.
respectivamente, aqueles de função sintática c) II apenas.
análoga à de cada uma das orações acima d) IV apenas.
assinaladas.
a) franco/ humano 36. Leia.
b) terrena/ do vinho “...enquanto escrevia a certo sujeito de minas,
c) do vinho/ para os outros recusando um negócio confuso de porcos e gado
d) dos convivas/ em pleno céu zebu, ouvi um grito de coruja e sobressaltei-me.”
Em relação à imagem do “pio da coruja”, que
34. Quanto ao romance São Bernardo de aparece repetidas vezes em São Bernardo, a única
Graciliano Ramos, é incorreto dizer que afirmação incorreta é que ela
a) um dos aspectos marcantes da obra é o seu a) traduz o abandono em que se encontra a
caráter de auto-análise: Paulo Honório procura fazenda.
entender a si mesmo e o que fez de sua vida. b) evoca a lembrança de Madalena.
b) a professora Madalena, esposa de Paulo c) traz à tona sentimentos de culpa e remorso para
Honório, passa a se interessar pela vida de miséria Paulo Honório.
e agruras dos empregados da fazenda e a interceder d) expõe um dado do mundo exterior que afeta o
por eles. mundo subjetivo da personagem.
c) o protagonista, ao fazer um balanço de seu
relacionamento com Madalena, afirma que, se 37. Observando a regência dos verbos, assinale
pudesse recomeçar sua vida com ela, tudo a alternativa em que o emprego do pronome
aconteceria de novo. oblíquo está em desacordo com a norma padrão da
d) a linguagem prolixa e rebuscada do narrador- língua.
personagem revela um homem de muitas ações, a) Azevedo Gondim chamou-lhe patriota.
obcecado pela vida agreste e pela luta em busca de b) Senti-me obrigado a informar-lhe o ocorrido.
riquezas e poder. c) O político pediu-me as fotografias, observou-as
e, ao se retirar, pagou-mas.

97
PROVAS DA AFA 1998 – 2010
d) Não queria presenciar a decadência de São literatura regionalista, que fez realçar a região
Bernardo, assistir-lhe seria demasiado penoso. focalizando o problema social, também apareceu
uma literatura urbana, muito intimista, em que a
38. Quanto aos poemas de A Rosa do Povo, de narração se construiu por registros de atmosferas.
Carlos Drummond de Andrade, é correto afirmar A poesia enveredou, no segundo tempo
que eles traduzem a(o) modernista, para a crítica social e para o
a) luta e a participação consciente do poeta nos entendimento das relações conturbadas do homem
acontecimentos de seu tempo. com o universo.”
b) posição política e social do poeta, por isso são
de pouco lirismo existencial. Samira Youssef Campedelli
c) autoconfiança do poeta e nos surpreendem por
rejeitarem o ceticismo e a ironia. Tomando por base a leitura do texto, pode-se
d) conflito angustiante da consciência, devido ao afirmar, sobre esse período de nossa literatura, que
sentimento de culpa que persegue o poeta, por não a) entre os autores “muito intimistas” não pode
participar das lutas sociais de seu tempo. faltar o nome de Raquel de Queiroz com o
romance Caminho de pedras, em que o enfoque
39. Leia psicológico sobrepõe-se ao social.
O grande amor b) Grande sertão: veredas e Os sertões estão entre
Tom Jobim e Vinícius de Moraes as obras desse decênio que fazem realçar uma dada
Haja o que houver “região focalizando o problema social”.
Há sempre um homem para uma mulher c) o modo típico de um escritor regionalista, dessa
E há de sempre haver “época de agonia”, conceber a personagem pode
Para esquecer um falso amor ser exemplificado pela caracterização de Paulo
E uma vontade de morrer Honório.
Seja como for d) o maior expoente dessa poesia que envereda
Há de vencer o grande amor para o “entendimento das relações conturbadas do
Que há de ser no coração homem com o universo” e para a “crítica social” é
Como um perdão para quem chorou. João Cabral de Melo Neto.

Sobre o texto acima, é correto afirmar que


a) possui interdependência entre elementos AFA 2001/2002 – Português
argumentativos e descritivos, os quais são
transformados em poesia. Leia atentamente os textos I e II; a seguir,
b) narra, poeticamente, a história de um responda às questões de no 01 a 07.
personagem que conseguiu esquecer um falso TEXTO I
amor quando encontrou um grande amor. Quem são nossos ídolos?
c) apresenta um narrador que expõe seu ponto de
vista sobre o relacionamento amoroso, usando o Eu estava na França nos idos dos anos 80. Ligando
procedimento de auto-referência. a televisão, ouvi por acaso uma entrevista com um
d) expressa a idéia, por meio de elementos jovem piloto de fórmula 1. Foi-lhe perguntado em
discursivos, arranjados numa linguagem poética- quem se inspirava como piloto iniciante. A
argumentativa, de que o verdadeiro amor sempre resposta foi pronta: Ayrton Senna. O curioso é que
vence. nessa época Senna não havia ganho uma só corrida
importante. Mas bastou ver o piloto brasileiro se
preparando para uma corrida: era o primeiro a
40. “A literatura que se produziu nos anos 30 e chegar no treino, o único a sempre fazer a pista a
nos anos 40 basicamente gravitou em torno da pé, o que mais trocava idéias com os mecânicos e
difícil realidade gerada pela ditadura que se o último a ir embora. Em outras palavras, sua
instalou no Brasil a partir de outubro de 1930, com dedicação, tenacidade, atenção aos detalhes eram
a ascensão de Getúlio Vargas ao poder. Cada autor tão descomunais que, aliadas a seu talento, teriam
passou a refletir essa época de agonia à sua de levar ao sucesso.
maneira. Assim, por exemplo, ao lado de uma
98
PROVAS DA AFA 1998 – 2010
Por que tal comentário teria hoje alguma fazê-lo. O cê-dê-efe é diminuído, menosprezado, é
importância? um pobre-diabo que só obtém bons resultados
Cada época tem seus ídolos, pois eles são a porque se mata de tanto estudar. A vitória
tradução de anseios, esperanças, sonhos e comemorada é a que deriva da improvisação, do
identidade cultural daquele momento. Mas, ao golpe de mestre. E, nos casos mais tristes, até
mesmo tempo, reforçam e ajudam a materializar competência na cola é motivo de orgulho.
esses modelos de pensar e agir. Parte do sucesso da educação japonesa e dos
Já faz muito tempo, Heleno de Freitas foi um Tigres Asiáticos provém da crença de que todos
grande ídolo do futebol. Segundo consta, jactava- podem obter bons resultados por via do esforço e
se de tomar uma cachacinha antes do jogo, para da dedicação. Pelo ideário desses países, pobres e
aumentar a criatividade. Entrava em campo ricos podem ter sucesso, é só dar duro.
exibindo seu bigodinho e, após o gol, puxava o O êxito em nossa educação passa por uma
pente e corrigia o penteado. O ídolo era a evolução semelhante à que aconteceu nos
genialidade pura do futebol-arte. desportos ⎯ da emoção para a razão. É preciso
Mais tarde, Garrincha era a expressão do povo, que o sucesso escolar passe a ser visto como
com sua alegria e ingenuidade. Era o jogador cujo resultado da disciplina, do paroxismo de
estilo brotava naturalmente. Era a espontaneidade, dedicação, da premeditação e do método na
como pessoa e como jogo, e era facilmente amado consecução de objetivos.
pelos brasileiros, pois materializava as virtudes da A valorização da genialidade em estado puro é o
criação genial. atraso, nos desportos e na educação. O modelo
Para o jogador “cavador”, cabia não mais do que para nossos estudantes deverá ser Ayrton Senna, o
um prêmio de consolação. Até que veio Pelé. supremo cê-dê-efe de nosso esporte. Se em seu
Genial, sim. Mas disciplinado, dedicado e modelo se inspirarem, vejo bons augúrios para
totalmente comprometido a usar todas as energias nossa educação.
para levar a cabo sua tarefa. E de atleta completo e
brilhante passou a ser um cidadão exemplar. (Cláudio de Moura e Castro ⎯ Ponto de Vista.
É bem adiante que vem Ayrton Senna. Tinha Revista Veja, 6 de junho, 2001. Edição 1703)
talento, sem dúvida. Mas tinha mais do que isso.
Tinha a obsessão da disciplina, do detalhe e da Glossário
dedicação total e completa. Era o talento a serviço
do método e da premeditação, que são muito mais apologia: louvor obsessão: idéia fixa
críticos nesse desporto. augúrio: prognóstico, presságio paradoxal:
Há mais do que uma coincidência nessa evolução. idéia ou atitude
Nossa escolha de ídolos evoluiu porque evoluímos. contrária
Nossos ídolos do passado refletiam nossa desdenhar: desprezar à opinião
imaturidade. Era a época de Macunaíma. Era a comum
apologia da genialidade pura. Só talento, pois descomunal: colossal, extraordinário paroxismo:
esforço é careta. Admirávamos quem era talentoso auge, apogeu
por graça de Deus e desdenhávamos o sucesso jactar-se: vangloriar-se premeditar:
originado do esforço. Amadurecemos. Cresceu o planejar
peso da razão nos ídolos. A emoção ingênua tenacidade:
recuou. Hoje criamos espaço para os ídolos cujo persistência, constância
êxito é, em grande medida, resultado da dedicação
e da disciplina ⎯ como Pelé e Senna. TEXTO II
Mas há o outro lado da equação, vital para a nossa
juventude. Necessitamos de modelos que mostrem No fundo do mato-virgem nasceu Macunaíma,
o caminho do sucesso por via do esforço e da herói de nossa gente. Era preto retinto e filho do
dedicação. Tais ídolos trazem um ideário mais medo da noite. Houve um momento em que o
disciplinado e produtivo. silêncio foi tão grande escutando o murmurejo do
Nossa educação ainda valoriza o aluno genial, que Uraricoera, que a índia tapanhumas pariu uma
não estuda ⎯ ou que, paradoxalmente, se sente na criança feia. Essa criança é que chamaram de
obrigação de estudar escondido e jactar-se de não Macunaíma.
99
PROVAS DA AFA 1998 – 2010
Já na meninice fez coisas de sarapantar. De sarapantar: o mesmo que espantar.
primeiro passou mais de seis anos não falando. Si jirau de paxiúba: estrado de varas (jirau) feito
o incitavam a falar exclamava: com fibras de palmeira (paxiúba).
⎯ Ai! que preguiça!... guaiamuns (ou guaimuns): espécie de
e não dizia mais nada. Ficava no canto da maloca, caranguejo.
trepado no jirau de paxiúba, espiando o trabalho cunhatã: moça, adolescente.
dos outros e principalmente os dois manos que macuru: na Amazônia, balanço feito
tinha, Maanape já velhinho e Jiguê na força de de pano e cipó, usado como berço.
homem. O divertimento dele era decepar a cabeça
de saúva. Vivia deitado mas si punha os olhos em 01 - Assinale a afirmativa INCORRETA a
dinheiro, Macunaíma dandava pra ganhar vintém. respeito do TEXTO I.
E também espertava quando a família ia tomar a) As salas de aula são, ainda, um espaço
banho no rio, todos juntos e nus. Passava o tempo retrógrado, onde se valoriza o gênio “iluminado” e
do banho dando mergulho, e as mulheres soltavam descompromissado.
gritos gozados por causa dos guaiamuns diz-que b) É preciso implementar na educação uma postura
habitando a água-doce por lá. No mucambo si mais racional, em que o esforço tenha o mesmo
alguma cunhatã se aproximava dele pra fazer valor que a genialidade.
festinha, Macunaíma punha a mão nas graças dela, c) Para parecer brilhante, muitas vezes o aluno
cunhatã se afastava. Nos machos guspia na cara. esconde seu esforço, camufla sua dedicação,
Porém respeitava os velhos e freqüentava com temendo rótulos.
aplicação a murua a poracê o torê o bacorocô a d) Haverá evolução a partir do momento em que, a
cucuicogue, todas essas danças religiosas da tribo. exemplo de Ayrton Senna, os jovens passarem a
Quando era pra dormir trepava no macuru valorizar a genialidade em estado puro.
pequeninho sempre se esquecendo de mijar. Como
a rede da mãe estava por debaixo do berço, o herói 02 - Considerando, ainda, o TEXTO I, assinale
mijava quente na velha, espantando os mosquitos a alternativa INCORRETA.
bem. Então adormecia sonhando palavras feias, a) O brasileiro, hoje, está mudando os critérios de
imoralidades estrambólicas e dava patadas no ar. escolha de um líder.
Nas conversas das mulheres no pino do dia o b) Os ídolos refletem, mas não reforçam a
assunto eram sempre as peraltagens do herói. As identidade cultural de um povo.
mulheres se riam muito simpatizadas, falando que c) A escolha de seus líderes retrata a maturidade e
“espinho que pinica, de pequeno já traz ponta”, e a evolução de um povo.
numa pajelança Rei Nagô fez um discurso e avisou d) Nas escolas ainda viceja a velha mentalidade do
que o herói era inteligente. gênio indolente.
(...)
03 - Assinale o trecho que caracteriza
(ANDRADE, Mário de. Macunaíma: O herói sem MELHOR a sagacidade de Macunaíma.
nenhum caráter. São Paulo, Ed. Círculo do Livro, a) “(...) o herói mijava quente na velha (...)”
1991. Cap. I) b) “(...) sonhando palavras-feias, imoralidades
estrambólicas.”
Glossário c) “Já na meninice fez coisas de sarapantar.”
d) “Porém respeitava os velhos e freqüentava com
Macunaíma: figura lendária da mitologia aplicação a murua (...)”
indígena, recolhida por Mário de Andrade no livro
Vom Roraima zum Orinoco, de Theodor Koch- 04 - Ayrton Senna se diferencia do herói
Grünberg, segundo o qual o nome Macunaíma é Macunaíma ESPECIALMENTE pela
composto de macku (= mau) e o sufixo a) inteligência em estado puro, inalterado.
aumentativo –ima (= grande). b) valorização da cultura, do saber.
tapanhumas: nome de origem tupi c) premeditação do esforço e da dedicação.
designativo dos negros filhos da África que d) prevalência da razão sobre a emoção.
moravam no Brasil; tribo lendária de índios
brasileiros, com características de negros.
100
PROVAS DA AFA 1998 – 2010
05 - Assinale a alternativa cujo par de palavras d) Só, é e dê devem ser acentuados por serem
NÃO caracteriza corretamente Macunaíma. monossílabos tônicos terminados em o e e.
a) Esperteza – cólera.
b) Preguiça – senso de humor. 10 - Leia atentamente este excerto de Álvares
c) Irreverência – sensualidade. de Azevedo.
d) Maldade – inteligência. “Que ruínas! que amor petrificado!
Tão antediluviano e gigantesco!
06 - Segundo o autor do TEXTO I, Macunaíma Ora, façam idéia que ternuras
a) é responsável, como reflexo de uma Terá essa lagarta posta ao fresco!”
mentalidade, pela derrocada do país. Assinale a alternativa correta.
b) ainda exerce fascínio em boa parte da sociedade a) No vocábulo antediluviano, verifica-se a
brasileira, notadamente nos mais idosos. presença do prefixo latino ante, que expressa
c) simboliza o ideário de um Brasil ingênuo, que precedência.
acreditava em soluções mágicas, miraculosas. b) No vocábulo gigantesco, verifica-se a presença
d) representa a postura da juventude atual que de um prefixo e sufixo italiano (esco).
desdenha a dedicação, considerando-a vergonhosa. c) No vocábulo lagarta, verifica-se a presença de
uma consoante de ligação.
07 - Assinale as afirmativas abaixo com V d) No vocábulo façam, verifica-se que a consoante
(verdadeiro) ou F (falso) e, em seguida, marque a m é uma desinência de gênero e número.
alternativa correspondente.
( ) Garrincha e Heleno de Freitas são 11 - Assinale a alternativa em que TODAS as
exemplos da genialidade em estado bruto, sem palavras formaram-se pelo mesmo e respectivo
lapidação. processo dos vocábulos bigodinho, totalmente,
( ) A atitude séria e dedicada de Pelé no espontaneidade e consolação.
futebol refletiu-se em sua postura de cidadão. a) Esforço, improvisação, cê-dê-efe, vitória.
( ) Senna reúne o que há de mais refinado em b) Ideário, criatividade, japonesa, dedicação.
um gênio: talento e dedicação extremos. c) Televisão, evolução, descomunais, futebol-arte.
( ) O sucesso depende mais da genialidade, é d) Pobre-diabo, genialidade, cavador, imaturidade.
difícil obtê-lo por meio do esforço e da tenacidade.
a) V – V – V – F 12 - Assinale a alternativa INCORRETA
b) F – V – F – V quanto à norma culta.
c) V – F – V – F a) “O certo é que ambos os dois monges
d) F – F – V – F caminhavam juntos.”
b) Estive com Sua Excelência em sua casa.
08 - A ortografia NÃO contraria a norma culta c) Trouxemos um livro para si.
em: d) “Mas, senhores, se é isso o que eles vêem (os
a) A sessão de terras aos semterra demorará muito. maus políticos), será isto, realmente, o que nós
b) É para assinar, não ponha rúbrica. somos? O Brasil não é isso. É isto. O Brasil,
c) Não reclame, você é previlegiado. senhores, sois vós.”
d) Minha estada em São Paulo se prolongará por
três dias. 13 - Assinale a alternativa que completa a
lacuna da frase abaixo:
09 - Todas as palavras estão corretamente “Chama belas às belas e feias às feias, e não te
justificadas quanto à acentuação, EXCETO na esqueças de contar anedotas que _______ as belas.
alternativa a) desfeiam b) desfeiem
a) Época, ídolo e Macunaíma são vocábulos c) desfeam d) desfêem
paroxítonos, portanto todos devem ser acentuados.
b) Pelé, Jiguê e bacorocô são vocábulos oxítonos 14 - Leia atentamente o informe publicitário da
terminados, respectivamente, em e e o, por isso revista Exame (Ed. 743 – 27/6/2001):
devem ser acentuados. “Então, quer dizer que a caldeira elétrica de sua
c) Água, vitória e ideário são vocábulos fábrica está sob ameaça? Que seus secadores,
paraxítonos terminados em ditongo crescente. máquinas de torrar café, fornos, queimadores,
101
PROVAS DA AFA 1998 – 2010
esterilizadores, reatores, bomba de calor, ar- a) F–F–F–V–V
condicionado, aquecedor de ambiente, luminárias b) V–V–V–F–F
ou dessalinizador de água podem parar de c) V–V–F–V–F
trabalhar por falta de energia? Ah! Você é dono de d) F–F–V–F–V
uma padaria e vai ter menos pãezinhos para
vender? Em qualquer situação, em empresas de 16 - Leia atentamente esta tira do Hagar:
qualquer porte, falta de energia significa prejuízo. (I)
E você precisa de uma solução já, agora, para
ontem.”
Com relação a ele, está correta APENAS a
alternativa
a) Os vocábulos: estorvo, corno e destroço são
flexionados no plural da mesma forma que fornos.
b) Os vocábulos: ancião, pagão e tabelião são (II) (III)
flexionados no plural da mesma forma que
pãezinhos.
c) Os vocábulos: feijão, capelão e cortesão são
flexionados no plural da mesma forma que
solução.
d) Os vocábulos: alferes, pires e ourives estão
flexionados no plural tal qual a palavra reatores.
(IV) (V)
15 - Examinando a linguagem utilizada por
Mário de Andrade em Macunaíma: o herói sem
nenhum caráter, pode-se observar que ele comete
várias infrações (propositais) à norma culta,
inclusive no que se refere à pontuação, pois se
preocupava em estabelecer, através de sua obra,
uma língua brasileira, que estivesse mais próxima
do falar do povo. (VI) (VII)
Analise as seguintes afirmativas a respeito do 2o e
do 4o parágrafos do TEXTO II, marcando (V)
para verdadeira e (F) para falsa.
( ) A expressão “Já na meninice” (2o
parágrafo) deve ser separada por vírgula, pois se
trata de um adjunto adverbial deslocado.
( ) O trecho “Si o incitavam a falar
exclamava” (2o parágrafo) apresenta erro de É correto afirmar que há ocorrência de apostos nos
grafia e de pontuação. quadrinhos
( ) No período “Vivia deitado mas si punha a) I, II, III, IV, V e VI apenas.
os olhos em dinheiro, Macunaíma dandava pra b) II, III, IV, V, e VI somente.
ganhar vintém.”, (4o parágrafo) há incorreção c) II, IV, V, VI, e VII apenas.
apenas no que se refere à ortografia. d) I, II, III, IV, V, VI e VII.
( ) Os termos a murua, a poracê, o toré, o
bacorocô e a cucuicogue (4o parágrafo) exercem a 17 - Da análise atenta do seguinte período: “No
mesma função sintática; portanto, devem ser fundo do mato-virgem nasceu Macunaíma,
separados por vírgula. herói de nossa gente.”, só NÃO se pode concluir
( ) No 4o parágrafo, a conjunção “e” está que a/o
corretamente grafada com letra minúscula, pois, a) expressão herói da nossa gente funciona como
embora inicie um novo parágrafo, está aposto de Macunaíma.
coordenando orações. b) termo No fundo do mato-virgem é um adjunto
Assinale, agora, a seqüência correta. adverbial de lugar.
102
PROVAS DA AFA 1998 – 2010
c) locução do mato-virgem é adjunto adnominal
de fundo.
sujeito do verbo nasceu é determinado e simples.

18 - Leia com atenção este soneto do poeta


barroco Gregório de Matos:

A Cristo Senhor Nosso crucificado estando 1 2 3


o poeta na última hora de sua vida I- No quadrinho 1, o primeiro período possui
três orações e é composto apenas por
1- Meu Deus que estais pendente em um subordinação.
madeiro, II - Ainda no primeiro período, no quadrinho 1,
em cuja lei protesto de viver, está correta a regência do verbo esquecer, pois o
em cuja santa lei hei de morrer mesmo não é pronominal.
animoso, constante, firme e inteiro. III - O segundo período, no quadrinho 1, é
formado por duas orações, que estão coordenadas
5- Nesse lance, por ser o derradeiro, assindeticamente.
pois vejo a minha vida anoitecer IV - No quadrinho 2, o segundo período é
é, meu Jesus, a hora de se ver constituído de uma oração absoluta; é, portanto,
a brandura de um Pai manso Cordeiro. considerado um período simples.
V- A segunda oração do período do quadrinho
9- Mui grande é vosso amor, e meu delito, 3 é uma coordenada aditiva; a última, subordinada
porém pode ter fim todo o pecar, substantiva objetiva direta.
e não o vosso amor, que é infinito. Estão corretas somente as afirmativas
a) I, II, e III.
b) II, IV, e V.
12 - Esta razão me obriga a confiar, c) II, III, e IV.
que por mais que pequei, neste conflito d) I, III, IV e V.
espero em vosso amor de me salvar.
20 - Assinale a alternativa INCORRETA
Analisando atentamente o seu conteúdo, só NÃO considerando a concordância verbal.
se pode afirmar que a) A União das Nações Unidas mandou invadir o
a) no verso 4, os termos animoso, constante, Iraque.
firme e inteiro funcionam como predicativo do b) Vossa Eminência já tivestes o ensejo de
sujeito. examinar a situação da diocese.
b) a oração “pois vejo a minha vida anoitecer” c) As Minas Gerais atraem os consumidores de
(verso 6) expressa uma explicação em relação ao minério.
fato expresso na oração que a antecede. d) Fala-se de festas em que se assiste a filmes
c) no verso 9, há uma relação de beneficentes.
proporcionalidade entre as expressões vosso amor
e meu delito. 21 - Assinale a alternativa correta em relação à
d) a oração “por mais que eu pequei” (verso 13) seguinte mensagem:
exprime uma conseqüência em relação ao fato “Dizem que a primeira e maior invenção foi o
apresentado na oração à qual ela se subordina. fogo.”
a) O período é simples, e a oração absoluta tem
19 - Analise as seguintes assertivas a respeito como sujeito a palavra fogo.
desta tira: b) Está evidenciada a presença de oração
subordinada adjetiva restritiva.
c) Identifica-se a presença de sujeito
indeterminado e oração substantiva.
d) Pode-se vislumbrar claramente um caso de
complemento nominal.
103
PROVAS DA AFA 1998 – 2010
25 - Assinale a alternativa correta quanto às
22 - Leia o seguinte excerto do conto A caçada, normas de regência verbal.
de Lygia Fagundes Telles: a) Devemos preferir mais lutar pela verdade do
“Era uma caçada(1). No primeiro plano, estava o que sucumbir à mentira.
caçador de arco retesado, apontando para uma b) Todo jovem, intimorato por natureza, aspira
touceira espessa(2). Num plano mais profundo, o pilotar um jato militar.
segundo caçador espreitava por entre as árvores do c) Falando com o filhinho, a mãe informou-lhe de
bosque, mas esta era apenas uma vaga silhueta, que estava na hora.
cujo rosto se reduzira a um esmaecido contorno(3). d) O comandante informou ao cadete do valor
Poderoso, absoluto era o primeiro caçador, a barba atribuído à ética militar.
violenta como um bolo de serpentes, os músculos
tensos, à espera de que a caça levantasse para 26 - Da análise atenta dos enunciados desta tira
desferir-lhe a seta(4).” Os bichos:

Analisando sintaticamente os seus enunciados, só (I)


NÃO se pode afirmar que, no período
a) (2), a oração “apontando para uma touceira
espessa” é uma adjetiva explicativa, reduzida de
gerúndio.
b) (3), a oração “cujo rosto se reduzira a um
esmaecido contorno” funciona como aposto de
silhueta. (II) (III)
c) (4), a oração “de que a caça levantasse”
funciona como complemento nominal da locução à
espera.
d) (4), ainda, os termos poderoso e absoluto são
predicativos do sujeito “o primeiro caçador”.

23 - Assinale a alternativa INCORRETA (IV) (V)


quanto à concordância nominal.
a) Os vestidos custaram barato, mas as saias
custaram caro.
b) Apesar de famoso, não são artistas de talento.
c) Eu já lhes pedi, bastantes vezes, que não
fizessem mais isso.
d) Eles caminhavam sós pela noite escura.
(VI) (VII)
24 - Assinale a opção cujo período foi reescrito
de acordo com as normas da língua culta.
a) Não queremos lhes incutir a idéia de que a
improvisação é a melhor saída.
Não queremos incutir aos alunos a idéia de que a
improvisação é a melhor saída.
b) Atualmente, está difícil disciplinar os maus só NÃO é correto afirmar que
alunos. a) por estar se dirigindo a um sapinho-aprendiz, o
Atualmente, custa-nos disciplinar-lhes. sapo-mestre mistura adequadamente a 2a e a 3a
c) É necessário premiar os alunos cê-dê-efes. pessoas gramaticais (respectivamente, tu e você).
É necessário premiar-lhes. b) o sujeito dos verbos lembra e espera
d) Informei aos estudantes a hora do exame. (respectivamente, nos quadrinhos I e III) é
Informei-lhes a hora do exame. desinencial.
c) nos quadrinhos IV e V, ocorre uma figura de
linguagem denominada onomatopéia.
104
PROVAS DA AFA 1998 – 2010
d) no último quadrinho, o sujeito do verbo d) Há a predominância da função fática no texto,
preocupe também é oculto (você). pois verifica-se o estabelecimento e a permanência
do contato entre o emissor e o receptor.
27- Analise os exemplos abaixo quanto ao
emprego da regência nominal: 30 - Relacione a 2a coluna à 1a e, a seguir,
I - Foi cruel e implacável com sua vítima. assinale a alternativa correta.
II - Manteve-se inexorável aos pedidos da filha. 1a coluna
III - Coração tão excessivo na ternura como 1 - Função referencial
desmedido no ódio. 2 - Função expressiva
IV - Severo para as pequenas culpas, brandíssimo 3 - Função conativa
para os grandes atentados. 4 - Função metalingüística
Estão corretas apenas as alternativas 2a coluna
a) I e II. b) II e III. ( ) “Só levo uma saudade ⎯ é dessas
c) I, II e III. d) I, II, III e IV. sombras”
Que eu sentia velar nas noites minhas...
28 - Leia atentamente esta estrofe de Manuel De ti, ó minha mãe! pobre coitada
Bandeira: Que por minha tristeza te definhas!”
E enquanto penso em ti, no meu sonho erradio, ( ) “O poeta é um fingidor.
Sentindo a dor atroz desta ânsia incontentada, Finge tão completamente
– Fora, aos beijos glaciais e cruéis da geada, Que chega a fingir que é dor
Tremem as flores e foge, ondeando o rio, A dor que deveras sente.”
E as estrelas tremem no ar frio, no céu frio... ( ) “O coração é composto de três tipos
Assinale a alternativa correta. principais de músculo cardíaco: músculo atrial,
a) A mensagem do texto está centrada nela músculo ventricular e fibras musculares
mesma, pois codificador e decodificador se condutoras excitatórias especializadas.
intercomunicam denotativamente. ( )
b) Está caracterizado um teste da eficiência do
canal, considerando que a função fática da
linguagem assim o demonstra.
c) A primeira estrofe suscita, através dos verbos e
pronomes, a expressividade, predominando a
função emotiva no texto.
d) Apontando para o sentido real das coisas e dos
seres, centrada no “eu”, sonho, dor, ânsia, beijos e
flores denotam função referencial.

29 - Leia atentamente o TEXTO II e, a seguir, a) 1 – 3 – 2 – 4 b) 2 – 4 – 1 – 3


assinale a alternativa correta quanto à função de c) 3 – 2 – 4 – 1 d) 4 – 1 – 3 – 2
linguagem.
a) A função poética é predominante no texto, pois 31 - Nos versos:
utilizam-se em sua elaboração recursos de forma e I - “Deus!, ó Deus! onde estás que não
de conteúdo que chamam nossa atenção, causando- respondes?”
nos surpresa, “estranhamento” e prazer estético. II - “Uma ilusão gemia em cada canto, / chorava
b) A função referencial é predominante no texto, em cada canto uma saudade.”
pois procura transmitir informações precisas sobre III - “Amor é fogo que arde sem se ver”,
um determinado assunto, a mensagem é objetiva e têm-se, respectivamente:
impessoal. a) prosopopéia, apóstrofe, pleonasmo.
c) Há a predominância da função emotiva no b) metáfora, metonímia, oxímoro.
texto, pois o emissor manifesta sua posição pessoal c) apóstrofe, prosopopéia, comparação.
diante do conteúdo transmitido, conferindo-lhe d) apóstrofe, prosopopéia, paradoxo.
certo grau de subjetividade.

105
PROVAS DA AFA 1998 – 2010
32 - Em “O cê-dê-efe é diminuído, Todas as alternativas abaixo apresentam vocábulos
menosprezado, é um pobre-diabo que obtém bons com relação antonímica, EXCETO
resultados porque se mata de estudar” têm-se, a) Venho – vou.
respectivamente, b) Nasceu – morreu.
a) hipérbole, metáfora, metáfora. c) Deslumbrado – fascinado.
b) metonímia, antonomásia, gíria. d) Coitado – feliz.
c) gíria, metáfora, hipérbole.
d) metáfora, hipérbole, metonímia. 36 - Minha colega teve uma súbita dor no peito.
O chefe pediu que eu fosse com ela ao
33 - Em todas as passagens, a palavra destacada cardiologista. Após examiná-la, o médico disse
está corretamente interpretada, EXCETO em: que não se tratava de nada grave. As dores eram
a) “Mais tarde, Garrincha era a expressão do provocadas por gases.
povo,...” – palavra, locução ou frase com que se O emprego da palavra médico é um recurso de
enunciam pensamentos. coesão denominado
b) “Era o talento a serviço do método e da a) hiperonímia. b) sinonímia.
premeditação...” – planejar antecipadamente. c) elipse parcial. d) elipse total.
c) “Pelo ideário desses países, pobres e ricos
podem ter sucesso, é só dar duro.” – planejamento, 37 - Leia os períodos abaixo, observando as
projeto. palavras em negrito.
d) ... “do paroxismo de dedicação, da I - “Para o jogador ‘cavador’, cabia não mais do
premeditação e do método na consecução de que um prêmio de consolação.
objetivos” – falta de seqüência lógica. II - “Cada época tem seus ídolos, pois eles são a
tradução dos anseios, esperanças, sonhos e
34 - Assinale a alternativa correta. identidade cultural daquele momento.”
a) “Nos machos guspia na cara” – a palavra III - O cê-dê-efe é diminuído, menosprezado, é um
destacada não foi empregada de acordo com o pobre-coitado que obtém bons resultados porque
padrão culto. se mata de estudar.”
b) ...“imoralidades estrambólicas...” – o termo Quanto aos fragmentos I, II e III, é
grifado, empregado na forma popular, significa de INCORRETO afirmar que
forma arruinada. a) no período I, a palavra cavador está delimitada
c) ... “espinho que pinica, ...” – o vocábulo por aspas para indicar, além de outros aspectos,
destacado tem como sinônimo a seguinte uma impropriedade vocabular.
expressão: ferir com peça metálica que serve de b) no período II, tem-se uma série sinonímica das
eixo. palavras anseios, esperanças e sonhos.
d) ... “dandava pra ganhar vintém...” – o vocábulo c) no período III, cê-dê-efe e pobre-diabo são
em destaque é uma forma coloquial do verbo vocábulos conotativos que caracterizam o bom
dançar. aluno.
d) há, no período III, emprego de linguagem
35 - Leia atentamente o excerto da “Canção do figurada, entretanto se nota o predomínio da
Expedicionário”: linguagem referencial.
“Venho do além desse monte
Que ainda azula no horizonte, 38 - Leia os poemas abaixo:
Onde o nosso amor nasceu; I- Infância
Do rancho que tinha ao lado O camisolão
Um coqueiro que, coitado, O jarro
De saudade já morreu. O passarinho
Venho do verde mais belo, O oceano
Do mais dourado amarelo, A visita na casa que a gente sentava no
Do azul mais cheio de luz, sofá.
Cheio de estrelas prateadas, (Oswald de
Que se ajoelham deslumbradas, Andrade)
Fazendo o sinal da cruz.” II - Não queiras ter Pátria.
106
PROVAS DA AFA 1998 – 2010
Não dividas a Terra. d) indireto e indireto livre.
Não dividas o Céu.
Não arranques pedaços de mar. 40 - Considere o seguinte texto:
Não queiras ter. A Festa de Santa Ifigênia
Nasce bem alto, Dias antes da festa reuniam-se na igreja centenas
Que as coisas boas serão tuas. de negras – traziam todas a carapinha empoada de
Que alcançarás todos os horizontes. ouro – e cantando lavavam as tábuas do templo,
Que o teu olhar, estando em toda parte. floriam os altares, vestiam as imagens, tapeçavam
Te ponha em tudo, o adro de folhas aromáticas. No dia da festa
Como Deus. famílias negras arranchavam-se nas imediações da
(Cecília Meireles) igreja e os tambores da África estrugiam, vinham
III - Lá vem a lua surgindo, os descantes crioulos e a mulata, airosa e trêfega,
Redonda que nem um tamanco. saía pela areia semeada de rosas, nos passos do
Pedaço de telha é caco. samba; mas, quando os coros sagrados
Caranguejo não tem pescoço. começavam, acudiam todas, as mulheres
Sobre os trechos acima, pode-se afirmar que descobriam as cabeças e o ouro reluzia ao sol
a) I é coeso, mas não é coerente; II é coeso, mas maravilhoso. Ao fim da cerimônia irrompia o
não é coerente; III é coerente, mas não é coeso. canto feminino e as negras, uma a uma, cantando,
b) I é coeso e coerente; II é coeso e coerente; III é baixavam as cabeças na pia e lavavam a carapinha,
coeso e coerente. e o ouro depositava-se no fundo do lavabo santo –
c) I é coerente, mas não é coeso; II é coeso e era a oferenda dos cativos à santa misericordiosa.
coerente; III não é coerente e não é coeso. E fora, à luz viva, os negros batucavam nos
d) I é coeso e coerente; II não é coeso, nem atabaques, saudando com alarido as mulheres que
coerente; III não é coerente, mas é coeso. voltavam gotejantes e louvando o Deus do Céu e a
santa da devoção.
39 - Leia os excertos abaixo: (Coelho Neto)
I- “Não se conformou: devia haver engano. Quanto aos modos de organização do discurso, o
Ele era bruto, sim senhor, via-se perfeitamente que texto acima pode ser classificado como
era bruto, mas a mulher tinha miolo. Com certeza a) narrativo. b) descritivo.
havia um erro no papel do branco. Não se c) dissertativo. d) instrucional.
descobriu o erro, e Fabiano perdeu os estribos.
Passar a vida inteira assim no toco, entregando o
que era dele de mão beijada! Estava direito aquilo? AFA 2002/2003 – Português
Trabalhar como negro e nunca arranjar carta de
alforria!” Leia atentamente os textos I e II; a seguir,
II - “Pobre do papagaio. Viajar com ela, na responda às questões referentes a eles.
gaiola que balançava em cima do baú de folha.
Gaguejava: ⎯ “Meu louro.” Era o que sabia dizer. TEXTO I
Fora isso, aboiava arremedando Fabiano e latia
como Baleia. Coitado. Sinhá Vitória nem queria O PRAZER DE BISBILHOTAR
lembrar-se daquilo. Esquecera a vida antiga, era
como se tivesse nascido depois que chegara à Na TV, internet ou na exibição de fotos
fazenda. A referência aos sapatos abrira-lhe uma reveladoras, especialistas constatam: as pessoas
ferida ⎯ e a viagem reaparecera. As alpercatas gostam de espionar a intimidade alheia.
dela tinham sido gastas nas pedras. Cansada, meio
morta de fome, carregava o filho mais novo, o baú Nunca a realidade fez tanto sucesso. O Brasil dos
e a gaiola do papagaio. Fabiano era ruim.” últimos meses viu uma explosão de audiência e
Nos trechos acima, pode-se encontrar, repercussão dos reality shows, programas que
respectivamente, discurso mostram o cotidiano das pessoas em detalhes
a) indireto e direto. prosaicos. Nas intrigas da Casa dos Artistas, do
b) indireto livre e direto. SBT, nas privações físicas de No limite, da Rede
c) direto e indireto. Globo, ou no embate psicológico de Big Brother
107
PROVAS DA AFA 1998 – 2010
Brasil, também, da Globo, o telespectador parece pensamento, não informam, nem sequer
ter se viciado na observação das pessoas que não representam a realidade, pois quem está
estão ali dramatizando, mas mostrando o que participando sabe muito bem que há câmeras e
realmente são. milhões de telespectadores observando."
Esse fenômeno levantou uma discussão entre (...)
pesquisadores do comportamento humano: até que Nem todos os especialistas são assim rigorosos. Na
ponto somos todos voyeurs ⎯ pessoas que têm opinião do pesquisador americano Clay Calvert, da
necessidade de observar a intimidade alheia? E até Universidade Estadual da Pensilvânia, por
que ponto essa curiosidade é saudável? Há limites? exemplo, esses programas são um entretenimento
Boas perguntas que poderiam ter sido feitas inofensivo. Autor do livro Voyeur Nation ⎯ um
também em outras épocas. "Essa curiosidade é estudo sobre o sucesso de programas como
milenar", afirma o psicanalista Armando Survivor e Dateline (uma espécie de "Namoro na
Colognese Jr., coordenador geral do departamento TV"), Calvert criou o termo "voyeurismo
de psicanálise do Instituto Sedes Sapientiae, de mediado" para descrever os reality shows.
São Paulo. "Só está se manifestando de uma forma "Ninguém sai prejudicado, pois todos ali
diferente agora." consentiram na violação de sua privacidade",
A palavra voyeur, que em francês significa algo afirma.
como "olhador", entrou para o vocabulário médico O psicanalista brasileiro Armando Colognese
no começo do século 20. Mas, o fenômeno é tão Jr. acrescenta que o espectador também não está
antigo como o homem ⎯ ou como outros primatas demonstrando comportamentos doentios ao sentir
que também gostam de espreitar os vizinhos, vontade de ver esses programas. "As pessoas
sobretudo em situações de intimidade. assistem porque têm a necessidade de investigar a
(...) vida do outro para estabelecer um padrão de
comparação", diz Colognese, ou seja, não estão
Componente perverso satisfeitas com suas vidas e procuram resposta da
própria insatisfação buscando respostas para a
Há quem veja nos reality shows algo mais sério do questão: "O que ele tem que eu não tenho?", diz o
que a tentativa de espantar a monotonia da própria psicanalista. Para ele, isso é normal, mas frisa:
vida. O psicólogo americano Philip Zimbardo, da "Desde que ver o programa não se torne
Universidade Stanford, por exemplo, acredita que fundamental para o prazer da pessoa".
"programas como Survivor (inspirador de No
limite) promovem os piores aspectos do (Paulo D'Amaro, Revista Galileu)
comportamento humano."
Zimbardo, que participou da produção do reality TEXTO II
show inglês Zoológico Humano, tem uma longa
história nessa área. Em 1971, ficou famoso por “(...) e as aldeias são a alheia vigilância."
confinar 24 estudantes voluntários, simulando uma
penitenciária ⎯ vigiada por câmeras e microfones. (Extraído do conto Desenredo, de Guimarães
O experimento foi interrompido no sexto dia para Rosa.)
impedir os atos de brutalidade protagonizados
pelos, até então, pacíficos alunos de psicologia. TEXTO III
Presidente da Associação Americana de
Psicologia, Zimbardo aprendeu a lição e hoje CIDADEZINHA QUALQUER
defende cautela no conteúdo dos reality shows.
A opinião é compartilhada pelo sociólogo 1 Casas entre bananeiras
Laurindo Leal Filho, presidente da ONG Tver, mulheres entre laranjeiras
criada para discutir a responsabilidade social da pomar amor cantar.
televisão no Brasil. "É lamentável que a TV se
aproveite de um componente perverso da Um homem vai devagar.
personalidade humana para conseguir audiência", 5 Um cachorro vai devagar.
diz, referindo-se a programas como Casa dos Um burro vai devagar.
Artistas e Big Brother. "Eles não estimulam o
108
PROVAS DA AFA 1998 – 2010
Devagar... as janelas olham. b) os reality shows estimulam o que há de pior na
8 Eta vida besta, meu Deus. natureza humana.
c) segundo Clay Calvert, embora os participantes
(Carlos Drummond de Andrade) consintam na violação de sua privacidade, alguns
saem prejudicados.
01 - O principal enfoque do TEXTO I é a d) há no voyeurismo algo de perverso, doentio.
a) crítica à exploração dos piores aspectos da
personalidade humana pelos meios de 05 - Assinale a alternativa que traduz com
comunicação. exatidão o trecho escrito por Guimarães Rosa
b) denúncia da pobreza intelectual vigente nos dias (TEXTO II).
atuais, responsável pelo sucesso dos shows de a) A vigilância à vida alheia é uma característica
realidade. das pequenas cidades.
c) justificativa para um comportamento humano b) Nas aldeias, as pessoas só pensam em vigiar a
perfeitamente natural, encontrado até mesmo em vida dos outros.
primatas. c) Só nas aldeias as pessoas pensam tanto em
d) análise do voyeurismo, um comportamento vigiar a vida dos outros.
milenar redescoberto, com pontos de vista de d) A vida alheia é a principal preocupação das
especialistas diversos. cidades pequenas, por falta de outras opções.

02 - O TEXTO I analisa a explosão de 06 - Assinale a única alternativa que NÃO está


audiência dos reality shows. De acordo com ele, de acordo com o TEXTO III.
pode-se afirmar que a) Sua linguagem é concisa, sem descartar certo
a) mais uma vez o Brasil atesta seu vazio cultural, lirismo.
sendo pioneiro nesse tipo de programa. b) O "prazer de bisbilhotar" é abordado com
b) o foco desses programas é o comportamento de suavidade no sétimo verso.
gente famosa em situações prosaicas, mas c) O eu-lírico demonstra uma visão irônica das
detalhadamente montadas. cidadezinhas do interior.
c) a base de seu sucesso está em explorar uma d) No último verso, percebe-se uma postura
tendência antiga do psiquismo humano: o nitidamente carinhosa em relação aos habitantes do
voyeurismo. interior.
d) após o sucesso desses programas no Brasil,
outros países aderiram à moda, conseguindo 07 - Tendo como base todo o contexto de
também grande repercussão. apresentação dos três textos, assinale as
afirmativas com V (verdadeiro) ou F (falso) e, a
03 - Assinale a alternativa que, corretamente, dá seguir, marque a alternativa correspondente.
continuidade ao trecho abaixo. ( ) O TEXTO I tem caráter
Os especialistas citados no TEXTO I predominantemente dissertativo.
( ) Os TEXTOS II e III são nitidamente
a) são unânimes em suas considerações sobre o literários.
"prazer de bisbilhotar". ( ) Os TEXTOS I e II foram escritos em
b) podem ser divididos em dois grupos prosa, enquanto o III o foi em versos.
equivalentes, de acordo com a postura adotada ( ) A argumentação é a principal
frente ao tema. característica do TEXTO II.
c) não encontram explicação plausível para o a) V – V – V – F b) F – F– V – V
sucesso desse tipo de programa. c) V – V – F – F d) F – F – F – V
d) concluíram que, até certo ponto, somos todos
voyeurs. 08 - A análise e comparação dos três textos (I,
II e III) acima demonstram que
04 - No TEXTO I, só NÃO se encontra a a) todos elegem o mesmo comportamento, como
seguinte afirmação: tema central.
a) a atitude dos participantes não é espontânea, b) o TEXTO II confirma a tese defendida no
porque eles sabem que estão sendo observados. TEXTO I.
109
PROVAS DA AFA 1998 – 2010
c) os TEXTOS II e III apresentam o mesmo Assinale a alternativa em que os vocábulos
contexto espacial. são acentuados, respectivamente, pela mesma
d) o TEXTO III adota a mesma postura analítica razão que os acima destacados.
do TEXTO I. a) Inglês, há, área, fé, lamentável.
b) Sociólogo, também, só, até, inútil.
09 - Observe a tirinha de Miguel Paiva e, a c) Igarapé, trás, Pensilvânia, má, juízes.
seguir, assinale a alternativa correta. d) Francês, pá, própria, dó, idéia.

12 - Leia o seguinte texto:


"Minha alma é como um deserto,
Por onde o romeiro incerto
Procura uma sombra em vão;
É como a ilha maldita
Que sobre as vagas palpita
Queimada por um vulcão."
I II

a) No 2o quadrinho, o pronome demonstrativo Assinale a alternativa INCORRETA,


pode ser substituído por esse. quanto à morfologia.
b) No 1o quadrinho, observa-se a presença de 4 a) O pronome possessivo identifica o eu lírico no
(quatro) ditongos decrescentes orais. primeiro verso da poesia.
c) Os pôr-do-sol é a flexão de plural correta de b) O adjetivo “incerto” foi formado pelo processo
pôr-do-sol. de derivação prefixal.
d) Apressar é uma palavra cognata de preço. c) O vocábulo "Queimada" se identifica como
forma verbal, particípio de “queimar”.
10 - Observe o quadrinho de Mort Walker d) A forma do verbo “ser”, no primeiro verso, é
(Recruta Zero). própria do modo subjuntivo.

13 - As afirmações a seguir se baseiam na


estrofe abaixo:

Via-se todo o monte revestido


de emaranhados troncos,
guedelha excelsa dos penedos broncos,
que tecendo entre [si] frondosos laços
I II eram das nuvens verdes embaraços.
(Eusébio de Matos)
I- Observa-se no 1o quadrinho a presença de 1
(um) neologismo. I- O se é pronome reflexivo. O sujeito é
II - Super é um prefixo de origem latina que “todo o monte revestido”. “Monte” é substantivo e
transmite a idéia de excesso. significa serra; é, também, o núcleo do sujeito.
III - O 2o quadrinho apresenta o vocábulo II - Em “de emaranhados troncos”, a
brevidade, composto por justaposição. preposição introduz um complemento nominal. O
Está(ão) correta(s) apenas a(s) questão(ões) substantivo “emaranhados” quer dizer espaçados.
a) I. b) II e III. c) I e II. d) I, II e III. III - No verso “guedelha excelsa dos penedos
broncos,” “excelsa” é adjetivo que significa
11 - Leia atentamente a frase do fotógrafo grandiosa, elevada; “broncos” também é um
peruano, que passou o réveillon no Rio. adjetivo e significa tosco, áspero.
"No Brasil, em ensaio de escola de samba você vê IV - O adjetivo “emaranhados” e o verbo
artista, modelo, milionário, bandido, tudo junto, é “tecendo” não se relacionam pelo sentido, pois
incrível." “tecendo” lembra urdidura e “emaranhados”
(Mário Testino) significa libertos.
Estão INCORRETAS as afirmações
110
PROVAS DA AFA 1998 – 2010
a) II e III. b) I e III. Um cachorro vai devagar.
c) I, II e IV. d) III e IV. Um burro vai devagar.”

14 - Identifique a alternativa em que a palavra Só é correto afirmar que


destacada está em DESACORDO com a relação a) se trata de períodos simples e, por isso, não
que transmite. constituem frases.
a) “- Como se chama? – perguntou-lhe o notário, b) em todos eles, o sujeito do verbo “ir” está
fitando-a por cima dos óculos, com a pena de pato determinado e possui apenas um núcleo.
apontada à página da nota.” (Direção) c) os predicados das três orações são verbo-
b) “...quando não dá mesmo para arrumar nominais.
desculpa, e ainda assim levando uma amiga a d) o termo “devagar”, em todos os versos,
tiracolo.” (Proximidade) funciona como predicativo do sujeito.
c) “A questão é ocupá-la à sua maneira,
reconhecendo aos poucos...” (Modo) 18 - Atente às afirmativas abaixo, referentes a
d) “...dirigir-se a um superior, ...” (Motivo) esta tira:

15 - Com relação ao TEXTO III, todas as


assertivas abaixo são corretas, EXCETO uma.
Assinale-a.
a) Posposto ao substantivo “cidadezinha”, o
pronome indefinido “qualquer” assume um valor
prosaico, realçando a temática do cotidiano e o I- A forma verbal “tem” é, no enunciado,
tom irônico que perpassa todo o poema. impessoal e corresponde ao verbo “haver” na
b) No último verso, a vírgula isola, linguagem culta.
obrigatoriamente, um vocativo. II - Nessa acepção, o verbo “ter" é pessoal e
c) Embora o verbo “olhar” seja, normalmente, possui como sujeito a expressão “uma árvore”.
transitivo direto, no penúltimo verso ele se torna III - A oração “quando se precisa” é
intransitivo. subordinada, porque funciona como adjunto
d) As expressões “entre bananeiras” e “entre adverbial em relação à oração principal.
laranjeiras” complementam o significado de IV - O sujeito da segunda oração é
substantivos concretos e uniformes. indeterminado, o que se comprova claramente pela
presença do pronome “se”.
16 -Com relação a este lead, inscrito no TEXTO I: V- Por se tratar de um período, o enunciado
“Na TV, internet ou na exibição de fotos não pode ser considerado como exemplo de frase.
reveladoras, especialistas constatam: as pessoas Somente estão corretas as afirmativas
gostam de espionar a intimidade alheia.” a) II, III, IV e V. b) I, III e IV.
Só é correto afirmar que a/o(s) c) II, III e IV. d) I, IV e V.
a) expressão “de fotos reveladoras” completa o
sentido do substantivo abstrato “exibição”; pode-se 19 - Leia com atenção esta passagem transcrita
considerá-la, portanto, como um termo integrante. do TEXTO I, assinalando, em seguida, a opção
b) adjetivos “reveladoras” e “alheia” qualificam os correta.
substantivos a que se referem, acrescentando-lhes, “Esse fenômeno levantou uma discussão
por isso, uma nova e indispensável informação. entre pesquisadores do comportamento humano:
c) dois-pontos separam, da oração principal, uma até que ponto somos todos voyeurs ⎯ pessoas que
subordinada substantiva, já que esta tem função têm necessidade de observar a intimidade alheia?”
sintática de aposto. a) Nele se percebe uma silepse de número.
d) período possui uma terceira oração, que, b) No período, o verbo “levantar” é impessoal e,
reduzida de infinitivo, funciona sintaticamente quanto à predicação, transitivo direto.
como complemento nominal. c) Os trechos “até que ponto somos todos
voyeurs” e “pessoas que têm necessidade de
17 - Pela análise destes versos do TEXTO III: observar a intimidade alheia” funcionam,
“Um homem vai devagar.
111
PROVAS DA AFA 1998 – 2010
respectivamente, como apostos explicativos de procurando a estrada, penetrando entre os arbustos,
“discussão” e “voyeurs”. espreitando, farejando.”
d) O pronome relativo “que”, destacado na frase É INCORRETO afirmar que
introduz uma oração que restringe o significado do a) ele possui oito verbos; sendo assim, é
estrangeirismo voyeurs. constituído de oito orações.
b) em “pressenti-a mais próxima”, os termos
20 - Atente ao TEXTO II para assinalar a destacados têm, respectivamente, função sintática
alternativa correta. de objeto direto e adjunto adverbial de lugar.
“(...) e as aldeias são a alheia vigilância.” c) dentre essas orações, quatro são reduzidas de
a) A concordância do verbo “ser” é especial; no gerúndio.
caso acima, está indo ao encontro da norma culta, d) a palavra “por que” está empregada
pois é patente a intenção de se realçar o sujeito. corretamente, por isso pode ser substituída, sem
b) O termo “a alheia vigilância”, cujo núcleo é alteração de sentido, pela expressão “por qual
“alheia”, exerce função sintática de predicativo do razão”.
sujeito.
c) O enunciado que o constitui, pelos aspectos 23 - Assinale, dentre os fragmentos abaixo,
gramaticais que apresenta, pode ser considerado aquele em que a ênclise NÃO é obrigatória.
um período. a) “Antes bonita, olhos de viva moça, morena mel
d) A palavra “vigilância”, no texto, é transitiva; e pão. Aliás, casada. Sorriram-se, viram-se. (...)
pede, pois, o complemento nominal “alheia”. Enfim, entenderam-se.”
(J. Guimarães Rosa. In: Desenredo)
21 - Com referência à seguinte passagem do b) “Somos muitos Severinos
TEXTO I: iguais em tudo e na sina:
“Nas intrigas da Casa dos Artistas, do SBT, nas a de abrandar estas pedras
privações físicas de No Limite, da Rede Globo, ou suando-se muito em cima”
no embate de Big Brother Brasil, também da (Cabral de Melo Neto. In: Morte e Vida Severina)
Globo, o telespectador parece ter se viciado na c) “Os outros homens afastaram-se como para
observação das pessoas que não estão ali deixar a arena livre, e Nicolau, atrás do balcão,
dramatizando, mas mostrando o que realmente começou a gritar:
são.” ⎯ Aqui dentro não! Lá fora! Lá fora!”
é INCORRETO sustentar que a(s)/o (Érico Veríssimo. In: Um certo Capitão Rodrigo.)
a) expressões “Nas intrigas da Casa dos Artistas”, d) “Era uma coisa de ver-se, grandiosa para eles,
“nas privações físicas de No Limite”, “no embate que os enchia de respeito e certo temor. Esse
de Big Brother Brasil” e “na observação das distante São Paulo devia de ser terra de muita
pessoas” expressam uma circunstância de lugar. riqueza realmente para exigir tanto sacrifício dos
b) locuções “da Casa dos Artistas”, “de No que para lá viajavam.”
Limite” e “de Big Brother Brasil”, por sua vez, são (Jorge Amado. In: Seara vermelha.)
adjuntos adnominais dos nomes aos quais se
referem. 24 - Leia com atenção os seguintes enunciados:
c) conjunção “mas” coordena duas idéias que se I- “Espero que os holofotes da CPI
opõem. continuem acesos quando ele chegar aos tubarões
d) pronome oblíquo “se”, por estar ligado a uma da lavagem de dinheiro.
forma verbal composta, pode também ser colocado (Extraído da seção VEJA ESSA, da revista VEJA.)
após o verbo principal. II - Você gosta que acariciem sua
barriguinha, não? Você é tão fofinho!
22 - Examine este período, extraído da obra O (Extraído de tira RECRUTA ZERO, de Mort
primeiro beijo e outros contos, de Clarice Walker – Jornal O Globo.)
Lispector: III - O senhor esqueceu de desligar o
“Não sabia como e por que mas agora se toupeira depois que o mandou cavar um buraco
sentia mais perto da água, pressentia-a mais para a sua roseira!
próxima, e seus olhos saltavam para fora da janela (Extraído de tira CROCK E OS LEGIONÁRIOS,
de Rechin & Wilder – Jornal O Globo.)
112
PROVAS DA AFA 1998 – 2010
Levando-se em consideração as normas de
regência verbal, pode-se afirmar que há 27 -
INCORREÇÃO em a) Está presente apenas o discurso indireto.
a) I e II apenas. b) III somente. b) Mesclam-se, nesse parágrafo, os três tipos de
c) II somente. d) I, II e III. discurso, (direto, indireto e indireto livre).
c) Ao discurso indireto, soma-se apenas o discurso
25 - Considere o fragmento. direto.
“No outro dia o meu primo Silvino nos d) São encontrados o discurso indireto e o discurso
contou que tinha se lembrado de dizer ao indireto livre.
cangaceiro que a Tia Sinhazinha não gostava dele.
É que nos falavam sempre de uma velha que 28 –
Antônio Silvino fizera dançar nua, dando a) Não existe defesa explícita de ponto de vista.
umbigada num pé de Candeiro, por motivo b) Não há abordagens psicológicas, o texto
semelhante. Se isto tivesse acontecido com a velha restringe-se à narração de fatos.
Sinhazinha, os moleques, as negras e os meninos c) É nítida a presença de um narrador personagem.
do Santa Rosa teriam dormido uma noite de d) A adjetivação expressiva deixa claro o caráter
grande.” descritivo do texto.
Nele ocorre(m) discurso(s)
a) indireto. b) direto. 29 - Tomando como base o texto não-verbal
c) indireto livre. d) direto e indireto. abaixo, assinale a opção correta.

26 - Marque a alternativa correta, levando em


consideração os tipos de discurso.
a) O discurso direto é característica da narração, já
o discurso indireto livre é necessidade básica da
dissertação.
b) No primeiro parágrafo, do TEXTO I, após o
subtítulo “Componente perverso”, a opinião de
Philip Zimbardo é transcrita pelo processo de a) O texto, propositadamente, apresenta falta de
discurso indireto. coesão.
c) O último verso de “Cidadezinha qualquer” é um b) Não há coesão, mas apenas coerência no texto.
exemplo do uso de discurso direto. c) A coerência do texto é garantida pela expressão
d) O trecho extraído de “Desenredo” TEXTO II, “a gente”.
conto de Guimarães Rosa, é um ótimo exemplo de d) A aparente incoerência visa a chamar a atenção
discurso indireto livre. do leitor.

Leia e analise o texto abaixo para assinalar a 30 - Assinale a alternativa cujo elemento de
alternativa correta das questões 27 e 28. coesão corrige o texto abaixo.

Mafra o consolou, batendo-lhe nas costas: tirara o As pessoas caminhavam pelas ruas,
terceiro lugar (numa prova de natação). Foi para despreocupadas, como se não existisse perigo
casa sozinho, a cabeça num tumulto. Por que afinal algum, MAS o policial continua tomando
tudo aquilo, Santo Deus? Que idéia descabida, que folgadamente o seu café.
estranha teimosia aquela, esquecer tudo durante
um mês, para dedicar-se como um louco a uma a) até mesmo b) por isso mesmo
experiência tão dura que não lhe traria proveito c) no entanto d) apesar disso
algum! Vaidade apenas? Solidariedade para com
seu clube? Ora, sabia muito bem que essas coisas 31 - Leia atentamente o trecho abaixo de
não existiam mais para ele. Por que então? O pai Oswald de Andrade.
lhe dissera apreensivo: “Você está exagerando, “E tia Gabriela sogra grasnadeira grasnou
meu filho. Isso não pode fazer bem.” graves grosas de infâmia.”
(Encontro Marcado, de Fernando Sabino.) Trata-se de um texto literário porque
113
PROVAS DA AFA 1998 – 2010
Diante do contexto apresentado no enunciado, a
a) o plano da expressão (sons) articula-se com o função de linguagem predominante é a
plano do conteúdo, contribuindo para a a) referencial. b) conativa.
significação global. c) metalingüística. d) poética.
b) o plano do conteúdo prevalece, priorizando,
assim, o que se diz, em vez de o modo como se 35 - Ao lado de cada estrofe abaixo, foi
diz. indicada uma figura de linguagem nela presente.
c) é possível fazer um resumo do texto, sem perder Assinale a opção cuja figura NÃO está
o essencial em nenhum dos planos. corretamente associada.
d) o uso estético da linguagem é sacrificado em a) “Santa Clara, padroeira da televisão,
função de uma abordagem mais denotativa. Que a televisão não seja o inferno, interno, ermo”
– Apóstrofe
32 - Assinale a afirmativa correta sobre texto b) A saudade abraçou-me, tão sincera
literário e texto não-literário. Soluçando no adeus de nunca mais – Pleonasmo
a) Conhece-se um texto literário pelo seu c) E as borboletas sem voz
conteúdo, pelos temas abordados. Dançavam assim veludosamente - Sinestesia
b) Só pode ser considerado literário um texto d) “Provisoriamente não cantaremos o amor
ficcional. Que se refugiou mais abaixo dos subterrâneos” –
c) O texto literário tem função estética, enquanto o Prosopopéia.
não-literário tem função utilitária.
d) Predomina, no texto não-literário, a linguagem 36 - Em “as aldeias são a alheia vigilância” e
conotativa, que é mais expressiva. “... as janelas olham”, ocorre, simultaneamente,
uma figura de
33 - Leia os fragmentos abaixo. a) construção, a redundância, cujo efeito estilístico
I- “(...) Voyeur, que em francês significa algo é realçar uma idéia.
como olhador.” b) sintaxe, que consiste em suprimir ou ocultar
II - “Casas entre bananeiras palavras expressas anteriormente e que se
mulheres entre laranjeiras encontram subentendidas.
pomar amor cantar.” c) palavra, que consiste na substituição de um
III - Na TV, internet, ou na exibição de fotos termo por outro, por haver certa relação de
reveladoras, especialistas constatam: as pessoas proximidade entre eles.
gostam de espionar a intimidade alheia.” d) sintaxe, que consiste numa alteração na ordem
Neles há, respectivamente, predominância das normal dos termos na oração, provocando uma
funções inversão.
a) conativa, emotiva, metalingüística.
b) referencial, apelativa, fática. 37 - Assinale a opção em cuja frase NÃO há a
c) fática, conativa, emotiva. figura de linguagem indicada ao lado.
d) metalingüística, poética, referencial. a) “O Brasil dos últimos meses viu uma explosão
de audiência e repercussão dos reality shows” –
34 - O conjunto musical Titãs foi a uma metonímia
determinada emissora de televisão e cantou, b) “(...) até que ponto somos todos voyeurs –
seguido da platéia, uma música de seu repertório, pessoas que têm necessidade de observar a vida
cujo fragmento está abaixo reproduzido. alheia (...). Essa curiosidade é milenar” –
“A mãe diz pra eu fazer alguma coisa mas hipérbole
eu não faço nada c) “(...) e as aldeias são a alheia vigilância” –
A luz do sol me incomoda, então eu deixo a metonímia
cortina fechada d) “Um homem vai devagar. / Um cachorro vai
É que a televisão me deixou burro, muito devagar. / Um burro vai devagar.” – anáfora
burro demais
E agora eu vivo dentro dessa jaula junto 38 - Leia os versos a seguir e responda o que se
dos animais.” pede logo abaixo.
“Atrás de portas fechadas,
114
PROVAS DA AFA 1998 – 2010
à luz de velas acesas, Horrendos a dançar...
entre sigilo e espionagem,
acontece a Inconfidência Negras mulheres, suspendendo às tetas
(...) Magras crianças, cujas bocas pretas
E os seus tristes inventores Rega o sangue das mães:
já são réus – pois se atreveram a falar em Outras, moças, mas nuas e espantadas,
liberdade No turbilhão de espectros arrastadas,
(...) Em ânsia e mágoa vãs!
E a vizinhança não dorme:
murmura, imagina, inventa. E ri-se a orquestra, irônica, estridente...
Não fica a bandeira escrita, E da ronda fantástica a serpente
mas fica escrita a sentença” Faz doudas espirais...
(Cecília Meireles) Se o velho arqueja, se no chão resvala,
Ouvem-se gritos... o chicote estala.
As palavras em negrito, das estrofes acima, E voam mais e mais...
foram substituídas, respectivamente, por uma série
sinonímia, EXCETO em Presa nos elos de uma só cadeia,
a) conjuração, acusados, alforria, segreda, decisão. A multidão faminta cambaleia,
b) conspiração, precitos, emancipação, rumoreja, E chora e dança ali!
condenação. Um de raiva delira, outro enlouquece,
c) insurreição, culpados, abolição, zunzuna, pena. Outro, que de martírios embrutece,
d) revolta, insontes, soltura, alaria, prisão. Cantando, geme e ri!

39 - Assinale a opção em que NEM TODAS as No entanto o capitão manda a manobra.


palavras relacionadas possuem uma mesma E após fitando o céu que se desdobra
significação de ordem geral. Tão puro sobre o mar,
a) Comichão, prurido, coceira, formigamento. Diz do fumo entre os densos nevoeiros:
b) Mensageiro, arauto, porta-novas, recadista. “Vibrai rijo o chicote, marinheiros!
c) Desestima, desdém, irreverência, deslouvor. Fazei-os mais dançar!...”
d) Submisso, subalterno, vassalo, ultrajado.
E ri-se a orquestra irônica, estridente...
40 - As palavras internet, shows, voyeurs, são E da ronda fantástica a serpente
importações lingüísticas, respectivamente, Faz doudas espirais...
denominadas como Qual num sonho dantesco as sombras
a) anglicismo, anglicismo, galicismo. [voam!...
b) galicismo, galicismo, anglicismo. Gritos, ais, maldições, preces ressoam!
c) anglicismo, galicismo, anglicismo. E ri-se Satanás!...
d) galicismo, anglicismo, galicismo.
1) A palavra luzernas, no segundo verso da 1ª
estrofe, pode ser substituída, sem alterar o
AFA 2003/2004 – Português contexto, por
a) embarcações b) lunetas
c) margens d) luzes
Leia atentamente a parte IV do poema “O
Navio Negreiro”, de Castro Alves, abaixo
2) Com relação ao poema, assinale a alternativa
transcrita e, a seguir, responda as questões de 1
correta.
a 5.
a) O argumento do poeta em favor da erradicação
da escravidão no Brasil tem como base a razão.
Era um sonho dantesco!... o tombadilho,
b) Castro Alves aproxima, por vezes, as idéias de
ue das luzernas avermelha o brilho,
som e movimento.
Em sangue a se banhar.
c) O poema é predominantemente narrativo.
Tinir de ferros... estalar de açoite...
Legiões de homens negros como a noite,
115
PROVAS DA AFA 1998 – 2010
d) O uso de antíteses é rico. O mesmo não ocorre a) cipós, álbum, anzóis
com as hipérboles. b) pés, raízes, heróico
c) nós, doído, vá
3) As metáforas “a orquestra” e “a serpente” nas d) após, Carandaí, pôs
3ª e 6ª estrofes, referem-se, respectivamente, ao(s):
a) som das águas/nevoeiros 10) O verbo está corretamente conjugado na
b) gritos e lamentos/chicote alternativa
c) ruídos do casco do navio/sangue dos escravos a) Tu és o futuro do Brasil! Cante teus versos
d) tinir de ferros/rastro do navio no mar alegremente!
b) Tu és bela! Não desperdice tua mocidade!
4) No verso: “Qual num sonho dantesco as c) Você é livre! Opta pelo melhor caminho!
sombras voam!...”, que sentido pode ser atribuído d) Vós sois o sal da terra! Sede felizes!
ao termo destacado?
a) Marinheiros fazem vibrar o chicote 11) Quanto ao emprego dos numerais, assinale a
b) Corpos negros tombam alternativa correta.
c) Energias negativas dominam o ambiente a) O Papa Paulo seis foi um mártir da Igreja
d) O navio esgueira-se pela escuridão da noite Católica.
b) O vigésimo primeiro século é a nossa realidade.
5) No verso: “E ri-se a orquestra irônica, c) O capítulo décimo nono fala sobre a História do
estridente...”, o termo destacado foi empregado no Brasil.
sentido d) O século cinco foi um facho de luz para a
a) denotativo b) polissêmico humanidade.
c) conotativo d) hiperonímico
12) “Quando cheguei à estação, o trem já partira”.
6) Assinale a alternativa em que todas as palavras A desinência modo-temporal do verbo destacado
estão corretamente grafadas. identifica o pretérito:
a) berinjela, xingar, regozijo a) perfeito do indicativo
b) Sergipe, xiste, proeza b) imperfeito do subjuntivo
c) girau, mexer, brasa c) imperfeito do indicativo
d) jeito, pechincha, despreso d) mais-que-perfeito do indicativo

7) Quanto a norma culta, assinale a alternativa 13) Numere a 2ª coluna de acordo com a 1ª e, a
correta. seguir, assinale a alternativa que corresponde à
a) A casa estava enfestada de pulgas. seqüência correta.
b) O submarino emergiu, desaparecendo assim nas Cada número pode ser usado mais de uma vez.
profundezas do mar. 1. Tu ( ) Mantêm-no
c) A honestidade roboriza as instituições 2. Ele ( ) Restituí-lo
permanentes de um país. 3. Vós ( ) Contém-lo
d) Meus olhos estão laços! 4. Eles ( ) Expõe-no
( ) Restitui-lo
8) Assinale a alternativa em que todos os ( ) Obtém-no
vocábulos devem receber acento gráfico. a) 4-1-1-2-3-2
a) germens, voos, torax, armazens. b) 4-3-1-2-1-2
b) papeis, transpo-lo, mes, por (preposição). c) 2-4-3-1-2-1
c) sabe-lo-emos, por (verbo), que (substantivo), d) 3-2-2-4-1-3
viuva.
d) pela (do verbo pelar), porque (conjunção), reus, 14) As palavras bonzinho, entardecer,
eter. expropriar e desigualdade são formadas,
respectivamente, por derivação
9) Assinale a alternativa em que as palavras são a) sufixal / parassintética / parassintética / prefixal
acentuadas, respectivamente, pela mesma regra de e sufixal
“sós”, “graúdo” e réu.
116
PROVAS DA AFA 1998 – 2010
b) regressiva / prefixal e sufixal / prefixal e sufixal a) pronome, partícula de realce, advérbio,
/ prefixal e sufixal conjunção, preposição
c) sufixal / prefixal e sufixal / prefixal e sufixal / b) advérbio, conjunção, advérbio, preposição,
prefixal e sufixal conjunção
d) sufixal / parassintética / prefixal e sufixal / c) conjunção, conjunção, preposição, preposição,
parassintética preposição
d) pronome, partícula de realce, conjunção,
15) Com relação aos radicais gregos, numere a 2ª conjunção, preposição
coluna de acordo com a 1ª e, em seguida, assinale
a alternativa correta. 19) Leia a primeira estrofe do poema “Marabá”, de
1. ánemos ( ) ar Gonçalves Dias:
2. báros ( ) nuvem Eu vivo sozinha; ninguém me procura!
3. aéros ( ) vento Acaso feitura
4. néfos ( ) exército Não sou de Tupá?
5. stratós ( ) peso, pressão Se algum dentre os homens de mim não se
6. crátos ( ) poder esconde
7. arque ( ) comando, governo – “Tu és”, me responde,
“Tu és Marabá!”
a) 3, 1, 4, 5, 2, 7, 6 b) 3, 1, 4, 5, 2, 6, 7
c) 4, 1, 3, 5, 2, 7, 6 d) 3, 4, 1, 5, 2, 6, 7 Os termos destacados foram empregados,
respectivamente, com o valor de
16) assinale a alternativa que completa correta e a) advérbio, adjetivo, pronome
respectivamente as lacunas abaixo. b) substantivo, substantivo, pronome
Se é para ________ dizer, afirmo-lhe que a escolha c) substantivo, advérbio, preposição
nunca se dará entre ________ e ________. d) advérbio, substantivo, preposição
a) mim / eu / tu b) eu / mim / ti
c) eu / mim / tu d) mim / eu / ti 20) Analise as orações abaixo:
I – Não o encontrei no escritório.
17) Quanto à flexão, os verbos apresentados são, II – Gosto de viajar em dias de chuva.
respectivamente, anômalo, defectivo, abundante e III – Coisas estranhas acontecem por lá.
irregular. IV – Todos estavam confiantes no veredicto.
a) ir, adequar, combalir, falir
b) ter, remir, ouvir, valer Os termos em destaque exercem, respectivamente,
c) ser, reaver, expulsar, caber a função sintática de
d) falir, haver, aceitar, ir a) objeto direto / complemento nominal / sujeito /
adjunto adnominal
18) Leia as frases abaixo: b) sujeito / complemento nominal / objeto direto /
“Ai que saudades que eu tenho da aurora de minha objeto indireto
vida...” (Casimiro de Abreu) c) objeto direto / adjunto adnominal / sujeito /
complemento nominal
“Oh! que altos são os segredos da Providência d) sujeito / objeto direto / objeto direto / objeto
divina.” (Vieira) indireto

“Finjo que o meu olho direito é de vidro...” 21) Observe o período abaixo:
(Rubem Fonseca) “Peço a V. Exa. que não associe o fato a nenhum
episódio anterior.”
“Não tenho que dar satisfação a ninguém,
tenho?...” (Guimarães Rosa) O termo em destaque

Os termos destacados nas frases acima devem ser a) exerce a função de objeto direto
classificados, respectivamente, como: b) não tem valor sintático
c) é partícula anunciadora de oração adjetiva
117
PROVAS DA AFA 1998 – 2010
d) exerce a função de objeto direto pleonástico b) dissertativo, quando analisa e interpreta dados
da realidade, através de argumentos.
22) Assinale a alternativa que apresenta os sujeitos c) descritivo, quando relata mudanças progressivas
dos verbos destacados abaixo: de estado, através do tempo.
“Não morrerá sem poetas nem soldados d) argumentativo, quando relata as características
A língua em que cantaste rudemente de uma pessoa, de um objeto ou de uma situação
As armas e os barões assinalados.” qualquer.
(Manuel Bandeira)
26) Observe o texto a seguir:
a) soldados / poetas; a língua “É sabido que o sistema do Império
b) tu; a língua Romano dependia da escravidão, sobretudo para a
c) armas / barões ; tu produção agrícola. É sabido ainda que a população
d) soldados / poetas; armas / barões escrava era recrutada principalmente entre
prisioneiros de guerra.
23) Leia o fragmento do poema “Tabacaria”, de Em vista disso, a pacificação das fronteiras
Fernando Pessoa. fez diminuir consideravelmente a população
escrava.
“Não sou nada. Como o sistema não podia prescindir da
Nunca serei nada mão-de-obra escrava, foi necessário encontrar
Não posso querer ser nada. outra forma de manter inalterada essa população.”
À parte isso, tenho em mim todos os sonhos do
mundo.” Analisando a conexão interna existente entre os
vários segmentos do texto, pode-se afirmar que a/o
Os termos destacados são, respectivamente, a) expressão em vista disso, no segundo parágrafo,
classificados morfológica e sintaticamente, como: estabelece uma relação de implicação causal entre
a) advérbio / objeto direto; adjetivo / núcleo do o dado anterior e o que vem a seguir.
predicativo do sujeito. b) conectivo como, que inicia o terceiro parágrafo,
b) pronome / objeto direto; substantivo / núcleo do manifesta uma relação de comparação.
predicativo do sujeito. c) palavra ainda, no primeiro parágrafo, (“É
c) advérbio / predicativo do sujeito; substantivo / sabido ainda que”...) serve para estabelecer uma
núcleo do objeto direto. relação de tempo, situando o texto no aspecto
d) pronome / predicativo do sujeito; substantivo / cronológico.
núcleo do objeto direto. d) conexão interna entre os vários enunciados do
texto evidencia uma relação de coerência.
24) Observe a frase abaixo, de A. Herculano:
1 27) Com relação às figuras de linguagem, numere
“Pois se o sabes, cumpre o teu dever, alcaide do a segunda coluna de acordo com a primeira e, em
Castelo de Faria!” seguida, assinale a alternativa correta.
2 1. “Eu sem você
Sou chama sem luz
Assinale a alternativa que aponta a correta Jardim sem 1uar
classificação morfológica do termo 1 e sintática do Luar sem amor.”
termo 2, respectivamente: (Vinícius de Moraes)
a) pronome indefinido / sujeito simples
b) pronome indefinido / aposto 2. “Trabalhava ao piano, não só Chopin
c) pronome demonstrativo / vocativo como ainda os estudos de Czerny.”
d) pronome pessoal / sujeito composto (Murilo Mendes)

25) Assinale a alternativa correta. 3. “O inútil choro das tristes águas


Um texto é predominantemente: Enche de mágoas a solidão...”
a) narrativo, quando relata ocorrências (Vicente de Carvalho)
simultâneas, em um momento estático do tempo.
118
PROVAS DA AFA 1998 – 2010
4. “Noite – montanha. Noite vazia. Noite indecisa. AFA 2004/2005 – Português
Confusa noite. Nada a procura, mesmo sem alvo.”
(Carlos Drummond de Andrade) Leia o texto de Clarice Lispector, abaixo transcrito
e, a seguir, responda as questões a ele referentes.
5. “Ó mar salgado, quanto do eu sal são lágrimas
de Portugal.” A quinta história
(Fernando Pessoa)
Esta história poderia chamar-se “As
( ) Prosopopéia Estátuas”. Outro nome possível é “O Assassinato”.
( ) Metonímia E também “Como Matar Baratas”. Farei então pelo
( ) Anáfora menos três histórias, verdadeiras porque nenhuma
( ) Apóstrofe delas mente a outra. Embora uma única, seriam mil
( ) Metáfora e uma, se mil e uma noites me dessem.
A primeira, “Como Matar Baratas”,
a) 3, 2, 4, 5, 1 b) 2, 3, 1, 4, 5 começa assim: queixei-me de baratas. Uma
c) 1, 2, 4, 3, 5 d) 5, 4, 3, 1, 2 senhora ouviu-me a queixa. Deu-me a
receita de como matá-las. Que misturasse em
28) Em relação à concordância ideológica, analise partes iguais açúcar, farinha e gesso. A farinha e o
as orações abaixo: açúcar as atrairiam, o gesso esturricaria o de-
I – “Há desenganos que fazem a gente velho.” dentro delas. Assim fiz. Morreram.
(Machado de Assis) A outra história é a primeira mesmo e
II – Os sobreviventes, emocionados, abraçamos os chama-se “O Assassinato”. Começa assim:
homens que vinham nos salvar. queixei-me de baratas. Uma senhora ouviu-me.
III – Essa turma é terrível! Como falam da vida Segue-se a receita. E então entra o assassinato. A
alheia! verdade é que só em abstrato me havia queixado
IV – Os brasileiros gostamos de futebol. de baratas, que nem minhas eram: pertenciam ao
andar térreo e escalavam os canos do edifício até o
Ocorre silepse de gênero e número, nosso lar. Só na hora de preparar a mistura é que
respectivamente, nas orações elas se tornaram minhas também. Em nosso nome,
a) I e II apenas b) II e IV apenas então, comecei a medir e pesar ingredientes numa
c) I e III apenas d) III e IV apenas concentração um pouco mais intensa. Um vago
rancor me tomara, um senso de ultraje. De dia as
29) “O filme ‘Cinema Paradiso’ é uma película baratas eram invisíveis e ninguém acreditaria no
que fala sobre o próprio cinema”. mal secreto que roía casa tão tranqüila. Mas se
A afirmativa caracteriza um exemplo da seguinte elas, como os males secretos, dormiam de dia, ali
função da linguagem: estava eu a preparar-lhes o veneno da noite.
a) fática b) cinematográfica Meticulosa, ardente, eu aviava o elixir da longa
c) metalingüística d) referencial morte. Um medo excitado e meu próprio mal
secreto me guiavam. Agora eu só queria
30) Leia as frases abaixo: gelidamente uma coisa: matar cada barata que
I – “Paulo pegou o ônibus correndo.” existe. Baratas sobem pelos canos enquanto a
II – “Sim, meu filho, neste momento tens meu gente, cansada, sonha. E eis que a receita estava
consentimento para o casamento.” pronta, tão branca. Como para baratas espertas
III – “Nosso hino é o mais belo do mundo.” como eu, espalhei habilmente o pó até que este
IV – “Há três meses atrás eu já previa o resultado.” mais parecia fazer parte da natureza. De minha
cama, no silêncio do apartamento, eu as imaginava
Há vício de linguagem nas frases subindo uma a uma até a área de serviço onde o
a) I, II, III e IV b) I, II e III apenas escuro dormia, só uma toalha alerta no varal.
c) I e III apenas d) II e IV apenas Acordei horas depois em sobressalto de atraso. Já
era de madrugada. Atravessei a cozinha. No chão
da área lá estavam elas, duras, grandes. Durante a

119
PROVAS DA AFA 1998 – 2010
noite eu matara. Em nosso nome, amanhecia. No vício de viver que rebentaria meu molde interno.
morro um galo cantou. Áspero instante de escolha entre dois caminhos
A terceira história que ora se inicia é a das que, pensava eu, se dizem adeus, e certa de que
“Estátuas”. Começa dizendo que eu me queixara qualquer escolha seria a do sacrifício: eu ou minha
de baratas. Depois vem a mesma senhora. Vai indo alma. Escolhi. E hoje ostento secretamente no
até o ponto em que, de madrugada, acordo e ainda coração uma placa de virtude: “Esta casa foi
sonolenta atravesso a cozinha. Mais sonolenta que dedetizada”.
eu está a área na sua perspectiva de ladrilhos. E na A quinta história chama-se “Leibnitz e a
escuridão da aurora, um arroxeado que distancia Transcendência do Amor na Polinésia”. Começa
tudo, distingo a meus pés sombras e brancuras: assim: queixei-me de baratas.
dezenas de estátuas se espalham rígidas. As baratas
que haviam endurecido de dentro para fora. 1) A leitura do texto de Clarice Lispector permite
Algumas de barriga para cima. Outras no meio de inferir que, EXCETO:
um gesto que não se completaria jamais. Na boca a) compreende vários textos que poderiam, cada
de umas um pouco da comida branca. Sou a um deles, constituir-se numa história completa.
primeira testemunha do alvorecer em Pompéia. Sei b) são quatro histórias; a quinta fica por conta da
como foi esta última noite, sei da orgia no escuro. imaginação de cada um.
Em algumas o gesso terá endurecido tão c) encaixadas uma nas outras, as histórias
lentamente como num processo vital, e elas, com compõem uma literatura lúdica, da qual todos
movimentos cada vez mais penosos, terão podem participar.
sofregamente intensificado as alegrias da noite, d) a primeira história não é propriamente uma
tentando fugir de dentro de si mesmas. Até que de narrativa, pois não apresenta os elementos
pedra se tornam, em espanto de inocência, e com essenciais do ato de narrar.
tal, tal olhar de censura magoada. Outras –
subitamente assaltadas pelo próprio âmago, sem 2) A rigidez das baratas mortas – que faz lembrar
nem sequer ter tido a intuição de um molde interno estátuas - é um evento que provoca a seguinte
que se petrificava! – essas de súbito se cristalizam, reflexão:
assim como a palavra é cortada da boca: eu te ... a) o processo de petrificação pelo gesso é lento e
Elas que, usando o nome de amor em vão, na noite gradativo.
de verão cantavam. Enquanto aquela ali, a de b) a morte “assalta”, flagra a vida sem aviso.
antena marrom suja de branco terá adivinhado c) as estátuas revelam gestos inacabados que ficam
tarde demais que se mumificara exatamente por eternizados pela petrificação.
não ter sabido usar as coisas com a graça gratuita d) a morte das baratas foi devida a ação do gesso
do em vão: “é que olhei demais para dentro de em contato com líquidos do corpo dos insetos.
mim! é que olhei demais para dentro de ...” – de
minha fria altura de gente olho a derrocada de um 3) No trecho: “Um medo excitado e meu próprio
mundo. Amanhece. Uma ou outra antena de barata mal secreto me guiavam”, o “mal secreto” pode ser
morta freme seca à brisa. Da história anterior canta interpretado por:
o galo. a) ímpeto assassino.
A quarta narrativa inaugura nova era no lar. b) enfermidade alérgica.
Começa como se sabe: queixei-me de baratas. Vai c) prazer de exterminar insetos.
até o momento em que vejo os monumentos de d) rito de morte.
gesso. Mortas, sim. Mas olho para os canos, por
onde esta mesma noite renovar-se-á uma 4) A narradora revela em seus atos o extravasar de
população lenta e viva em fila indiana. Eu iria um instinto cruel. Que passagem do texto confirma
então renovar todas as noites o açúcar letal? como essa afirmação?
quem já não dorme sem a avidez de um rito. E a) “Estremeci de mau prazer à visão daquela vida
todas as madrugadas me conduziria sonâmbula até dupla de feiticeira.”
o pavilhão? no vício de ir ao encontro das estátuas b) “Um vago rancor me tomara, um senso de
que minha noite suada erguia. Estremeci de mau ultraje”.
prazer à visão daquela vida dupla de feiticeira. E c) “Meticulosa, ardente, eu aviava o elixir da
estremeci também ao aviso do gesso que seca: o longa morte”
120
PROVAS DA AFA 1998 – 2010
d) “De minha cama, no silêncio do apartamento, 1 - Um e meio ( ) perigeu
eu as imaginava subindo uma a uma até a área de
serviço...” 2 - Posição anterior ( ) hemistíquio

5) Atenção! Para a resolução da questão, foram 3 - Metade ( ) pródromo


omitidos os acentos gráficos das palavras.
Assinale a alternativa correta. 4 - Além de ( ) sesquiáltera
a) “jacares”, “atraves”, “baus”, “alguem” e
“Parana” devem receber acento gráfico pela regra 5 - Em torno de ( ) preternatural
das palavras oxítonas.
b) “zefiro”, “bavaro”, “egide”, “ariete” e “amago” a) 5, 3, 2, 1, 4 c) 4, 1, 5, 3, 2
devem receber acento gráfico pela regra das b) 4, 3, 2, 1, 5 d) 5, 3, 4, 1, 2
palavras proparoxítonas.
c) “vatapa”, “domino”, “papeis”, “cafe” e Leia o texto abaixo:
“tambem” devem receber acento gráfico pela regra
das palavras oxítonas. O pastor e o lobo
d) “hifen”, “semen”, “gratis”, “itens” e “orfã”
devem receber acento gráfico pela regra das Um pastor encontrou uma vez um filhote de lobo
palavras paroxítonas. que a mãe abandonara. Levou-o para casa, tratou
dele e ensinou-o a roubar carneiros dos rebanhos
6) Apenas uma das alternativas abaixo NÃO vizinhos. O lobo cresceu e aprendeu tão bem que
apresenta erro de ortografia. Assinale-a. um dia roubou um carneiro do rebanho do próprio
a) “Houve sessão extraordinária, porém a ata pastor.
apresentou deslises que exigiram pronta ⎯ Por que fizeste isso comigo? – perguntou o
retificação.” pastor.
b) “Sempre considerou prazeiroso caminhar pelas ⎯ Por que me ensinaste a roubar? – retrucou o
ruas de Londres e nunca deixou de desfrutar desse lobo.
previlégio.”
c) “Coalisaram-se as oposições mas o presidente Fábulas de Esopo. Texto em português de
interveio e obteve consenso.” Guilherme de Figueiredo
d) “O esmaecimento das cores, no outono,
imprime excessiva melancolia em seu espírito, 9) A classificação morfológica das palavras
tornando-o infenso às depressões.” destacadas é, respectivamente:
a) conjunção, substantivo, conjunção, pronome.
7) Assinale a alternativa que NÃO apresenta erro. b) conjunção, adjetivo, pronome, advérbio.
a) Quando o advogado falava de modo respeitoso c) pronome, substantivo, conjunção, pronome.
era porque referia-se ao meretíssimo senhor juiz. d) pronome, adjetivo, conjunção, pronome.
b) Para as crianças, a fazenda era um paraíso.
Fartavam-se de frutas: amoras, laranjas, Leia a tira abaixo:
jaboticabas e, principalmente, goiabas.
c) O monumento da Praça Sete, em Belo 1º QUADRINHO 2º QUADRINHO 3º
Horizonte, carinhosamente chamado de “pirulito” QUADRINHO
é um obelisco ponteagudo.
d) O menino, irrequieto, movia-se de um lado para
outro, dentro do carro do tio.

8) Em relação ao sentido dos prefixos, numere a 2ª


coluna de acordo com a 1ª e, em seguida, assinale
a alternativa que apresenta a seqüência correta.

121
PROVAS DA AFA 1998 – 2010
10) Marque a alternativa INCORRETA, em 14) Nas frases:
relação à tira acima. I- O jogo final do campeonato brasileiro foi
a) Há, no 1º quadrinho, dois advérbios. muito emocionante.
b) A palavra “quieto”, no 1º quadrinho está no II - O resultado tornou o Cruzeiro campeão.
grau superlativo absoluto analítico.
c) As palavras do 2º quadrinho devem ser Unindo-se as duas orações num só período,
classificadas, morfologicamente, como usando um pronome relativo, tem-se:
interjeições. a) O jogo final do campeonato brasileiro o qual
d) A palavra “melhor”, no último quadrinho, deve resultado tornou o Cruzeiro campeão foi muito
ser classificada, morfologicamente, como adjetivo. emocionante.
b) O jogo final do campeonato brasileiro cujo
11) Leia o fragmento abaixo transcrito. resultado tornou o Cruzeiro campeão foi muito
emocionante.
"⎯ Por que Laís não chegou ainda? c) O jogo final cujo resultado tornou o Cruzeiro
"⎯ Ela não chegou porque o carro que a campeão do campeonato brasileiro foi muito
conduzia quebrou. emocionante.
"⎯ Gostaria que ela chegasse logo porque d) O jogo final do campeonato brasileiro cujo o
preciso dela aqui!” resultado tornou o Cruzeiro campeão foi muito
emocionante.
Os termos destacados devem ser classificados,
respectivamente, como 15) Em relação aos advérbios destacados, numere
a 2ª coluna de acordo com a 1ª e, em seguida,
a) advérbio; conjunção coordenativa explicativa; assinale a alternativa que apresenta a seqüência
conjunção subordinativa causal. correta.
b) advérbio; conjunção subordinativa causal; Cada número pode ser usado mais de uma vez, ou
conjunção coordenativa explicativa. não ser usado.
c) preposição acidental; conjunção subordinativa
causal; conjunção coordenativa conclusiva. 1- Afirmação
d) preposição acidental; conjunção subordinativa 2- Modo
causal; conjunção coordenativa explicativa. 3- Tempo
4- Intensidade
12) Em relação ao emprego do particípio, assinale
a alternativa correta. ( ) “O poeta é um fingidor.
a) O comandante havia aceito a proposta do aluno. Finge tão completamente
b) Próximo à pia batismal, a água foi benta pelo Que chega a fingir que é dor
padre. A dor que deveras sente.” (F.Pessoa)
c) A autoridade fiscal havia isento o contribuinte. ( ) A criada sentiu-se realmente esgotada.
d) Os bombeiros haviam salvo o desesperado ( ) O filme era terrivelmente assustador.
banhista. ( ) Os alunos não são mais crianças.
( ) O padre chegou mais cedo que os fiéis.
13) Apenas uma das alternativas abaixo apresenta
uma locução prepositiva. Assinale-a. a) 1, 1, 4, 3, 4 c) 1, 4, 4, 3, 4
a) Desde que ela passou a dirigir a companhia, b) 2, 2, 2, 4, 4 d) 1, 1, 4, 3, 3
tudo se fez às claras.
b) O sucesso esperado veio à custa de muito 16) Os substantivos compostos estão corretamente
sacrifício. flexionados, EXCETO em:
c) Por fim, Antônia decidiu entregar-se à atividade a) Este auditório está reservado para os tenentes-
na roça. coronéis.
d) “Aquele que vem a mim, de modo algum b) A praia amanheceu repleta de estrelas-do-mar.
lançarei fora.” c) O tempo está bom, meninos. Não há
necessidade de guarda-chuvas.
d) Em menos de três meses foram nomeados
122
PROVAS DA AFA 1998 – 2010
guardas-mor. a) I – complemento II - adjunto
nominal; adnominal;
17) Assinale a alternativa correta. III – adjunto IV - adjunto
a) As esquadras luso-brasileiras conquistaram o adnominal; adnominal.
mundo há 500 anos.
b) As borboletas azuis-claras enfeitavam o jardim. b) I – adjunto II - complemento
c) O vendedor ambulante oferecia gravatas adnominal; nominal;
amarelo-palhas aos transeuntes. III – adjunto IV - predicativo do
d) Os casacos abóboras eram ridículos. adnominal; objeto

18) Leia os períodos abaixo: c) I – complemento II - complemento


I- O aluno requereu, junto ao Comando, nominal; nominal;
providências. III – predicativo do IV - predicativo do
II - O comerciante reaveu o dinheiro que objeto; objeto.
havia perdido.
III - Precavei-vos vós, ó pecadores! d) I – adjunto II - complemento
IV - Torna-se imperioso que eu precaveja o adnominal; nominal;
futuro da nossa escola. III – predicativo do IV - adjunto
objeto; adnominal.
Pode-se afirmar que os períodos
a) I, II, III e IV estão corretos. 21) Leia as orações abaixo:
b) I, II, III e IV estão incorretos. I- Tristeza, desilusão, ingratidão, nada o
c) I e III estão corretos e II e IV estão incorretos. derrubava.
d) I, II e III estão corretos e IV está incorreto. II - A cidade de Belo Horizonte é a mais
populosa de Minas Gerais.
19) Analise as orações abaixo quanto à III - O clima de Porto Seguro é sempre muito
classificação do sujeito e, a seguir, assinale a agradável.
alternativa correta.
I- Jamais se precisou de tantas precauções Os termos destacados devem ser sintaticamente
como atualmente. classificados, respectivamente, como:
II - Comentam coisas inacreditáveis a respeito a) aposto, aposto, aposto.
daquele jogador. b) sujeito, adjunto adnominal, adjunto adnominal.
III - Choviam sobre o indesejado visitante c) aposto, adjunto adnominal, adjunto adnominal.
tomates e ovos. d) aposto, aposto, adjunto adnominal.
IV - Durante a madrugada, esfriou bastante.
22) Assinale a alternativa INCORRETA quanto à
a) Classifica-se como indeterminado o sujeito das classificação do período e da oração destacada.
orações I e III. a) Parece que vai chover novamente. (Período
b) Na oração II o sujeito é simples, elíptico. composto por subordinação; oração subordinada
c) Na oração III, o sujeito é composto, pois substantiva objetiva direta.)
apresenta mais de um núcleo. b) Tenho certeza de que vou viajar nas férias.
d) O sujeito da oração IV é inexistente, assim (Período composto por subordinação; oração
como ocorre na oração III. subordinada substantiva completiva nominal.)
c) O fato é que a escola mudou bastante.
20) Leia as orações abaixo e, em seguida, assinale (Período composto por subordinação; oração
a opção correta, em relação aos termos destacados. subordinada substantiva predicativa.)
I- Reverenciai o amor de mãe. d) Antônia saiu cedo e voltou no fim da tarde.
II - Ele tinha muito respeito ao pai. (Período composto por coordenação; oração
III - Contei uma história emocionante. coordenada sindética aditiva.)
IV - Considero a história emocionante.
23) Leia os períodos abaixo transcritos.

123
PROVAS DA AFA 1998 – 2010
I- “Como anoitecesse, recolhi-me e pouco c) “Eis teu romance: fantástico enredo e
depois deitei-me.” personagens, mas estilo pobre e imaturo.”
(Monteiro Lobato) d) “Desde que se confirmaram as ameaças
II - “Poucas eram as árvores que o inverno anteriores os soldados estão alertas.”
despira.”
(Érico Veríssimo) 27) As orações abaixo podem apresentar erros de
III - “Bentinho seria metido no seminário, para concordância nominal, regência verbal ou
não mais se encontrar com Capitu.” colocação de pronomes. Apenas uma alternativa
(Machado de Assis) contempla uma oração gramaticalmente correta.
IV - “Há no país uma legenda, Assinale-a.
que ladrão se mata com tiro.” a) Um laudo duvidoso de uma pseuda-perita não
(Carlos Drummond de Andrade) pode ser levado em consideração.
b) Embora meio cansada pela longa noite de
As orações destacadas devem ser classificadas, vigília, a juíza encontrava forças bastantes para
respectivamente, como subordinada conduzir a audiência.
a) adverbial consecutiva; adjetiva restritiva; c) Por ser esta uma decisão difícil, oxalá o
adverbial causal; substantiva objetiva direta. momento encontre-vos preparados.
b) adverbial causal; adjetiva restritiva; adverbial d) Enquanto seguia a audiência, permaneciam
final; substantiva apositiva. silenciosas a juíza, as partes, as testemunhas e os
c) adverbial consecutiva; substantiva apositiva; advogados.
adverbial causal; substantiva completiva nominal.
d) adverbial causal; substantiva apositiva; 28) Leia os trechos abaixo transcritos.
adverbial final; substantiva apositiva.
I) “Um dia hei de ir embora (eufemismo)
24) Assinale a alternativa em que o emprego da Adormecer no derradeiro
vírgula está INCORRETO. sono...”
a) Gomes, o melhor goleiro do Brasil, defende as Manuel Bandeira
cores do Cruzeiro.
b) Valdete, depois de muita reflexão, devolveu as II) “O inútil choro das tristes (prosopopéia)
provas, ao advogado. águas
c) Homem de pouca fé, acaso duvidas da Enche de mágoas a solidão...”
Providência Divina? Vicente de Carvalho
d) Que cada um realize seu trabalho, o chefe
espera. III)“Não basta inda de dor, ó (preterição)
Deus terrível?”
25) A utilização do acento grave que indica a crase Castro Alves
depende do conhecimento de determinadas regras.
Em relação ao emprego ou não da crase, assinale a IV) “Na chácara abandonada (prosopopéia)
alternativa INCORRETA. O velho poço olha a lua
a) Calçava sapatos à Luís XV. Suspensa no ar, e toda
b) Os alunos permaneceram indiferentes àquele Noite com a sua visão
evento. Na água retratada
c) O avô fechou a porta a chave. Leva a sonhar...”
d) Antes que chegassem à casa, apertou-a nos Alberto de Oliveira
braços.
Em relação as figuras de estilo, pode-se dizer que a
26) Em todas as frases abaixo, a concordância correspondência está correta em:
nominal está correta, EXCETO em: a) I, II, III e IV. c) II, III e IV somente.
a) “Passara a cuidar do jardim depois de muito b) I, II e IV somente. d) I e II somente.
sofrer e aprender por longes terras.”
b) “O governador do estado conversa a sós com o 29) Associe as colunas e, a seguir, assinale a
secretário de educação.” alternativa que apresenta a seqüência correta.
124
PROVAS DA AFA 1998 – 2010
A intensa busca da perfeição
1- Metáfora Nos enganaram. É isso mesmo. Nem tudo
2- Antítese termina em beijo. Quase nada, na verdade. E por
3- Paradoxo que é que insistimos que sim? Talvez eu não
4- Hipérbole devesse atribuir a dúvida a todos nós. Só a mim
5- Prosopopéia mesma já está de bom tamanho.
Faço parte da massa de pessoas viciadas
( ) “...porque outro mar mais copioso em comédias românticas do tipo água-com-açúcar.
Largando de meus olhos a corrente, Sabe que aqueles filmes podem fazer um mal e
Lhe formará meu pranto saudoso”. tanto? Tudo é sempre tão perfeito, tão
Jacinto Freire de Andrade maravilhoso! E, claro, no meio do filme há sempre
uma crise entre o casal e, no último quarto, no
( ) “O prazer com a pena se embaraça; final, tudo começa a se ajeitar. Tudo caminhando
Porém quando um com outro mais porfia, para o grande final. O beijo! A câmera
O gosto corre, a dor apenas passa.” fecha no casal se beijando na chuva ou na praia, ou
Gregório de Matos entre as flores. Ou ainda todas as anteriores ao mês
mo tempo.
( ) “Se és fogo, como passas brandamente? O cara é sempre bonito, gostoso, simpático,
Se és neve, como queima com porfia? sorridente, carinhoso, cheiroso, bom cozinheiro,
Gregório de Matos bem-vestido, inteligente – todas as variáveis
existentes. A moça é sempre maravilhosa,
( ) “Agora que se cala o surdo vento determinada, inteligente, bem-humorada, bem-
E o rio enternecido com meu pranto vestida, delicada, meiga, romântica – e todas as
Detém seu vagaroso movimento...” outras variáveis existentes. Então, só o que posso
Francisco Rodrigues Lobo concluir é que, afinal, nos enganaram.
Nem todas somos maravilhosas, meigas,
( ) “Ardor em firme coração nascido; determinadas e tudo o mais ao mesmo tempo. E,
Pranto por belos olhos derramado; acreditem, nem todos eles são lindos, gostosos e –
Incêndio em mares de água disfarçado; ao mesmo tempo – inteligentes, simpáticos e tudo
Rio de neve em fogo convertido. o mais... E aí, como é que fica a vida real? Como é
Gregório de Matos que nos mostram tudo isso e, depois – como se
fosse um belo prêmio de consolação – nos dão
a) 5, 2, 3, 4, 1 c) 2, 3, 4, 5, 1 isso. Acho que acabo de descobrir por que os
b) 4, 2, 1, 5, 3 d) 3, 5, 1, 2, 4 filmes românticos terminam quando o casal dá o
beijo definitivo. É porque, a partir daí, começa a
30) Leia as frases abaixo: realidade. E eles não vão querer nos mostrar a
I- Juarez nunca gasta mais do que ganha. realidade. Não vende.
II - Ao analisarmos os crustáceos, o Daí, por que esses filmes podem fazer um
carangueijo surge como um dos mais interessantes. mal e tanto. Ficamos esperando a perfeição. E ela
III - Aluno e professor brigaram por causa de deve ser realmente como nos filmes. Não
sua má conduta. aceitamos qualquer amostra barata. E então, um
IV - Há quinhentos anos atrás não havia tem mau hálito, outro uma barriguinha, outro usa
tecnologia tão poderosa. meia de ursinho, outro é um pouco lerdo, outro usa
aparelho, outro gosta do É o Tchan! E, se por
É correto afirmar que há vício de linguagem em: acaso vocês saem – se é que se chega a tal ponto –
a) I, II, III e IV. c) I, III e IV somente. nada de passeios ao luar, velas, beijos debaixo da
b) I e III somente. d) III somente. chuva. No dia seguinte, nada de telefonemas,
mensagens ou e-mails apaixonados, nem mesmo –
muitos menos – flores.
AFA 2005/2006 – Português Então, quando parei para pensar nisso,
depois de uma maratona de três desses retrat os da
perfeição, achei que talvez devêssemos nunca mais
Leia o texto abaixo:
125
PROVAS DA AFA 1998 – 2010
assistir a eles. Greve às comédias românticas! Mas, comédias românticas. Condena suas
no fim, acho que isso não resolveria. Devemos, características, porém, atesta que as pessoas não
isso sim, deixar de ser tão covardes. Levantar do consumiriam um filme que retratasse a realidade.
sofá, desligar a TV e dar a cara a tapa. É tão b) na constante busca da perfeição, exige -se que a
cômodo sentar e dizer “nada é bom o bastante para vida seja como nos filmes, não se tolerando nada
mim” e não correr o risco de se machucar. Nada menos que isso.
mais perfeito do que um amor de verdade, com c) imperfeições e falhas são o que nos tornam
todas as suas falhas e imperfeições. Com todas as perfeitos, sem os artifícios de uma mera fantasia,
brigas, encontros e desencontros, mau hálito e mas plenos de nós mesmos, atraentes por sermos
meias de ursinho. Sabe por quê? Porque, no fim, uma eterna descoberta.
descobrimos que somos perfeitos pelo simples fato d) agir no sentido de dar novos rumos à vida e às
de não o sermos. relações humanas é conseqüência do expor -se
freqüentemente a esse tipo de manifestação
(Annita Veslasque. Publicado no jornal Estado de artística.
Minas.29.03.2005)
4) Há ERRO de colocação pronominal em:
1) Depreende-se do texto que a busca da perfeição, a) Não devo dar-lhe mais explicações.
sobretudo nos relacionamentos humanos, é o b) O aluno se vem aplicando.
objetivo de muitas pessoas. Que passagem do texto c) Em se dizendo coordenador do evento, nos
confirma esta afirmação? ofereceu dois ingressos.
a) “Faço parte da massa de pessoas viciadas em d) Tudo fez para agradar -lhe durante a viagem a
comédias românticas do tipo água-com-açúcar.” Ouro Preto.
b) “Tudo caminhando para o grande final. O beijo!
A câmera fecha no casal se beijando na chuva ou 5) A cena final do seriado “Anos Rebeldes” foi
na praia, ou entre as flores.” mesmo antológica. Num breve momento retratou
c) “Eles não vão querer nos mostrar a realidade. toda uma época, com fidelidade, poesia e arte. O
Não vende.” termo destacado pode ser substituído, sem alterar o
d) “Nada mais perfeito do que um amor de contexto, por:
verdade, com todas as suas falhas e imperfeições.” a) completa c) notável
b) perfeita d) peculiar
2) A narrativa, feita em primeira pessoa, enfatiza
os sentimentos e opiniões da autora. São 6) Assinale a alternativa em que todas as palavras
argumentos que apresenta ao defender a idéia de estão grafadas corretamente.
que as comédias românticas podem fazer “um mal a) Contextar, estender, hesitar, expelir.
e tanto”, EXCETO: b) Pixe, mexer, estrebuchar, chuchu.
a) Por apresentar situações e pessoas ideais, o c) Majestoso, sarjeta, sargento, ogiva.
filme retrata uma mentira, pois a perfeição não d) Catorze, xácara, espletivo, pretexto.
existe na vida real.
b) As comédias românticas sugerem situações 7) Analisando os sufixos das palavras abaixo,
desejáveis, que atraem as pessoas; entretanto, de assinale a alternativa em que o sentido expresso
volta à realidade, essas pessoas são lançadas à entre parênteses NÃO corresponde ao termo ao
situações que evidenciam limitações diversas e qual se refere.
sentem-se frustradas. a) ancoradouro (indicador de lugar)
c) A idealização apresentada nos filmes tornam as b) afetuoso (provido de)
pessoas c) materialismo (ação, estado ou qualidade)
d) A busca da perfeição nas comédias românticas d) realismo (ciência, arte, doutrina)
permite perceber que o mais perfeito é um amor de
verdade, com suas falhas e imperfeições. 8) Só NÃO há processo de derivação imprópria na
palavra destacada em:
3) Da análise do texto, só NÃO se pode inferir que a) Pedro pisava forte sobre as tábuas da
a) há um paradoxo no texto acerca da produção das construção.

126
PROVAS DA AFA 1998 – 2010
b) Julgando estar muito longe do acampamento, b) O pronome “me” exerce a função de objeto
resolveu falar alto a fim de que o escutassem. indireto da forma verbal “flagrei”, função típica
c) Mandei o cujo me procurar na esquina da rua dos pronomes do caso oblíquo.
das Flores. c) O pronome “você” exerce a função de objeto
d) Registrou-se às doze horas a renúncia do indireto da forma nominal do verbo “pensar”,
presidente. função típica dos pronomes do caso oblíquo.
d) O pronome “te” exerce a função de objeto
9) Analise o slogan de um comercial, abaixo indireto da forma nominal do verbo “dizer”,
transcrito. função típica dos
“Vem pra Caixa você também”. pronomes do caso oblíquo.
Em relação ao emprego do verbo “vir”, é correto
afirmar que está conjugado 14) Analise as orações abaixo:
a) corretamente no presente do indicativo. I - Afora tu, ninguém mais admite esse
b) corretamente no imperativo afirmativo. comportamento.
c) incorretamente no imperativo afirmativo. II - A respeito de gramática, todas as informações
d) incorretamente no presente do indicativo. são importantes.
III - Ela está esperando desde cedo.
10) Em relação ao plural dos substantivos IV - Nem sempre se pode vestir conforme os
compostos, assinale a alternativa correta. costumes.
a) Os guardas-florestais eram escolhidos mediante Pode-se afirmar que
rigorosa seleção. a) na oração I o termo destacado é preposição
b) Os guardas-chuvas pretos nunca saem de moda. essencial.
c) O quarto era tão grande que abrigava três b) há preposição acidental nas frases I e IV.
guardasroupas. c) o termo em destaque na oração III é advérbio.
d) Eles não eram vigias, eram guarda-noturnos. d) existe locução adverbial na frase II.

11) Assinale a alternativa que apresenta a classe 15) Leia a orientação dada por um jornal aos
gramatical do termo destacado na oração abaixo: estudantes que, ao participarem de um vestibular,
“Parece que a cada momento se me rebenta o fariam uma prova de redação.
coração.” Como escrever
a) pronome relativo. Olho vivo para não maltratar o Português. Preste
b) conjunção subordinativa integrante. atenção ao enunciado. Se fugir do tema, copiar o
c) pronome indefinido. texto apresentado ou fazer uma narração [...], a
d) conjunção subordinativa temporal. redação será anulada.
Identifique a passagem em que o redator do texto
12) “Eu não tinha estas mãos sem força, acaba, ele próprio, “maltratando o português”.
tão paradas e frias e mortas...” a) “Preste atenção ao enunciado.”
As palavras destacadas nos versos acima devem b) “Se fugir do tema, copiar o texto apresentado ou
ser classificadas, respectivamente, como: fazer uma narração [...].”
a) advérbio, preposição, advérbio. c) “Olho vivo para não maltratar o português”.
b) pronome, preposição, advérbio. d) “A redação será anulada.”
c) advérbio, conjunção, pronome.
d) pronome, conjunção, pronome. 16) Em relação à forma nominal do verbo
destacado na tirinha abaixo, assinale a alternativa
13) “Eu me flagrei pensando em você correta.
em tudo que eu queria te dizer”
(Zeca Baleiro)
Com relação aos pronomes pessoais destacados
nos versos acima, é INCORRETO afirmar que
a) O pronome “eu” exerce a função de sujeito das
formas verbais “flagrei” e “queria”, função típica
dos pronomes do caso reto.
127
PROVAS DA AFA 1998 – 2010
a) Apresenta o fato verbal em processo, isto é, b) A presença do artigo confirma ser a palavra
enquanto ele está se desenvolvendo ao longo do “verão” um substantivo. Sua função sintática é
tempo. adjunto adverbial de tempo. “Rosa”, nas duas
b) Exprime o fato verbal em si, sem limitá-lo no ocorrências, é substantivo
tempo, isto é, sem indicar seu começo ou seu fim. precedido do artigo que lhe atribui gênero
c) Apresenta o resultado do fato verbal, ou seja, o masculino.
fato concluído. Exerce a função de sujeito dos verbos chegou e
d) Determina uma diferença de sentidos entre o impera.
conceito de uma atividade dinâmica e uma atitude c) A palavra “verão” associa-se à preposição e ao
estática. artigo, compondo uma locução adjetiva. “Rosa” é
adjetivo e exerce a função sintática de adjunto
17) Apenas uma das alternativas abaixo está adnominal, no segundo parágrafo, e sujeito, no
absolutamente correta quanto à regência verbal. terceiro parágrafo.
Assinale-a. d) Tanto “verão” como “rosa”, nas duas
a) Que filme você assistiu ontem à noite? ocorrências, são substantivos e exercem,
b) Vi e gostei do filme. respectivamente, as funções
c) O político trovejava insultos. sintáticas de complemento nominal e sujeito.
d) Não desobedeça este regulamento.
19) Aponte a alternativa em que o sentido expresso
Leia: entre parênteses NÃO corresponde à regência
MODA estabelecida.
O verão é rosa. a) Quero uma cópia deste documento. (desejar, ter
Roupas, sapato, batom – a cor, nos mais vontade ou intenção)
variados tons, domina tudo. b) O irmão lhe queria mais do que a si mesmo.
Rosa-choque, rosa-light, pink, sorbet, (amar, gostar de)
fúcsia, chá, tangerina, rosé, salmão. A variedade c) A assembléia resolveu remeter a decisão do
de nomes é tão impressionante quanto o impacto acordo para o mês seguinte. (enviar, destinar)
da invasão cor-de-rosa que começa a dar o tom do d) O governo deve proceder aos ajustes fiscais
verão. (...) necessários. (realizar, dar início a)
Clarinho para as meninas, mais chocantes
para as adolescentes, o rosa, cor oficial de toda 20) Apenas uma das alternativas abaixo apresenta
patricinha de Beverly Hills a Belo Horizonte, ERRO de concordância verbal. Assinale-a.
também chegou ao guardaroupa das senhoras a) Davam três horas quando ela chegou ao
maduras – acompanhado de um segredo das titias: dentista.
o tom rejuvenesce. (...) b) Hoje existem poucas chances de conquistarmos
Além de roupas e sapatos, o rosa impera a liderança do grupo.
nos lançamentos de bolsas, cintos, colares, brincos, c) “Um gesto, um olhar, um sorriso bastariam para
pulseiras e maquiagem em geral. (...) que o rapaz se sentisse feliz.”
(Revista Veja, ano 36, 24 set 2003.p.82 d) Faltou víveres no acampamento da escola.
Editora Abril)
21) Observe as frases abaixo:
18) No corpo da notícia, tem-se uma ocorrência da I - Os povos não podem viver à mercê de
palavra verão e duas da palavra rosa (segundo e ditadores.
terceiro parágrafos). Que alternativa apresenta e II - Os marinheiros ficaram felizes, pois
justifica corretamente a função morfológica e resolveram ir a terra.
sintática dessas palavras? III - Quando o maestro falta ao ensaio, o flautista
a) A palavra “verão” é um substantivo exercendo a faz as vezes de regente.
função sintática de adjunto adnominal. A palavra IV - “Tudo vale a pena quando a alma não é
“rosa”, nas duas ocorrências é substantivo. pequena...”
Desempenha função de sujeito, estabelecendo É correto afirmar que
concordância com as formas verbais chegou e a) a frase I está correta, pois nas locuções
impera. prepositivas a crase é obrigatória.
128
PROVAS DA AFA 1998 – 2010
b) a frase II está incorreta porque a palavra terra c) Sabia de cor mil e trezentas orações.
significa oposto de mar e, por isso, a crase é d) As propostas são para o segundo semestre.
obrigatória.
c) a frase III está incorreta porque a expressão 25) Leia este poema, de Adélia Prado, e responda a
destacada não é locução prepositiva e, por isso, questão proposta.
deve ocorrer crase.
d) a expressão “vale a pena” (no sentido de valer o Explicação de poesia sem ninguém pedir
sacrifício, o esforço) exige crase pois o “a” é artigo Um trem-de-ferro é uma coisa mecânica,
definido. Por isso, a frase IV está incorreta. Mas atravessa a noite, a madrugada, o dia
Atravessou minha vida,
22) Analise as orações abaixo: Virou só sentimento
I - Eles talvez nos apóiem novamente. (Poesia reunida. São Paulo: Siciliano, 1991. p.48)
II - Meus pais convidaram-te para a festa.
III - Meus amigos, preparem-se para a grande Ao lado do sentido comum de “veículo
reunião. ferroviário”, a
IV - Deus nos livre desse problema. expressão trem-de-ferro, no poema, tem também o
Quanto ao posicionamento dos pronomes oblíquos sentido conotativo de
destacados, pode-se afirmar que a) lembrança, sentimento. c) força, poder.
a) a frase I está correta porque, segundo a norma b) tristeza, angústia. d) solidão, vazio interior.
culta, ocorre próclise quando, antes do verbo,
aparece um pronome indefinido. 26) - Leia:
b) há erro de colocação na frase II porque ocorre No espelho da linguagem encontramos a
próclise quando o verbo não está iniciando a nós
oração. mesmos por meio da fala, da escrita e do
c) a ênclise se justifica na frase III porque esse pensamento. E o desafio da linguagem, expressão
posicionamento é opcional com formas verbais do e apreensão das idéias está presente em cada
imperativo afirmativo. momento da nossa vida. Lutamos pela palavra e
d) a frase IV está correta porque nas expressões aprendemos a negociar com ela. Dominar a
optativas o pronome oblíquo deve ficar linguagem é um poder – um poder que precisamos
obrigatoriamente antes do verbo. adquirir e usar da melhor forma possível.
(Extraído de “Língua Portuguesa”. Harry Vieira
23) As orações subordinadas substantivas Lopes e Editora do Brasil: São Paulo. p 97)
destacadas no período abaixo estão corretamente
classificadas, conforme indicado entre parênteses, Da análise do texto, só NÃO se pode inferir que
EXCETO: a) a maioria das situações comunicativas que
a) Convém que nos entendamos melhor. (objetiva vivemos tem uma intenção avaliativa e de
direta) convencimento do
b) Revelara apenas isto: jamais entendera interlocutor de que nossas posições são as
Madalena. (apositiva) melhores.
c) Ficou com a impressão de que jamais b) o uso da linguagem também indica o que
conheceria Jéssica inteiramente. (completiva pensamos ou achamos que pensamos sobre o
nominal) mundo.
d) E, falando assim, compreendo que perco o c) usamos os símbolos sonoros com naturalidade,
tempo. (objetiva direta) esperando que nossos ouvintes nos compreendam,
nos respeitem e, de preferência, façam o que
24) Assinale a alternativa em que o numeral estamos querendo.
empregado na frase tem valor indeterminado e d) o poder de persuasão da linguagem é tão intenso
enfático. que, ainda que o interlocutor não deseje ouvir, ela
a) Cerca de dois milhões de crianças passam fome alcançará seu objetivo.
na América Latina.
b) “Do alto dessas pirâmides quarenta séculos nos 27) Existem figuras de linguagem que
contemplam.” potencializam o ritmo, a musicalidade dos textos.
129
PROVAS DA AFA 1998 – 2010
Analise os exemplos abaixo e assinale a alternativa a) 1-2-3-4 c) 3-1-2-4
em que a figura expressa entre parênteses NÃO b) 2-1-3-4 d) 3-2-4-1
corresponde ao enunciado ao qual se refere.
a) “Felizmente era tempo de jabuticabas. No sítio Leia a reportagem abaixo transcrita, extraída da
de Dona Benta havia vários pés, mas bastava um edição nº 347 da revista “Época” e, a seguir,
para que todos se regalassem até enjoar. responda as 2 questões a ela referentes.
Justamente naquela semana as jabuticabas tinham Por um Davi limpinho
chegado ‘no ponto’ e a menina não fazia outra A limpeza da escultura Davi, (1) de Michelangelo,
coisa senão chupar jabuticabas. Volta e meia (2) que tem mais de 500 anos, foi concluída em
trepava à árvore, que nem uma macaquinha. maio e já não está tão brilhante. Especialistas
Escolhia as mais bonitas, punha-as entre os dentes acreditam que a sujeira e a umidade encontradas na
e tloc! E depois do tloc, uma engolidinha de caldo obra são levadas pelos turistas que a visitam em
e pluf! – caroço fora. E tloc, pluf – tloc, pluf, lá Florença. Agora, eles estudam maneiras de
passava o dia inteiro na árvore.” conservar a obra de arte sem ter de colocá-la atrás
(onomatopéia) de um vidro. Algumas das opções são (3) a
b) “Que falta nesta cidade?.................................. instalação de um jato
Verdade. de ar sobre a escultura, carpetes especiais para
Que mais por sua desonra?.............................. “limpar” os turistas ou ainda um corredor arejado
Honra. para remover a sujeira dos sapatos e roupas dos
Falta mais que se lhe ponha?........................... visitantes.
Vergonha.
(...) 29) Com relação à estrutura “1”, pode-se dizer que
E que justiça a resguarda?............................... a) é uma oração.
Bastarda. b) é uma frase.
É grátis distribuída?......................................... c) é um período simples.
Vendida. d) não é nem uma frase, nem uma oração.
Que tem, que a todos assusta?.........................
Injusta.” 30) Sintaticamente, as estruturas “2” e “3” devem
(eco) ser classificadas, respectivamente, como orações
c) “Viola violeta violenta violada óbvia subordinadas
vertigem...” a) adjetiva restritiva e substantiva subjetiva.
(aliteração) b) substantiva apositiva e substantiva predicativa.
d) “Aquela que era moça c) adjetiva explicativa e substantiva predicativa.
no mar vira peixe d) substantiva apositiva e substantiva subjetiva.
mas peixe sem mexer
peixe que não nada. Nada!”
(anáfora) AFA 2006/2007 – Português
28 ) Associe as frases abaixo aos vícios de Texto I
linguagem que apresentam. A INVEJA
1 – Soube que ele surpreendeu o ladrão em sua Tomás de Aquino define a inveja como “a
casa. tristeza por não possuir o bem alheio”. Invejam-se
2 – Corrija já aquelas provas. a cor dos olhos, o tom da voz, a erudição, os
3 – Há alguns dias que eu o ouço cantar à hora do títulos, a função, a riqueza ou as viagens de
almoço. outrem. “Onde há inveja, não há amizade”,
4 – Desde cedo já apresentava ares de moço alertava Camões.
fremoso. O invejoso é um derrotado. Perdeu para a
( ) Hiato sua auto-estima. Lamenta, no íntimo, ser quem é e
( ) Anfibologia nutre a fantasia de que poderia ter sido outra
( ) Colisão pessoa. O inimigo do invejoso é ele próprio.
( ) Arcaísmo (...)
A seqüência correta é
130
PROVAS DA AFA 1998 – 2010
A inveja é a tristeza de ser o que se é. A Texto II
advogada sonha que poderia ter sido atriz, o Contente Está Quem Assim se Julga de Si
engenheiro imagina-se no lugar do empresário, o Mesmo
rapaz chora por não pilotar um carro de Fórmula
l. Mal sabem que o invejado também sofre de A abastança e a indigência dependem da
invejas, pois o desejo é insaciável. Centrado nos opinião de cada um; e a riqueza não mais do que a
bens objetivos, escraviza o ser humano. glória, do que a saúde têm tanto de beleza e de
(...) prazer quanto lhes atribui quem as possui. Cada
Só quem se gosta não tem inveja. É capaz, qual está bem ou mal conforme assim se achar.
portanto, de reconhecer e aplaudir o sucesso Contente está não quem assim julgamos, mas
alheio. Faz sua a alegria do outro. quem assim julga de si mesmo. E apenas com isso
(Frei Betto, O Estado de S. Paulo, 1998.) a crença assume essência e verdade.
(Michel de Montaigne)
1) Considerando o Texto I, ordene corretamente as
idéias do citado fragmento e, a seguir, assinale a 4) Da leitura do Texto II, pode-se depreender que .
alternativa correta. a) a abastança e a indigência dependem da opinião
( ) O desejo escraviza o ser humano. dos demais.
( ) Pessoas com auto-estima não têm inveja. b) riqueza, glória, saúde, beleza e prazer são
( ) O invejoso é inimigo de si mesmo. medidos de acordo com os critérios de cada um.
( ) O invejoso deseja possuir o bem alheio. c) está nas mãos do indivíduo o direito de
a) 1 – 3 – 4 – 2 c) 2 – 1 – 3 – 4 conquistar situações favoráveis para si.
b) 3 – 4 – 2 – 1 d) 4 – 2 – 1 – 3 d) depende do ser humano encontrar alegria
naquilo que é ou possui.
2) No Texto I aparecem as seguintes
considerações para o termo inveja: 5) O trecho abaixo foi reescrito de diversas
I - “a tristeza de não possuir o bem alheio.” maneiras. Assinale a opção em que essa reescritura
II - “Onde há inveja, não há amizade.” mantém a mensagem original.
III - “A inveja é a tristeza de ser o que se é.”
Quanto a essas considerações, pode-se afirmar que “... a riqueza não mais do que a glória, do que a
a. saúde têm tanto de beleza e de prazer quanto lhes
a) primeira e a segunda são excludentes. atribui quem as possui.”
b) primeira e a terceira são contrastantes.
c) segunda e a terceira são relacionáveis. a) A riqueza menos do que a glória e a saúde têm a
d) primeira e a segunda são complementares. mesma beleza e prazer que lhes atribui quem as
possui.
3) Assinale a alternativa INCORRETA relativa ao b) A glória tanto quanto a saúde e menos do que a
Texto I. riqueza detêm a beleza e o prazer que lhes são
a) Verifica-se a presença de voz passiva no emprestados por seus possuidores.
primeiro parágrafo. c) Só quem possui a riqueza, a glória, a saúde,
b) Tanto a oração “de que poderia ter sido outra tanto quanto beleza e prazer é que sabe o valor que
pessoa” quanto a expressão “do invejoso”, no esses atributos possuem.
segundo parágrafo, possuem a mesma função d) A beleza e o prazer conferidos pela riqueza
sintática. tanto quanto pela glória e a saúde dependem de
c) Se as orações “Onde há inveja, não há amizade” quem as possui.
(. 4) tivessem a ordem invertida e não fosse usada
6) Leia o seguinte trecho do Texto II: “Contente
vírgula entre elas, não sofreriam qualquer alteração está quem assim se julga de si mesmo.” A frase
semântica ou sintática. destacada pode ser substituída, sem prejuízo de seu
d) As orações “Centrado nos bens objetivos, sentido e da língua padrão escrita, por
escraviza o ser humano.” (. 14 e 15) referem-se à a) dessa forma sentencia por si mesmo.
palavra “desejo”. b) igualmente se demonstra consigo mesmo.
c) desse modo considera-se a si mesmo.
131
PROVAS DA AFA 1998 – 2010
d) assim sendo tem em conta para si mesmo. “De todas as características que são vulgares na
natureza humana a inveja é a mais desgraçada; o
7) Assinale a alternativa INCORRETA. invejoso não só deseja provocar o infortúnio e o
a) No último parágrafo do Texto I, a conjunção provoca sempre que o pode fazer impunemente,
“portanto” (. 17) pode ser substituída por “no como também se torna infeliz por causa da sua
inveja.”
entanto” sem que o período sofra qualquer Marque a afirmativa, a respeito do fragmento
alteração. acima, que está
b) No Texto II, os pronomes lhes e as (. 3) INCORRETA.
desempenham o papel de complementos verbais. a) Os verbos constantes do período (são, é, torna)
c) A expressão “apenas com isso” (. 6) (Texto II) nos transmitem a idéia de algo em processo, que
perdura.
poderia ser isolada por vírgulas. b) Que, sempre que e como também exercem
d) De acordo com o Texto II, o ser tem íntima função de elementos coesivos.
dependência do crer. c) O fragmento reforça a idéia do seguinte
provérbio: “A inveja toma todas as formas para
Texto III ferir.”
O Desgaste da Inveja d) O período iniciado após o ponto-e-vírgula (ƒÜ.
De todas as características que são vulgares 2) explica a afirmativa do período anterior.
na natureza humana a inveja é a mais desgraçada;
o invejoso não só deseja provocar o infortúnio e o 10) Após a leitura atenta dos três primeiros textos
provoca sempre que o pode fazer impunemente, da prova, marque as afirmativas abaixo com (V)
como também se torna infeliz por causa da sua verdadeiro ou (F) falso. Em seguida, assinale a
inveja. Em vez de sentir prazer com o que possui, opção correspondente.
sofre com o que os outros têm. Se puder, priva os ( ) Os três textos apresentam “antídotos” contra a
outros das suas vantagens, o que para ele é tão inveja: auto-estima, contentamento e admiração ao
desejável como assegurar as mesmas vantagens próximo.
para si próprio. Se uma tal paixão toma proporções ( ) O conceito de inveja do Texto III é mais
desmedidas, torna-se fatal a todo o mérito e “corrosivo” que o do Texto I.
mesmo ao exercício do talento mais excepcional. ( ) De acordo com os três textos, o invejoso é um
(...) infeliz, um derrotado.
Afortunadamente, porém, há na natureza ( ) Conforme o Texto II, abastança e indigência
humana um sentimento compensador, chamado não se situam nas circunstâncias, no concreto. Ao
admiração. Todos os que desejam aumentar a contrário, elas são relativas, subjetivas, pertencem
felicidade humana devem procurar aumentar a ao terreno do abstrato.
admiração e diminuir a inveja. ( ) O Texto II afirma que cada criatura é artífice de
(Bertrand Russell, in ‘A Conquista da seu próprio estado de espírito.
Felicidade’.) a) V – V – V – F – V c) V – F – V – V – F
b) V – V – F – V – V d) F – V – V – F – F
8) Da leitura atenta do Texto III, só NÃO se pode
inferir que 11) Com relação aos Textos I, II e III pode-se
a) o invejoso tem olhos para a falta e não para a afirmar que o Texto
plenitude. a) I comprova o Texto II.
b) para o invejoso, impingir sofrimentos ao b) II suplementa o Texto I.
invejado proporciona tanto prazer quanto c) III contradiz o Texto II.
conquistar o objeto de sua cobiça. d) II exemplifica o Texto III.
c) a inveja pode causar a destruição de seu
portador, a menos que ele possua algum talento 12) Observe os fragmentos e analise-os.
extraordinário. I - “eu sei, cê não pôde ser o que sempre quis então
d) sobrepor à inveja a admiração pode contribuir não suporta ver ninguém feliz.” (Ultraje a Rigor)
para sanar essa limitação. II - “A inveja é a homenagem que a inferioridade
tributa ao mérito.” (provérbio)
9) Observe o período abaixo do Texto III.
132
PROVAS DA AFA 1998 – 2010
III - “Não só quem nos odeia ou nos inveja c) Qualquer uma das palavras a seguir pode
Nos limita e oprime; quem nos ama Não menos substituir “incitaram” (versículo 11): instigaram,
nos limita.” estimularam, impeliram, açularam.
(Ricardo Reis) d) No versículo 9, ficaria também adequada a
redação “Quereis que solte-vos o Rei dos Judeus?”
Marque a alternativa correta.
a) Em I e II prevalece a linguagem coloquial. 15) Só NÃO é correto afirmar, em relação ao
b) I e II apresentam pontos de vista opostos. Texto IV que
c) O poema de Ricardo Reis tem sua mensagem a) o trecho do evangelho de Marcos exemplifica o
centrada na limitação que nos é imposta por quem tipo de inveja apontado no Texto III.
nos odeia ou inveja. b) o trecho “por inveja”, no versículo 10, poderia
d) A oração “portanto o invejado poderia sentir-se vir cercado de vírgulas.
até lisonjeado” completaria adequadamente o c) “solto”, no versículo 11, não poderia ser
fragmento II (provérbio). substituído por soltado.
d) o trecho “e, açoitado Jesus”, no versículo 15,
Texto IV pode ser substituído por “e tendo açoitado Jesus”
8 E a multidão, dando gritos, começou a pedir sem prejuízo semântico.
que fizesse como sempre lhes tinha feito.
9 E Pilatos lhes respondeu, dizendo: Quereis que Texto V
vos solte o Rei dos Judeus? ROMANCE XXVIII ou DA DENÚNCIA DE
10 Porque ele bem sabia que por inveja os JOAQUIM
principais dos sacerdotes o tinham entregado. SILVÉRIO
11 Mas os principais dos sacerdotes incitaram a No palácio da Cachoeira,
multidão para que fosse solto antes Barrabás. com pena bem aparada,
12 E Pilatos, respondendo, lhes disse outra vez: começa Joaquim Silvério
Que quereis, pois, que faça daquele a quem a redigir sua carta.
chamais Rei dos Judeus? De boca já disse tudo
13 E eles tornaram a clamar: Crucifica-o. quanto soube e imaginava.
14 Mas Pilatos lhes disse: Mas que mal fez? E
eles cada vez clamavam mais: Crucifica-o. Ai, que o traiçoeiro invejoso
15 Então Pilatos, querendo satisfazer a multidão, junta às ambições a astúcia.
soltou-lhe Barrabás e, açoitado Jesus, o entregou Vede a pena como enrola
para ser crucificado. arabescos de volúpia,
(Marcos, 15,8:15) entre as palavras sinistras
desta carta de denúncia!
13) “... começou a pedir que fizesse como sempre
lhes tinha feito.” Que letras extravagantes,
O trecho destacado do Texto IV pode ser com falsos intuitos de arte!
substituído, sem prejuízo de sentido, por Tortos ganchos de malícia,
a) fizesse sempre como havia feito por eles. grandes borrões de vaidade.
b) sempre fizesse como tinha feito para eles. Quando a aranha estende a teia,
c) fizesse como havia sempre feito com eles. não se encontra asa que escape.
d) fizesse como tinha feito sempre para eles.
Vede como está contente,
pelos horrores escritos,
14) Uma das afirmativas abaixo, sobre o Texto IV, esse impostor caloteiro
está INCORRETA. Assinale-a. que em tremendos labirintos
a) No versículo 11, a expressão “antes” tem valor prende os homens indefesos
semântico semelhante a “preferencialmente”. e beija os pés aos ministros!
b) A palavra demagogia pode ser aplicada com (...)
bastante precisão à atitude de Pilatos.
(No grande espelho do tempo,
133
PROVAS DA AFA 1998 – 2010
cada vida se retrata: Que letras extravagantes,
os heróis, em seus degredos com falsos intuitos de arte!
ou mortos em plena praça; ....................
– os delatores, cobrando
o preço das suas cartas...) Quando a aranha estende a teia,
(Cecília Meireles) não se encontra asa que escape
....................
16) O fragmento de Cecília Meireles (Texto V) prende os homens indefesos
revela a denúncia de Joaquim Silvério dos Reis. e beija os pés aos ministros!
Relacione as idéias constantes em cada estrofe aos
comentários abaixo e, a seguir, assinale a As conjunções não devem ser vistas como meros
alternativa correta. conectivos (palavras aparentemente sem carga
( ) Felicidade por prestar favores a ministros. significativa que ligam outras palavras ou
( ) Local onde se redigiu a carta. orações), mas como elementos capazes de
( ) Palavras maliciosas. estabelecer relações de significado.
( ) Cada pessoa exerce um papel na sociedade. Considerando as palavras destacadas, aponte a
( ) Comporta-se como um invejoso. alternativa que estabelece a relação correta.
a) 5 – 4 – 2 – 3 – 1 c) 2 – 3 – 4 – 1 – 5 a) Que (1º verso) e quando (3º verso) expressam a
b) 3 – 2 – 1 – 4 – 5 d) 4 – 1 – 3 – 5 – 2 mesma relação temporal.
b) E (6º verso) estabelece relação de adição entre
17) “Vede como a pena enrola as orações.
arabescos de volúpia,” c) Que (1º verso) e que (4º verso) estabelecem a
Os versos retirados do Texto V podem ser mesma relação conclusão.
reescritos, em termos atuais, sem que haja perda ou d) Que (1º verso) tem função de retificação.
alteração de sentido conforme alternativa:
a) Contemplem como a caneta traça rabiscos de 20) Considerando a última estrofe do Texto V,
prazer. analise as afirmativas abaixo.
b) Olhem o prazer com que a caneta traça os I - Os parênteses indicam uma reflexão acerca das
rabiscos. conseqüências das atitudes heróicas ou vis dos
c)Observem como os rabiscos são prazerosamente homens.
escritos. II - A presença das reticências pressupõe o caráter
d) Vejam o prazer dos rabiscos traçados pela inconcluso das reflexões feitas acerca do tempo e
caneta. da morte.
III - A presença da vírgula, após o termo delatores,
18) Analise as afirmativas sobre o Texto V. indica supressão de um termo facilmente
I - Todas as orações que compõem a quarta estrofe perceptível.
poema têm como sujeito “esse impostor caloteiro”. IV - A presença do travessão, após um ponto-e-
II - Na segunda estrofe “às ambições a astúcia” vírgula, reforça a idéia de exclusão a que os
encontram- dois complementos verbais. delatores são submetidos.
III - Esse trecho do Romanceiro da Inconfidência é Estão corretas somente
todo dedicado a descrever o prazer, a volúpia, com a) I e III. c) II e III.
que Silvério dos Reis teria escrito a carta de b) I e IV. d) II e IV.
delação famoso movimento.
IV - A terceira estrofe possui apenas frases 21) Considerando-se o texto do evangelista Marcos
nominais, ao passo que na segunda não há uma (Texto IV) e texto de Cecília Meireles (Texto V),
frase nominal sequer. pode-se inferir que a) em ambos os textos, os
Estão corretas somente delatores fizeram suas denúncias movidos por
a) I, III e IV. c) I, II e III. sentimentos que mesclam inveja e ambição.
b) II, III e IV. d) II e III. b) o delator é incitado pelos detentores do poder à
denúncia oral escrita, no Texto V.
19) Leia os fragmentos de Cecília Meireles, Texto c) em Cecília Meireles, o algoz insinua para os
V. delatores que pena cabível é a capital.
134
PROVAS DA AFA 1998 – 2010
d) em ambos os textos, o resultado da delação é, parece, que tudo o que os outros têm, nós o
obrigatoriamente, a morte em praça pública. merecíamos melhor; por isso olhamos com
desgosto para as cousas alheias, por nos parecer,
Texto VI que deviam ser nossas; que é isto senão vaidade?
Não só quem nos odeia ou nos inveja Não podemos ver luzimento em outrem, porque
imaginamos, que só em nós é próprio: cuidamos,
Não só quem nos odeia ou nos inveja que a grandeza só em nós fica sendo natural, e nos
mais violenta: o esplendor alheio passa no nosso
Nos limita e oprime; quem nos ama conceito por desordem do acaso, e por miséria do
Não menos nos limita. tempo. Quem diria aos homens, que no mundo há
Que os deuses nos concedam que, despido outra cousa mais do que fortuna, e que, nas honras,
De afetos, tenha a fria liberdade há predestinação?
Dos píncaros sem nada. (Matias Aires, in 'Reflexões Sobre a Vaidade dos
Quem quer pouco, tem tudo; quem quer nada Homens e Carta Sobre a Fortuna'.)
É livre: quem não tem, e não deseja,
Homem, é igual aos deuses 24) Assinale a alternativa correta, de acordo com o
(Ricardo Reis) Texto VII.
a) Luzimento é um vocábulo formado por
22) Da leitura do Texto VI, pode-se inferir que a/o derivação prefixal e refere-se a algo que produz
a) ausência de sentimentos fortes liberta o homem claridade.
das opressões. b) A análise da inveja nesse texto parte de um
b) amor é limitante, porém em menor grau que o pressuposto totalmente diverso dos textos
ódio e a inveja. anteriores.
c) homem é igual aos deuses na medida em que se c) Predestinação é um vocábulo de formação
liberta da opressão de quem o ama. parassintética e refere-se ao efeito de determinar
d) querer pouco e o querer nada constituem a antecipadamente. d) O último período do texto é
ausência de opressão. estruturado com apenas uma oração.

23) Considerando o poema de Ricardo Reis, 25) Ao analisar o Texto VII, pode-se afirmar que
analise as proposições abaixo. a) nós, só e têm recebem o sinal gráfico pelo
I - Ricardo Reis, numa linguagem clássica, remete- mesmo motivo.
nos à idéia de que “a inveja combate sempre a b) Zuenir Ventura, em Inveja – Mal Secreto, diz o
elevação”. oposto ao texto de Matias Aires.
II - Píncaros tem como sinônimo: cume, auge, “O invejoso tem um desgosto de si mesmo,
apogeu e estabelece relação antonímica com complexo de inferioridade, carência de auto-
pisotear. estima. Ele vive uma insatisfação permanente,
III - Em “Que os deuses nos concedam que, porque depende do sofrimento alheio para sentir
despido de afetos, tenha a fria liberdade dos prazer.”
píncaros sem nada.”, o autor emprega uma frase c) as expressões a seguir complementam termos
optativa diante de uma hipótese ... se despido de nominais: desordem do acaso, injustiça da fortuna
afetos... e diria aos homens.
Está(ão) correta(s) d) Inveja e Vaidade possuem relação sinonímica,
a) I, II e III. c) I e III apenas. além de terem o mesmo número de sílabas e a
b) I e II apenas. d) II apenas. mesma posição da sílaba tônica.

Texto VII 26) Relacione a 2ª coluna à 1ª e, a seguir, assinale


Inveja é Vaidade a alternativa correta.
O que chamamos inveja, não é senão 1ª coluna
vaidade.Continuamente acusamos a injustiça da (1) Invejoso
fortuna (sorte), e a consideramos ainda mais cega (2) Invejado
do que o amor, na repartição das felicidades.
Desejamos o que os outros possuem, porque nos 2ª Coluna
135
PROVAS DA AFA 1998 – 2010
( ) Possui elevada estima, sente-se Com teu esforço insigne as tens ganhado,
vitorioso. No teu escudo eternamente estejam
( ) Sente-se derrotado e infeliz por ser o Por elas conhecido, e afamado
que é. Serás entre os Heróis, que mais se invejam,
( ) Deseja provocar infortúnio aos outros. Que bem merece ter armas por glória.”
( ) Une a ambição à astúcia.
Relacionado os fragmentos acima, assinale a
a) 1 – 2 – 2 – 1 c) 2 – 1 – 1 – 1 alternativa correta.
b) 1 – 2 – 1 – 2 d) 2 – 2 – 2 – 1 a) Zuenir Ventura dá ênfase à inveja que é citada
por Gregório de Matos.
27) As afirmativas abaixo referem-se à tirinha de b) Gregório de Matos em tom de exaltação ao seu
Allan Sieber. Assinale-as com (V) verdadeiro ou herói emprega o sentimento da inveja.
(F) falso e, em seguida, marque a alternativa c) Para Zuenir, a inveja pode ser manifestada em
correspondente. qualquer atitude ou pessoa, até mesmo num herói
barroco.
d) Gregório de Matos ilustra a inveja referida por
Zuenir Ventura — o herói é um invejoso.

29) O escritor Zuenir Ventura em sua obra Inveja


– Mal Secreto diz
que
( ) As expressões faciais dos personagens denotam “O antídoto contra a inveja está no amor. E você
a impassibilidade e a intolerância típicas da pode extrair o amor da inveja, como tira o soro do
timidez e da inveja respectivamente. veneno da cobra. Todo mundo carrega um pouco
( ) O personagem Invejoso é incapaz de desse veneno. A melhor maneira de lidarmos com
compreender a posição física e a situação ele é admitirmos que fomos inoculados.”
emocional do personagem Tímido.
( ) A ausência de reação apresentada pelo A conclusão que se pode tirar desse fragmento é
personagem Tímido se deve exclusivamente à que
agressividade demonstrada pelo Invejoso. a) todos podemos combater a inveja depende
( ) O muro de tijolos que separa os dois apenas de nós.
personagens simboliza b) antes invejado do que lastimado.
a distância e as dificuldades de relacionamento de c) a inveja “é a admiração da malevolência”.
ambos. d) a inveja é “a homenagem que a inferioridade
a) F – V – V – F c) V – F – F – V tributa ao mérito”.
b) F – V – F – V d) V – F – V – F
30) O teólogo Santo Tomás de Aquino listou
28 - Zuenir Ventura em Inveja – Mal Secreto algumas metáforas sobre a inveja. Numere a
escreve: segunda coluna observando a explicação para a
metáfora da primeira coluna. Em seguida, assinale
“A inveja não se manifesta só pelos olhos. Às a opção correta.
vezes vemos que uma pessoa está com inveja da 1 - A inveja é podridão
gente por uma determinada frase. Ou por um 2 - Roer-se de inveja
silêncio? Com certeza. A inveja também aparece 3 - O arroto da inveja
num gesto, num suspiro, num tom de voz, numa 4 - O espinho da inveja
expressão facial.” 5 - A inveja avinagra
( ) A inveja não pode ser sufocada por muito
Gregório de Matos também se refere a um herói tempo, pois ela acaba por vir à tona e é
invejado no fragmento: desagradável.
( ) A inveja degrada as relações humanas e revela o
“Esse despojo, ó Herói sublimado, lado fraco daquele que a sente.
Como de armas te foi, armas te sejam,
136
PROVAS DA AFA 1998 – 2010
( ) A inveja traz em seu bojo um elemento que fere [solidão
e espicaça quem a sente.
( ) A inveja é um sentimento que dilacera, além no pobre coração já cheio de compromissos...
das relações humanas, o próprio coração do Se bem que seja difícil compreender o que querem
invejoso. [esses homens,
( ) A inveja azeda, exaspera as relações entre as
pessoas, transformando-as em pessoas infelizes. por que motivo eles se ajuntaram e qual a razão de
a) 1 – 3 – 2 – 5 – 4 c) 2 – 3 – 5 – 4 – 1 seus sofrimentos.
b) 3 – 1 – 4 – 2 – 5 d) 4 – 5 – 1 – 2 – 3
Precisamos, precisamos esquecer o Brasil!
Tão majestoso, tão sem limites, tão
AFA 2007/2008 – Português despropositado, ele quer repousar de nossos
terríveis carinhos.
O Brasil não nos quer! Está farto de nós!
Leia atentamente o Texto I para responder às
Nosso Brasil é no outro mundo. Este não é o
questões de 31 a 36.
Brasil.
Nenhum Brasil existe. E acaso existirão os
Texto I
brasileiros?
Hino nacional
(Andrade, Carlos Drummond de. Sentimento do
Mundo – 12a ed.
Precisamos descobrir o Brasil!
Rio de Janeiro: Record, 2001.)
Escondido atrás das florestas,
com a água dos rios no meio,
1) O título dado ao poema, remete-nos ao Hino
o Brasil está dormindo, coitado.
Nacional Brasileiro escrito por Duque Estrada.
Precisamos colonizar o Brasil.
Pode-se dizer que, com isso, Drummond procurou
a) conservar algumas idéias afins em relação ao
O que faremos importando francesas
Hino Nacional Brasileiro.
muito louras, de pele macia,
b) fazer uma imitação cômica, através do recurso
alemãs gordas, russas nostálgicas para
intertextual paródia.
garçonettes dos restaurantes noturnos.
c) absorver e transformar o poema em uma
E virão sírias fidelíssimas.
multiplicidade de outros textos.
Não convém desprezar as japonesas...
d) criar um canto nacional de conscientização em
detrimento do canto ufanista de Duque Estrada.
Precisamos educar o Brasil.
Compraremos professores e livros,
2) De acordo com o texto, assinale a alternativa
assimilaremos finas culturas,
INCORRETA.
abriremos dancings e subvencionaremos as elites.
a) A expressão se bem que (verso 27) determina
uma mudança de direção na idéia desenvolvida no
Cada brasileiro terá sua casa
texto.
com fogão e aquecedor elétricos, piscina,
b) O eu-lírico apresenta duas posturas em relação
salão para conferências científicas.
ao brasileiro: a princípio otimista e por fim
E cuidaremos do Estado Técnico.
desalentada.
c) Até parte da sexta estrofe há no texto uma
Precisamos louvar o Brasil.
intensificação progressiva de significados e
Não é só um país sem igual.
emoções.
Nossas revoluções são bem maiores
d) A expressão terríveis carinhos (verso 34)
do que quaisquer outras; nossos erros também.
apresenta um contra-senso e foi utilizada para
E nossas virtudes? A terra das sublimes paixões...
reforçar um aspecto negativo de nosso
os Amazonas inenarráveis...os incríveis João-
nacionalismo.
Pessoas...
3) Assinale a alternativa correta, em relação ao
Precisamos adorar o Brasil!
texto.
Se bem que seja difícil caber tanto oceano e tanta
137
PROVAS DA AFA 1998 – 2010
a) O verbo “descobrir” é considerado um verbo gerúndio apresentadas exercem funções próprias
irregular, pois seu particípio não se faz de forma do adjetivo.)
regular.
b) Os verbos presentes nos versos de 12 a 15 6) Assinale a opção INCORRETA em relação à
exigem o mesmo tipo de complemento, uma vez análise dos seguintes versos.
que possuem a mesma transitividade. a) “Precisamos colonizar o Brasil / o que faremos
c) O ponto e vírgula utilizado no verso 23 não está importando francesas.” (O pronome o resgata a
de acordo com a norma culta padrão. O seu idéia expressa no verso anterior e dá sentido ao
emprego é justificado pela licença poética. verso a que pertence.)
d) Em todas as palavras a seguir, encontra-se um b) “O Brasil está dormindo, coitado” (A palavra
ditongo crescente: “água”, “macia”, “sírias”, coitado traduz um sentimento do eu-lírico e está
“conferências”, “igual”, “maiores”, “paixões” e sintaticamente desvinculada dos outros termos da
“inenarráveis”. oração.)
c) “Se bem que seja difícil caber tanto oceano e
4) Marque (V) para as proposições verdadeiras e tanta solidão / no pobre coração já cheio de
(F) para as falsas. compromissos.” (A oração e o termo destacados
( ) A segunda e terceira estrofes apresentam uma são complementos das palavras difícil e cheio.)
ironia em relação às necessidades e interesses do d) O Brasil não nos quer! Está farto de nós! (Os
Brasil. termos grifados referem-se, respectivamente, a
( ) As palavras inenarráveis, incríveis (verso 25) e brasileiros e a Brasil.)
despropositado (verso 33) originaram-se pelo
mesmo processo de formação das palavras. Leia atentamente o Texto II e, a seguir, responda
( ) O texto valoriza as importações utilizadas na às questões de 37 a 44.
formação da cultura brasileira.
( ) A substituição de “precisamos descobrir o Texto II
Brasil” por precisamos de descobrir o Brasil está
de acordo com a norma culta. Sinais de vida e morte no planeta verde
( ) O eu-lírico posiciona-se contrariamente ao
tratamento idealizado dado ao Brasil. A ocupação da Amazônia tira o sono do mundo,
A seqüência correta é mas é um desafio do qual os brasileiros não podem
a) V, F, F, V, V. c) F, V, V, F, F. escapar
b) F, V, F, F, V. d) V, F, V, V, F.
“O homem está aqui para o bem do homem”
5) Analise as proposições abaixo, em relação ao Albert Einstein
texto, e, a seguir, assinale a alternativa em que se
apresenta um comentário INCORRETO. Nas vésperas de inaugurar com orgulho da raça seu
a) “... com fogão e aquecedor elétricos...”. (Se feita terceiro milênio cristão e o centésimo da espécie, a
a concordância atrativa, o enunciado não sofreria humanidade descobriu um novo mundo
nenhuma alteração, já que a norma culta padrão aparentemente hostil, o planeta verde da Amazônia
permite as duas formas de concordância.) brasileira. A opinião pública mundial eletrizou
b) “Não convém desprezar as japonesas...”. (O com as afirmações de que as entranhas da
trecho apresenta um termo sobre o qual se faz uma Amazônia ardiam em chamas, um fogo capaz de
declaração em forma de oração reduzida.) gerar com suas labaredas fumaça suficiente para
c) “Se bem que seja difícil compreender o que fazer cair sobre a Terra um castigo bíblico, o
querem esses homens, por que motivo eles se aquecimento irreversível de sua superfície num
ajuntaram e qual a razão de seus sofrimentos”. verão tórrido e perpétuo. A aldeia global
(Entre as orações do trecho, há três que exercem a convenceu-se de que o inferno verde existe aqui e
mesma função sintática, portanto, estão agora no Brasil e de que só uma campanha
coordenadas entre si.) internacional pode salvar o pulmão do mundo de
d) “... o Brasil está dormindo...“ “... o que faremos seus agressores, as queimadas e os desmatamentos.
importando francesas...” (As formas nominais de Essa imagem, a de um gigante ameaçador que
engole florestas e cospe fogo, correu o mundo.
138
PROVAS DA AFA 1998 – 2010
Feita de brumas, ela é apenas uma ilusão perversa “...a humanidade descobriu um novo mundo
e exagerada que esconde uma realidade complexa. aparentemente hostil, o planeta verde da Amazônia
Esconde as batalhas pela vida empreendidas por brasileira...”
uma civilização de brasileiros que, "depois de a) “A aldeia global convenceu-se de que o inferno
vagar por ali três séculos numa agitação verde existe aqui e agora no Brasil...”
tumultuária e estéril" – como escreveu Euclides da b) “...só uma campanha internacional pode salvar o
Cunha –, começa a aprender a conciliar a pulmão do mundo de seus agressores...”
preservação com o progresso. c) “A opinião pública mundial eletrizou com as
(Veja – 5 de julho de 1989) afirmações de que as entranhas da Amazônia
ardiam em chamas.”
7) Associando o texto à tirinha abaixo, só se pode d) “Feita de brumas, ela é apenas uma ilusão
inferir que perversa e exagerada que esconde uma realidade
complexa.”

10) Assinale a alternativa cuja alteração não


prejudicou o sentido original do texto.
a) “A aldeia global convenceu que ....” (linha 10)
b) “...– do modo que escreveu Euclides da
Cunha....” (linha 21)
c) “...por uma civilização de brasileiros onde,
a) ambos os textos são pessimistas: aquele por suas “depois de vagar por ali...” (linhas 19 e 20)
figuras de linguagem se referindo à Amazônia d) “Tal imagem, a de um gigante ameaçador que
sempre de forma negativa; este pela fala engole...” (linhas 14 e 15)
amedrontada do fantasminha.
b) o objetivo principal desses textos é 11) Assinale a alternativa que NÃO contém uma
conscientizar a população da necessidade de oração com a mesma função sintática da que foi
preservar a natureza. destacada no trecho abaixo.
c) as queimadas e os desmatamentos são a “... mas é um desafio do qual os brasileiros não
principal causa de destruição das florestas. podem escapar.”
d) o homem da tira representa os brasileiros que a) “A opinião pública mundial eletrizou com as
“depois de vagar por ali por três séculos começa a afirmações de que as entranhas da Amazônia
aprender a conciliar a preservação e o progresso.” ardiam em chamas...”
b) “Essa imagem, a de um gigante ameaçador que
8) Assinale a alternativa INCORRETA. engole florestas e cospe fogo...”
a) A expressão “Feita de brumas” (linha 16) leva- c) “... por uma civilização de brasileiros que,
nos a inferir que a imagem construída da ‘depois de vagar por ali três séculos numa agitação
Amazônia é obscura. tumultuária e estéril’ – como escreveu Euclides da
b) Para enfatizar o exagero da mídia em relação às Cunha –, começa a aprender a conciliar...”
informações sobre a Amazônia, o locutor se vale d) “... ela é apenas uma ilusão perversa e
de expressões do discurso religioso apocalíptico. exagerada que esconde uma realidade complexa.”
c) Depreende-se por parte do locutor uma
confiança nos brasileiros no que se refere ao 12) Analise as proposições abaixo.
cuidado com as riquezas naturais da Amazônia. I - Em “A aldeia global convenceu-se de...” (linha
d) Infere-se que a Amazônia é o pulmão do 10) observa-se a pronominalização verbal
mundo, portanto patrimônio da humanidade e enfatizando a idéia reflexiva do agente.
deve-se defendê-la de uma administração II - A vírgula utilizada na linha 21 é necessária,
irresponsável. embora haja um travessão no período.
III - As expressões gigante ameaçador, brumas e
9) Assinale a alternativa cuja palavra grifada realidade complexa fazem referência aos aspectos
possui a mesma figura de linguagem destacada na negativos divulgados sobre a Amazônia.
oração abaixo.

139
PROVAS DA AFA 1998 – 2010
IV - Em “aprender a conciliar a preservação” os que reservas financeiras sirvam para queimar
artigos confirmam a transitividade direta dos países inteiros na volúpia da especulação.
verbos ao qual completam.
Estão corretas apenas as preposições Antes mesmo da Amazônia, eu gostaria de ver a
a) I e II. b) III e IV. c) II, III e IV. d) I, II e III. internacionalização de todos os grandes museus do
mundo. O Louvre não deve pertencer apenas à
13) Assinale a alternativa correta. França. Cada museu do mundo é guardião das
a) A vírgula após a palavra agressores (linha 13) mais belas peças produzidas pelo gênio humano.
justifica-se por separar elementos de uma Não se pode deixar que esse patrimônio cultural,
enumeração. como o patrimônio natural amazônico, seja
b) Em “... as afirmações de que as entranhas da manipulado e destruído pelo gosto de um
Amazônia ardiam em chamas...” (linhas 5 e 6), a proprietário ou de um país.
oração destacada exerce a mesma função sintática ..................
que o termo destacado em “... o aquecimento Nos seus debates, os atuais candidatos à
irreversível de sua superfície...” (linhas 8 e 9). presidência dos EUA têm defendido a idéia de
c) O texto estrutura-se em duas partes, na primeira internacionalizar as reservas florestais do mundo
encontra-se a imagem que se construiu da em troca da dívida.
Amazônia no exterior; na segunda, o alerta para a
necessidade urgente de preservar a Amazônia. Comecemos usando essa dívida para garantir que
d) A epígrafe está diretamente relacionada à cada criança do mundo tenha possibilidade de
necessidade de promover o progresso que comer e de ir à escola. Internacionalizaremos as
propiciará condições de vida mais dignas aos crianças tratando-as, todas elas, não importando o
habitantes da Amazônia. país onde nasceram, como patrimônio que merece
cuidados do mundo inteiro. Ainda mais do que
14) Assinale a alternativa em que ocorre alteração merece a Amazônia.
no significado da expressão, de acordo com o
texto, devido à reescritura proposta. Quando os dirigentes tratarem as crianças pobres
a) ... a humanidade descobriu um mundo novo do mundo como um patrimônio da humanidade,
aparentemente hostil... (linhas 2 a 4) eles não deixarão que elas trabalhem quando
b) ... o verde planeta da Amazônia brasileira. (linha deveriam estudar, que morram quando deveriam
4) viver.
c) ... um castigo bíblico, o irreversível
aquecimento de sua superfície num verão tórrido e Como humanista, aceito defender a
perpétuo. (linhas 8 a 10) internacionalização do mundo. Mas, enquanto o
d) Feita de brumas, ela apenas é uma exagerada e mundo me tratar como brasileiro, lutarei para que a
perversa ilusão... (linhas 16 e 17) Amazônia seja nossa.
Só nossa!
As questões de 15 a 19 referem-se ao Texto III. (http://www.culturabrasil.org/amazoniadobrasil.ht
Leia-o atentamente para respondê-las. m)

Texto III 15) Pode-se inferir do texto que a/o a) humanidade


precisa pensar apenas em seu ecossistema.
Durante debate recente em uma universidade, nos b) maior valor do nosso planeta são os museus e o
Estados Unidos, o ex-governador do Distrito ecossistema.
Federal e ex-Ministro da Educação, Senador c) humanidade precisa se respeitar mais, pois
Cristovam Buarque, foiquestionado sobre o que cuidar do planeta e de seus moradores é
pensava da internacionalizaçãoda Amazônia. responsabilidade de todos.
.................. d) capitalismo é o único responsável pelos graves
Queimar a Amazônia é tão grave quanto o problemas vividos pelo globo terrestre.
desemprego provocado pelas decisões arbitrárias
dos especuladores globais. Não podemos deixar 16) Analise as proposições a seguir.

140
PROVAS DA AFA 1998 – 2010
I - A postura humanista a que se refere Cristovam (Apresenta um termo que especifica outro termo
Buarque está diretamente condicionada à exigência anterior e não apresenta agente para a locução
social de preservar o patrimônio cultural e natural verbal.)
da humanidade. b) “Antes mesmo da Amazônia, eu gostaria de ver
II - Em “Internacionalizaremos as crianças, a internacionalização de todos os grandes museus
tratando-as, todas elas, não importando...“ (linhas do mundo.”
24 e 25) a expressão destacada relaciona-se ao (Apresenta um termo de natureza temporal e um
complemento verbal, a fim de enfatizar o termo cuja função é completar o sentido de outro
tratamento desigual dado às crianças. termo de natureza nominal.)
III - No trecho destacado em “Mas, enquanto o c) “Quando os dirigentes tratarem as crianças
mundo me tratar como brasileiro...” (linha 33) pobres do mundo como um patrimônio da
Cristovam Buarque insinua a postura unilateral e humanidade...”
materialista daqueles que defendem a (Apresenta termos que estão, no texto, numa
internacionalização da Amazônia. relação de conformidade entre si.)
IV - O sintagma “... sobre o que pensava da d) “Comecemos usando essa dívida para garantir
internacionalização da Amazônia.” (linhas 4 e 5) que cada criança do mundo tenha possibilidade de
acrescenta uma circunstância à locução verbal foi comer e de ir à escola.”
questionado (linha 3). (Apresenta termos que têm a mesma função
V - O verbo queimar (linha 9) vem acrescido de sintática, por isso estão coordenados entre si.)
outro significado, paralelo ao de sua incidência
anterior. 19) Leia as afirmações sobre o texto e assinale
Estão corretas apenas como (V) verdadeira ou (F) falsa.
a) I, II e IV. c) I, IV e V. ( ) Se feito o plural de educação,
b) II, III e V. d) III, IV e V. internacionalização e guardião, em apenas duas
delas obter-se-á a mesma forma encontrada no
17) Considerando a relação semântica, relacione a plural de decisão.
1a coluna com a 2a e, a seguir, assinale a ( ) Na linha 6 encontramos um adjetivo no grau
alternativa com a seqüência correta. comparativo de igualdade.
( ) A próclise existente na linha 15 é facultativa.
Coluna 1 ( ) Em “Quando os dirigentes tratarem as crianças
(1) Volúpia da especulação pobres do mundo como patrimônio...” (linhas 28 e
(2) Patrimônio natural amazônico 29) a substituição de as crianças pelo pronome
(3) Reservas florestais oblíquo ficará tratarem-as.
(4) Especuladores globais ( ) Mantendo a mesma voz verbal, pode-se dizer
que “... belas peças produzidas pelo gênio
Coluna 2 humano” (linhas 14 e 15) corresponde à
( ) Conservação da fauna e flora. produziram-se belas peças.
( ) Desejo avultado de exploração.
( ) Exploração das reservas do planeta. Assinale a seqüência correta.
( ) Reconhecimento da floresta como bem tombado a) V – V – F – F – V c) V – F – F – V – F
para usufruto dos cidadãos. b) F – F – V – V – F d) F – V – F – F – V

a) 2 – 1 – 4 – 1 c) 3 – 1 – 4 – 2 Responda às questões de 20 a 29 de acordo com o


b) 4 – 2 – 3 – 1 d) 1 – 4 – 2 – 2 Texto IV.

18) Assinale a alternativa cuja afirmação entre Texto IV


parênteses NÃO está em consonância com o trecho Carta do Índio Chefe Seatle, “Manifesto da
analisado. Terra-Mãe”
a) “...o ex-governador do Distrito Federal e ex-
Ministro da Educação, Senador Cristovam “Como podeis comprar ou vender o céu, o calor
Buarque, foi questionado sobre o que pensava da da terra? A idéia não tem sentido para nós.
internacionalização da Amazônia.”
141
PROVAS DA AFA 1998 – 2010
Se não somos donos da frescura do ar ou do brilho teia da vida; é antes um dos seus fios. O que quer
das águas, como podeis querer comprá-los? que faça a essa teia, faz a si próprio.
Qualquer parte desta terra é sagrada para meu ..................
povo. Qualquer folha de pinheiro, cada grão de Esta terra tem um valor inestimável para Ele, e
areia nas praias, a neblina nos bosques sombrios, ofender a terra é insultar o Criador. Também os
cada monte e até o zumbido do inseto, tudo é brancos acabarão um dia talvez mais cedo do que
sagrado na memória e no passado do meu povo. A todas as outras tribos. Contaminai os vossos rios e
seiva que percorre o interior das árvores leva em si uma noite morrerão afogados nos vossos resíduos.
as memórias do homem vermelho. Contudo, caminhareis para a vossa destruição,
................... iluminados pela força do Deus que vos trouxe a
Nós sabemos que o homem branco não entende o esta terra e por algum desígnio especial vos deu o
nosso modo de ser. Ele não sabe distinguir um domínio sobre ela e sobre o homem vermelho. Este
pedaço de terra de outro qualquer, pois é um destino é um mistério para nós, pois não
estranho que vem de noite e rouba da terra tudo de compreendemos como será no dia em que o último
que precisa. A terra não é sua irmã, mas sua bisonte for dizimado, os cavalos selvagens
inimiga, depois de vencida e conquistada, ele vai domesticados, os secretos recantos das florestas
embora, à procura de outro lugar. invadidos pelo odor do suor de muitos homens e a
.................. visão das brilhantes colinas bloqueada por fios
O ar é inestimável para o homem vermelho, pois falantes. Onde está o matagal? Desapareceu. Onde
dele todos se alimentam. Os animais, as árvores, o está a águia? Desapareceu. Termina a vida começa
homem, todos respiram o mesmo ar. O homem a sobrevivência.”
branco parece não se importar com o ar que (http://mnemosyne.blog-city.com/a-carta-do-indio-
respira. Como um cadáver em decomposição, ele é eatllemanifesto-da-terramae.htm )
insensível ao mau cheiro. Mas se vós venderdes
nossa terra, deveis recordar que o ar é precioso 20) Pode-se inferir do texto que
para nós, que o ar insufla seu espírito em todas as a) tudo o que existe faz parte do patrimônio
coisas que dele vivem. O vento que deu aos nossos humano, logo os homens têm o direito de dispor
avós o primeiro sopro de vida é o mesmo que lhes dele como desejar.
recebe o último suspiro. b) a terra é sagrada, devemos preservá-la e
................... respeitá-la como meio de subsistência humana.
Sou um selvagem e não compreendo como o c) o índio ensina a seus descendentes a sugar e
fumegante cavalo de ferro possa ser mais retirar da terra tudo aquilo que ela pode lhes
importante que o bisonte, que nós caçamos apenas proporcionar.
para sobreviver. Que será dos homens sem os d) mais importante é a preservação dos animais,
animais? Se todos os animais desaparecem, o pois são eles que alimentarão as gerações futuras.
homem morrerá de solidão espiritual.
Porque o que suceder aos animais afetará os 21) O homem branco só NÃO é comparado a um
homens. a) estranho que vem da noite.
Tudo está ligado. b) cadáver em decomposição.
Deveis ensinar a vossos filhos que o solo que c) solitário espiritual.
pisam são as cinzas de nossos avós. Para que eles d) fio da teia que tece a vida.
respeitem a terra, ensina-lhes que ela é rica pela
vida dos seres de todas as espécies. Ensinai aos 22) Assinale a alternativa em que a reescritura dos
vossos filhos o que nós ensinamos aos nossos: Que trechos, retirados do texto, provocou a alteração
a terra é a nossa mãe. Quando o homem cospe sintática e/ou semântica.
sobre a terra, cospe sobre si mesmo. De uma coisa a) Vender o céu, o calor da terra é atitude
nós temos certeza: A terra não pertence ao homem inconcebível para os índios. (linhas 1 e 2)
branco; o homem branco é que pertence à terra. b) O homem vermelho se alimenta do ar, portanto
Disso nós temos a certeza. Todas as coisas estão este tem valor incomensurável. (linhas 17 e 18)
relacionadas como o sangue que une uma família. c) O bisonte é mais importante que o cavalo de
Tudo está associado. O que fere a terra fere ferro fumegante, e um índio, por sua natureza, não
também aos filhos da terra. O homem não tece a
142
PROVAS DA AFA 1998 – 2010
entende que isso seja visto de outra forma. (linhas c) “Posto que, caminhareis para a vossa destruição,
27 a 29) iluminados pela força do Deus que vos trouxe a
d) As cinzas dos antepassados estão impregnadas esta terra e por algum desígnio especial...”
no solo e as gerações mais novas devem d) “De sorte que, caminhareis para a vossa
compreender isso. (linhas 34 e 35) destruição, iluminados pela força do Deus que vos
trouxe a esta terra e por algum desígnio especial...”
23) Pode-se afirmar que em
a) “fumegante cavalo de ferro” (linhas 27 e 28), 26) No trecho “Nós sabemos que o homem
observa-se a presença da metáfora que enfatiza a branco não entende o nosso modo de ser”, o Índio
inversão de valores percebida pelo índio. Chefe Seatle emite um juízo de valor que mostra o
b) “cinzas de nossos avós” (linha 35), fica clara a quão distinta é a cultura do homem branco e a do
desobrigação que o homem tem para com seus índio.
antepassados. Dentre as passagens abaixo, assinale aquela que
c) “O que fere a terra fere também aos filhos da NÃO ilustra esse choque cultural.
terra.” (linhas 43 e 44), mostra o respeito que se a) “O homem não tece a teia da vida; é antes um
deve ter pelo homem, tudo é menor diante da sua dos seus fios. O que quer faça a essa teia, faz a si
supremacia. próprio. Esta terra tem um valor inestimável para
d) “Contaminai os vossos rios e uma noite Ele, e ofender a terra é insultar o Criador.”
morrerão afogados nos vossos resíduos.” (linha b) “Sou um selvagem e não compreendo como o
50), depreendese que o homem polui seus rios e fumegante cavalo de ferro possa ser mais
possui capacidade para evitar que isso ocorra. importante que o bisonte, que nós caçamos apenas
para sobreviver.”
24) O texto é concluído com interrogações e c) “O ar é inestimável para o homem vermelho,
afirmações. Pode-se dizer, com isso, que o locutor pois dele todos se alimentam.” / “O homem branco
a) nos revela que devemos cuidar do nosso parece não se importar com o ar que respira.”
ecossistema para que a raça humana seja d) “Deveis ensinar a vossos filhos que o solo que
preservada. pisam são as cinzas de nossos avós (...) Ensinai aos
b) não interfere na argumentação por se tratarem vossos filhos o que nós ensinamos aos nossos...”
de perguntas e respostas que não conduzem ao
questionamento. 27) Os emissores do Texto I, II, III e IV guardam,
c) responsabiliza o Criador por tudo que ocorre na predominante e respectivamente, as seguintes
Terra, fazendo-nos crer que só Ele pode fazer algo. características:
d) apregoa que o grande responsável é o destino, a) indignação, ironia, incredulidade e ingenuidade.
pois já estava planejado que o homem é um grande b) intolerância, sabedoria, esperteza e calma.
predador. c) ironia, preocupação, indignação e perplexidade.
d) cólera, desconforto, intolerância e obediência.
25) Considere o excerto abaixo.
“Contudo, caminhareis para a vossa destruição, 58) Os trechos abaixo foram modificados do texto
iluminados pela força do Deus que vos trouxe a original. Assinale a opção cuja pontuação NÃO
esta terra e por algum desígnio especial...” ficou de acordo com a norma padrão da Língua.
a) Nas vésperas de inaugurar com orgulho da raça
Assinale a alternativa em que a substituição do seu terceiro milênio cristão e o centésimo da
conectivo NÃO espécie, a humanidade descobriu um novo mundo:
interfere semanticamente na idéia proposta. o planeta verde da Amazônia brasileira. (Texto II,
a) “Pois, caminhareis para a vossa destruição, linhas 1 a 4)
iluminados pela força do Deus que vos trouxe a b) O homem morrerá de solidão espiritual se todos
esta terra e por algum desígnio especial...” os animais desaparecerem. (Texto IV, linhas 30 e
b) “Todavia, caminhareis para a vossa destruição, 31)
iluminados pela força do Deus que vos trouxe a c) Internacionalizaremos as crianças, tratando-as,
esta terra e por algum desígnio especial...” todas elas, não importando o país onde nasceram,
como patrimônio que merece cuidados do mundo
inteiro. (Texto III, linhas 24 a 26)
143
PROVAS DA AFA 1998 – 2010
d) Quando os dirigentes tratarem as crianças AFA 2008/2009 – Português
pobres do mundo, como um patrimônio da
humanidade, eles não deixarão que elas trabalhem Preces por São Francisco
quando deveriam estudar, que morram quando Por meio de uma greve de fome de 23 dias, o
deveriam viver. (Texto III, linhas 28 a 31) bispo dom Cappio voltou a chamar a atenção
para os problemas inerentes à transposição das
29) Assinale a alternativa INCORRETA. águas do Velho Chico para o semi-árido
a) No texto IV, a forma verbal leva (linha 9) nordestino.
deveria estar no futuro do presente do indicativo a
fim de se adequar semanticamente ao texto. Em dezembro de 2007, um religioso fez
b) No texto I, a forma verbal precisamos (linha 1) jejum de 23 dias por São Francisco. Esse, que
encontrase no presente do indicativo, mas seu poderia ser uma manifestação qualquer de algum
valor, no texto, equivale ao modo imperativo. dos milhões de católicos brasileiros, ultrapassou os
c) No texto II, ao se substituir a forma verbal limites da Igreja e virou um problema de Estado.
descobriu (linha 3) por descobre mantém-se a Afinal, não era um São Francisco qualquer, mas o
correção gramatical, mas altera-se o valor rio brasileiro que atravessa cinco estados;
aspectual do verbo. tampouco se tratava de um religioso qualquer, mas
d) No texto III, 6o parágrafo, encontram-se três de dom Luís Flávio Cappio, bispo de Barra,
formas verbais no futuro, sendo que a última delas histórico ambientalista, defensor e conhecedor das
se refere a processos de realização desejável. populações pobres do sertão nordestino. A greve
de fome foi amplamente divulgada pela imprensa
30) Leia a tirinha e marque a afirmativa nacional e esquentou o debate sobre as obras no
INCORRETA. VelhoChico.
(...)
O Velho Chico, como o rio é conhecido
Brasil afora, nasce em João de Roque de Minas, na
serra da Canastra, e lança suas águas entre Alagoas
e Sergipe no oceano Atlântico. O São Francisco foi
descoberto por Américo Vespúcio em 4 de outubro
de 1501, quando o navegador mapeava o litoral
a) A expressão de surpresa dos indiozinhos brasileiro. Sua bacia hidrográfica representa 8% do
corresponde ao sentimento “Sou um selvagem e território nacional e 57% da área do semi-árido.
não compreendo como o fumegante cavalo de São 32 sub-bacias, 168 afluentes e envolve 505
ferro possa ser mais importante que o bisonte...” municípios – 101 no vale do rio – onde vivem
(linhas 27 e 28) do Texto IV. aproximadamente 15 milhões de pessoas.
b) Um dos terríveis carinhos de que nos fala o eu- (Revista Discutindo Geografia, Ano 3 – adaptado.)
lírico do Texto I pode ser exemplificado através da
ação dos caraíbas que visam principalmente ao 1) Sobre o texto acima, é correto afirmar que o/a
progresso do país. a) forma carinhosa como o rio São Francisco é
c) O tema utilizado nessa tirinha colabora na conhecido caracteriza uma figura de linguagem
divulgação da idéia exposta no Texto II: a conhecida como eufemismo.
vinculação de uma imagem de destruição da b) pronome esse (l. 2) funciona como um elemento
natureza que esconde uma realidade complexa. de coesão textual e substitui o termo religioso.
d) O público-alvo de Maurício de Sousa é infanto- c) repercussão da atitude do frei não era esperada
juvenil, logo pode-se dizer que um dos objetivos pela Igreja Católica que delegou o problema para o
da tirinha é despertar a consciência das crianças Estado.
para um problema mundial: o desmatamento. d) descoberta do rio São Francisco está
condicionada a uma situação exploratória.

2) Assinale a opção cujo termo grifado NÃO está


coerente com o sentido original do Texto I.

144
PROVAS DA AFA 1998 – 2010
a) Não era, pois, um São Francisco qualquer, mas ( ) Do trecho “...no vale do rio São Francisco –
o rio brasileiro que atravessa cinco estados (...) (l. onde vivem
5 e 6) aproximadamente 15 milhões de pessoas...”,
b) O Velho Chico, como o rio é conhecido ao depreende-se que o número de pessoas que vivem
longo do Brasil (...) (l. 14) no vale do rio pode ser inferior a 15 milhões.
c) Dom Cappio voltou a chamar a atenção para os A seqüência correta é
problemas contrários à transposição das águas do a) V, F, F, V. c) V, V, V, F.
Velho Chico (...) (lide) b) F, F, F, V. d) F, V, V, F.
d) Muito menos se tratava de um religioso
qualquer, mas de dom Luís Flávio Cappio, bispo Texto II
de Barra, histórico ambientalista (...) Degradação
(l. 6 a 8) O rio São Francisco tem enorme
importância socioeconômica atribuída à utilização
3) Dentre as preposições abaixo grifadas, assinale de suas águas. Dentre os diversos usos, destacam-
aquela cuja classificação entre parênteses está de se: o consumo humano, a produção de energia
acordo com o sentido utilizado no texto. elétrica distribuída para aproximadamente 97% da
a) “Por meio de uma greve de fome de 23 dias, o região Nordeste, a agricultura irrigada . praticada
bispo dom Cappio voltou a chamar a atenção para em diversas agrovilas espalhadas ao longo das
os problemas...” margens . a piscicultura . como a criação de
(INSTRUMENTO) tilápias no cânion do rio, no município baiano de
b) “A greve de fome foi amplamente divulgada Paulo Afonso . a navegação e o turismo, entre
pela imprensa nacional...” (LUGAR) outras funções.
c) “Em dezembro de 2007, um religioso fez jejum O São Francisco se encontra em estágio
de 23 dias por São Francisco...” (CAUSA) avançado de degradação. O problema é imputado
d) “...e esquentou o debate sobre as obras no Velho às ações humanas como, por exemplo, devastação
Chico.” (MEIO) das nascentes e matas ciliares, despejo de esgoto e
lixo . doméstico e industrial . ; às atividades
4) “Esse, que poderia ser uma manifestação agropecuárias, mineradoras e de garimpo; às
qualquer de algum dos milhões de católicos queimadas etc.
brasileiros, ultrapassou os limites da Igreja e (...)
virou um problema de Estado”.(l. 2 a 5) Esses impactos ambientais na região
Dessa passagem pode-se inferir a/o drenada pelo Velho Chico e seus afluentes
a) devoção dos brasileiros a São Francisco de começaram a ocorrer por volta de 500 anos atrás,
Assis. época em que se iniciou o povoamento no vale do
b) grande dimensão do problema com a São Francisco. Hoje, o rio está debilitado e em
transposição. diversos setores da bacia hidrográfica a degradação
c) jejum como prática corriqueira entre muitos preocupa, aflige e entristece.
brasileiros. (Revista Discutindo Geografia, Ano 3 – adaptado.)
d) estreita relação entre Igreja e Estado no Brasil.
6) De acordo com o Texto II, é correto afirmar que
5) Assinale (V) verdadeiro ou (F) falso nas a/o(s)
afirmativas abaixo, retiradas de trechos do Texto I. a) impactos ambientais na região acentuaram-se
( ) A expressão “Por meio de uma greve de fome sobremaneira nos últimos anos em decorrência da
de 23 dias...”, mantém uma relação de causa com o industrialização.
restante do período. b) rio São Francisco tem uma importância capital
( ) A palavra sublinhada em “O Velho Chico, para o desenvolvimento do Nordeste brasileiro.
como o rio é conhecido Brasil afora,...” estabelece c) degradação do rio São Francisco e seus
uma relação comparativa. afluentes causa muita preocupação
( ) O termo destacado em “...ultrapassou os limites prioritariamente aos ribeirinhos.
da Igreja e virou um problema de Estado.” é um d) processo de povoamento desordenado da região
complemento direto da forma verbal virou. foi o responsável pela degradação do rio.

145
PROVAS DA AFA 1998 – 2010
7) Sobre o 2º parágrafo, é correto afirmar que II. Às atividades agropecuárias (...), às queimadas
a) em “O São Francisco se encontra em estágio complementam um termo que exerce a função de
avançado de degradação” há um termo elíptico e predicativo.
um predicativo do sujeito. III. Observa-se que o período é simples e nele há
b) a expressão por exemplo (l. 12) poderia ficar sujeito, complemento nominal e aposto.
sem as vírgulas, uma vez que é explicativa. IV. Ciliares, doméstico e garimpo são vocábulos
c) o verbo do 1º período está flexionado na voz que caracterizam
passiva. um termo imediatamente anterior.
d) as ações humanas, as atividades agropecuárias e Estão corretas apenas
as queimadas são, dentre outras, as conseqüências a) II e IV. c) I e II.
da degradação do rio. b) III e IV. d) I e III.

8) Assinale, entre as expressões sublinhadas Texto III


abaixo, aquela cujo valor semântico NÃO foi Prós e contras
corretamente indicado. A idéia de transferir parte das águas do São
a) “...a produção de energia elétrica distribuída Francisco remonta aos finais do século XIX e vem
para aproximadamente 97% da região Nordeste...” à tona, neste início de século XXI, como uma das
(l. 3 a 5) – Valor restritivo. ações prioritárias de governo na gestão do
b) “...a agricultura irrigada – praticada em diversas presidente Lula.
agrovilas espalhadas ao longo das margens – a O debate em torno da obra de transposição
piscicultura...” (l. 5 a 7) – Valor explicativo. segue cada vez mais acalorado. Uns dizem que a
c) “...enorme importância socioeconômica obra de engenharia é viável tecnicamente e deve
atribuída à utilização de suas águas...” (l. 1 e 2) – trazer geração de empregos, aumento de renda e
Valor conclusivo. redução da migração populacional, com a
d) “O problema é imputado às ações humanas...” implantação de agronegócios. Propaga-se que 12
(l. 11) – Valor de decorrência. milhões de pessoas serão beneficiadas. Outros
defendem a necessidade de revitalização do rio,
9) Assinale a alternativa correta. por causa do quadro avançado de degradação do
a) Em “O problema é imputado às ações São Francisco, além da urgência de estudos
humanas”, o termo destacado exerce a função de técnicos com fins de se prever os possíveis
agente da ação verbal. impactos socioambientais. Argumenta-se ainda a
b) O sintagma “de suas águas” (l. 2) exerce a incerteza da viabilidade econômica do projeto e a
mesma função sintática que o termo destacado em falta de clareza quanto aos benefícios sociais e à
“O São Francisco se encontra em estágio avançado distribuição de renda que poderá trazer.
de degradação”. (l. 10 e 11) (Revista Discutindo Geografia, Ano 3 – adaptado.)
c) Os vocábulos degradação, irrigada, consumo, e
piscicultura foram formados pelo mesmo processo 11) Pode-se inferir da leitura global do texto acima
de formação de palavras. que o/a
d) O vocábulo problema (l.11) é um item de a) viabilidade do projeto depende de muita
coesão lexical que retoma a expressão “estágio discussão relacionando os prejuízos e benefícios da
avançado de degradação”. transposição.
b) número de pessoas que serão prejudicadas com
10) Leia atentamente o excerto e analise as a mudança de curso do rio é relativamente
proposições a seguir. pequeno.
c) idéia de transposição, desde o século XIX,
“O problema é imputado às ações humanas como, sempre ocupou lugar de destaque nas discussões
por exemplo, devastação das nascentes e matas públicas.
ciliares, despejo de esgoto e lixo – doméstico e d) transposição do rio São Francisco trará mais
industrial – às atividades agropecuárias benefícios do que prejuízos.
mineradoras de garimpo, às queimadas etc...”
I. O vocábulo imputar pertence ao campo 12) Sobre o Texto III, assinale a alternativa
semântico de responsabilizar. INCORRETA.
146
PROVAS DA AFA 1998 – 2010
a) Prioritárias (l. 3), populacional (l. 8) são cisternas e na revitalização permanente do próprio
vocábulos adjetivos que caracterizam rio.
sintaticamente um termo nominal. Considerando-se que, em épocas de chuvas
b) Em propaga-se (l. 9), há ocorrência de torrenciais, as quais chegam todo ano até os vários
passividade verbal. sertões do semiárido, as cisternas funcionariam
c) Em “O debate em torno da obra de transposição como estoque de água para o longo período de
segue cada vez mais acalorado”, houve a estiagem, aumentar a quantidade de poços proveria
intercalação de um termo adverbial entre o sujeito um maior número de famílias sertanejas,
e a ação verbal. amenizaria impactos sócio-ambientais e reduziria o
d) Um forte argumento a favor da transposição das atual custo estimado para a obra de transposição.
águas do rio é o elevado número de pessoas (...)
beneficiadas por essa ação. Sendo assim, toda cautela, destreza e
preparo técnico serão convenientes na execução de
13) Leia as assertivas abaixo sobre o Texto III. uma obra com a magnitude da transposição. Todos
I. No segundo parágrafo, usam-se os pronomes uns os setores da sociedade deverão opinar sobre qual
(l. 6) e outros (l. 10) para indefinir os falantes que o melhor projeto hídrico para o bravo e dócil povo
estão em lados opostos da divergência. dos sertões, a fim de evitar equívocos e agravar
II. O texto apresenta, entre seus recursos ainda mais a delicada situação de vida no
argumentativos, pontos semiárido nordestino.
de vista antagônicos e contextualização histórica. (Revista Discutindo Geografia, Ano 3 – adaptado.)
III. Em “... remonta aos finais do século XIX...” (l.
2) e em “... a incerteza da viabilidade econômica 14) Pode-se afirmar do texto acima que
do projeto” (l. 14 e 15) os termos grifados exercem a) a oração “tutelado pelo Ministério da Integração
a mesma função sintática. Nacional” (l. 1 e 2) possui valor substantivo.
IV. O debate sobre a transposição segue cada vez b) no último período do primeiro parágrafo, a
mais acalorado por causa das incertezas em relação oração principal encontra-se na voz reflexiva.
à viabilidade técnica e aos impactos da distribuição c) a expressão em vez de (l. 13) poderia ser
de renda. substituída por ao invés de, mantendo-se a
Estão corretas apenas correção gramatical e o sentido original.
a) I e II. c) II e IV. d) a expressão sendo assim (l. 25) é um elemento
b) I e III. d) III e IV. de coesão e estabelece com o texto uma relação de
conclusão.
Texto IV
Desenvolvimento nocivo 15) Assinale a alternativa em que a palavra
O objetivo do projeto de transposição, sublinhada pode ser substituída pela palavra entre
tutelado pelo Ministério da Integração Nacional, é parênteses, mantendo-se o sentido original.
levar água para o interior seco e abastecer açudes e a) “(...) é levar água para o interior seco e
rios intermitentes dos estados de Pernambuco, abastecer açudes e rios intermitentes dos estados
Paraíba, Rio Grande do Norte e Ceará, no semi- de Pernambuco...” (l. 2 a 4) – (intermediários)
árido setentrional e oriental, a partir da represa de b) “Esse valor (...) numa obra faraônica, poderia
Sobradinho, na Bahia. O volume de água ser utilizado de outra maneira...” (l. 13 e 14) –
transferido para “integrar outras bacias” será de 26 (suntuosa)
metros cúbicos por segundo (Ciência Hoje, julho c) “(...) na execução de uma obra com a magnitude
de 2005). Para o eixo Norte está prevista a da transposição.” (l. 26 e 27) – (magnanimidade)
construção de 400 quilômetros de canais e, para o d) “O objetivo do projeto de transposição, tutelado
eixo Leste, um pouco mais de 200 quilômetros. A pelo Ministério da Integração Nacional...”(l. 1 e 2)
obra, estimada em 2 bilhões de dólares, deverá ser – (reportado)
executada em 20 anos.
Esse valor, em vez de ser investido numa 16) Coloque (V) verdadeiro ou (F) falso e, a
obra faraônica, poderia ser utilizado de outra seguir, assinale a seqüência
maneira, como na ampliação da construção de correta, em relação ao Texto IV.

147
PROVAS DA AFA 1998 – 2010
( ) No primeiro parágrafo do texto, a presença do
futuro do presente do indicativo enuncia a certeza 18) Sobre o Texto V, assinale a alternativa correta.
da transposição do rio, apesar dos esforços a) O eu-lírico personifica todos os lugarejos e
daqueles que são contra. estabelece uma interlocução com cada um deles.
( ) A presença constante do futuro do pretérito do b) Há o predomínio da função poética da
indicativo no terceiro e quarto parágrafos indica linguagem.
outras possibilidades viáveis para a solução do c) O nível lingüístico utilizado é a norma padrão
problema da seca. da língua.
( ) Da análise dos argumentos apresentados, infere- d) Em “... a profecia do beato que dizia que o
se que o emissor, apesar dos custos exorbitantes, sertão ia alagar”, há ambigüidade, que não poderá
acredita na transposição como forma de resolver o ser evitada pela simples substituição do pronome
problema da seca no relativo.
sertão nordestino.
19) Leia as assertivas abaixo relativas ao texto
Sobradinho.
( ) A expressão “para o bravo e dócil povo dos I. As formas verbais põe (v. 2), vai mudar (v. 2),
sertões” constrói sua carga semântica através do diz (v. 4) têm como sujeito a expressão o homem.
paradoxo dos atributos do sertanejo. II. Na expressão o homem (v. 1), o artigo o define
a) V – V – F – F c) F – F – F – V o substantivo.
b) F – V – V – F d) V – F – V – V III. A enumeração de nomes de localidades (v. 11
e 12) demonstra a pouca importância desses
Texto V vilarejos.
Sobradinho IV. As expressões debaixo d’àgua (v. 13) e por
O homem chega e já desfaz a natureza cima (v. 14) denotam as transformações no
Tira gente põe represa, diz que tudo vai mudar cotidiano provocadas pela construção da barragem.
O São Francisco lá pra cima da Bahia Estão corretas apenas
Diz que dia menos dia vai subir bem devagar a) I e III. c) II e III.
E passo a passo vai cumprindo a profecia b) II e IV. d) III e IV.
Do beato que dizia que o sertão ia alagar
O sertão vai virar mar 20) Assinale a opção cuja análise morfossintática
Dá no coração está correta.
O medo que algum dia a) Todos os verbos presentes nos versos “O
O mar também vire sertão homem chega e já desfaz a natureza/ Tira gente
Adeus Remanso, Casa Nova, Sento Sé põe represa” necessitam de complementação
Adeus Pilão Arcado, vem o rio te engolir direta.
– Debaixo d’água lá se vai a vida inteira b) Em “Adeus Remanso, Casa Nova, Sento Sé,
Por cima da cachoeira o Gaiola vai subir Adeus Pilão Arcado vem o rio te engolir”, o
Vai ter barragem no Salto do Sobradinho pronome pessoal, além de resgatar os nomes
E o povo vai se embora com medo de se afogar próprios citados, funciona como complemento
(http://letras.terra.com.br/sa guarabyra/487321 - direto do verbo vir.
Acesso em 15/05/08 às 14h) c) Em “O povo vai se embora com medo de se
afogar” as duas ocorrências do pronome se servem
17) Pode-se inferir do texto acima que de realce, podendo ser retiradas sem prejuízo
a) o povo se retira do sertão quando o beato prediz semântico.
o que vai ocorrer por lá. d) A palavra mar nos versos “O sertão vai virar
b) o homem quando chega anuncia que o sertão vai mar” e “O mar também vire sertão” é,
virar mar em detrimento do mar virar sertão. respectivamente, predicativo e sujeito.
c) tudo no sertão vai mudar conquanto o povo
compreenda a necessidade de colocar represa no 21) Assinale a alternativa INCORRETA.
lugar onde moram ribeirinhos. a) O Texto V apresenta como recursos estilísticos
d) a barragem do Salto do Sobradinho é a causa do hipérbato e paradoxo.
êxodo dos sertanejos.
148
PROVAS DA AFA 1998 – 2010
b) Enquanto o advérbio lá, no verso 3, veicula b) termos enumerados das linhas 19 a 21 referem-
idéia de lugar; no verso 13, ele traz idéia de modo. se à palavra todas (l. 19), exercendo a função de
c) O advérbio já, (v. 1), acrescenta ao texto uma adjunto adnominal desse vocábulo.
idéia de prepotência à ação humana. c) pronome em destaque no sintagma “deixai-me
d) No enunciado “o medo que algum dia o mar levar” (l. 22) exerce a função sintática de sujeito da
também vire sertão”, o pronome relativo que ação verbal “levar”.
introduz uma oração restritiva. d) expressões destacadas em “o sabor de minhas
águas, juninas e seus fogos de artifícios” (l. 23)
Texto VI exercem a mesma função sintática.
Oração do rio São Francisco em tempos de
poucos rios 23) No segundo fragmento do texto, é
Onde houver a dúvida dos que fraquejam, que eu INCORRETO afirmar que
leve a fé dos que constroem seu tempo. Na a) a expressão “os canhões abrindo fendas...” (l.
adversidade, meio ao deserto e ao clima árido, a fé 11) constitui uma metonímia e a expressão “ferida
dos que colhem uvas e mangas em minhas da loucura” (l. 12 e 13), uma metáfora.
margens. Dos que colhem arroz em minhas b) o conectivo e nas linhas 12 e 13 tem valor
várzeas, dos que criam peixes com minhas águas adversativo e na linha 14, valor aditivo.
em açudes feitos. A fé dos xocós lá em Poço c) nas expressões “emprego rarefeito” e “dignidade
Redondo. A fé que cria cabras nos Escuriais. Dos estuprada” (l. 15), o uso das formas nominais
que colhem cajus e criam gado em Barreiras e possui valor passivo, no entanto, o agente da ação
outros cafundós. verbal é entrevisto no contexto.
d) do enunciado “O bom senso dos conselheiros de
Onde houver o erro dos governantes que eu leve a encontro à sanidade dos totalitários” (l. 10 e 11),
verdade de Canudos. O bom senso dos pode-se inferir que o bom senso se contrapõe à
conselheiros de encontro à insanidade dos insanidade.
totalitários. Os canhões abrindo fendas na cidade
sitiada e a verdade expondo cada vez mais a ferida 24) Considerando o 2o fragmento do Texto VI, é
da loucura na caricatura da História. O confisco da correto afirmar que
poupança e o rombo na previdência. O fim da a) refere-se apenas às questões históricas ocorridas
inflação e o pão escasso, o emprego rarefeito, a em Canudos.
dignidade estuprada em cada lar de nordestinos. b) a responsabilidade de todos os problemas nele
citados é exclusivamente dos governantes.
Onde houver a tristeza dos solitários que eu leve a c) os pontos empregados poderiam ser substituídos
alegria das festas de São João. Solitário eu banho por vírgulas e isso não acarretaria nenhum prejuízo
muitas terras e em todas, das Gerais, do sintático ou semântico.
Pernambuco, das Alagoas e do Sergipe, não há d) o fragmento faz uma crítica social, política e
tristeza ao pé da fogueira, nas núpcias entre a histórica aos problemas vividos pela comunidade
concertina e o repente, entre a catira e o baião. Das nordestina.
festas do Divino ao Maior São João do Mundo,
deixai-me levar, Senhor o sabor de minhas águas 25) Assinale (V) verdadeiro e (F) falso, e, a seguir,
juninas e seus fogos de artifícios. assinale a alternativa
(http://adercego.blogsome.com/2006/12/04/oracao- correta em relação ao 1o fragmento do Texto VI.
do-rio-sao-francisco-emtempos-de-poucos-rios - ( ) Os sintagmas “que constroem o seu tempo” (l.
Acesso em 15/05/08 às 14h) 2) e “que cria cabras” (l. 6 e 7) exercem a mesma
função restritiva em relação ao vocábulo fé.
22) Sobre o 3o fragmento do texto acima, é correto ( ) No último período do fragmento, há um
afirmar que as/o(s) expediente de coesão, a elipse, que retoma um
a) sintagma “nas núpcias entre a concertina e o termo facilmente depreendido do contexto.
repente, entre a catira e o baião” (l. 20 e 21) exerce ( ) Em “...a fé dos que colhem uvas e mangas em
uma função adverbial e apresenta uma catacrese minhas margens” (l. 3 e 4) o vocábulo que tem
como recurso expressivo. como referente “os” e introduz uma oração com
função adjetiva.
149
PROVAS DA AFA 1998 – 2010
( ) Em “...a fé dos que constroem...” (l. 1 e 2) o
vocábulo fé possui um sentido passivo e em “...a fé
que cria cabras...”, (l. 6 e 7) um sentido ativo.
( ) Há, na linha 1, o uso da antítese que se repete
em todos os outros períodos.
a) F – F – V – F – V c) V – V – F – V – F
b) F – V – V – V – F d) V – F – V – F – F

AFA 2009/2010 – Português


Leia o Texto I para responder as questões de 1 a 9.

Texto I
Darwin no Brasil

Encanto com a Natureza e choque com a


escravidão Na passagem pelo Brasil,
especialmente no Rio de Janeiro, Darwin
descobre um mundo novo de sedução e (CAPAZZOLI, Ulisses. Scientfic American
horrores. Brasil. Fev. 2009, nº 81, ano 7. Edição Especial.
Pág.90.(Adaptado)

1) Da leitura do texto, conclui-se que


a) a viagem iniciada em 27 de dezembro foi longa
e sem transtornos, contudo revolucionária.
b) em 1832, apesar de bastante jovem, Darwin já
acumulava alguns sucessos profissionais.
c) a exuberante natureza tropical fez com que
Darwin se apaixonasse por Salvador.
d) a estada no Rio de Janeiro foi o momento mais
marcante da viagem, por despertar em Darwin
intensos sentimentos.

2) Assinale a alternativa que apresenta a relação


INCORRETA entre o parágrafo e a ideia nele
contida.
a) O 1º parágrafo justifica a hipótese de que o
Brasil não estava nos planos de Darwin, que
somente chegou aqui por um desvio de percurso.
b) O 2º parágrafo apresenta o grande cientista da
teoria do evolucionismo.
c) No 3º parágrafo, fica patente o encanto de
Darwin diante da diversidade da natureza do Rio
de Janeiro.
d) No 6º parágrafo, o autor descreve a paisagem e
os ruídos da mata.

3) De acordo com o texto, entende-se que Darwin.


a) era, desde a mocidade, um evolucionista
convicto.
b) se encantou com o Brasil, escandalizando-se
apenas com questões de natureza antropológica.
150
PROVAS DA AFA 1998 – 2010
c) presenciou um ato aterrador: a morte de uma
velha que se atirou contra as rochas. 7)Assinale a opção cuja justificativa para o uso dos
d) desejou permanecer no Brasil, depois de se termos sublinhados está correta.
surpreender com a natureza e a vida social intensa. a) No trecho “Em Viagens de um naturalista ao
redor do mundo (...), em que faz um detalhado
4)Leia o trecho abaixo, considerando-o no registro...” (ℓ. 29 a 31), a expressão em que pode
parágrafo em que ele se insere. ser trocada por em cujo, já que ambas são
compostas de pronomes relativos.
“Praticado por uma matrona romana esse ato seria b) Em “Charles Robert Darwin... cujo nome seria
interpretado e difundido como amor à liberdade, sinônimo de evolucionismo...” (ℓ. 12 a 15), o
mas da parte de uma pobre negra, limitaram-se a pronome relativo cujo traz uma ideia de posse e
dizer que não passou de um gesto bruto.” (. 62 a sempre concorda em gênero e número com a
68) palavra que o antecede.
c) No período “A incursão começou em 8 de abril,
É possível afirmar que, no fragmento acima, o formada por uma equipe de sete pessoas.” (ℓ. 43 e
autor utiliza uma estratégia argumentativa 44), o verbo formar está no particípio e introduz
denominada. uma oração substantivada reduzida.
a) especificação. c) exemplificação. d) Em “Um personagem, que a história tornaria o
b) conceituação. d) citação. passageiro mais importante a bordo do Beagle...”
(ℓ. 8 e 9), o tempo do verbo denota um estado não
5) Os termos abaixo destacados representam lugar. concluído no passado.
Porémadverbial. Assinale-o.
a) “..., para uma viagem de quatro anos e nove 8) Assinale a opção em que a palavra entre
meses ao redor do mundo.” (ℓ. 5 a 7). parênteses substitui a destacada sem alterar o
b) “... à exceção de uma mulher, já velha, que se sentido da expressão original.
atira contra as rochas.” (ℓ. 60 e 61). a) “... havia sofrido alguns reveses profissionais
c) “Em Viagens de um naturalista ao redor do antes de se envolver com a história
mundo, (...) em que faz um detalhado registro de natural.”(insucessos) (ℓ. 10 a 12).
sua longa exploração...” (ℓ. 29 a 32). b) “Mas foi no Rio de Janeiro, especialmente por
d) “Com a lua cheia, que nasce cedo no céu, o uma incursão de alguns dias pelo interior...”
grupo decide prosseguir viagem para dormir. (dispersão) (ℓ. 24 a 26).
na Lagoa de Maricá...” (ℓ. 52 a 54) c) “Praticado por uma matrona romana esse ato
seria interpretado e difundido como amor à
6) Assinale a opção correta. liberdade...” (escrava) (ℓ. 64 a 66).
a) De acordo com a norma padrão da língua em d) “Darwin dedica menos de dez páginas a
“Um grupo de soldados teria sido enviado...” (ℓ. 58 Salvador, na Bahia, aonde chegou... mas já
e 59), a flexão do verbo ter pode ser feita no plural, fascinado pela exuberância da natureza tropical”.
concordando com o substantivo soldados. (singeleza) (ℓ. 32 a 35).
b) No período “Em 4 de abril, o Beagle atracou no
Rio de Janeiro e aí começaram as descobertas 9) Assinale com V (verdadeiras) ou F (falsas) as
que...” (ℓ. 36 e 37), o sujeito do verbo começar está afirmativas abaixo e, a seguir, marque a alternativa
indeterminado. correta.
c) Em “Então, ele faz uma das observações que ( ) Os vocábulos arquipélago, Atlântico e sinônimo
revelam sua profunda repulsa à escravidão”(ℓ. 62 e são acentuados pela mesma razão.
63), o verbo revelar foi flexionado de acordo com ( ) A presença de encontros vocálicos é observada
o núcleo do sujeito. nas palavras, a seguir, Lagoa, exceção e Praia.
d) Na oração “A vista e as cores na passagem de ( ) Estão corretas as seguintes divisões silábicas: ve
Praia Grande (atual Niterói) absorvem toda a – lha, pros – se – guir, I – tha – ca – ia,.
atenção de Darwin...” (ℓ. 47 a 49) o verbo absorver ( ) Os vocábulos: três, canhões e Niterói recebem a
pode ficar no singular, concordando com o núcleo mesma classificação quanto à posição da sílaba
do sujeito. tônica.
a) F – V – F – V. c) V – V – V – F.
151
PROVAS DA AFA 1998 – 2010
b) V – V – F – F. d) F – F – V – V.

Leia o Texto II para responder as questões de 10 a


20.

Texto II

Desconforto com a Escravidão

CAPAZZOLI, Ulisses. Scientfic American Brasil.


Fev. 2009, nº 81, ano 7. Edição Especial – Pág.92-
93 (Adaptado)

10) O título do texto remete-nos à ideia de que


a) o Brasil, apesar de ser um país livre, ainda
permitia a escravidão em alguns momentos.
b) Darwin ficou incomodado com algumas cenas
presenciadas por ocasião de sua estada no Brasil.
c) no Brasil, a natureza foi o único aspecto
apreciado por Darwin.
d) os negros participaram da expedição como
simples carregadores do material necessário para a
pesquisa de Darwin.

11) Assinale a alternativa que apresenta uma


inferência FALSA.
a) A exuberância da natureza brasileira prejudica
os trabalhos de caracterização daqueles que se
interessam por história natural.
b) O evolucionismo, para vir à tona, exigiu de seu
criador coragem e desprendimento de sua vida
pessoal.
c) A agressão era, no Brasil, um tratamento
rotineiro nas relações do branco com o negro.
d) A estada de Darwin no Rio de Janeiro foi
marcada basicamente por profundos sentimentos
de vergonha e desconforto.

12) A partir das ideias presentes no texto, assinale


com (V) as assertivas verdadeiras ou com (F) as

152
PROVAS DA AFA 1998 – 2010
falsas e, a seguir, marque a alternativa a) As aspas simples, usadas para introduzir a fala
correspondente. de Darwin, iniciam o discurso indireto livre.
( ) A dificuldade do percurso relaciona-se à b) Se o substantivo cavalo fosse substituído por
ingremidade do terreno. outro feminino, como, por exemplo, carroça, a
( ) A falta de comunicação entre Darwin e os ocorrência da crase seria obrigatória.
habitantes da terra leva-o a uma atitude violenta. c) No trecho “o caminho ficou cada vez mais
( ) O encantamento com a fauna e flora brasileira difícil...”, o verbo ficar é intransitivo.
são exaltados no penúltimo parágrafo. d) A palavra como pode ser substituída, sem
( ) Mais tarde, Darwin retornou ao Brasil prejuízo semântico, pela preposição por.
reforçando suas pesquisas sobre o evolucionismo.
a) V – V – F – F c) F – F – V – F 16) Considere as afirmativas a seguir.
b) V – F – F – V d) F – V – V – V I. O vocábulo verbal relata (ℓ. 2) exige uma
complementação, no caso, um termo oracional.
13) Assinale a alternativa INCORRETA. II. Êxtase e espécime (ℓ. 38) são vocábulos
a) Darwin acredita que nem sempre os atos determinantes que se referem à Natureza.(ℓ. 38)
tirânicos cometidos contra os escravos eram III. Cartas angustiadas (ℓ 59) denotam um estado
conscientes. emocional do emissor.
b) O tetraneto de Darwin interessou-se em Estão corretas apenas
percorrer a mesma rota do naturalista por a) I e II. c) I e III.
interesses econômicos. b) II e III. d) I, II e III.
c) As descobertas são numerosas, principalmente,
em regiões com fauna e flora tão rica. 17) Assinale a alternativa em que a análise da
d) Darwin fica maravilhado com sua estada no Rio palavra sublinhada está correta.
de Janeiro e impressiona-se com a exuberância do a) “O retorno ao Rio marca uma nova fase de
Brasil. êxtase...” (ℓ. 37 e 38) - (Palavra substantivada,
proveniente de derivação imprópria).
14) Leia as proposições abaixo. b) “O negro abaixou imediatamente as mãos,
I - A principal intencionalidade discursiva do semicerrou os olhos e dirigiu-lhe um olhar
locutor do texto é conscientizar o leitor do horror temeroso...”(ℓ. 30 a 32) (Pronome pessoal na
da escravidão. função de objeto indireto)
II - O futuro do presente foi utilizado no último c) “... os interessados em história natural têm
parágrafo para indicar ações realizadas num vantagem no sentido de que sempre
momento futuro em relação ao narrado. descobrem...”(ℓ. 45 e 46) (Elemento de coesão que
III - As descobertas em Galápagos deram origem retoma a palavra sentido).
ao evolucionismo, que surpreendeu e assustou o d) “Nesse período, Darwin ficara hospedado em
próprio Darwin. uma casa à beira-mar (cottage) em Botafogo.” (ℓ.
IV - O conectivo e (ℓ. 16) possui valor adversativo 39 e 41) (Particípio do verbo hospedar que,
e introduz uma crítica de Darwin ao tratamento juntamente com o verbo ficar, forma voz passiva).
dado aos escravos.
Estão corretas apenas 18) De acordo com a pontuação do texto, é
a) I, III e IV. c) II, III e IV. INCORRETO afirmar que as vírgulas do trecho.
b) II e III. d) I e IV. a) “O grupo cruzava um rio, provavelmente, o rio
Macaé, conduzido por um negro, quando...”
15) Leia o trecho e, a seguir, assinale a opção separam, respectivamente, circunstância adverbial
correta. e oração reduzida.
b) “Num desses movimentos, conta ele, suas mãos
Nas últimas horas de viagem a cavalo, Darwin passaram próximo ao rosto do homem...” separam
relata que ‘o caminho ficou cada vez mais difícil, uma oração intercalada.
porque atravessa uma terra selvagem e pantanosa’. c) “... Darwin relata um desencontro, que não
O grupo viaja na região hoje conhecida como Serra detalha, e que...” separam uma oração adjetiva
da Tiririca... (adaptado). explicativa.

153
PROVAS DA AFA 1998 – 2010
d) “Insetos luminosos, provavelmente vaga-lumes, AFA 1997/1998 – Inglês
segundo o relato, voam em torno...” separam,
respectivamente, aposto e oração intercalada. Instructions for questions 1 to 32.
Mark the alternative which best answers the
19) Sobre o modo como as ideias no texto se item.
relacionam, é correto afirmar que a.
a) expressão ainda que (ℓ. 57) introduz uma 1. Jane took the book although she knew it was
concessão ao período que a precede. _____ and she left it an hour ago with a friend of
b) conjunção que (ℓ. 46) inicia uma oração ______.
adverbial causal. a) mine / hers
c) conjunção porque (ℓ. 3) antecede uma oração b) my own / her
adverbial final. c) my / his own
d) palavra como (ℓ. 59) traz uma comparação ao d) me own’s / hers own
período no qual está inserida.
2. They intended to keep all the money for
20)Dentre as análises abaixo, assinale a _______ Jack ________ gave me this information.
INCORRETA. What is going to happen to ______ now ?
a) No período “...descobertas novas são feitas a a) they / his own / ours
cada instante e são tão numerosas que só se pode b) themselves / himself / us
avançar com dificuldade.” c) theirselves / himself / we
(ℓ.49 a 51), a conjunção adverbial estabelece uma d) they own / by him / ourselves
relação de conseqüência com a oração anterior a
ela. 3. She _____ by the end of the next year, when I
b) Em “Darwin relata o profundo sentimento de ____ to Europe.
surpresa, desconforto e vergonha que se apoderara a) will have graduating / travel
dele e que jamais b) will be graduating / will be traveling
iria esquecer.” (ℓ. 32 a 35), ambos os pronomes c) will can be graduating / am traveling
relativos destacados substituem a palavra d) had been graduating / had already traveled
sentimento e são, respectivamente, classificados
como sujeito e complemento verbal. 4. _____________ you survive in that forest ?
c) No trecho “O negro tinha alguma dificuldade de I had a survival course three years ago so I knew
comunicação, o que fez com que Darwin tentasse how to get food.
comunicar-se com ele por mímica...” (ℓ. 24 a 26) o a) How did
pronome se é uma partícula de realce que pode ser b) Why didn’t
retirada sem prejuízo sintático. c) What can
d) Em, “...os interessados em história natural têm d) Who could
vantagem no sentido de que sempre descobrem
alguma coisa que lhes chama a atenção...”,(ℓ 45 a 5. You _______ say such bad things about your
47) o pronome lhes refere-se aos interessados e, parents.
sintaticamente, completa o substantivo atenção. a) could to
b) have got
c) shouldn’t
d) don’t ought to

6. I - _____ the boys pass or they fail.


II -_____ being run over many times, he became
more careful.
III - The dog _______ barks does not usually bite.
IV - She joined a military academy _______
become an officer.
a) Whether / Which / after / either
b) Either / After / which / in order to
154
PROVAS DA AFA 1998 – 2010
c) After / Either / in order to / which b) loves
d) Which / Before / after / in order to c) hates
d) used to
7. The airplane will land ______ half an hour.
a) by off 14. Robert never had an accident though he drives
b) at here very _____.
c) through a) fast
d) in about b) slow
c) carefully
8. England and France have had powerful military d) cautious
forces but the United States has _____ powerful
forces of them. 15. Cabral _________ Brazil many years ago and
a) more his discovery opened new gates for Portugal,
b) the most __________ ?
c) many more a) discovered / didn’t it
d) much more b) has discovered / did it
c) had discovered / didn’t him
9. A house-breaker could easily go into that d) has been discovered / did they
building because of ________ windows were
broken, but he might have hurt ________ The 16. We can’t do without him, _____________ ?
police could see blood on the ground. They are a) can us
going to find out what has happened to _______. b) can we
a) his / itself / his c) can ours
b) their / itself / it d) can’t he
c) my / myself / it
d) its / himself / him 17. By the time we got to the cinema, the movie
_______ already ________ .
10. Michael played the piano _________ when he a) had / started
was _________. Now he stopped practicing. b) has / started
a) better / younger c) didn’t / started
b) very well / newest d) would / started
c) well / more young
d) more right / more young 18. If
he likes that van, he ________ buy it or he
_______ never take that long trip.
11. He could play tennis very well but today he a) will / will
doesn’t play it ________ he used to b) could / had
c) would / would
a) as often as d) can / wouldn’t
b) more often as
c) more frequently 19. IfI had _____ harder, I ______ have ______
d) the most frequently that the Math test.
a) studied / would / fail
12. I like that teacher. He explains things very b) study / would / passed
_______. c) study / wouldn’t / failed
a) slow d) studied / wouldn’t / failed
b) easy
c) clearly 20. The old lady looks for ____ can give her minute
d) intelligent of attention.
a) whoever
13. He ___________ smoke a lot but he stopped b) whatever
because he was running a big risk. c) wherever
a) has d) whichever

155
PROVAS DA AFA 1998 – 2010
a) Why are you here so late ?
21. Bob’s opponent was hitting him hard but Bob b) Why are you there so late ?
didn’t ________ . He was sure of ________ his c) What have you been doing?
own record. d) What are you doing here so late ?
a) take in / get over
b) break up / giving up 28. I________ ten days in Europe in my last
c) give up / breaking up vacation and I _______ it was wonderful.
d) take back / get through a) spend / think
b) spent / thought
22. _____ man I don’t know has called you but c) had spended / thought
didn’t leave any message. ______ man just told me d) spended / have thought
he’d call again during __________ week.
a) An / A / an 29. -I can’t lend you my car this weekend.
b) A / The / the - What did he say ?
c) The / A / the - He _________ this weekend.
d) The / The / a a) said he can’t lend you my car
b) said he cannot lend you his car
23. How do you write the numbers 1572 and 339 in c) said you he can’t lend you his car
full ? d) told us he cannot lend you my car
a) one thousand and fifty two ; three thousand
thirty nine 30. Jennifer,__________ , didn’t do a good job but
b) one hundred thousand five hundred seventy two the _______ was very well in that movie.
; three and thirty nine a) the actriss / artist
c) one thousand five hundred and seventy two ; b) the actrix / author
three hundred and thirty nine c) the actress / actor
d) one thousand five hundred seventeen two ; three d) the actoress / actor
hundred and thirteen nine
31. Unknown to her parents, Daisy was at a
24. _____ he ____ studied _____ have _____ his Christmas party at the office last night. She hadn’t
examination. come back home until 4 a.m. There was a
a) If / hasn’t / would / pass telephone set in the office, so _________.
b) If / had / wouldn’t / passed a) She can hardly phone them tonight
c) If / hadn’t / wouldn’t / passed b) She will probably phone them later on
d) When / had / wouldn’t / pass c) She should have forgotten going back home
d) She could have phoned that she was going to be
25. You should have called to say you’d be late. late
I _________ but there was nobody home.
a) did call 32. Which sentence is correct ?
b) tried call a) Never I have heard your name.
c) can’t call Not once he did mentioned it.
d) try phone b) I have heard your name never.
Did not once he mention it.
26. The victim ______ walking along the bridge c) I have never heard your name.
with a tall man. Once not he did mention it.
a) has seen d) Never have I heard your name.
b) was seen Not once did he mention it.
c) didn’t see
d) wasn’t seeing Instructions for questions 33 to 35.
Read these sentences and choose the right
27. The sentence Joe asked what I was doing there alternative.
so late. can be equivalent to the following direct
speech:
156
PROVAS DA AFA 1998 – 2010
33. I- Jane traveled to San Francisco by train last armored regiment is based nearby, as is one of its
January. key infantry and tank-training camps.
II - Jane traveled by train to San Franscisco last Nevertheless, the Khmer Rouge force was able to
January. hit it along a kilometer-long front for 40 minutes
III - Last January Jane traveled to San Francisco before it was driven off by the district’s defenders.
by train. Only two people were killed, but the rebels
IV - To San Francisco Jane traveled by train last overran the clinic and carried all the medicine
January. away.
a) Only sentence II is wrong.
b) Sentences I and III are correct. The gunfight at Phnom Sruoch
c) Sentences II and Iv are correct. demonstrated the Khmer Rouge’s growing courage
d) Sentences III and IV are correct. - and its increasing ability to attack near the center
of the government supported by the Vietnamese.
34. I - From 8 to 10 p.m. frequently Robert studies. Khmer Rouge fighters - still under the command of
II - Robert frequently studies from 8 to 10 p.m. Pol Pot, whose government the Vietnamese
III - Robert frequently from 8 to 10 p.m. studies. extinguished in 1979 - don’t intend to take power.
IV - Robert studies frequently from 8 to 10 p.m. Yet in the 11 months since Hanoi moved the bulk
a) Only sentence I is correct. of its combat forces and military advisers out, the
b) Only sentence II is correct. Khmer Rouge has gone deeper and deeper inside
c) Only sentence III is wrong. the country. The guerrillas have taken several
d) Sentences III and IV are correct. thousand kilometers along the Thai border and
now have two new strategically located targets:
35. I- My brother were sent to a new station by the Kampong Spew and Kampong Thom.
commander.
II - Scarlet was promised a new computer for 36. The guerrillas have ____________
Christmas. a) lost some battles
III - All the students must be told that the new b) been losing for 11 months
teacher is to arrive in an hour c) shown courage of its Air Force
IV - Great ! Where the doctor has being gone all d) been having important victories
those weeks !
a) I and II are correct. 37. Kampong Spew and Kampong Thom are two
b) I and III are correct. ____________
c) I and IV are correct. a) guerrillas’ bases
d) II and III are correct. b) guerrillas’ commanders
c) towns the soldiers attacked
Instructions for questions 36 to 40. d) towns the guerrillas intend to attack
Read the text below and choose the best
alternative to complete the statement. 38. Phnom Sruoch’s market, health clinic and
military and administrative offices were
THE KHMER ROUGE IS MOVING DEEPER __________________
INSIDE CAMBODIA - BUT IT IS NOT ABOUT a) targets hit by the guerrillas
TO TAKE POWER b) burned one morning last month
c) Phnom Sruoch’s defense forces
You don’t have to travel far from d) a difficult target for the Cambodians
Phnom Penh to see the war. Early one morning last
month Khmer Rouge guerrillas attacked the well- 39. Because of the two hundred and fifty soldiers’
defended town of Phnom Sruoch - just 70 attack, Phnom Sruoch’s inhabitants may have
kilometers from the Cambodian capital. Some 250 ________________
rebel soldiers opened fire with automatic weapons a) to run over the clinic again
and B-40 rockets on the town’s market, health b) serious health assistance problems
clinic and military and administrative offices. c) made the people lose their courage
Phnom Sruoch is not an easy target: a government d) more attacks by the district’s defenders

157
PROVAS DA AFA 1998 – 2010
40. We could observe that the Khmer Rouge’s men 6. The doctor advised her ________________
________________. while pregnant.
a) moved the bulk of its forces a) stops to smoke
b) have no intention to take power b) to stop smoking
c) command the Vietnamese Forces c) to stop to smoke
d) set fire on Cambodian automatic weapons stopping smoking

7. Read the text below:


AFA 1998/1999 – Inglês Paul McCartney eulogized his wife, Linda, before
more than 700 mourners - including Sting, Elton
John, and former Beatles George Harrison and
1. Only one of the words from the item has O with
Ringo Starr - at a memorial service Monday
the sound /ou/. Which is this word ?
evening at a West End church. “She was my
a) lover
girlfriend. I lost my girlfriend. I still can’t believe
b) cover
it, but it’s true. I have to believe it.” McCartney
c) clover
said Linda, 56, died of breast cancer in April.
d) discover
Observe the underlined words. The pronoun IT
refers to:
2. Every word in this item, except one, rhymes
a) I have to believe.
with weak. Identify this word in which the EA
b) I still can’t believe.
sounds like /ei/.
c) I lost my girlfriend.
a) leak
d) She was my girlfriend.
b) beak
c) steak
8. Choose the WRONG comparison.
d) peaky
a) The more he thinks, the more he smokes.
b) The most he has got, the more he wants to get.
3. Complete the following text:
c) The diamond was worth more than its high
And the World Cup is ________ from a global
price.
economic perspective. It generates ________ of $
d) As the time went by she became more and more
428 million for sponsor rights and $ 450 million
beautiful.
for TV rights. Licensed merchandise _______ are
projected at $ 1.2 million.
Based on the text below answer questions 9 and
a) big / taxis / sells
10.
b) huge / fees / sales
“Defense Secretary William Cohen refused
c) huge / rates / sizes
Monday to segregate male and female recruits in
d) gigantic / changes / prices
Army, Navy and Air Force training camps. After
nearly six months of review, Cohen announced
4. Choose the correct alternative.
training the sexes together from the start
The ship struck an iceberg, which tore a huge hole
__________(I) prepare troops to work together
in _______ side.
later. ______(II) 14%______(II) the force female,
a) its
“we cannot run a military today ________(II)
b) his
women, ”Cohen said.”
c) her
d) their
9. Choose an alternative to complete blank number
(I) on the text.
5. “(...) still in the doorway, I heard a mission bell
a) was a good way to
and I was thinking _________________ this could
b) were better ways to
be heaven or this could be hell.”
c) was the best way for
a) with me
d) were the best way to
b) of myself
c) to myself
d) about my own
158
PROVAS DA AFA 1998 – 2010
10.Complete the blank marked with number (II) 15.Complete the text.
with prepositions: “Mr. Brooks __________ well since last year and
a) at / in / on ________________ help from any doctor until he
b) with / from / over _______________ at the dinner party.”
c) with / of / without a) had felt / had sought / fainted
d) both / in / without b) hasn’t felt / as sought / has faint
c) hasn’t felt / hasn’t sought / fainted
11.This hot weather has affected my d) had felt / hadn’t sought / had been fainting
___________________.
a) concentrated ability 16.The passive voice is correct in:
b) ability concentrating I- 719.521 firearms had been imported by the
c) ability concentration USA since 1991.
d) ability to concentrate II- 719.521 firearms were imported by the USA in
1991.
12.Complete the text. III-719.521 firearms have been imported by the
“Airbus’ success _______________ its US USA since 1991.
competitors and could cause a bitter transatlantic IV-719.521 firearms have being imported by the
trade war. Jean Pierson _________ like a true USA in 1991.
capitalist. “We are in this business to build aircraft a) all sentences above
and make money,” _________ the head of Airbus b) sentences I and II only
Industrie”. c) numbers I, II and III only
a) worry / to talk / say d) numbers II, III and IV only
b) worries / talks / says
c) has worried / talks / to say 17.Use the right tense of the verbs in parenthesis to
d) has been worrying / talking / saying complete the text below:
“The comic-book Superman _________ (to die)
13.Complete the text. about three years ago ________ (to kill) in a
“Of course, they are no angels, but they colossal fight with an alien archfiend named
__________________ us pretty well and they Doomsday. Seven issues later he __________ (to
__________________ to kill us, like the Russian.” be) back in action, resurrected by an artist’s ink
(Yelena Kapran, one of the hundreds of Russian and imagination. If only silver-screen Superman
civilians taken hostage in Budyonnovsk, after her Christopher Reeve could be as lucky. Reeve, 42,
release by Chechen Guerrillas.) _____ (to lie) last week in a Virginia hospital,
a) cared / attacked paralyzed and breathing only with help of a
b) treated / didn’t try respirator after he __________ (to injure) himself
c) haven’t treated / haven’t tried in an equestrian competition over the holiday
d) have worked / haven’t shooted weekend.”
a) died / killed / was / lay / injured
14.Complete the text. b) died / killed / was / lied / injuring
“City residents tired of noisy car alarms that go off c) has died / killing / is / lied / injured
at all hours of the night _____________ this: an d) died / killed / has been / lied / injured
auto-security system that uses smoke, not noise.
Called the Dragon Vehicle Defense Machine, it 18.Complete the text below.
_____________________ robbery by filling the “A bright light ______ the plane”, __________ Lt.
car with a cloud of smoke so dense that the thief Col. Paul Tibbets, the pilot of the Enola Gay, the
can’t see. It ___________ at car stores in June. B-29 that _________ the first atomic bomb. “We
Cost 35 dollars.” _______ back to look at Hiroshima. The city was
a) love / will prevent / is hidden by that awful... cloud boiling up
b) will love / prevents / will be mushrooming.”
c) won’t love / is preventing / won’t be a) firing / told / made / went
d) loving / is going to prevent / is being b) filled / wrote / dropped / turned
c) shone / said / exploded / come
159
PROVAS DA AFA 1998 – 2010
d) fired / described / dropped / got c) be referred to as her daughter’s mother.
d) be called by the name of her daughter’s.
19.Complete the text below.
“We ____________________ you today which 23.A lot of young people in the USA
players will start the game against Scotland. We a) believe in the future with great hope.
____________________ a secret as long as we b) are drug addicted and feel little hope.
can,” said Brazilian soccer team’s coach Zagallo c) will follow Chelsea’s solutions themselves.
when asked about Brazil’s line-up. d) believe that a plague will get the teenagers.
a) told / were keeping
b) will not tell / might keep 24.In the text Chelsea:
c) had told / are going to keep I- uses the floor to sit down.
d) are not going to tell / will keep II- made many questions about the problems in the
USA.
Based on the text below, answer questions from 20 III-creates a shelter in a fan club of her own.
to 24. IV-was asked about young Americans’
In Masai culture, when you really want to honor a problems.
woman, you refer to her as the mother of her oldest V- said the Clintons can make the future.
daughter. So last week, when a group of Masai a) Only one sentence is true.
schoolgirls in northern Tanzania held up a sign b) Only two sentences are true.
saying KARIBU MAMA CHELSEA, it c) Only two sentences are false.
_____________(I) more than just “Welcome d) Four of the sentences are true.
Chelsea’s mom”. Hillary and Chelsea Clinton, on a
good-will tour trip of several African countries, 25.Considering that the papers were locked in the
understood the compliment. They also understood desk we can say that:
that the sheltered First Daughter is developing a I- She locked up the papers in the desk.
fan club of her own. Mrs. Clinton quickly noted II- She locked the papers up in the desk.
that the Masai girls III-She locked up them in the desk.
________________________(II) to someone IV-She locked them up in the desk.
“more active than her mother,” and Chelsea took V- She locked up in the desk the papers.
floor with ease. Asked about the problem of a) All the sentences are correct.
American youth, she mentioned the drug abuse and b) Only two sentences are correct.
the ”hopelessness and cynicism” that plague many c) Only four sentences are correct.
of her fellow teenagers. “The solutions ultimately d) Only three sentences are correct.
have to come from the youth people themselves,”
Chelsea concluded confidently. “We are the future, 26.Choose the best structured sentence.
and we make of our future what we make of it.” a) Never I have a cigarette before breakfast.
b) Before breakfast I have never a cigarette.
20.The verbal form to complete blank (I) is: c) I never have a cigarette before breakfast.
a) means d) Never before breakfast I have a cigarette.
b) meant
c) will mean 27.Considering the sentences below mark the
d) has meant correct alternative.
I- The librarian recommended that he return the
21.The blank (II) is going to be filled with: book sooner.
a) may enjoy talking II- What does he suggest that John do?
b) might enjoy talking III-Perhaps the soldier wants to visit the museum.
c) may to enjoy talking IV-It is necessary that the lady see a doctor at
d) might enjoyed to talk once.
a) Only number III is correct.
22.The honor to a woman is to b) All of them are not correct.
a) refer to the daughter of her mother’s. c) All these sentences are right.
b) have the same of her daughter’s name. d) Only numbers I and II are right.
160
PROVAS DA AFA 1998 – 2010
next room. He’d been having this problem for
28.Given: almost two hours and he finally had enough of it.”
- When can I see you again to decide this ? a) He walked up to the door, opened it and said:
- Meet me ______________________. Can you turn for God’s sake this music off? You
I. by the post office know I’m studying.
II. tomorrow afternoon b) He walked up to the door, knocked, put his head
III.at a quarter to four inside and said: Why don’t you turn that off?
IV.on the corner c) He walked up to the door, knocked, opened it
V. sharp and said: Turn off, I told you I had to study!
Choose the correct alternative which orders the d) He walked up to the door, knocked, opened it
adverbial phrases to complete the meaning of the and said: Would you mind turning the radio off? I
sentence above. need to study.
a) I - V - III - IV - II
b) III - IV - II - V - I 32.My mother said to me : “Don’t leave your little
c) IV - I - III - V - II brother alone!”
d) III - II - V - I - IV The reported speech of the above sentence is:
a) My mother told my little brother not to be left
29.“Come back to me and you will really know alone.
what happiness can be.” means b) My mother told me don’t leave my little brother
a) if you come back to me, you will know what alone.
happiness can be. c) My mother said me to not leave my little
b) if coming back makes you happy, it could bring brother alone.
happiness to me. d) My mother told me not to leave my little
c) if you come back to me, you would know what brother alone.
happiness could be.
d) come back to me or else you won’t know what 33.“(...) and so all the pilots decided not to fly that
could be happiness. strange day, and it was extremely lucky because
we had the most heavy thunderstorm of the year in
30.Complete the text below. that region.”
“Deaths among men _______ took the wildly From the text we can say:
popular impotence pill Viagra have climbed to 16, a) that strange day was extremely lucky, and all
including seven men ________ reportedly died the pilots decided not to fly then.
during or after sex, the Food and Drug b) all the pilots decided not to fly that day, which
Administration said Tuesday. The FDA said there was extremely lucky for them.
is no evidence ________ Viagra itself is c) all the pilots decided not to fly that day which
dangerous, but again warned ________ was extremely lucky, in the year.
nitroglycerin and the impotence drug are a d) that lucky day which was extremely strange,
dangerous mix.” had the most heavy thunder storm of the year.
a) that /  / who / that
b) who / which /  / who 34.Read the text below.
c) which / who / that /  “For three and a half years, the Bosnian Serbs
d) who / who / that / that strangled Sarajevo in the belief that one day it
would be theirs. From outlying districts of the city,
31.Read the following text and choose the best they cut off electricity, gas and water and shelled
construction that Johnny should use to make his the government, killing more than 10,000 people.
friend obey him in a clear imperative form. Serb marksmen in the Grbavica district picked off
“Johnny had a very important test for a job the hapless civilians a few hundred yards away on a
next day. If he passed, he’d get himself a good job thoroughfare that came to be known as Aleja
and the chance to make some real good money. He Snajpera (Sniper Alley). Then, in Dayton, the
had a notebook in his hands, but couldn’t Serbs lost it all. The peace agreement made
concentrate because of the music coming from the Sarajevo a united city, ruled by Muslim-
dominated-Bosnian government. Now, because
161
PROVAS DA AFA 1998 – 2010
they are Dayton’s biggest losers, the Serbs who d) if you are able to see how different is a green
besieged Sarajevo could become the biggest field from a cold steel rail.
troublemakers as NATO troops try to enforce the
peace agreement. Some Serbs promise to fight. Read the text below and answer questions 37 to 40
Others say they will move away, leaving scorched
earth behind them. “We will never live with LUSTING AFTER WALL STREET
Muslims,” vows Srpko Tobica, a 52-year-old If you have a big piece of your money in the stock
soldier.” market these days, you must feel the way Hugh
Based on the text you can say that Grant felt when he went for his famous drive down
a) Aleja Snajpera was known as a city after that. Sunset Boulevard: your brain is telling you to play
b) the peace agreement besieged Sarajevo at all. it safe and go home, but your lust is so great that
c) Sarajevo received its supplies from the Serb you can’t help yourself.
district. The lust we’re talking about, of course, is money
d) some Serbs would burn the earth they were lust. The market has risen so far so fast - the 30-
leaving. stock Dow Jones industrial average up 23 percent
for the year through Friday, the broader Standard
35.Choose the correct alternative to complete the & Poor’s 500 up 22 percent - that almost anyone
text. who’s been in the market has made a ton of money
“In the beginning the Earth was inhabited only by this year. Including dividends, the gains through
men. At that time ________ gods Zeus and last Friday work out a rate of around 45 percent a
Prometheus had ________ quarrel because Zeus year. If things get a little hotter, 1995 could
had hidden fire away from the men. However, become the best year in market history, topping the
Prometheus found it and took it back to Earth. sap’s 53 percent return in 1993. This kind of thing
Zeus got very angry with _________ Prometheus, makes investors with a sense of history very
and decided to take revenge on him and all men. jumpy, because it’s too good to last. You’re afraid
So Zeus made the first woman - Pandora. He have to pull your money out of the market and miss the
her _________ small box, but told her not to open rest of the fun, but you’re also afraid of getting
it. caught with your money exposed if the market
Pandora married the brother of Prometheus. One crashes. You don’t dare be in the stock market at
day she became curious about ________ box - too these prices, but you don’t dare not be in. What’s a
curious and opened it. And a great number of tiny greedy investors to do?
monsters flew out. They were: hate, envy, anger, The short answer: no one knows. Even though
jealousy, revenge, selfishness, greed cruelty, stock prices are very high by important standards
disease, and all of the curses that often make us like dividends, the market isn’t necessarily heading
humans miserable. for a fall. But there are warning flags flying. The
Pandora quickly closed the box, but it was too late. two most worrisome signals: dividends are at their
The only thing left in ________ box was hope.” lowest level relative to stock prices in at last 70
a) a / the / the /  / a / a years, and the four most dangerous words in
b)  /  / a / the /  / the finance, “This time it’s different,” are abroad in the
c) the / a /  / a / the / the land.
d)  / the /  / a / a / the Dividends matter because they have historically
accounted for almost half of investors’ returns. If
36.“Can you tell a green field from a cold steel you own stocks, you make money from price
rail ?” increases and from dividends. If dividends are
The question asks lower than usual, then stock prices have to rise
a) if you can tell a story about the field and the more than usual for you to make your normal
cold steel rail. return.
b) if you are able to say how to go from the green
field to the rail road. 37.Through the comparison with Hugh Grant the
c) if you know one difference between the green text tells us that the desire to invest in the stock
and the cold steel rail. market has become
a) famous.
162
PROVAS DA AFA 1998 – 2010
b) charming. 1. Select an alternative to complete the blanks.
c) profitable. a) that / of that / _______ / that
d) irresistible. b) who / _______ / who / which
c) which / that / which / _______
38.The market results have been d) that / of which / _______ / who
a) scary.
b) terrific. 2. In the second paragraph, the pronoun “it” refers
c) terrible. to
d) reasonable. a) an indication.
b) something about you.
39.What’s investor’s biggest doubt? c) being an Air Force cadet.
a) When they should crash. d) nothing of real lasting value.
b) What they should invest in.
c) The average of the Dow Jones rates. 3. The correct interrogative form of the sentence
d) To leave capital in or out of the market. “The Air Force is not just airplanes.” is:
a) Isn’t the Air Force just airplanes?
40.What factors are an alert that the market may be b) Is there just airplanes in the Air Force?
going down? c) Are not just airplanes in the Air Force?
a) investors are greed and stock prices are very d) Does the Air Force isn’t just airplanes?
high this time.
b) dividends aren’t high and investors’ belief that 4. According to the text we could say that “nothing
“this time it’s different”. of real and lasting value can be accomplished by
c) stock prices are very high and investors’ belief a) the Air Force”
that “this time it’s different”. b) seeking people”
d) dividends are lower than 70 years ago and c) dedicate people”
investors’ belief that “this time it’s different”. d) their imagination”

5. Tell where the phrasal verb is not correctly used


AFA 1999/2000 – Inglês according to the meaning of the sentence.
a) Because of the rain they put off the final game
for Saturday.
Read the text below to answer questions from 1 to
b) In spite of the bad weather this aircraft will take
4.
over on time.
IT’S MORE THAN FLYING!!
c) I didn’t know these words until I looked them
The Air Force is not just airplanes. It's men and
up in the dictionary.
women _______ are integral parts of a closely knit
d) We’ve had quite a few problems before, but we
organization.
get along with each other just fine now.
Being an Air Force cadet says something about
you. It’s an indication _______ you have
6. Which alternative completes meaningfully the
imagination and drive to get things done. Nothing
sentence below?
of real and lasting value can be accomplished
“_______ the possibility of an awful storm they
without dedicated people. That’s why the Air
decided not to _______ the match that _______
Force is seeking people _______ are selflessly
scheduled.”
loyal to their country, the Air Force and
a) Because / win / is
themselves.
b) Although / play / was
You can become an Air Force officer proudly
c) However / cancel / isn’t
working at a demanding and challenging
d) In spite of / call off / had been
profession _______ calls for dedication, hard work
and discipline, but also offers many personal
7. Observe the articles in these following
rewards.
sentences:
Accept the challenge.
I – Sandra is an engineer and her sister is a teacher.
Be an Air Force cadet.
II – Please, don’t give me an advice.
The experience can change your life.
163
PROVAS DA AFA 1998 – 2010
III – I’ve never seen such a beautiful day! blanks.
IV – My father is going to give you a money. “The airplane _______ at 3000 feet _______ the
V – I can’t imagine such patience as the Browns airport when the pilot _______ something was
had with those three babies together. happening with one of the engines.”
a) only sentence IV is wrong a) had flown/ over/ repaired
b) sentences I, III and V are correct b) was flying/ above/ noticed
c) sentences I, II and III are correct c) had been flying/ on/ had been
d) sentences II, III and V are correct d) has flown/ up/ had communicated

8. Talking about numbers, “six“ is to “twelve” the 14. Jake began to work out at 5:00 p.m. It's 6:30
same way "_______" is to “one thousand”. p.m. now and Jake's still working.
a) five dozen The sentence tells us that Jake
b) five hundred a) might quit working soon and go home.
c) half a million b) has been working long hours at the job.
d) five thousand c) has been exercising for 1:30 hours at all.
d) will have gone back inside at any moment.
9. 08:00 p.m. - Mike had several glasses of wine.
12:00 a.m. - Mike had an accident. 15. Find the alternative that best completes the
What could you say about Mike at this present three sentences below.
moment? “Microsoft's Explorer aims at _______ its users
a) Mike will have gone home earlier then. with quick access to the web.”
b) He has been driving drunk for many hours. “As the class was almost over the students didn't
c) Mike can't drive very well after the accident. bother _______ the exercise.”
d) After he'd drunk several glasses of wine, Mike “She offered _______ us and I think that was very
had an accident. nice of her.”
a) to give / doing / her help
10. “Growth is a risky business. One cannot stay b) providing / to start / to help
safe and grow. To grow means that one must take c) to provide / answering / helping
chances and push oneself beyond yesterday’s d) program / finishing / her helping
limits.”
Which of these ideas can be inferred from the Questions from 16 to 19 refer to the text below.
paragraph? The subject of flight has fascinated us for
a) You must grow to take chances. thousands of years. Our distant ancestors first
b) Don’t take chances and you may never grow. envied the birds and scratched pictures of them
c) Without growth you can have chances to take. _______ rock walls. That was a primitive way of
d) You should stay safe to push yourself beyond expressing wonder at the phenomenon of flight
your limits. and a desire _______ share it. _______ the
classical myth, the architect Daedalus and his son
11. The sentence “just do it”, which is Nike's Icarus fastened wings _______ their bodies with
slogan, is in wax. Icarus, flying godlike too near the sun, was
a) the present tense. destroyed when the wax melted.
b) the present perfect tense.
c) the imperative affirmative. 16. Complete the text with the missing
d) the contracted emphatic form. prepositions.
a) in / of / In / at
12. The past tenses are correct in: b) on / to / In / to
a) meant – shook – understood – shaved c) for / to / To / in
b) taught – touched – lent – earnt – burnt d) over / in / From / on
c) kept – trusted – bought – stealed – owned
d) dreamt – spoke – hurted – realized – thought 17. We can say that our distant ancestors
a) wished wings like a bird’s.
13. Choose the right alternative to complete the b) sent the bird’s scratches away.
164
PROVAS DA AFA 1998 – 2010
c) flew fast with wings like gods’. a) also / or / neither / also
d) broke the bird’s pictures on the wall. b) nor / either / neither / too
c) either / also / either / either
18. In the text we can find d) neither / nor / either / either
a) one relative pronoun.
b) two personal pronouns. 24. The teacher always says: "Open your books,
c) four objective pronouns. _______? Let's finish this exercise now, _______?
d) three adjective possessive pronouns. a) will you / shall we
b) don't you / let's not
19. The word “when” belongs to c) couldn't you / won't we
a) a noun clause. d) open not you / would you
b) a relative clause.
c) an objective clause. 25. “My friend would still be living in New York
d) an adverbial clause. if his father hadn’t died in the countryside.”
So you can come to a conclusion that he
20. “Some friends of mine say Robert is dating my a) would live in New York if his father died.
cousin Jane. I’ve never seen them together yet. b) is taking care of his father in the countryside.
This either is or is not so. In any way it’s none of c) still lives in New York and not in the country
my business.” now.
You can infer from this paragraph that d) lost his father in the countryside, not in New
a) there won’t be any way to know if Robert is York.
dating.
b) Jane’s cousin can’t say whether or not Robert is 26. Read the paragraph to tell what the underlined
dating her. words are respectively.
c) if Robert is dating, Jane’s cousin must not care “Spit it out! People who let the world know how
about his own business. they feel may be healthier than those who keep
d) if Jane’s cousin saw either Robert or Jane they their remorse to themselves, experts say. People
would be actually dating. who never complain may not have enough self-
respect. If you don't complain when you've got a
In questions 21 to 24 choose the alternatives which legitimate grievance, it implies you don't think you
fill in the blanks correctly. deserve good things.”
a) relative pronoun / conjunction / relative pronoun
21. The Brazilian economy seemed to be getting / conjunction
_______ when the Asian financial crisis took place b) conjunction / interrogative determiner /
and drastically affected FHC’s government. conjunction / relative pronoun
a) better and best c) interrogative pronoun / conjunction / relative
b) better and better pronoun / interrogative determiner
c) the best financial d) relative pronoun / interrogative determiner /
d) heavy and heavier interrogative pronoun / conjunction

22. That day Michael managed to control himself 27. “Prince Charles, heir apparent to the throne of
because he's not _______ his brother. He is England, had a story of a deeply troubled
_______ he should be at his age. marriage with princess Diana.” The
a) as impulsive as / much more reasonable than pronunciation of the word “heir” is just like
b) the most impulsive of / more reasonable than the word
c) the least impulsive than / the most reasonable of a) “air”
d) less impulsive than / very much reasonable than b) “ear”
c) “hair”
23. After class we went _______ to the movies d) “hare”
_______ to the theater. We didn’t go to bed 28. Select a sequence of words to complete the
_______. We started studying for the next test. following paragraph correctly:
Didn’t you do it _______ ? “We don’t pronounce the sound of the consonant
165
PROVAS DA AFA 1998 – 2010
T in the words _______ and _______. The sound Everyone has something to learn from _______
of the vowel E after the consonant F in the Kosovo. But _______ keenest students of the war
words _______ and _______ isn’t usually should be those who live in Western Europe. Other
pronounced either. places in _______ world have seen as much or
The H is also omitted when we say ______. more bloodshed since _______ end of the cold
These are things we must learn in English!” war. But only _______ Europe has medieval
a) startle / often / differentiate / referee / haste hatred taken hold so close to the heart of what
b) whistle / turtle / conference / preferment / boastfully supposes itself to be _______ rational,
honest sophisticated civilization.
c) castle / fasten / difference / reference /
honorable 32. Complete the text above with some articles
d) wrestle / mostly / preference / different / horizon where they are necessary or just omit them when
they are not supposed to be used.
29. Choose the alternative which express the same a) the / ___ / the / the / a / the
idea of the sentence below, but changes all the b) the / the / ___ / an / the / a
nouns to its corresponding gender, masculine or c) ___ / the / the / the / ___ / a
feminine. d) ___ / the / the / an / ___ / the
“The bachelor, who is the widower's son,
should date with our niece, a bright student from 33. The pronoun “itself” in the text refers to
Colorado University.” a) the heart of a medieval continent.
a) The bacheless, who is the widow's son, should b) the European continent as a whole.
date our nephewer... c) a rational, sophisticated civilization.
b) The spinster, who is the widower's son, should d) a continent which is the heart of the world.
date with our nephew...
c) The spinster, who is the widow's daughter,
should date with our nephew... Read the text below to answer questions from 34
d) The spinteress, who is the widower's daughter, to 40
should date with our niecetor...
A NEW PARK THEME: GLUT
30. “Although the ultimate meaning of life is Welcome to troubleland. With seven large parks
mysterious, it affects every moment that we live. on the ground and more on the way, industry
What we believe about the meaning of life analysts are issuing dire warnings: “Orlando is
influences what we value and every choice we now a zero-sum game”, says a media analyst. The
make.” theme-park glut promises a brutal shakeout that
The word “ultimate” in the text is used as could pound the earnings of park owners Disney,
a) a synonym of "mysterious". Seagram (Universal) and Anheuser-Busch (Busch
b) a noun that means “last one”. Gardens, Sea World).
c) an adverb of time, the same as "lately". Disney alone has built at least one park every
d) an adjective that means “fundamental”. decade since the Magic Kingdom appeared in
1971. Last year the company not only opened the
31. Choose the alternative which has its adverbs in gates to Animal Kingdom (reported cost:
the most usual order following a verb of $800 million) but also launched the Disney Magic
movement. ($350 million), its first cruise ship in Florida. A
a) There go our bus now down the lane. second ship, the Disney Wonder, is on its way.
b) Vicky danced beautifully yesterday at the party. Analysts see so little economic rationale for these
c) He sat on the chair very quickly when the expenditures that they've begun to label the ships
teacher called his name. “Tragic” and “Blunder”. Disney has enough land
d) Mike entered fast by the open door when he in central Florida to add three more parks. Indeed
heard she was crying his name. there are rumors that a sports-themed park is on
the drawing board.
Questions 32 and 33 refer to the text below Determined to loosen Disney's choke on hold area
tourism, Seagram's Universal division bought an
166
PROVAS DA AFA 1998 – 2010
additional 770 hectares. That's enough for two a) american cruise ships' high prices.
more parks. Anheuser-Busch's Sea World, the third b) Animal Kingdom's gates opening cost.
big player in central Florida, is adding a splashy c) work beginning on Disney ships prices.
new park in which visitors will be able to swim d) Disney Magic and the Disney Wonder high
with the residents. That interactive attraction is costs.
scheduled to open next year.
The “can-you-top-this?” mentality has not only 37. Which idea is not mentioned in the text?
raised the ante for thrill rides but also driven up the a) Universal intends to cut Disney's monopoly.
cost of construction. Finding workers to operate b) Disney's building 3 new parks in central Florida.
the parks is another headache. Disney, Universal c) Analysts don't advise investing more in Orlando.
and Sea World have had to raise their starting d) In a new park tourists will have close contact
hourly pay to more than $6 an hour (the U.S. with sea animals.
minimum is $5.15) to attract and retain employees.
This building boom is happening just when 38. Construction is becoming more expensive in
consumer demand for theme parks is softening. Orlando because of
The economic slump overseas slashed tourism to a) the price of land hectares.
Orlando. But experts wonder whether the whole b) the world's economic crisis.
theme-park business is maturing, as the children of c) the hard competition among theme parks.
U.S. baby boomers get older and hence reduce the d) the unemployment problem in the United States
number of repeat trips. “I just don't think it makes
a lot of sense to build more parks in Orlando”, says 39. The word hence in the 5th paragraph is
a media analyst. “They've reached the saturation _______ that could be replaced by _______.
point, and profits are going to come down.”
So will prices. The typical family spends about a) a verb / intend
five days and more than $1000 in Orlando's parks. b) a noun / adults
It's nearly impossible to see everything. As a c) a conjunction / because
result, every park is feverishly baiting tourists d) an adverb / consequently
away from rivals. If you've ever had to spend an
ungodly amount of money to wait two hours for a 40. “To get even” in the last line of the text means
six-minute ride, this may be your chance to get a) to revenge yourself.
even. b) to have a good time.
c) to save yourself from rivals.
34. The text can make you think that d) to look after new prices and costs.
a) people's interest in theme parks is increasing
lately.
b) a tourism prophecy predicted that theme parks AFA 2000/2001 – Inglês
were coming down.
c) it's not extraordinary when a family spends more Instructions for questions 01 through 06.
than 1000 dollars visiting parks.
d) the U.S. children are maturing when they Choose the correct alternative to fill in the blanks:
wonder about the park business around.
1. Sliding boards are fun
35. According to the text, Orlando I like climbing higher and higher
a) already has more parks than it should have. __________ the sky,
b) needs two or three more parks to be complete. and when I slide __________ the bottom
c) will close one or two parks as soon as possible. I laugh when I feel the air
d) won't see a new attraction sooner or later this rush __________me.
year. a) to / up to / against
b) toward / down to / past
36. “Tragic” and “Blunder” are words used by c) forward / back to / toward
analysts to talk about: d) toward / past to / down on

167
PROVAS DA AFA 1998 – 2010
2. After complaining about the bus driver’s d) kid / bitch / pill / thick
behavior during the trip, the students reported
everything they had observed to the principal. As a 8. BOMB (final):
matter of fact, he drove __________ but a) same / dumb / dim / dam
__________. b) gamb / team / map / might
a) faster / safe c) time / some / ambush / became
b) fastly / well d) succumb / come / muck / autumn
c) fast / safety
d) fast / safely 9. “It seems impossible to love people who hurt
and disappoint us. Yet there are no other kinds of
3. All stars have similar life cycles. They form, people.” That is why it’s not easy to love. Haven’t
use up their energy as heat and light and finally you realized that yet? Have you ever loved?
die. In the paragraph above there are _________.
The simple tense is used in this sentence because it a) two relative pronouns
talks about __________. b) one interrogative pronoun
a) things that happened at an indefinite time c) three adverbs of frequency
b) a fact that is ever in progress in the universe d) two demonstrative pronouns
c) general truth or laws of nature in the universe
d) a fact that will have happened at a certain 10. Considering the sentence: “That is why it’s not
future time easy to love” from the paragraph above, “that”
refers to the __________.
4. No one could avoid noticing my __________ a) fact that everybody hurts everybody
or __________dress during the party. b) impossibility of people loving all kinds of
a) wives / daughter people
b) wife / daughter’s c) kind of people who love to hurt and disappoint
c) wife’s / daughter’s us
d) wive’s / daughter’s d) disappointments that other kind of people can
5. Choose the alternative in which there are only bring
uncountable nouns:
Yesterday I went to the supermarket and bought 11. Complete the sentences below:
_________, __________, __________, She sang beautifully __________ he played.
______________________. I won’t go __________ they insist on it.
a) tea , cream, paper, beer and ice You had better eat something __________ you
b) fruit, salmon, rice, sugar and oil leave.
c) lettuce, milk, jam, egg and cucumber __________ he lived in Germany for 8 years, he
d) beans, cabbage, fish, carrot and water did not acquire a good German accent.
You can go by plane. __________, if that scares
6. __________ milk and __________ meat are you, you can take the train.
good for __________ our health. a) unless / except / if / Therefore / Although
a) ⎯ / ⎯ / ⎯ b) while / unless / before / Although / However
b) ⎯ / the / ⎯ c) as / however / before / Until / Notwithstanding
c) The / the / ⎯ d) why / whereas / despite of / Neither / In spite
d) The / the / the of

Instructions for questions 7 and 8: Read the text to answer questions 12 through 14.
Choose the alternative with words that have
the same sound of the underlined letters: “Second Wind – This is a feeling of relief that
occurs after exercise has become strenuous.
7. SIT (medial): Whereas breathing was labored and the work felt
a) bee / sea / file / bite painful before, breathing becomes easier and the
b) bic / pit / dim / dime work more tolerable after athletes experience a
c) dim / sigh / lean / line second wind. The reasons for this reduction in
168
PROVAS DA AFA 1998 – 2010
effort are not known. The early distress may be d) two relative clauses and one imperative clause
associated with the temporary use of anaerobic
metabolism until oxygen consumption has 17.Although the pressure in the trial at that
increased and aerobic metabolism is providing a moment was very high, they didn’t __________
larger percentage of the energy for work. There is and won in the end, when the man who seemed to
some support for this notion in the fact that second be the only innocent __________ to be the
wind only occurs during endurance efforts. murderer.
Second wind usually occurs when athletes are just a) stop by / took out
beginning their training program after a long b) run into / come off
layoff. Well-trained athletes rarely experience this c) give in / turned out
phenomenon, probably because their circulatory d) come off / got over
system adjust more rapidly after they become
conditioned.” (“Swimming even faster”, Ernest 18.⎯ Hey Jackie are you there?! Jackie?!
Maglisho) ⎯ I’m here!
⎯ The phone is ringing!
12.According to the excerpt, second wind is ⎯ __________ it.
_______. a) I’ll get
a) a painful exercise b) I’ve rung
b) a good sensation c) I ought answer
c) an exercise training d) I’d been taking
d) the aerobic metabolism
19. I hate __________ up early in the morning.
13. When does second wind happen? Mary suggested __________ to the country club
a) During endurance efforts next week.
b) In the beginning of a layoff I’m trying __________. Please stop __________.
c) When athletes reduce efforts I don’t mind __________ by train sometimes.
d) When doing anaerobic exercises Goodbye. I hope __________ you again.
a) to get / to go / to work / to quarrel / to travel /
14.__________ do not feel second wind often. to see
a) Athletes who have a good preparation b) to get / going / working / to quarrel / traveling
b) Athletes who have strenuous breathing / seeing
c) Athletes who have increased the use of c) getting / going / to work / quarreling /
anaerobic consumption traveling / to see
d) Athletes who have a good experience with this d) getting / to go / to work / to quarrel / to travel /
kind of phenomenom to seeing

Instructions for questions 15 through 24: 20. Things are going really well for him, he
Fill the blanks by choosing the correct answer. __________ be so worried. No one __________
think that his luck will let him down.
15.I like being happy because I feel big inside a) shall / might
__________ a balloon that gets __________. b) shan’t / could
a) in a way / as big as c) ought not / will
b) just how / big and bigs d) shouldn’t / would
c) as well / rather bigger than
d) just like / bigger and bigger 21.When I told him he should visit his brother
Thomas he told me he __________ there and
16. “When eating fruit, think of the person who Albert __________ a great problem with his ex-
planted the tree.” wife then.
The saying contains __________. a) was gone / has already
a) a conditional clause and a reflexive clause b) shall pass / would have
b) an imperative clause and an adverbial clause c) had already been / had had
c) two adverbial clauses and one relative clause d) was going after / will have had
169
PROVAS DA AFA 1998 – 2010
to circumvent your trap. Improperly set up, it can
22.“Can we throw a party in here, Jill?” go off accidentally and injure or kill you or a
Albert asked Jill __________. member of your family, or your household pet.(...)
a) to throw the party Booby traps are for guerrilla soldiers. They do not
b) if they can throw a party here belong in American homes and business. If you try
c) where they can throw the party to use them you will be the “booby” who
d) whether they could throw a party there ultimately gets “trapped”. (“The truth about booby
traps”, Massad F. Ayoob)
23. Five years ago this issue __________ seriously
but now that damage __________ in large areas of 25. According to American laws a citizen has the
forest, our politicians __________ to take actions. right to __________.
a) was been taken / has been reported / are been a) run away only during a fire
forced b) kill only to protect his house
b) hasn’t being taken / has being reported / has c) revenge only particular intruders
being forced d) hurt only if he receives threatens in his house
c) was not being taken / has been reported / are
being forced 26. “Booby trap” is a (an) ___________.
d) hadn’t being taken / had been reported / have a) proper weapon for revenges
being forced b) proper device for guerrillas
c) home device safe against robbers
24. We __________ early for our flight. d) excellent mechanism of protection
I __________ because my shoes were too big for
me. According to the text:
She doesn’t know how to __________ children.
Can you __________ the kids while I’m away? I. Booby traps only hurt innocents.
I ___________ Chemistry because I didn’t like it, II. Your own family can be hurt.
but I __________ Physics instead. III. You can be your own victim.
a) switched in / fall for / bring off / look down / IV. An accident may hurt your house in fire.
gave in / took in
b) checked in / fell over / bring up / look after / 27. From the statements above, the right one (s)
gave up / took up are (is)_________.
c) signed for / fell to / bring away / look back / a) only II
gave off / took into b) II and III
d) put up / fell away / bring back / look out of / c) III and IV
gave away / took away d) I, II and III

Read the text below to answer questions 25 28. Which is the meaning of the underlined words
through 28. in these sentences from the text?
Improperly set up, it can go off.
“I can empathize with the person who has his You will be the “booby” who ultimately gets
home violated and seeks both revenge on burglars trapped.
in general, and a painfully deterring experience for a) fix / be injured
the next burglar in particular who chooses him for b) imagined / tune in
a victim. But booby traps are not the way.(...) c) deployed / be caught
Under American law you can only hurt the burglar d) tuned in / be confused
if he threatens the life and limb of innocent
persons inside. A booby trap, something that 29.“You won’t discover the limits of your soul,
physically harms an intruder, can maim or kill a however far you go” (Heraclitus)
fire fighter breaking in to rescue your home and What do you infer from this saying?
possessions from a blaze. It can spring shut on a) You must go far to discover your limits.
your spouse or child, or even on you if you come b) As a matter of fact, your soul has no limits.
home preoccupied with something else and forget
170
PROVAS DA AFA 1998 – 2010
c) The limits of your soul aren’t however very c) If thinking of the difficulties realized you it’s
far. because the marriage decisions time.
d) Walk a long way through and you shall find d) If you thought of the difficulties, you would
your limits. realize why marriage decisions take time.

30. Choose the correct question for the context: Read the text below to answer questions 35
You are speaking to your daughter. You want to through 40.
make sure that she turned off the stove. You ask
her: We know dinosaurs only by their bones. The
a) You turned off the stove, did you? largest, most powerful animals to walk the earth
b) You do turn off the stove, didn’t you? are extinct. Their “arrogance of power ”was of no
c) You didn’t turn off the stove, did you? use.
d) You did turn off the stove, didn’t you? There is an irony here. If we had been their
contemporaries, we would never have suspected
31.What’s the right answer for the numerical that theirs would be such a sad and inglorious end.
expressions below? The stronger the better, we assume, in the struggle
1 3 16 for existence. The more powerful a species is, the
; 2 ;
5 5 15 greater should be its chances of survival.
a) One fives / two thirty-five / sixteen fifteen But this did not prove to be true. Animals of much
b) One five / two and third fifth / sixteen fifteens more fragile structure, whose bodies were weaker
c) One fifth / two and three fifths / sixteen and smaller beyond comparison are still around.
fifteenths But dinosaurs are nothing more than memories of
d) First fifths / second thirty-five / sixteenth one of life’s experiments that failed.
fifteenths The dinosaurs disappeared not because they were
too weak, but because they were too strong. Their
32.Choose the alternative that is correct: fantastic power came from a biological framework
a) I hid the remote control of my son. which was basically absurd, and the result was
b) We can say that fortune smiled to him at last. annihilation. Can you cure an insane person by
c) Whip the eggs and the butter and then add making his body physically fit? Obviously not.
sugar to the mixture. This would add power to insanity, making it more
d) After thinking about the matter for a long insane still. The power generated by an irrational
time, he arrived to decision. structure only tends to aggravate the very
irrationality from which it springs. By adding
33. What’s the passive voice for “Mr. McCannigan power to the absurd one does not abolish it; on the
left this leather jacket in the back seat of my car contrary, it becomes still more hopelessly
last night”? entangled in itself. Power is like a mathematical
a) Mr. McCanningan had been leaving this jacket number inside a bracket. If the bracket is preceded
in the back seat of my car. by a minus sign, it is not possible to transform into
b) Last night this leather jacket in the back left in a plus by making the number bigger and bigger.
my car by Mr. McCannigan. This simply increases its negativity.
c) This leather jacket was left in the back seat of Power is a simple potentializing factor. It can
my car by Mr. McCannigan last night. never go beyond the logic of the structure that
d) This leather jacket had been left by the back generates it. This is why dinosaurs had to die.
seat of Mr. McCannigan’s car last night. Their “arrogance of power” entrapped them in the
very absurdity of their organic structure. They
34. “If you think of the difficulties, you will realize were thereby made incapable of responding in
why marriage decisions take time.” different ways to the new challenges their
Another way to express the same idea can be: environment presented.
a) Think of the difficulties or else you will Our civilizations is behaving just like the dinosaur.
realize the marriage decisions time. Underneath everything it does, one finds the
b) Think of the difficulties and you will realize ultimate certainty that there is no problem that
why marriage decisions take time. cannot be solved by means of a little more power.
171
PROVAS DA AFA 1998 – 2010
It is not by accident that for years detergent makers 40. The text tells us that dinosaurs were _____ .
have been advertising “stronger”, “faster”, “more a) present in an environment that was very
concentrated” and improved formulas. They know absurd
that these values control our collective b) killed by other species that went beyond their
unconscious. What is stronger must be better. Love structure
of power has become our obsession, and power c) unable to adapt to natural changes that had
itself our sole god. (“Tomorrow’s child” Rubem occurred
Alves) d) arrogant because they entrapped other organic
structures
35. “But this did not prove to be true”. The
underlined word refers to the __________.
a) irony of us being dinosaurs contemporaries AFA 2001/2002 – Inglês
b) best dinosaurs struggled for a contemporary
existence Directions: Read the text below and answer
c) fact that we believed we were stronger than questions number 01 to 03 according to it.
dinosaurs
d) assumption that the more powerful the longer Frequent flying slightly raises the odds of getting
it’ll live cancer.
Cosmic rays were once the province of
36. According to the text we can conclude that: science fiction, feared only by space travelers
a) Power is not the best remedy for absurd. rocketing to unknown worlds. Here on earth the
b) A fit body obviously does not belong to a sane danger posed streams subatomic particles from the
person. sun and stars was considered insignificant. Now
c) Frameworks are so absurd that they’re that assurance has been shaken, at least for those
basically destroyed. who spend much of their lives flying at high
d) Insanity ought to be cured by the acquisition altitudes. Last week the U.S Department of
of more power. Transportation reported that radiation penetrating
the thin skins of jetliners can pose a hazard for
37.Animals that were much less powerful than passengers and crews.
dinosaurs ___________ . For the occasional traveler the danger is
a) were experiments that failed slight and not worth worrying about. But the
b) did not become extinct at all Government study suggests that radiation is
c) couldn’t prove they were true causing a small and measurable increase in cancer
d) were around the smaller bodies rates among crew members and the most frequent
flyers. As a result, the Federal Aviation
38. The text tells us that ___________ . Administration is sending airline workers an
a) human beings are too devoted to power advisory on the health risks of cosmic radiation.
b) the strongest things have an obligation to be The Government estimates that among
better 100,000 crew members who spend 20 years on an
c) God is solely the power that controls our average route, such radiation will cause 600
unconscious “premature cancer deaths.” By comparison, cancer
d) we are solely obsessed with God because of from all causes may kill 22,000 of that same
His power group. Frequent travelers who spend half as much
time in the air as pilots do, or about nine hours a
39. We learn from the text that power _______ . week, will incur half the risk. Exposure varies
a) must be logical, well structured and simple dramatically with altitude and proximity to the
b) can potentialize the absurd, but cannot solve it poles. A year spent flying the great-circle route
c) becomes more entangled when preceded by from New York City to Tokyo will subject a crew
certain signs to 30 times the risk of a year of flying between
d) is negative and cannot be transformed into Austin and Houston.
something bigger Pregnant crew members have special
concerns. The FAA says women who fly
172
PROVAS DA AFA 1998 – 2010
throughout the first seven months of pregnancy b) II and IV
may exceed the recommended radiation doses for c) I and III
fetuses and slightly boost the risk of birth defects. d) I and II
Some experts feel such women should consider
taking leaves or working on less exposed routes, 04 -The item in which the word has the same
especially from the eighth to the 15th week of peculiarity of the “H” in the word HONOR is:
pregnancy. Others emphasize that the radiation is a) hole
much less hazardous than other dangers of b) hostility
everyday life. Says Dr. Wallace Friedberg, head of c) hour
radiobiology at the FAA: “If my wife were a flight d) honeymoon
attendant and pregnant, I would not tell her to stop
flying.” 05 -The alternative in which both words have the
(Adapted from Time) same vowel sound is:
a) antique – seek
01 - According to the text: b) wear – weary
a) Subatomic particles from the sun and stars c) quick – quite
can cause cancer for those who frequently fly. d) tip – tiny
b) People who spend much time flying at high
altitudes raise the odds of getting cancer. 06 -“Albert says that his wife Rose can’t stand it
c) Flying from New York City to Tokyo is when he turns on the radio while she’s performing
really dangerous when the crew is exposed. any written activity.”
d) The Government estimates that a large The paragraph means that Rose
number of people who spend 20 years on the same a) listens to the radio without complaining.
route will die of cancer. b) hates when her cousin turns the radio on.
c) doesn’t appreciate her husband’s attitude.
02 - Which idea is mentioned in the text? d) asks him to decrease its volume.
a) Crew members don’t have to worry about
the special concerns in this text. 07 -In the following alternatives, all the nouns are
b) If radiation penetrates the crew members’ feminine, EXCEPT:
skin it will cause a damage and consequently they a) heiress / jewess / empress / mare
will get cancer. b) spinster / niece / vixen / hen
c) This article is not so important for people c) daughter / cock / witch / aunt
who don’t fly very often because the danger is d) lioness / gander / ewe / bee
slight and they will never get cancer.
d) For people who don’t fly very often the 08 - Mark the correct set of pronouns to complete
danger is small and it’s not necessary to worry the sentences below.
about it. Duchess Sharon couldn’t find any jewels in the
jewel case because _____ children had taken ____
03 - According to the text which ideas are correct? to perform a play at school. ____ didn’t tell it to
I. Half of the travelers who frequently fly _____ mother and ____ got very irritated with
have double risk of getting cancer because of the ____ because of ____.
cosmic radiation. a) her / them / They / their / she / them / it
II. In the Government research radiation is b) your / him / They / them / your / her / them
damaging the health of crew members and the c) their / they / Them / their / her / them / him
travelers who seldom fly. d) his / their / Their / they / he / they / its
III. Frequent travelers and pilots spend half of
their time in the air. Directions: Read the text below and then
IV. Pregnant women who fly a lot have to answer the questions 09 to 11 according to it.
worry about radiation because of the fetus. A fox was walking through an orchard
From the statements above, the right one (s) is when he saw a bunch of grapes which had just
(are) turned ripe. They were on a vine that was growing
a) only IV over a lofty branch. “Just the thing to quench my
173
PROVAS DA AFA 1998 – 2010
thirst”, he thought. Drawing back a few paces, he a) six adjectives in the superlative form and
took a run and a jump and just missed the brunch. five ones in the comparative of superiority.
Turning round, he again tried to jump, but with no b) five adjectives in the comparative of
better success. Again and again he tried after the equality and six in the comparative of superiority.
tempting morsel. At last he had to give it up and c) six adjectives in the comparative of
walked away with his nose in the air, saying to superiority and five in the superlative form.
himself, “I am sure they are sour.” d) five adjectives in the superlative form and
(Adapted from Stairway to English) six in the comparative of inequality.

09 - Who was the fox trying to convince? 13 -In the sentence “...one of the worst problems
a) Herself today”, the underlined words in the comparative of
b) Myself equality would be
c) Himself a) worse than
d) Ourselves b) not so good as
c) as well as
10 -In the sentence “...a vine that was growing d) as bad as
over a lofty branch”, the underlined word means
a) tiny 14 - The sentence “...so people will be healthier
b) huge than they are today”, means that
c) high a) there will be no more illness in the world.
d) distant b) people will have a better quality of life and
less sickness than nowadays.
11 - The opposite of “sour” in the fable is c) it will be a time with a lot of diseases in the
a) salty world.
b) green d) everybody will have the chance to live in a
c) bitter clean world.
d) ripe
Directions: Read the text below and
Directions: Read the following text and answer the questions 15 to 18 according to it.
answer the questions 12 to 14. A scientific exhibition was taking place in
The world will be a better place in a Philadelphia in 1876. There were many important
hundred years, optimists say. Life will be more guests and judges. Among them, D. Pedro II,
comfortable than today. Computers, the most Emperor of Brazil, the most important guest.
sophisticated machines, will do much of our work. There were lots of electrical apparatuses for
We will have more time for sports and the judges to see. So, they decided to postpone the
entertainment. Cities will not continue to grow. examination of the inventions that could not be
People will live in smaller communities and seen that day. Alexander Graham Bell was one of
everybody will be happier. Pollution, one of the the exhibitors. Mr. Bell felt that he had wasted his
worst problems today, will be a thing of the past in time bringing his telephone.
a hundred years. The air in the cities will be The judges began to disperse. Suddenly
cleaner, so people will be healthier than they are Emperor D. Pedro saw Mr. Bell and recognized
today. Rio de Janeiro will again be one of the most him. They had already met once in a school for
beautiful cities in the world. Brazil, the largest deaf and dumb in Boston.
country in the Southern Hemisphere, will also be “Mr. Bell, how are the deaf-mutes in
one of the richest in the world. Optimists say all Boston?” asked the Emperor.
these things will happen in a hundred years. But “They are very well, Your Majesty. I dare
we’ll never know. We won’t be here. tell thee that I have an exhibit here.”
(Adapted from Graded English Exercises and The Emperor decided to see it and the
Texts) judges followed him. When D. Pedro heard Mr.
Bell’s voice from another room, coming out of the
12 - In the text above we can find iron-box receiver, he exclaimed surprised, “It
talks!”
174
PROVAS DA AFA 1998 – 2010
The judges approached to confirm what the In 1993, two English boys were considered guilty
Emperor had said. All of them became surprised. of one of the most shocking crimes in British
Mr. Bell’s invention was saved. history. The boys, at that time, had taken away 2-
“Thanks to Dom Pedro” – wrote A. G. Bell year-old James Bulger from a shopping center on
– “I went to bed the night before as an unknown the outskirts of Liverpool. First, they ______ to
man, and woke up to find myself famous.” push him into a nearby canal. Then, they dragged
(Adapted from Our Wonderful World) him two and a half miles to a railway line. They
______ the child with stones and bricks. They
15 - What did Dom Pedro have to do with the kicked him in the head. They ______ him with an
invention of the telephone? iron bar. Trying to disguise their crime as an
a) He asked Graham Bell to see his invention accident, they finally _____ his dead body on the
because they were close friends. rail where it ______ in two by a passing train.
b) He wanted to see the invention of Graham (Adapted from Newsweek)
Bell who he had already known, after the judges
had decided to postpone the new inventions 19 – The correct verbal forms to complete the
examination. blanks are:
c) He helped Graham Bell to invent the a) tried / pelted / hit / laid / was cut
telephone since they had met before in a special b) try / had pelted / hit / lain / cuts
school. c) tried / were pelted / have hit / lay / had cut
d) He was the first person to test the d) had tried / pelted / hitted / laid / was cut
invention and admit it really worked.
20 – Consider the following statements:
16 – According to the text, the judges wanted to
postpone the examination of the inventions. The Phil is going to Denver tomorrow.
underlined verb means Phil’s plane will depart at 10 tomorrow morning.
a) to put off It’s 10 a.m. now.
b) to put out
c) to put away Based on the information above, we can say that at
d) to put on this time tomorrow Phil
a) is going to be in Denver.
17 – What did the judges do after they had seen b) will be departing to Denver
Graham Bell’s invention working? They c) will have arrived in Denver.
a) got amazed and believed it was a good d) would have gone to Denver.
invention.
b) went to see the invention only because it was D. Since 1961, the World Wildlife Fund has saved
Pedro who had gone to check it. lots of animals and birds from extinction. It has
c) asked Graham Bell to demonstrate the invention given more than 40 billion dollars to conservation
to them. projects around the world. It has created or
d) felt curious about the invention and went to test supported 260 National Parks on five continents.
it. But there’s still much more work to do.
(Adapted from Time)
18 – In the text we have some irregular verbs in
the Simple Past Tense: WAS – SAW – HAD – 21 – According to the context and the verbal tenses
THOUGHT – TOOK. The infinitive form of these used in the article, it’s correct to say that
verbs are respectively: a) The World Wildlife Fund’s work may be
a) am / see / has / thank / taking considered a finished action.
b) be / see / have / think / take b) Animals and birds were saved by the Fund in
c) is / seen / has / thinks / taken 1961 and before then.
d) are / sees / have / thinking / takes c) The fund is going to support National Parks in
the future but not now.
Read the following passage: d) The help given by the World Wildlife Fund
began in the past and it’s still active.
175
PROVAS DA AFA 1998 – 2010
22 – Mark the sentence in which the use of the According to the pieces of information above, the
modal verb doesn’t correspond to the meaning in following sentences describe the situation
parentheses. correctly, EXCEPT:
a) You really ought to stop bothering people who
are working.  (advice) a) Although his instructor knew his capacity, the
b) Privates mustn’t pass by their superiors without cadet refused to pilot the plane alone because he
saluting them.  (absence of obligation) thought it was a hard task for him.
c) I told you it couldn’t be Sally at the door since b) Though the cadet thought it was a hard task for
she’s in Paris.  (complete certainty) him and refused to pilot the plane alone, his
d) “Employees must fill in the forms by noon”, instructor knew he could do it alone.
said the boss.  (strong obligation) c) The cadet refused to pilot the plane alone
because his instructor knew he would be able to do
23 – The numbers (1) to (6) are parts of a sentence. it by himself, unless he thought it was a hard task
Mark the alternative in which the sequence forms for him.
the sentence in a correct order. d) His instructor knew he could pilot the plane
(1) to Austin (4) we alone, but the cadet refused to do it by himself
(2) hope since he thought it was a hard task for him.
(5) for a vacation
(3) next month (6) to go 27 – On December 7th, 1941, Pearl Harbor, an
a) 4, 2, 6, 1, 5, 3 American Naval base in the Pacific Ocean, was
b) 1, 4, 6, 2, 5, 3 attacked by Japanese planes, causing the United
c) 4, 2, 3, 6, 5, 1 States to enter the war the next day. Nineteen ships
d) 1, 4, 2, 5, 6, 3 and about 200 planes were destroyed or seriously
damaged in the attack.
24 – Mark the alternative in which the calculation
2 1 3 Mark the correct alternative to complete the
− = is expressed correctly.
10 8 40 sentences below so that the questions are correct
a) Two tenth less one eighth is three fortieth. and can be answered based on the text above.
b) One eighth into two tenths leaves three fortieths. ______ planes were destroyed or damaged in the
c) Two over ten from one over eight is three over attack?
forty. ______ was Pearl Harbor attacked by?
d) Two tenths minus one eighth equals three ______ caused the US to enter the war?
fortieths. ______ did Japanese planes attack Pearl Harbor?
a) What / Why / Who / How
25 – Mark the correct set of prepositions to b) Whose / When / How / What time
complete the following paragraph: c) How many / Who / What / When
Charles Darwin, the English naturalist who d) Which / Where / What / How long
revolutionized the study _____ Biology, was
_____ Brazil _____ some time _____ his 5-year Directions: Read the following text and answer
voyage _____ the world _____ board the questions 28 to 30 according to it.
“Beagle”. Scenes from the nuclear theater of a long cold
a) about – in – at – for – within – on war:
b) of – in – for – during – around – on In the 1940s and ‘50s, when the Americans
c) in – at – for – along – through – in still trusted their doctors and their government,
d) of – in – by – during – among – on researchers subjected hundreds of ill-informed
people to doses of nuclear radiation, in order to
26 – Read the following information: study the effects on human beings. Later on, in two
The cadet refused to pilot the plane alone. The experiments on the West Coast, 131 prison
cadet thought it was a hard task for him. His inmates, many of them black, had their testicles
instructor knew the cadet would be able to pilot by irradiated.
himself.
176
PROVAS DA AFA 1998 – 2010
From 1963 on, the U.S. government My daughter was in doubt whether to drink coke
conducted hundreds of unannounced nuclear tests. or orange juice.
The Russians weren’t fooled, but Americans were. If we had time to call her, we would have avoided
Washington’s last secret underground blast the accident.
occurred as recently as April 4, 1990. My family will travel next weekend unless my
By the 1990s, America was awash in father has other plans.
nuclear waste. Tons of plutonium from arms If you want to, you could go.
factories and spent fuel from nuclear reactors were How can I guess if it’s going to rain tonight?
stored haphazardly and unsafely, sometimes
threatening workers and nearby residents. The The correct alternative is:
public still has not been told the true dimensions of a) Sentences I and IV are correct
the toxic mess. b) Sentences II and IV aren’t correct
(Slightly adapted from Newsweek) c) Sentences III and IV are correct
d) Sentences I and V aren’t correct
28 – Read the statements below and mark the
correct alternative according to the text. 32 – Choose the option that completes the
Handicapped people were subjected to nuclear sentences below correctly:
radiation. The ladder ________ I was standing began to slip.
131 black prisoners had their testicles exposed to My roof leaks ________ it rains.
nuclear tests. The wine, ________ was in the cellar, was ruined.
The public is still unaware of the real dimensions _________ happens don’t forget to call us.
of the nuclear problem. a) on which / whichever / when / wherever
a) All the sentences are incorrect. b)  / when / what / whenever
b) All the sentences are correct. c) which / when / that / whichever
c) Only one sentence is correct. d) that / whenever / which / whatever
d) Two sentences are correct.
33 – The underlined sentence in which the
29 – According to their meaning in the text, mark connecting word was erroneously omitted is:
the antonyms of haphazardly and unsafely, Sam is not a man we can trust at all(1), believe it
respectively. or not. He told me the cheese comes from his city
a) carefully / safely is quite good(2).He’s a liar. In fact it tastes pretty
b) carelessly / dangerously bad. He also told me the land he sold me is good
c) disorderly / safely for farming(3). It’s an arid region. And he insists
d) carefully / in jeopardy on saying that he didn’t tell me anything of this.
But he can’t forget the things he said(4). Now I’m
30 – According to the text, it’s correct to affirm terribly disappointed.
that a) 1 b) 4 c) 3 d) 2
a) In the ‘40s and ‘50s not only the government but
also the doctors had already lost their credibility 34 – The item that correctly explains the sentences
before the American people. below is:
b) In 1963 hundreds of secret nuclear tests,
including the last unannounced underground blast, She was too excited, so she took a sleeping pill.
took place in the USA. You can’t drive a car since you are not eighteen
c) The Americans who were chosen to undergo the yet.
tests knew too much about their government and Although the waiter had a very sore throat, he
the nuclear experiments. managed to answer in a hoarse whisper.
d) Nuclear tests have been made for more than As soon as I have finished, I’ll explain him that I
fifty years and nuclear waste storage has don’t feel up to tidying the kitchen now.
sometimes threatened Americans.
a) concession / time / reason / comparison
31 – Read the sentences below carefully: b) result / reason / concession / time
c) reason / result / cause / time
177
PROVAS DA AFA 1998 – 2010
d) cause / time / result / comparison b) Nothing is wrong, aren’t they?
c) You’ll have to pay for this, haven’t you?
35 – One of these options below contains a passive d) You’d better be careful, hadn’t you?
construction. Mark it.
a) Permission to use the site for the festival was 39) There’s an alternative in which DO(ES) can’t
eventually given . be used. Mark it.
b) They have transferred the money to my bank a) Which does type of battery do you use?
account. b) Who do you want to speak to?
c) She’s eventually permitted the use of the site for c) They don’t do very well in their exams.
the festival. d) Diana Ross really has a very beautiful voice,
d) They used to transfer the money to my bank doesn’t she?
account. Yes, she does have a beautiful voice.

36 – Mark the option in which the Indirect speech 40) Mark the item that completes the following
doesn’t correspond to the Direct one: passage CORRECTLY with the appropriate
a) “If I had the directory I would know what to missing articles when required.
do”, said Joe. – He said that if he had had the “Immediately after _____ assault, rape or robbery,
directory he’d have known what to do. victims or witnesses can usually summon up
b) “Get out of here at once”, John said angrily. – _____ mental image of _____ assailant. But by
He told me angrily got out there at once. _____ time they sit down with _____ sketch artist
c) “I shall be 18 tomorrow”, said Ann. – She told at _____ police station, their memories often have
me she would be 18 the following day. faded.“
d) “We were thinking of selling our old car but we a)  / the / an /  / the / the
have decided not to”, said Paul and Peter. – They b) an / a / the / the / a / the
told me they had been thinking of selling their old c) the /  / the /  / the / 
car but had decided not to. d) an / the /  / the /  / a

37 – Mark the correct alternative to complete the


gaps: AFA 2002/2003 – Inglês
The thieves ________ our apartment last night, but
Directions: Read the text below and answer the
when they ________ the police were waiting for
questions from 01 to 05, according to it.
them.
I didn’t ________ studying last night until almost
Amelia Earhart – The Sky’s the Limit
eleven o’clock, so Mary and Leslie ________
helping our mother.
In the 1930s Amelia Earhart was one of the most
The case was _______ of court because of
famous women in the world.
insufficient evidence.
As the first woman passenger and the first female
We may ________ relations with that country.
pilot to cross the Atlantic Ocean by air, she has a
place in the record books that can never be taken
a) broke off / ran over / break down / gave up /
(1). In those days women simply didn’t do that sort
crossed out / look into
of things (2). But the fame that her achievements
b) broke into / came out / get through / took turns /
brought her(3) are now also eclipsed by the
thrown out / break off
mystery of her disappearance.
c) took off / got away / make out / looked after /
In 1932, she became the first woman to fly solo
put away / look up
across the Atlantic Ocean. Over the next few years
d) picked out / got off / tried on / took part /
Earhart set and broke many other aviation records
looked out / call on
before announcing that she was going to face the
biggest challenge of all, to fly around the globe.
38 – The question tag is CORRECT in the
She would pilot the aeroplane accompanied only
following sentence:
by her navigator Fred Noonan.
a) Let’s have dinner now, haven’t us?
178
PROVAS DA AFA 1998 – 2010
She made her first attempt in March 1937, which a) Earhart and Noonan’s plane had an
ended in a crash in Hawaii (4), but within three accident in Hawaii.
months they were ready to another attempt. b) Both Earhart and her navigator were
Together Earhart and Noonan flew across the involved in a conspiracy together with the US.
United States, South America, North Africa, India, c) Earhart’s plane went down into the ocean
Asia, Australia and finally to New Guinea. But since it had no fuel.
then they had to face the most difficult part of the d) Earhart and Noonan were victims of a US
journey: 2,500 miles across the Pacific to Howland government’s secret plan.
Island.
Earhart and Noonan never arrived at Howland 04 - Mark the word from the text which has the
Island. Some believe the two of them drowned following definition:
because their plane ran out of fuel. But the most a) achievement
intriguing theory is that it was all part of a US b) challenge
government conspiracy. Some said that the US c) theory
Navy had used Earhart’s disappearance as an d) myth
excuse to send aircrafts over the Marshall Islands
where it was suspected that the Japanese were 05 - Observe the underlined clauses (1), (2), (3)
building military installations, others suggested and (4) in the text.
that Earhart and Noonan were part of the plot. It’s correct to say that we have Adjective Clauses
The myth of Earhart’s disappearance had begun, in
and it would never end. a) numbers (1), (3) and (4).
(Adapted from Speak Up – September 1999) b) numbers (1), (2) and (3).
c) numbers (2) and (4).
01 - Read the following statements: d) all of them.
I- There are evidences of a US conspiracy
related to Earhart’s disappearance. 06 - Think you feel bad about those lost
II - Earhart tried twice to fly around the world, vacation photos? Consider (1) Barry Mathews.
unsuccessfully. In 1993 (2) British geologist traveled to (3)
III - Amelia Earhart was a very unusual kind of Arctic Circle. There, he photographed soils and
woman for her time. vegetation for (4) study on climate change.
According to the text, Back in (5) England, he took (6) film to his
a) all the statements are true. local pharmacy for (7) developing. That was
b) two statements are false. (8) last he saw of it. Three years later, (9)
c) two statements are true. pharmacy admits it lost (10) film. Mathews, 59,
d) all the statements are false. is suing for £30,000 – (11) cost of another trip.
Our advice: take (12) sketch pad.
02 - All the alternatives below are correct About the 12 gaps on the text above, mark the
definitions for the word PLOT. Mark the only one false alternative.
which describes the meaning of PLOT in the text. a) All the gaps can be completed with definite
a) Measured piece of ground for building or or indefinite articles;
growing things. b) We cannot use articles in three of the gaps;
b) To follow the position of an aircraft using a c) Gaps number (4) and (12) are correctly
map or radar. completed with indefinite articles;
c) Line or curve drawn on a paper showing d) It’s correct to use definite article in seven
certain facts. gaps.
d) Secret plan to do something harmful in a
combined action. 07 - Student pilot Paul Marko __________
unexpectedly at Phil Brown’s home when he
03 - There are many theories trying to explain landed his plane on the roof.
Earhart’s disappearance. “I see this as a learning experience,” quipped
Mark the alternative which is not mentioned in the Marko, who was uninjured.
text as a possible explanation for this fact.
179
PROVAS DA AFA 1998 – 2010
The incredible rooftop landing in Tuscaloosa, Ala., (3) Contrast;
occurred on February 23 as Marko, 21, was on one (4) Addition; and
of his first solo flights. He’d ___________ from (5) Condition.
Atlanta to Jesper. As he neared his destination, his
single-engine Cessna ___________ of fuel and Chicago is known the world over as the birthplace
plunged toward earth. When Marko ___________ of the skyscrapers, the atom bomb, electric blues
guide it to a field near Brown’s three-bedroom and thick-slice pizza. The city is also renowned for
home, a wing clipped a tree and the plane landed its cruel climate, which can be unbearably cold in
belly down on the roof of Brown’s living-room. winter, _________(1) unbearably hot in summer.
Brown had to get a ladder to help Marko down, And yet, _________(2) this impressive list of
then he told his wife what happened and added: achievements, the “Windy City” is more
“Call the insurance man.”Tuscaloosa Sheriff Ted immediately recognizable for having been the
Sexton said Marko had “the Lord for his copilot.” home of a notorious gangster, Al “Scarface”
Mark the alternative in which the verbs complete Capone. _________ (3) some Chicagoans are
correctly the blanks in the text above. proud of the Capone’s heritage, members of the
a) dropped in / taken of / run out / attempted on city’s large Italian-American community tend to
b) dropped on / took off / ran up / attempted to resent any mention of him. _________(4), Al
c) dropped over / taken over / ran off / attempted Capone also raises the question of _________(5)
in Chicago is still run by “the mob”.
d) dropped in / taken off / ran out / attempted to (Adapted from Speak Up – Issue 110 – 1996)

08 - A sentence is underlined in the text. The correct expressions to complete the text above
This sentence in the Passive Voice would be: are:
a) A ladder has been gotten by Brown to help a) (1) and; (2) although; (3) Despite; (4) And;
Marko down. (5) if
b) A ladder had to be gotten by Brown to help b) (1) but; (2) furthermore; (3) Yet; (4) But; (5)
Marko down. whereas
c) Marko had to be gotten down by the Brown’s c) (1) despite; (2) whereas; (3) Though; (4) In
ladder. addition; (5) if
d) Marko was helped by a ladder gotten by Brown. d) (1) but; (2) in spite of; (3) Whereas; (4) Besides,
(5) whether
09 - Read the following passage and, then, mark
the alternative which contains the appropriate Directions: Read the text below and answer the
question tags to complete the gaps. questions from 11 to 14, according to it.
“You think I’m nuts, _________? You’ve just had
enough of my voice, _________? I’m such a Fear of Loose Nukes
hindrance in your life, _________? Let’s make a
last deal, __________? Leave now and never look Forget ANTHRAX; NOW it’s nukes. On Saturday
back over your shoulder, _________?” Osama bin Laden told Dawn newspaper in
a) don’t you / hadn’t you / am I / will we / shall Pakistan that Al Qaeda has nuclear weapons to be
you used “as a deterrent.” Although unconfirmed, his
b) aren’t I / haven’t you / am not I / shall we / claim can hardly be ignored. The United states has
don’t you long worried about what nuclear weapons Al
c) do you / have you / isn’t me / would we / won’t Qaeda could get its hands on.
you The main U.S. concern is that Russia is the source.
d) don’t you / haven’t you / aren’t I / shall we / Moscow has responded to fears that terrorists
will you could use stray nuclear material to build a “dirty
bomb” by promising to thighten security around its
10 - Complete the text below, so that the nuclear submarines and nuclear research centers.
sentences express, respectively: The United States has long encouraged such steps.
(1) Opposition; But last week the White House threatened to block
(2) Concession; a $40 billion appropriations bill if extra allocation
180
PROVAS DA AFA 1998 – 2010
– some designed to enhance security in Russia – 13 - The terrorists have nuclear weapons to be
were added, choosing instead to focus the package used “as a deterrent.” The underlined word in the
on domestic economic stimulus. text means:
Even if extra funds are eventually allocated to help a) something murderous and difficult to be
the Russians, it may be a case of too little, too late. controlled.
Two Turkish men – Aydin Duran, 32, and Mehmet b) something that has complete control over
Aydin, 47 – were arrested last Tuesday near unaware people.
Istanbul in possession of 1,150 grams of uranium, c) a substance that can cause serious damage
which they were allegedly trying to sell. Turkish and explosions.
police say the enriched uranium could have been d) something that stops people doing
used to build a bomb or, packed in a conventional something because they are afraid of what will
weapon, to spread radiation. And it’s known that happen if they do.
the two men traveled frequently to Eastern Europe
and Ukraine, where they probably obtained the 14 - The main U.S. concern is that Russia is the
uranium. (They have already admitted to buying source. Moscow has responded to fears that
the uranium from a Russian supplier.) The good terrorists could use stray nuclear material to build a
news is that Duran and Aydin don’t appear to have dirty bomb by promising to tighten security around
terrorist connections. They’re more likely a couple its nuclear submarines and nuclear research
of entrepreneurs out to make a buck, say police. centers.”
The bad news: Others like them may well have
already connected with Al Qaeda. In others words, Considering the underlined words as C (Count
bin Laden could be telling the truth. Noun) and NC (Noncount Noun), how can you
(Newsweek, November 19, 2001) classify them?
a) concern – C / source – NC / research – C
11 - We can deduce from the text that: b) concern – NC / source – C / research – C
a) Al Qaeda assured they have chemical weapons. c) concern – NC or C / source – C / research – NC
b) The United States have to face up the reality d) concern – NC or C / source – NC / research –
that they became feeble. NC
c) bin Laden claims to hinder the United States but
it’s not necessary to have faith in him. 15 - I’m still doubtful about the characters of
d) bin Laden has a different nuclear weapon and the new novel. You probably agree that the
maybe he can use it if the United States don’t suspects reactions were totally unexpected as their
respond to his remarkable exploit. explanations didn’t make their intentions clear
enough in the testimony. We could first deduce
12 - Mark the option which contains the correct that the fake nun was the prime suspect for having
alternative. poisoned the stallion and the sow. On the other
I- The Russians gave the United States an hand, the cunning steward and the wily widower
assurance that nothing would happen to them. had extravagant posture after the judgement.
II - It’s supposed that the Turkish weren’t
terrorists. They were just people on their own Considering the underlined words in the text, you
business. could say that:
III - Although the Turkish were connected to a a) Three words are female.
charitable organization, they weren’t under a b) All of words are male.
terrorist agreement. c) Two words are female.
IV - The terrorists may have something to d) Just one word is female.
inhibit the United States.
a) Statements II and IV are correct. 16 - Mark the option that completes the text
b) Statements III e IV are correct. correctly:
c) Statements I and II are correct. My little boy is just 8 but completely curious about
d) Statements I and III are correct. the study of different _________ and theories. He
got the surprise of his life when he discovered at
school that ________ are the smallest living
181
PROVAS DA AFA 1998 – 2010
things. He is enchanted with hundreds of 19 - According to the Possessive Case rules,
________ about the universe and surprised for mark the only option in whick the justification in
sometimes not finding answers to the ________ parentheses is correct:
and the fantastic development of nature. Therefore, a) Thomas’s knapsack was sewn by a
I suppose he knows his ________ can also explain hospitable old woman in the last hostel he was.
to him some doubts he has about thousand of (Incorrect – If a noun ends in s, you put just ’
________ of the universe and procreation, after the s )
considering the existence of God, of course. b) Next week’s meeting has been canceled
a) formula / bacteria / mysterys / thesis / because of the last stormy night. (Incorrect – you
believes / phemenon can’t use ’s with time expressions)
b) formulas / bacterium / mystery / theses / c) Thousands of passers–by went to the city’s
believes / phenomenos new theater to shelter from the rain. (It’s possible
c) formulae / bacteria / mysteries / theses / to use ’s for places )
beliefs / phenomena d) Jack and Jill need eight hours’ sleep a
d) formulas / bacterias / mysterious / thesis / night. (Incorrect – you can’t use ’s with plural
beliefs / phenomenon words and with periods of time)

17 - The funniest scene of the comedy is the one 20 - Read the text to complete the blanks
when the host spills red wine on the heir’s coat and correctly with the right option:
by mistake kneels down beside a shy spinster to I’m used to _________ with the window open
offer _____ apologies. So, a problem of because I appreciate _________ during the night.
communication arises between them because the My husband can do nothing but _________ the
shy spinster is deaf. Besides, _____ can’t fact. I have trouble _________ I’m wrong, but I
understand what’s happening. While the heir is in realize the situation is sometimes a little hard
the bathroom cleaning the coat, a water pipe bursts because nights are windy in this time of the year.
and floods the whole place where the guests are. Harold has frequently encouraged me ________
The heir _____ gets confused about what to do. I this vice as he is obliged _________ the poison I
burst into laughter. put into the air. I fell sorry _________him.
a) sleep / to smoke / accepts / to admit /
Choose the item that completes the gaps correctly: quitting / to inhale / bothering
a) his / he / by herself b) sleeping / smoking / accept / admitting / to
b) her / she / herself quit / to inhale / to bother
c) him / he / by himself c) sleep / smoking / to accept / to admit / to
d) his / she / himself quit / inhaling / bothering
d) sleeping / to smoke / accepting / admitting /
18 - Read the sentences carefully: quitting / inhaling / bothering
I- The longer Sue stays in Canada, the less
likely she will ever go back to England. Directions: Read the text below and answer the
II - The warmest the weather, the best I fell. questions from 21 to 25, according to it.
III - My brother-in-law is elder than I am but he
looks younger. Central Station
IV - Her illness was far more serious than we
thought at first. The reality of everyday life that “Central Station”
V- The most electricity you use, the higher shows is harsh. In the film, Dora (Fernanda
your bill will be. Montenegro) is a bitter woman who makes her
The correct alternative is: living in Rio’s Central Station writting letters for
a) Two sentences are correct. illiterate people. She takes their money but
b) Two sentences are incorrect. discards the letters.
c) Just one sentence is incorrect. One day she writes a letter for a mother and her
d) Just one sentence is correct. little boy
(Vinícius de Oliveira). When the mother is killed
in an accident outside the station, Dora tries to sell
182
PROVAS DA AFA 1998 – 2010
the boy for adoption. Then she realizes her c) damaged d) considered
mistake, rescues the boy and the two set out on a
bus 25 - “Dora tries to sell the boy for adoption.
trip to find his father. For Walter Salles, the Then she realizes her mistake, rescues the boy and
director of the film, Dora is a good example of the two set out on a bus trip to find his father.” In
modern Brazil, with its culture of “cynicism”. But the paragraph above we may conclude that Dora
as Dora gradually develops a bond with the boy a) would have sold the boy for adoption if she
“she begins to understand that the boy’s route and hadn’t had notions of her mistake.
the boy’s problems are b) would sell the boy for adoption if he didn’t
comparable to her own”, he said. help her during the trip.
The growing friendship between these two is, for c) would have sold him if he hadn’t gone to
Mr.Salles, a symbol of Brazil where solidarity and her on a bus trip
compassion may be buried but are still present. His d) wouldn’t sell him unless he wanted to go
film is not utopian, but it celebrates the diversity with her on a bus trip.
both of the land and of what Mr. Salles calls the
“human geography” that Dora and Josué encounter 26 - In the sentence “In fact, it’s my concern
on their journey. that this problem must be solved as soon as
possible”, the underlined expression could be
(From “A Searching Journey into the Heart of replaced by:
Brazil,”by Laura Winters, a) Unless b) On the whole
In The New York Times, c) Indeed d) Moreover
November 22,1998.)
Adapted from New Password English, 27 - Read the following sentences carefully:
MARQUES, Amadeu (Vol.1) I- The Earth is the only planet we have.
Therefore, we must take care of it.
21 - In the sentence, “But as Dora gradually II - Although Galileo had to deny it because of
develops a bond with the boy...” We deduce that the Inquisition, he believed the sun was the center
she of the universe.
a) unites him to her as a bitter stepmother. III - The humankind won't have a good future
b) feels linked to him. unless we stop destroying our environment.
c) connects her feelings with his since they IV - Neither Gandhi or Luther King were
weren’t tramps. understood the way they should have been.
d) shows a marked lack of affection in him. According to the use of the underlined words,
mark the correct alternative.
22 - “The reality of everyday life that Central a) All the sentences are correct.
Station shows is harsh.” b) All the sentences are incorrect.
The underlined word may be replaced by c) Only one sentence is incorrect.
a) cunning b) harmful d) Only two sentences are correct.
c) strict d) neglectful
28 - Read the paragraph,
23 - “She takes their money but discards the
letters.” The underlined words mean that she “On Friday, my classmates and I _________ do an
a) sends them to people English exam at school. They invited me to study
b) uses them to her own with them and I _________ arrive there earlier in
c) writes them to people order to relax and concentrate. If we have time to
d) gets rid of all them check all the program before it, I’m sure I
_________ do better. Probably, I _________ finish
24 - In the sentence ”...where solidarity and it on time because I am not good at foreign
compassion may be buried but are still present”, languages at all, but I _________ try to improve it
the underlined words say that the two feelings can looking for a personal instructor. You know, I
be _________ travel to Australia next semester and I
a) destroyed b) ignored need to learn this language urgently.”
183
PROVAS DA AFA 1998 – 2010
The correct verb tenses to fill in the blanks are, 3- Just_________knowledge is needed to
a) am going to/ won’t / will / am going to / solve such elementary technical problem.
will not / ‘ll 4- Do you get_________chance to travel in
b) were going to / are going to / will / will not your job?
/ will / will 5- After_________deliberation the judges
c) are going to / am going to / will / won’t / awarded the prize to her.
‘m going to / ’m going to a) few – Many – little – much – much.
d) was going to / will go / will not / won’t / b) a lot of – Few – many – lots of – little.
was going to / won’t c) much – Lots of – a few – a little – a lot of.
d) lots of – Little – lots of – little – few.
29 - Mark the correct set of prepositions to
complete the following passage. Directions: Read the following text and answer
A couple ____ the United States reported that they questions 32 and 33, according to it.
were driving _____ a country road _____ 10:00
p.m. when they were blinded ____ a very bright “Ela é carioca,
light ____ the sky. The next thing they remember She’s a carioca
is that they woke up 300 miles away _____ the Just see the way she walks.
place they were before. They believe they were Nobody else can be
abducted ____ aliens. What she is to me.
a) in - along - at - by - in - from - by I look and what do I see,
b) from - at - around - for - on - in - for When I look deep in her eyes?
c) of - in - until - by - to - through - with I can see the sea,
d) in - towards - to - for - from - at - of A forgotten road,
The caressing skies,
The Lord’s Prayer And not only that I’m in love
With her.
Our father who art in heaven The most exciting was
Hallowed be thy name It’s written on my lips
Thy kingdom come Where her kisses stay.
Thy will be done She smiles and all of a sudden
On earth as it is in heaven The world is smiling for me,
Give us this day our daily bread And you know what else, she’s a carioca
And forgive our trespasses Ela é carioca.”
As we forgive those who trespass against us (Antônio Carlos Jobim/ Vinicius de Moraes/Ray
And lead us not into temptation Gilbert)
But deliver us from evil
Amen 32 - In the text above, we can find
a) seven regular verbs, five irregular and one
30 - According to the prayer you have just modal.
read, all the sentences below are in imperative b) six regular verbs, six irregular and two modals.
form,except c) four regular verbs, four irregular and one
a) Give us this day our daily bread modal.
b) Forgive our trespasses d) four irregular verbs, five regular and two
c) As we forgive those who trespass against us modals.
d) Lead us not into temptation
33 - The sentence “Nobody else can be what
31 - Taking the following sentences mark the she is to me” means that
option which fills the blanks correctly. a) she’s more important than his life.
1- Is it true that _________suggestions were b) he loves her more than himself.
declined by the senators? c) no one is like her to him.
2- _________manholes could be severely d) she’s the most important person to everybody.
depleted over the storms.
184
PROVAS DA AFA 1998 – 2010
Directions: Read the dialog below and answer 36 - "...Jonathan isn't sure he'll go into the
questions number 34 and 35 according to it. family business. He might want to become an
astronuclear physicist, he says: 'Then I could make
Office: "Control tower, this is the Captain on duty a rocket ship go to Mars, instead of just writing
speaking. The jumbo jet 104 isn't allowed to take about it'."
off. Cancel clearance. I repeat: cancel clearance According to the paragraph above, mark the
right away." correct statement:
a) Jonathan is sure about his vocation and his
Controller: "I'm afraid it's impossible, Sir. The future career.
jumbo jet 104 departed two minutes ago." b) Jonathan admits the possibility of being a
physicist.
34 - According to the dialog above, mark the c) Jonathan has no idea about which career to
correct statement. follow.
a) The control tower could cancel the d) Jonathan is sure he won't go into his family
clearance though the jumbo jet had departed two business.
minutes later.
b) It was impossible to cancel the clearance 37 - A woman was patiently putting some
because the officer wasn't allowed to take off. moisturizer on her face as she usually did at night,
c) When the officer ordered to cancel the when her five-year-old son asked what she was
clearance, the jumbo jet would depart in two doing. She told him that the cream was for
minutes. wrinkles. The boy said innocently, "It's doing a
d) When the officer on duty ordered the good job, Mommy. You're getting a lot of them."
control tower to cancel the clearance, the jumbo jet
had already taken off. The kinds of adverbs we can find in the passage
above are:
35 - The dialog in the Indirect Speech would be:
a) The Captain advised the Controller that the a) only adverbs of time, intensity and place.
jumbo jet 104 was not allowed to take off and b) adverbs of manner, frequency, place and time.
canceled clearance at that moment. Then, the c) adverbs of place, time, manner and intensity.
Controller replied he was afraid and that it was d) only adverbs of manner, place and time.
impossible as the jumbo jet 104 have departed two
minutes before. 38 - Mark the alternative in which the numerical
b) The Captain warned the Controller that the expression is correctly described in the
jumbo jet 104 wasn’t allowed to take off and parentheses.
ordered him to cancel clearance immediately. So, a) 1,500  100 = 15  (fifteen hundred divided by
the Controller answered he was afraid that it was one hundred equals fifty)
impossible as the jumbo jet 104 had departed two b) 17 + 23 = 40  (seventeen more twenty and
minutes before. three is forty)
c) The Captain reassured the Controller that c) 64 - 64 = 0  (sixty-four minus sixty-four equals
the jumbo jet 104 isn’t allowed to take off and to zero)
cancel clearance right away. Then, the Controller d) 30 x 3 = 90  (thirty into three is ninety)
mentioned he had been afraid and that it was
impossible as the jumbo jet 104 departed two 39 - Mark the correct alternative to complete the
minutes before. dialog:
d) The Captain commanded the Controller A: "What a terrible trip I had."
that the jumbo jet 104 hadn’t been allowed and B: "Come with me. I'll show you a place to sleep."
took off to cancel clearance immediately. So, the A: "Thanks. By the way, (I) ?"
Controller said he was afraid that it had been B: "It's Linda's. She's living in London now. So,
impossible as the jumbo jet 104 had departed two you can use it as long as you want."
minutes before. A: " (II) ?"
B: "Since last summer."

185
PROVAS DA AFA 1998 – 2010
a) (I) Whose room is this? 1) We can deduce from the text that it is
(II) How long has she been living there? I- A supplication in which we somehow emphasize
our devotion to Jesus Christ.
b) (I) Who's this in the picture? II – A sermon in which someone who is god1ly
(II) How much was she away? declares his feeling for a devilish woman.
III – A pray in which a diveness is magnified.
c) (I) Whose is this bedroom? IV – A piece of praise in which we clumsily beg
(II) When did she go to London? forgiveness for our faults.

d) (I) Who had lived in this room? The correct statement(s) according to the text is
(II) When has she lived in London? (are) only
a) I and III b) II c) III and IV d) I, II and IV
40 - Observe the following dialogs:
I- Sarah: “What about my new skirt? Does it 2) Mark the option in which all the words have the
suit me?” same meaning of Holy.
Joe: “Sure. You do look great in it. It does a) adorable / hale / devilish
suit you.” b) hallowed / sacred / blessed
c) sanctified / venerable / renowned
II - Captain: “Your negligence caused an d) renewable / vengeful / pious
airmiss during the mission. What do you say about
this?” 3) How many different prepositions can you find
Controller: “Sorry, Captain, but I did try to in the text?
copy your message. Put the blame on a a) four b) five c) six d) seven
malfunction in the radio operation.”
Read the notice below and answer questions 4 and
III - Boss: “Do take good care of these jewels. 5
They’re really valuable.”
Employee: “ Please, don’t ask me to be that “An unnamed boy aged 14 in Washington DC
responsible.” _______ false fire alarms. He _______ psychiatric
treatment to cure him of that, and he _______ it.
According to the use of DO and DID in the Apparently he learned _______ the glass and pull
sentences above, mark the correct alternative. the hook _______ there actually was a fire. So –
a) All the dialogs are correct. now he sets the house on fire first, and has done it
b) Only one dialog is correct. four times.”
c) Only two dialogs are correct. (Baltimore Sam)
d) All the dialogs are incorrect.
4) Choose the right words to complete the text
AFA 2003/2004 – Inglês above so that it’s grammatically correct.
a) used to turning in / undergoes / stops to do / not
to break / if
Hale Mary
b) used to turn in / underwent / stopped doing /
never to break / unless
Hale Mary full of grace
c) is used to turning in / has undergone / has
the Lord is with thee
stopped doing / doesn’t break / since
Blessed are thou
d) was used to turning in / had undergone / stopped
among sinners
to do / rarely breaks / whether
and blessed is thy
Womb Jesus.
5) The text can be considered
Holy Mary Mother of God
a) an insane plot b) a fateful joke
pray for our sinners now
c) a witty account d) a literary tale
and at the hour of our death
Amen.

186
PROVAS DA AFA 1998 – 2010
Read the following paragraph and answer 10) What’s the definition of Monastery?
questions 6, 7 and 8 a) the place where a judge decides whether
someone is guilty of a crime.
“An elderly German decided to commit suicide. b) a building where men live as a religious group.
Took a lot of pills, tied a briefcase full of stones c) a large building where government is based.
around his neck, rowed out into the middle of the d) an official court.
Rhine and was found sound asleep in his boat.”
(Buffalo News) 11) After reading the two stories below, mark the
option that best explain the tenses used in each of
6) In the first sentence “An elderly German them.
decided to commit suicide”. The word elderly is
used as Text A
a) a more polite form for old Unemployed Terry Fitton has applied for an
b) a synonym for eldest amazing 2.350 jobs… and he’s still out of work.
c) the comparative form of the adjective elder Terry, 50, has posted applications at the rate of
d) the comparative form of the adjective old nearly four a day for the past two years.
usually used when we compare members of a
family Text B
Superstar Paul McCartney last night watched a
7) In the sentence “Took a lot of pills”, which heart-stopping sea search for his 15-year-old son
other expression of quantity couldn’t be used in it? James. The ex-Beatle and his wife Linda stood
a) a large number of b) a couple of ashen-faced on a beach after the youngster was
c) a great deal of d) plenty of swept out of sight while surfing. But thirty minutes
later they were joyfully hugging James as he
8) According to the end of the paragraph we could stepped unharmed from the waves.
see that
a) the man was discovered in deep sleep in his a) Text A has a time phrase: for the past two years,
craft. which sets the time as time coming up to now. And
b) the man was encountered in a boat by the sound Text B has the time phrase last night, which sets
of his snore. the time as time finished, separated from now.
c) the man was met in a bow soundly fainted. b) Text A has no explicit time phrase used so it
d) the man was stumbled upon in the seashore. doesn’t matter which tense you choose when there
is no time phrase. And in Text B, past is used
Read the text below and answer the questions 9 because the speaker considers it important in
and 10 relation to now.
c) In Text A it doesn’t matter which tense you
“_______ a possession next Sunday afternoon in choose because there is no time phrase, so both are
the grounds of the Monastery, but _______ in the always possible. In Text B, past is used because
afternoon, the possesion _______ into the things have happened recently.
morning.” d) In Text A, the present perfect is used because
(Notice in Irish Paper) things are separated in the speaker’s mind from
now. In Text B, the past is used as things happened
9) Use the right grammatical structures to a long time ago.
complete the text above.
a) There will have / unless it rains / have to take
place
b) There’ll be able / when it rains / is going to take
place
c) There’s going to be / whereas it rains / won’t
take place
d) There will be / if it rains / will take place

187
PROVAS DA AFA 1998 – 2010
Read the cartoon and answer questions 12, 13 and On this endless ocean
14 Finally lovers know no shame
Turning and returning
“I’m learning how to relax, doctor – but I want to to some secret place inside
realax _______ and _______! Watching in slow motion
I WANT TO BE ON THE CUTTING EDGE OF As you turn around and say
RELAXATION!” Take my breath away
Take my breath away
12) Complete the dialog with the appropriate
words. Watching I keep waiting
a) more quickly / best Still anticipating love
b) further / the higher Never hesitating
c) the sooner / the better To become the fated ones
d) better / faster Turning and returning
To some secret place to hide
13) “I want to be on the cutting edge of Watching in slow motion
relaxation!” We could understand by the As you turn to me and say:
underlined expression that the patient wants to be “Take my breath away”
a) very modern and with all the newest
development of relaxation. Through the hourglass I saw you
b) on the largest amount that is allowed or possible In time you slipped away
of his relaxation. When the mirror crashed I called you
c) on his best relaxation. And turned to hear you say:
d) on the upper limit of his relaxation. “If only for today I am unafraid
Take my breath away”
14) We could deduce from the dialog that
a) as the patient is not considerably anxious he’s Watching every motion
going to be relaxed very soon. In this foolish lover’s game
b) as the patient is very strict, being on top of Haunted by the notion
relaxation is going to be stimulating for him. Somewhere there’s a love in flames
c) it’s quite impossible for anyone to relax in this Turning and returning
way. To some secret place inside
d) indeed, the patient has already learnt how to Watching, in slow motion
relax. As you turn my way and say:
“Take my breath away”
Read the text below and answer questions from 15
to 21 15) Mark the item which contains the right
affirmative(s) according to the text:
Take my Breath Away I – The lyrics shows a love story between a pilot
(G. Moroder / T. Witlock) and his superior in an American Air Force base.
II – The song expresses the dates’ dizzy feeling
Among eighteen recorded CD’S by the London which is not reciprocal.
Starlight Orchestra, eleven are dedicated to some III – The context shows that someone is infatuated.
good movies. They are considered veracious IV – It’s clear that the lovers hold their breath
musical photographs as part of the story of cinema. whenever they see each other.
“Take My Breath Away”, from Moroder and V – The mates have inconclusive and secret blind
Witlock, is the love theme of the remarkable dates as they are forbidden.
movie “Top Gun” and also from the album that
contains more than seventeen of the same type. a) Only statements II and V are correct
b) Only statement II is correct
Watching every motion c) Only statements I and V are correct
In my foolish lover’s game d) Only statements II and IV are correct
188
PROVAS DA AFA 1998 – 2010
16) Mark the word from the second verse which 22) Read the text below and then complete the
has the following definition. gaps with correct words.
“If people _______ their mates, the mates would
Something that seems certain to look, act, and think very much like _______.
happen because mysterious force is _______ attracted to people _______ ourselves,
controlling events.
we are in effect validating _______ saying to
ourselves that we are worthy of being liked, that
a) game b) foolish c) fated d) hide we are attractive. _______ there are exceptions, we
generally like people _______ are similar to
17) Extracting the word ones from the second ourselves in nationality, race, ability, physical
stanza, we may say that it refers to characteristics, intelligence, attitudes, and so on.
a) the lovers We are _______ attracted to mirror images of
b) their feelings ourselves.”
c) their secret places
d) their breath a) could construct / themselves / By being / like /
ourselves / Açthough / who / often
18) According to the title of the song, what’s the b) constructed / it / To be / enjoy / them /
best definition for the lover’s feeling? Otherwise / that / generally
a) you keep air in my lungs and not let it out. c) ‘d constructed / ourselves / Being / involve / us /
b) you make me feel shocked by surprise. Therefore / whom / frequently
c) you make me feelbrand new. d) should construct / we ourselves / Been / as / it /
d) you hold my breath deeply. But / that / se]ldom

19) What’s meant by the sentence “Watching in Read the text below and answer question 23
slow motion”?
a) Being reduced to tears. A Rosa de Hiroshima
b) Loking for more exciting emotions.
c) Concerning about one’s sluggish emotions. Pensem nas crianças mudas telepáticas
d) Observing carefully. Pensem nas meninas cegas inexatas
Pensem nas mulheres rotas alteradas
20) Considering the sentence “in my foolish Pensem nas feridas como rosas cálidas
lover’s games.” The boldface structure is Mas, oh, não se esqueçam da rosa, da rosa
grammatically correct in the context because Da rosa de Hiroshima, a rosa hereditária
a) ‘s is used with singular and plural nouns not A rosa radioativa, estúpida e inválida
ending in s. A rosa com cirrose, a anti-rosa atômica
b) a simple apostrophe s (‘s) is used with abstract Sem cor, sem perfume, sem rosa, sem nada.
nouns. (Vinícius de Moraes)
c) it’s a classical name not ending in s.
d) referring to feelings, ‘s shoujld be used. 23) In this song Vinícius de Moraes indignantly
expresses his disappointment about atomic bombs
21) How can we classify the first verb in the first dropped on the cities of Hiroshima and Nagasaki
verse? It’s during the World War II. In the sentence “A rosa
a) a present participle used as part of the present radiotiva, estúpida e inválida”, he tries to let us
progressive. know that Americans
b) a gerund and it is used as the subject of the a) had done such a foolish attack and now they
sentence. have been seen as the winners.
c) not used as infinitive, just as gerund because it’s b) were responsible for the rose he refers to which
a special expression. was formed by a smoking cloud whose shape
d) a gerund used with certain idiomatic canvassed them.
expressions, for the most part, recreational c) could get anything but lots of deaths and
activities. crippled people for years on.
189
PROVAS DA AFA 1998 – 2010
d) realized how stupid the war was attaining its a) assured Jack not to go there.
aim which was to subvert Hiroshima and Nagasaki b) donated Cindy not to go there.
and allure their population. c) claimed Cindy not to go there.
d) warned Jack not to go there.
24) Not only sad poems and songs but also happy
ones wrote our great poet Vinícius de Moraes in 27) Read the paragraph below and answer the
which he appreciates alove and happiness like the question that follows it.
following we have below:
“That’s incredible!! Since Nathalia’s gone to Italy
“É melhor ser alegre que ser triste her little dog Valeska doesn’t eat anything at all.
A alegria é a melhor coisa que existe So, how can we interrupt her during her
É assim como a luz no coração...” honeymoon trip once it’s not a sensible reason to
(Samba da bênção) tell her to return home as soon as she can?”

Taking this part of the song the author Observing the inderlined words, it’s correct to say
a) uses the comparative of superiority and the that the sentence
superlative forms to show us how life is anguish. a) has a verb in the present perfect tense since it
b) compares the beuty in equality with life in order doesn’t have an adverb of time.
to show us the former is the most important. b) has a verb in the contracted form to be more
c) is so happy that he says the light in our heart is informal.
more important than our happiness. c) has the main verb in the past participle because
d) only wants to show us how good feeling happy of the contracted form of to be.
is. d) doesn’t have any auxiliary because it’s an
affirmative one.
25) Read this paragraph and mark the correct
option. Read the text below and answer questions 28, 29
and 30
“I give up telfling our grandpa going on a diet. The
nutritionist has advised him to eat only organic Turning the world into a better one is our
food but he insists on doing the other way around.” mission because we are the world. Observe the
way Michael Jackson and Lionel Ritchie used to
The correct passive form to say the sentence “The show people their point of view about it.
nutritionist has advised our grandpa to eat only
organic food”, is WE ARE THE WORLD
a) Only organic food has been advised by our
grandpa to the nutritionist. There comes a time
b) Only organic food were advised our grandpa. When we hear a certain call
c) The nutritionist advised our grandpa to eat only When the world must come
organic food by him. together as one
d) Our grandpa has been advised to eat only There are people dying
organic food by the nutritionist. And it’s time to lend a hand to life
The greatest gift of all
26) Read the dialog below
We can’t go on
Cindy: “You’d better not to go there. It could get Pretending day by day
you into trouble, Jack”. That someone, somewhere will soon make a
Jack: “Don’t be afraid. Since I’m sure about my change
purposes no one will make me give up of doing it.” We are all a part of
God’s great big family
The sentence: “You’d better not to go there. It And the truth, you know, love
could get you into trouble, guy.” Means that the is all we need
speaker
190
PROVAS DA AFA 1998 – 2010
(Chorus) 30) Mark the correct option after reading these
We are the world lines from the song WE ARE THE WORLD.
We are the children
We are the ones who make a “WE CAN’T GO ON
brighter day PRETENDING DAY BY DAY
So let’s start giving THAT SOMEONE, SOMEWHERE
There’s a choice we’re making WILL SOON MAKE A CHANGE”
We’re saving our own lives
It’s true we’ll make a better day It let us understand that
Just you and me a) none of us cannot wait a long time for better
Send them your heart days.
b) we can’t continue living making believe that
So they’ll know that someone cares someone in a near future will change the world.
And their lives will be stronger and free c) people all over the world cannot live day by day
As God has shown us by turning waiting for someone to vex the world sooner.
stone to bread d) someone one day will surely make a huge
so we all must lend a helping hand change in the world.

(Chorus)
We are the world AFA 2004/2005 – Inglês
(…)
When you’re down and out Read the text below and answer questions 01 and
There seems no hope at all 02.
But if you just believe
There’s no way we can fall Ampicilim
Well, well, well, well, let us realize
That a chance will only come Indications: urinary-tract infections, otitis
When we stand together as one
media, sinusitis, chronic bronchitis.
(Chorus)
We are the world Cautions: history of allergy: renal impairment:
(…) erythematous rashes common in glandular fever.
Interactions: Appendix 1 (penicillin)
28) In the sentence “When the world must come
together as one”, the singers mean that Contra-indications: penicillin hypersensitivity.
a) their country has to be together with others Side-effects: nausea, diarrhea: rashes (discontinue
giving them a hand to change. treatment): rarely, antibiotic-associated colitis.
b) there are needy people around the wor]ld who
must be succoured. Dose: by mouthy, 0.25 – 1 g every six hours, at least
c) all the peoples should join their effort in order to 30 minutes before food.
live as a unique one. By intramuscular injection or intravenous injection or
d) popu]lation from different countries ought to
infusion, 500mg every 4-6 hours: higher doses in
keep the world whimsically alive.
meningitis.
29) According to their way to express the world’s CHILD under 10 years, any route, half adult dose.
situation LIFE is
a) the biggest truth of all.
b) the most important point to be given attention 1) Mark the option that completes the text in the
to. best way.
c) a very brittle human need.
d) choices to be made. It’s a ________ indicated for people ________
have ________ diseases. ________ it’s necessary
caution because according to the contra-indications

191
PROVAS DA AFA 1998 – 2010
, the ones ________ have penicillin IV - Immediately after an assault, rape or
hypersensitivity ________ take it. robbery, victims or witnesses can usually summon
It’s important ________ the side effects. Thus, up a mental image of the assailant. But the time
take the right dose in order to get ________. they sit down with a sketch artist at the police
station, their memories often have faded.
a) prescription / whom / any / Therefore / whose / (Adapted from Time)
can’t / remembering / best.
b) recipe / that / some / But / Who / shouldn’t / to 3) In which extract(s) above you cannot find
remind / well. Passive Construction(s)?
c) piece of information / who / some / However / a) Only in extracts I and III.
that / mustn’t / to remember / better. b) Only in extracts II and IV.
d) piece of advice / whose / any / Although / which c) Only in extract IV.
/ mightn’t / reminding / good. d) Only in extract III.

2) Which sentence(s) below is (are) grammatically 4) What are the real topics mentioned in the
correct according to the context and the use of If extracts you’ve just read above?
Clause? I - The reason why the house was called
Commonwood House.
I- If you have any contra-indication in relation to II - The fact that a hospital can feed the poor
penicillin hypersensitivity, it’s better not to take it. patient with good supply.
II - It will be supposed that the results won’t be III - How a great part of a work was completed in
so efficient, if the patient wouldn’t take the the nineteenth century, how it was farmed in some
medicine at least 30 minutes before food. places and the changes they suffered.
III - If the side-effects of this remedy were nausea IV - There’s an evidence that after an assault or
and diarrhea rashes it were prudent doesn’t make robbery, the victims get frightened of telling the
use of it. police the true facts.
IV - It won’t be necessary to use the liquid a) I and IV. c) II, III and IV.
ampicillin if the users would prefer to make use of b) I and II. d) I and III.
this by intramuscular injection.
V - If the patient is going to be a child under 10 Read the ad and answer question 05.
years, he or she should have used just half adult
dose. Lockheed Martin and Sikorsky. Working together
for three decades, this team ________(I) and
a) Only IV and V. c) Only I. integrated __________(II) any other team in the
b) Only III. d) Only II and IV. world. That includes the SH-60B, ________(III)
the new MH-60R and MH60S programs. This
Considering the extracts of some texts, answer __________(IV) delivers the proven Naval Hawk
questions 03, and 04 airframe with advanced multi-mission avionics
capabilities to the U.S. Navy and navies worldwide
I- The woods were visible from the back – together we deliver ________(V) naval
windows and it was clear why the house was capability to ________(VI) customers.
called Commonwood House, because there were ( SiKorsKy )
views of the common from all sides. A United Technology Company
II - The hospital where Maggie in Little
Dorrit was treated for fever is able to provide its 05 - Fill in the blanks correctly with
impoverished patients with fruit and soft drinks... grammatical subjects that have been asked in the
III - Most of the work was completed before questions below:
the start of the nineteenth century. Thus the next I- Use the Present Perfect Tense of the verb to
chapter will focus on how land was farmed in build.
Northamptonshire and Bedfordshire in the II - Use a Comparative form of Superiority.
eighteenth century. Changes after 1700 will not be III - Use a Comparative form of Equality.
discussed. IV - Use the right Possessive case of nouns.
192
PROVAS DA AFA 1998 – 2010
V- Use the Superlative form of an adjective. Before the 1930’s folk songs were almost
VI - Use the right Pronoun. unknown in American cities. By the end of World
War II , in 1945, young people became interested
a) has built / more naval helicopters than / as in traditional songs. Perhaps they were tired of hit
well as / team’s synergy / the best / our –parade tunes. Perhaps they wanted to hear
b) have build / many naval helicopters as / so meaningful songs that made sense of them.
good as / teams’ synergy / the better / ours
c) had build / much more helicopters than / 08 - How many different prepositions can you
less well as / team synergy’s / more and more / us find in the paragraph?
d) has builded / less helicopters than / not so a) eight b) four c) five d) seven
bad as / team synegys’s / the most / we
09 - Reading the sentence: “Perhaps they were
06 - Which responses would you choose for tired of hit- parade tunes”, we can conclude that
these questions in the dialog below to make it a) the Americans weren’t used to listening to
correct? new songs on the radio but only the live ones.
Larry: “Paola has different customs. Is she b) the American people wanted to hear a
organized? different sort of rhythm that valued their traditions
Sandy: “_______________” and was related to their way of living.
Larry: “Do you know where ____________ my c) they didn’t like folk songs and after the
new ring?” Second World War these songs became recognized
Sandy: “Maybe she’s lost it.” around the world .
Doris: “_______________.” d) in America , they just listened to songs that
a) She may / she keeps / she should have were produced in their cities and the traditional
b) She may be / she kept / she might have ones had no sense of them.
c) She is / does she keep / she might
d) Yes / did she keep / she may be Read the text below and answer questions 10 to 13
according to it.
Read the text below and answer question 07.
One day in 1895 a German scientist named
A woman was so fed up of having to wait two Wilhelm Roentgen discovered an astonishing
hours in a physician’s waiting room that she billed thing. He ______________ with the electron rays,
him $50 an hour for her wasted time – and shooting them at a coated glass screen. Roentgen
amazingly, he agreed to pay! happened to pass his hands between the rays and
(adapted from The National Enquirer) the screen. To his amazement, the shadow of the
bones inside his hand ________________ instead !
07 - We could understand from the text that Most of the rays _______________ right through
a) the unlucky woman obliged the man the flesh of his hand lighting the screen, but
eventually to pay her an amount of money for she enough of them were stopped by the bones to
has been waiting for him long hours. produce a clear shadow of the hand’s skeleton.
b) surprisingly a doctor paid a woman for being Roentgen ______________ the X-ray.
waiting for him such a long time. (From ATOMS – The Golden Library Of
c) the physicist had to apologize to his patient Knowledge)
by paying her some money because she waited for
him a lot. 10 - Fill in the blanks in the text. Choose the
d) a cheerful woman got so impatient into the best verbs to complete it
man’s room that she made him pay her fifty a) experimented / appears / passes /
dollars. discovered
b) had experimented / appeared / has passed /
Read the text below and answer questions 08 to 09 has discovered
according to it. c) is experimenting / is appearing / passed /
was discovering

193
PROVAS DA AFA 1998 – 2010
d) was experimenting / appeared / were California man whose wife of 23 years left him
passing / had discovered after meeting a man online, Chatcheaters.com
averages 400 visitors (mostly women) a day.
11 - The underlined words in the text are (Adapted form Online Detectives, Speak Up, ed.
respectively 202, mar. 2004)
a) gerund form / possessive pronoun /
preposition / possessive adjective / preposition 14 - Mark the option that shows the correct
b) adjective / object pronoun / preposition / statements according to the reading:
possessive adjective / conjunction I- Spying activities have improved as well as
c) noun / subject pronoun / conjunction / the technology.
possessive pronoun / adverb II - Chatcheaters is a web site that concerns
d) continuous tense / adjective pronoun / unfaithful matters.
adverb / possessive pronoun / preposition III - The news is a result of a wife who was left
for a younger woman.
12 - The sentence: ”The rays were stopped by IV - The Internet has become a way of finding
the bones to produce a clear shadow of the hand’s partners.
skeleton”, in the ACTIVE VOICE is a) All the statements are correct.
a) To produce the rays by the bones the rays b) Only statements I and II are correct.
stopped a clear shadow of the hand’s skeleton. c) Only statement III is incorrect.
b) A clear shadow of the hand’s skeleton d) Statements II and III are incorrect.
produced the bones by the rays stopped.
c) The bones stopped the rays to produce a 15 - The underlined expressions from the text
clear shadow of the hand’s skeleton. (once upon a time and but) mean according to the
d) To produce a clear shadow the rays of the context, respectively:
hand’s skeleton stopped the bones. a) something happened a long time ago /
opposite information
13 - It’s correct to say that the text b) repeated action / connecting phrases
a) relates an accidental discovery of the x-ray c) at the same time / explaining why
by a German scientist, describing how the d) sometimes but not often / showing surprise
experiment was done and its unexpected result.
b) details the production of clear shadows at a Read the part of the lyrics and answer questions 16
coated glass, appearing the scientist’s skeleton on and 17:
the screen.
c) describes how Roentgen was shooting Slip inside the eye of your mind
electron rays on his hand lighting the bones and Don't you know you might find
the way it made him astonished. A better place to play
d) shows how an amazing experiment could You said that you've never been
make a scientist become famous only trying to But all the things that you've seen
reflect the shadow of the flesh of his both hands. They slowly fade away
(Don’t look back in anger – Oasis)
Read the paragraphs below and answer questions
14 and 15, according to it. 16 - Mark the option that justifies the use of the
underlined verbal tense in the context:
Once upon a time, suspicious husbands and wives a) indefinite time b) reported speech
called in private detectives to check on a cheating c) finished action d) beginning of an action
spouse. But in today's high-tech world, where the
Internet fuels online chatting and many affairs, a 17 - The sentence “They slowly fade away”
market for electronic spying has developed. means:
Web sites describe a variety of products that can a) The places are sometimes harmful to people.
be used for tracking a spouse's e-mails and online b) The things gradually disappear.
chats, including a great variety of vehicle trackers c) Persons in love may die.
and computer spying programs. Founded by a d) Minds are mainly confused.
194
PROVAS DA AFA 1998 – 2010
joelhos, banhando os pés de sua ama, plantou no
18 - Complete the dialog with the appropriate meu coração um sentimento muito forte. Amo-a!
sequence of prepositions/adverbs: Tudo naquela ilha fatal se combinou para
Mike: “Have you ________ traveled abroad?” enfeitiçar-me, tudo!”
Joshua: “Unfortunately, ________. ________ (MACEDO, Joaquim Manoel de, A Moreninha )
I’ve started studying a foreign language I want this
kind of effective practice. I hope I won’t have to 20 - From the paragraph above we may
wait ________ a long time.” conclude that Augusto
a) ever / yet / Already / since a) from the moment the girl was bathing her
b) already / never / Ever / for mistress feet on, he couldn’t help loving her even
c) ever / never / Since / for after he had bored her.
d) already / yet / For / since b) loved watching the girl’s moistening her
mother’s feet that’s why he got so impressed with
Read the text below and answer question 19 her.
according to it. c) walked around the room trying to explain how
deep his feeling in relation to that girl was.
For seven frustrating years, Ayrton Senna’s dream d) fell in love with the girl because she was
of a victory at home, in the Brazilian Grand Prix, washing her baby-sitter’s feet and he had regretted
eluded him. He finally won the race in 1991, but to hurt her feelings.
only after a cliffhanger finish and typical heroics.
He was leading comfortably, with only a few 21 - “Não posso mais esconder meus
kilometers to go, when the gearbox of his car sentimentos, Leopoldo.” The Indirect Speech of
jammed.By the last lap only one gear was this sentence in English is
functioning, but Senna wrestled his McLaren- a) Leopoldo said to Augusto that he mustn’t hid
Honda to the checkered flag. So exhausted he his feelings.
could barely lift his arms, grimacing from the pain, b) Leopoldo asked Augusto if his feelings
he hoisted the trophy and the flag. shouldn’t be hidden by him.
(Adapted from Speak up) c) Augusto said that he wouldn’t hid his feelings
at all.
19 - In the sentence “So exhausted he could d) Augusto told Leopoldo that his feelings could
barely lift his arms, grimacing from the pain, he not be hidden anymore.
hoisted the trophy and the flag”, the underlined
word is 22 - All the sequences of nouns below are in the
a) a verb expressing that he could just put his plural form, except one. Mark it.
hands high to show happiness. a) sheep / series / women / loaves / fruit
b) an adverb and it means that Senna was able b) means / children / mice / lice / wolves
only to lift his arms grimacing from the pain. c) geese / thieves / oxen / buzzes / feet
c) an adjective which shows how painful his d) news / electronics / measles / nephew / widower
movement of grimacing his face was.
d) a pronoun used to explain how Senna presents Observe the cartoon below and answer questions
the flag in order to show his hoisted victory. 23 and 24 according to it

Read the text below and answer questions 20 and


21 according to it.

“Houve um momento de silêncio, em que Augusto


passeou pela sala inquieto. Finalmente, veio
sentar-se de novo junto de Leopoldo.
– É verdade – disse , não é a minha cabeça, a causa
está no coração. Não posso mais esconder meus
sentimentos. Leopoldo, aquela menina, que
aborreci no primeiro instante, quando a vi de
195
PROVAS DA AFA 1998 – 2010
26 - In the text above, the pronoun they in the
"I don't care what planet you're from. You can't 8th line refers to:
run around Earth stark naked!" a) H.G.Wells and Michael J. Fox
b) M.J.Fox’s parents in the film
23 - Which sentence has the same idea as the c) The science-fiction writers
modal can’t in the dialogue above? d) The films about science-fiction
a) You don’t have to run around Earth...
b) You mustn’t run around Earth... Read the dialogue and answer questions 27 and 28
c) You needn’t run around Earth...
d) You aren’t able to run around Earth... A: “Sue has been playing volleyball a lot
recently”!
24 - What’s the best definition for the sentence B: “That’s because she wants to be a professional.”
“I don’t care” in the context?
a) It’s a difficult situation if someone is naked 27 - Mark the option that best explains the use
in front of me. of the underlined verb tense.
b) I don’t have to take care of people who a) It expresses a habit or a regular past
walk naked. activity.
c) It really upsets me when someone comes b) It’s a situation that started and finished in
naked from another planet. the past.
d) It’s not important if someone comes from a c) It’s an activity that started in the past and
different planet. hasn’t finished yet.
d) The action is happening right now and will
25 - Mark the alternative that best completes the go on in a near future.
text below:
28 - Mark the correct question tag for the
“Would you like to travel back in time and see second sentence:
knights in armor fighting battles? Or go forward to a) Hasn’t that? c) Doesn’t she?
the 21st century and see the world of the future? b) Isn’t she? d) Doesn’t that?

Time travel ________ the dream of science-fiction 29 - Read the following article and answer
writers for many years. Perhaps the first story questions 29 and 30 according to it.
about time travel was The Time Machine by H.G.
Wells, but there has been many more stories, films
and TV series since then. In the movie BACK TO
THE FUTURE, Michael J. Fox plays a teenager
who accidentally travels back 30 years in time.
There he meets his parents, ________ are still
teenagers and ________ in love yet.
He has to help them get together, because if they
don’t get married, he _______ exist! When he gets
back to the present, he finds that he has improved
their lives – and his own. The Origin of the Names of the Months
You may think that time travel is just fantasy, but
some scientists believe it’s possible – at least January Janus, the Roman god of doors and
traveling into the future. gates
(adapted from the Internet) Frebruary Februa, the Roman period of
a) was / that / have fallen / wouldn’t purification
b) has been / who / haven’t fallen / won’t March Mars, Roman god of war
c) is / whom / have fallen / won’t April aprerire, the Latin word which means to
d) have been / in which / haven’t fallen / would open:
April was the month when spring began
May Maia, Roman goddess of growth
196
PROVAS DA AFA 1998 – 2010
June Juno, Roman goddess of marriage recent industrial disaster. Chances are that it was
July After Julius Caesar caused at least in part by sleep deprivation.
August After Augustus Caesar, first emperor of Being an insomniac myself, I asked Dr.
Rome Thorpe for some suggestions. He told me to stop
September septem, Latin for seven* drinking coffee. He said to have a warm glass of
October octo, Latin for eight* milk instead. “A lot of old-fashioned remedies
November novem, Latin for nine* work. Have a high- carbohydrate snack like a
December decem, Latin for ten* banana before you go to bed,” he said. But he
advises patients not to eat a heavy meal before
* September, October, November, and December turning in for the night. What about exercise?
were originally the seventh, eighth, ninth, and “Regular exercise helps, but don’t exercise too
tenth months of the year. close to bedtime”, he suggested. Finally, he told
me not to despair. “Don’t worry about not
From the article we can conclude that: sleeping. It’s the worst thing to do,” he said. I
I- one third of the names were created to don’t know. After thinking about those industrial
celebrate the gods or goddesses. accidents, I doubt I’ll be able to sleep at all!
II - astronomers wanted to praise only Caesar. (Taken from Grammar Express, Longman)
III - there was a concern about cardinal
numbers. 1) Classify the statements as correct or not,
IV - the names of the months have no relation to according to the text.
some ancient gods. I - Being an insomniac is a problem which affects
a) only one of the sentences is correct. great part of the population.
b) only two of them are incorrect. II - Lack of sleep is one of the causes of car
c) only three of them are correct. accidents.
d) all of them are correct. III - The interviewed doctor believes just in well-
known medication from drugstores.
30 - The preposition FOR referred to the IV - The more you worry about sleeping the worse.
months from September to December in the text is a) Only II and III are correct.
used to show b) Only I and II are correct.
a) a period of time. c) Only I, II and IV are correct.
b) how some words are derived from Latin. d) Only III and IV are correct.
c) a relation among all months and numbers.
d) the ways in which Latin and English were 2) The underlined sentence in the direct speech is
not linked. “The doctor told me: ...”
a) You should to think about what can happen if
people drive when they’re tired.
AFA 2005/2006 – Inglês b) Think about what can happen if people drive
when they’re tired.
c) You should think about what could happen if
Read the text and answer questions 1 to 4
people drove when they’re tired.
according to it.
d) Think about what could happen if people drive
when they’re tired.
Tossing and Turning
Can’t sleep? You’ re not alone. Millions of
3) Mark the opt ion that completes the sentence
people are up tossing and turning instead of getting
correctly.
their zzzz’s. Dr. Ray Thorpe, Director of the Sleep
The doctor advised the reporter__________ about
Disorders Clinic, says, “Don’t think that loss of
not sleeping.
sleep is just a minor inconvenience.” During an
a) don’t worry c) not worry
interview he told me tothink about what can
b) not to worry d) to not worry
happen if people drive when they’re tired. Every
year up to 200,000 car accidents are caused by
4) The title of the text means in its context that
drowsy drivers. Then he asked me to think about a

197
PROVAS DA AFA 1998 – 2010
a) many people are oft en trying desperately to
sleep.
b) the lack of sleep causes other health problems.
c) millions of people can’t sleep because of being
tossing and turning.
d) sleep disorders are defined by the medicine as
tossing and turning.
(Taken from Advanced Writing with English in
5) Joshua has some problems with his uniform Use, Oxford).
and he’s very worried about causing a bad
impression on his first day in the new Air Force 6) Mark the alternative with the correct modal to
base. He sent an e-mail to his mother asking for complete the gap in the text, considering what it
some advice. expresses in the context.
a) must (suggestion)
b) might (certainty)
c) may (possibility
d) had better (prediction)

7) Read the conditional sentences from I to IV and


check if they have a similar meaning to the
underlined ones in the text.
Now read his mother’s response. I - Unless everybody always told the truth, the
result would be disastrous.
II - Whether everybody lied all the time, modern
civilized society couldn’t exist.
III - The result would be disastrous if nobody told
lies.
IV - Modern civilized society could exist, unless
everybody didn’t lie all the time.
a) All the sentences are correct.
b) Only sentences II, III, IV are correct.
Complete the mother’s e-mail with the appropriate c) Only sentences I and II are incorrect.
passive causative sentences in the correct d) Only sentences I and IV are incorrect.
sequence. There are two extra ones.
a - it’ll be cleaned 8) In the author’s opinion, schools develop
b - have your uniform dry cleaned a) a useful ability in their students.
c - have it ironed b) a disastrous skill to the society.
d - have it dyed and also tightened c) the truth as a virtue to students keep during their
e - you’ll have a new uniform made lives.
f - had it done d) a skill children should forget. Read the dialog
g - it’ll get the stain removed between Charlie Brown and the tree and answer
question 9 according to it.
a) 1d / 2g / 3a / 4 c / 5 f
b) 1c / 2e / 3g / 4 f / 5 b
c) 1b / 2a / 3g / 4 d / 5 f
d) 1f / 2g/ 3e / 4 c / 5 b

Read the paragraph below and answer questions 6


to 8 about it.

198
PROVAS DA AFA 1998 – 2010
11) Which sentence(s) is (are) correct according to
the grammatical use of the relative pronouns and
modal verbs?
I - In China, even people who have common jobs
should learn English because of an important sport
event in few years.
II - Ordinary Chinese, which work in simple jobs,
ought to learn the English language in order to
become fluent until the Olympics games.
III - In India, the one whose English is good must
9) In the sentences "It's been a long winter, get a better job, as it is an important ability there.
______________?" and "You'd like to eat this kite, IV - Companie s in India offer a job to the one
______________?", which pair of question tags whom speaks English because they have to
complete the passage? consider it as an important skill.
a) isn't it / hadn't you a) Only I and III are correct.
b) doesn't it / wouldn't you b) Only II and IV are correct.
c) wasn't it / hadn't you c) Only I is correct.
d) hasn't it / wouldn't you d) Only II is correct.

Read the text below and answer questions 10 to 13 12) “Throughout organizations, more people need
according to it. more English.”
The number of English- speakers in Asia The boldfaced word can be replaced by
roughly equals total in America, Canada and ______________ in the context.
Britain. a) across c) whereas
To achieve fluency, non-native speakers are b) in every part of d) beyond
learning at
ever-younger age. 13) Mark the wrong question for the affirmative
Why such enthusiasm? In a word, jobs. A below.
generation ago, only elites like diplomats and “To achieve fluency, non-native speakers are
CFOs needed English for work. The ante and learning at everyounger
what’s needed is going up year by year. age.”
Throughout organizations, more people need more a) How old are people from other countries trying
English. In China The Beijing Organizing to speak English ?
Committee for the 2008 Olympics is publishing b) How do non -native speakers have looked for
English among staff, guides, taxi drivers and being fluent in English?
ordinary citizens. For lower- middle classes in c) In what age are mutual speakers achieving the
India, English can mean a ticket to a prized call- learning of a language?
center job. With call centers, no longer is speaking d) When do people from different countries learn
English one of the important skills to get a good English?
job.
(Adapted from Newsweek – March 27/2005) Read some fragments of an exclusive extract from
Jeffrey Achs’s new book, explaining how we can
10) What’s clear in the text? change the conditions that consign 1 billion people
a) Nowadays the learning of English is not only an to poverty and then answer questions 14 to 17
advantage of leading circles. according to each paragraph.
b) Enterprising people are learning English (Extracted and adapted from Time - March 14,
because of their jobs. 2005)
c) Undeveloped countries became English more
useful because of clerical workers. 14 - This paragraph shows us that
d) More and more non-natives are searching a
rowdy way to get employment opportunities.

199
PROVAS DA AFA 1998 – 2010
17) We finally deduce after reading the last extract
above that it
a) tries to make a suggestion to the readers.
b) emphasizes the failure of a country not keeping
its words.
a) Asia is less miserable in leadership than Africa. c) shows the solution to the evident problem.
b) The rate of poor Asian in numbers is not as d) insists on demonstrating optimism about the end
large as the rate of African. of poverty in some countries.
c) In a numerical calculation Asia has the control Read the paragraph below and answer questions 18
of poverty. On the other hand, Africa guides the and 19 according to it.
calamity.
d) In Asia, the quantity of poor people is huge and “During the Trojan War, the Greeks hid inside a
in Africa the proportion is larger. big wooden horse to trick their enemies into letting
______________ inside the city of Troy. The trick
15 - Considering the correct use of the words a few was successful and once the horse was inside the
and much, choose the item in which one of them is city gates, ______________ jumped out and
wrongly applied. caught the Trojans by surprise.”
(by Longman Dictionary of Culture)

18)According to the paragraph above, mark the


option which contains the appropriate words that
a) She's staying at the cottage for a few days. / complete the gaps and whom they are referring to
Much work remains to be done. respectively.
b) I met a few of the other employees at my a) them / they (the soldiers)
interview. / It would take much effort to tidy my b) they / themselves (the Greeks)
bedroom. c) they / they (the Wooden Horse)
c) A few people have had the same problem. / d) them / he (the Trojans)
There's much choice.
d) I only know a few about my grandparents. / He 19) The text says that
has much chance of winning. a) inside the city there was a horse which caught
the soldiers by surprise.
16 - One of the options is the correct explanation b) the Greeks hid themselves inside the horse in
for the use of the underlined verbal tenses. Mark it. order to catch their enemies by surprise.
c) the Trojans were caught by surprise because the
Wooden Horse was hidden inside the city.
d) surprisingly, the trick was successful because
the Trojans were caught during the war by the
hidden Wooden Horse.

a) An action which is completed before a time in 20) Taking the sentences:


the past. I- When the police arrived at the accident they had
b) An ongoing action which began in the past and already lain the injured man on the grass after
is still continuing. realizing he was passing away.
c) A single completed action in the past. II- At the moment the athletes were lying the
d) An action on progress at a point of time in the coach ordered them to concentrate and do their
past. best to be champions.
III- He was lying when he told us he hasn’t seen
the thief.
IV- Sarah laid a plan to get at the meeting earlier
so that she could finish her report first.
Which option bears the correct infinitive forms of
the underlined verbs ? Mark it.
200
PROVAS DA AFA 1998 – 2010
a) lay / lie / lie / lay c) lie / lie / lie / lay (Robert Frost)
b) lay / lie / lay / lie d) lie / lay / lie / lie
23) By observing the boldfaced statement the
Read the fragment of a song and answer question author of this poem means that he
21 according to it. a) holds those who want to set the world on fire.
b) would prefer ice because it satisfies him more.
“Não tenho nada com isso nem vem falar c) agrees with people who prefer fire.
eu não consigo entender sua lógica d) thinks that the destruction of the world by ice
minha palavra cantada pode espantar would be enough.
e a seus ouvidos parecer exótica”
(Caetano Veloso, Muito Romântico) 24) The underlined lines in the poem mean that the
author
21) Only one option, containing appropriate words, a) wishes the world ended once more to suffice
can translate into English the main idea of the both the author and others.
paragraph above. Mark it. b) preferred the world perished twice for those
a) The author says he’s nothing to do with that who favored ice.
because he can’t understand his logic and that his c) also favored fire instead of ice because it would
words can startle someone’s ears. be greater and suffice both.
b) He affirms not to have anything at all with it but d) supposes that perishing the world twice would
he also couldn’t get in mind how his singing words give him more than one way to understand it.
may be exotic.
c) The man doesn’t have nothing to say because 25) Mark the alternative that contains an
his words may be logical and frighten someone’s inappropriate use of pronouns.
ears. a) The doctor herself talked to the patient about
d) He has no way to make someone understand his that serious disease.
logical and singing words because they are exotic b) As nobody helped me, I prepared the food by
and could scare someone’s ears. myself.
c) We are annoyed with ourselves after such an
22) Mark the alternative that suits the paragraph embarrassing situation.
below. d) As he couldn’t think of any solution to the
Alice always _________ home from work at 6:30 problem, he himself committed suicide.
in the evening.
She _________ at home at 5 o’clock – she Read the comment of “The Art of War” and
_________ from work. answer questions 26 to 29 according to it.
a) gets / won’t be / won’t have arrived
b) will get / isn’t / will arrive The Art of War
c) get / hasn’t been / wouldn’t arrive The Art of War, from The Chinese
d) is going to get / will be / will be arrived philosopher warrior, Sun Tzu, was written
approximately in century III before Christ. It is one
After reading the poem, answer questions 23 and of the wisest military strategy agreements.
24 according to it. Its text is applied to the competition and to
the general conflict in all levels of life. Its aim is
Fire and Ice victory without combat.
Some say the world will end in fire Their strategic basis are: win without
Some say in ice fighting, avoid strength to attack weakness and
From what I’ve tasted of desire before winning the others, win oneself.
I hold with those who favor fire The Art of War has been used, as a
But if it had to perish twice metaphor, in the battles that has become the
I think I know enough of hate competitions among the companies.
To say that for destruction Sun Tzu’s book became a best seller and
Ice is also great nowadays, the greatest presidents of corporations,
And would suffice
201
PROVAS DA AFA 1998 – 2010
political leaders and successful businessmen are
using it as a marketing manual.
(Adapted from The Art of War – Sun Tzu)

26) Change the first paragraph into the Active


Voice.
a) About three centuries Before Christ, Sun Tzu, a
Chinese philosopher warrior wrote The Art of War.
b) Sun Tzu, who was a Chinese philosopher
(Adapted from Speak up – Set. 2003)
warrior, about three centuries BC, had written The
a - which
Art of War.
b - who
c) The book Art of War which has been written by
c - where
Sun Tzu, a Chinese philosopher warrior, has just
d - when
three centuries before Christ.
e - whose
d) Three centuries B.C, the Chinese Sun Tzu, who
f - if
had been a philosopher warrior wrote the Art of
g - whether
War.
h - unless
a) 1e / 2h / 3a / 4 a c) 1c / 2g / 3e / 4 a
27) According to the text above, The Art of War
b) 1a / 2f / 3e / 4 b d) 1d / 2f / 3a / 4 c
a) teaches strictly military strategies used in battles
by different kinds of leaders.
b) is compiled in a manual and can be used in
different situations of our lives. AFA 2006/2007 – Inglês
c) though it’s very old, it is still used as an aim and
all leaders make use of it to win some battles. Let's Not Spend The Night Together
d) shows technical competitions and special way s "He used to love her, but it's all over now. A
of dealing with conflicts and chaos. couple for more than two decades, Mick Jagger,
and Jerry Hall, said they would have their arriage
28) “Some of its strategic basis are: win without annulled. Hall filed for divorce after a Brazilian
fighting, avoid strength to attack weakness and model claimed Jagger had made her pregnant. But
before winning the others, win oneself.” Jagger challenged the action, saying his marriage
The underlined word is _____________ and refers to Hall in Bali was not legally valid. Undoubtedly
to the hangup was money. London papers claimed
______________. that the Rolling Stones' lead singer will pay $11
a) a possessive adjective / the content of the book. million to $15 million. That shouldn't be a
b) a possessive pronoun / the art of war. problem. The Stones, even after rolling for a
c) an objective pronoun / the competitions and third of a century, are still the world's top musical
general conflict in all levels of life. attraction, taking in nearly $88 million so far from
d) a subject pronoun / the military strategies. this year’s touring."
(Adapted from Time, July, 1999)
29) The third paragraph has a metaphorical
meaning and we can summarize it in few words. 1)The statement from the text, "But Jagger
Which option shows the right summary? challenged the action, saying his marriage to Hall
a) The knowledge of oneself. in Bali was not legally valid." means that Jagger.
b) The power of inner battle. a) expressed his disagreement about the authorities
c) The triumph of strength. rules, affirming his speech was true and there were
d) How to avoid the fear of living . some mistakes with the laws.
b) displayed how his determination in relation to
30) Read the paragraph below, based on a text his marriage in Bali was not valid and legal in
about obesity. Complete it with the appropriate court.
relative pronoun or conjunction, in the correct
sequence. There are four extra ones.
202
PROVAS DA AFA 1998 – 2010
c) wanted to challenge Hall by saying their 4)Another way to say the statement, “Streets,
marriage had happened as a result of her avenues and schools are named after astronauts”,
pregnancy. would be:
d) divorced his wife after confirming her a) Astronauts gave their names to streets, avenues
judgment, and schools.
accepting the consideration of a court of law she's b) The names of astronauts give them a place of
brought an action against him. heroes on streets, avenues and schools.
c) Russians honor their astronauts by giving their
2)The word hangup in the text means, names to streets, avenues and schools.
a) something you wait for a short time and solve d) Streets, avenues and schools are given to
tightly. astronauts by the meaning of their names.
b) a particular place where people get money after
requiring it in court. 5) Taking the sentence, “There are a lot of statues
c) a subject you have to solve. and museums all over the country. There's also a
d) an informal way to keep something updated date to celebrate them.”, the word in boldface
instead of asking for money judicially. refers to
a) the statues and museums in Russia.
b) the astronauts from Russia.
Where Astronauts Are Gods c) streets, avenues and schools in Russia.
"In a country which learned not to believe in God, d) scientists from Russia.
it
reigns the belief in science. Like saints, the June 12th, 1997. Diana, Princess of Wales,
cosmonauts(the way Russians call astronauts) addresses a seminar on landmines. Here are some
occupy a place of eminence in the pantheon of the excerpts of her speech.
national Russian heroes. They have multiple
talents, being the greatest one, the capacity of I -"I welcome this conference on landmines (...)
going to space. Streets, avenues and schools are because the world is too little aware of the waste of
named after astronauts. There are a lot of statues life, limb and land which anti-personnel landmines
and museums all over the country. There's also a are causing among some of the poorest people on
date to celebrate them, April,12. earth. Indeed, until my journey to ngola early this
Up to now, ninety nine Russians have year, (...) I was largely unaware of it too". (...)
already gone into space since 1961, when Yuri
Gagarin became the first man to go into orbit. Not 6) According to the excerpt above the underlined
by chance, he's adored among the conquerors of statement means that a) anti-personnel landmines
Cosmo. His premature death when he was 34 are causing awareness among not only the oorest
years old (seven years after his first and unique people but the landmines, too.
space flight in a tragic plane accident whose causes b) the poorest people on earth are wasting their
are still mysterious) contributed to create the lives in landmines that's why they waste their
myth." healthy, too.
(Adapted from O Globo, April, 02-2006.) c) limbs and lands that are among the poorest
population are not aware of the world.
3) According to the text, it's understood that d) many people not even know or care about limbs
a) Yuri Gagarin went into orbit and since then and lands that are being lost due to landmines
science is a great subject to be discussed. among the poorest people.
b) Russia believes that God may be more
important than its capacity to send cosmonauts to II. “The world, with its many other preoccupations,
space. remains largely unmoved by a death roll of some
c) nowadays astronauts occupy a high position in 800 people every month, many of them women
the news. and children. Those who not killed outright, and
d) Russians are people whose religion isn't so they number another 1,200 a month, suffer terrible
strong as it is in the rest of the world. injuries and are handicapped for life. I was in
Angola in January with the British Red Cross, a
203
PROVAS DA AFA 1998 – 2010
country where there are 15 million landmines in a b) plenty of. d) a small number of.
population, ladies and gentlemen, of 10 million.”
(Adapted from Sun Vol. 1, Ed. Moderna) 10) Mark the option that means "Heating bills can
be reduced when double glazing is installed."
a) Double glazing can reduce heating bills when it
7) According to Diana's speech we may observe is installed.
that her first concern is related to the a) ones who b) When double glazing is installed heating bills
are not killed outright. can reduce it.
b) little information the world has about the deaths c) Double glazing is installed to reduce heating
caused by landmines. bills.
c) terrible injuries people from Angola suffer. d) When you install double glazing you reduce
d) world that remains largely aware of the heating bills.
handicapped ones.
11) Mark the item that contains the right
Read the paragraph below to answer question 8. information about the text.
"Os brasileiros são os grandes beneficiados com a a) Houses become more secure if the windows are
autosuficiência na produção nacional de petróleo e glazed because of the condensation.
com os investimentos que estão sendo feitos pela b) A safe house is recognized when we can check
Petrobras." that it's glazed indeed and not so noisy.
(Revista PETROBRAS, Abril de 2006.) c) Double glazed houses are less noisy and prevent
us from the high temperature.
8) According to the text, d) When double glazed is installed in a house it
a) our fuel is being produced in our country and we gets warmer and with less noise.
need to import it.
b) Brazilians now have their own production of Read the article below carefully and answer
fuel that's why we may be proud of Petrobras. questions 12 to 14.
c) a lot of investment should be done in order to
put Brazil in a high position. Fireworks
d) Petrobras has been the first company to produce Fireworks constitute a variety of devices used for
fuel in great quantity. religious or entertainment purposes. They contain
incendiary compounds that produce sound, color,
The following extract is from a children's school smoke, or movement, or combination of these. The
Science book. Read it and answer questions 9 to ingredients consist of fuels, oxidizers, and
11. modifying agents. The fuels used are charcoal,
When houses are double glazed, only a relatively sulfur, antimony sulfide, and powdered metals.
small amount of heat is lost through the windows. Chlorates and nitrates usually provide oxygen for
Double glazed windows have two panes of glass the reaction, since fireworks do not use
and air is trapped between them, preventing atmospheric oxygen. Various colors are obtained
escaping of the heat. Double glazing also ensures from metals salts. Sodium produces yellow;
that condensation is reduced and noise is copper, blue green; calcium, red; strontium,
decreased. Heating bills can be reduced when scarlet; and barium, green. Picric acid or sulfur
double glazing is installed. People living near busy tends to intensify the colors, and ammonium salts
roads or airports also find that double glazing has increase the shades obtainable. Addition of iron
to be fitted. and aluminum powder provides sparks and fiery
(Taken from Exploring Grammar in Context - displays. The craft of blending mixtures packaging
Cambridge them is an ancient one in China, where fireworks
University Press) are used for religious purposes. In Europe
fireworks for entertainment have been
9) “only a relatively small amount of heat is lost manufactured since the 13th century. In many
through the windows." The boldfaced expression places throughout the United States and Canada
in the sentence canonly be substituted for the sale of fireworks is restricted by law
a) a gretat deal of. c) a little. because of the danger of injury.
204
PROVAS DA AFA 1998 – 2010
(The International Encyclopedia)
15) It's clear that "Who moved my cheese?" is
12) Que altemativa está correta em relação ao a) a self-confidence book that tries to make us
texto? smarter to modify plentiful situations in our lives.
a) Fogos de artifício vêm sendo produzidos na b) a self-improvement reading that shows how we
Europa há oito séculos. should deal with ourselves and make life better.
b) O oxigênio atmosférico, na verdade, é um dos c) a piece of advice for the ones who can't solve
ingredientes dos fogos de artifício. their problems at all, and still insist on being stuck.
c) Religiosos usavam fogos de artifício somente d) a special speech for the ones that are involved
para diversões. with the great changes that occur when they get
d) Nos Estados Unidos e Canadá as pessoas não amazed.
podem comprar fogos de artifício.
16) "Two little mice and two little men live in a
13) "The craft of blending mixtures packaging labyrinth searching for some cheese."
them is an ancient one in China, where fireworks The word little can only be substituted for
are used for religious purposes." The word one in ____________and refers to the ____________.
this sentence. a) few / size of the men and mice
a) denotes the quantity of fireworks used in China. b) brief / height of them
b) is used to refer to a particular thing within a c) short / the lack of importance showed by the
group. author
c) is used to emphasize a situation. d) small / emphatic opinion given by the author to
d) refers to the word craft. the characters.

14) Change the sentence extracted from the text 17) “Who moved my cheese?”
into passive voice. What's expected from the one who reads it?
"Chlorates and nitrates usually provide oxygen for a) Knowing life and the labyrinths to succeed more
the reaction." than usual.
a) Oxygen is provided for the reaction usually by b) Searching spiritual peace and living as the
chlorates and nitrates. characters, always complaining about the career
b) Oxygen for the reaction is usually provided by and life in general.
chlorates and nitrates. c) Dealing with changes, living better and
c) The ones that usually provide oxygen for the achieving success.
reaction are chlorates and nitrates. d) Scribbling new experiences and admitting the
d) Chlorates and nitrates are usually provided by ones success to use them deeply.
oxygen for the reaction.
Read a letter and answer questions 18 to 20.
Read the passage below and answer questions 15
to 17. "As a survivor of the Holocaust, I lost the life I led
more than 60 years, when the world didn't give a
Who Moved My Cheese? damn or, like today, acknowledge that a storm of
Who Moved My Cheese? It's a parable that reveals destruction was imminent (‘The Lortable, dry
deep truths about changes. Two little mice and two home
little men live in a labyrinth searching for some watching the horror of New Orleans made me cry
cheese - a metaphor used to express what we wish the tears I didn't have when I was a child losing
to have in life, from a good job to spiritual peace. everything, fleeing with just the clothing I wore
One of them is successful and writes what that day. More than 60 years ago, I was on a
he learnt from his experiences on the walls of the different continent. The disaster that unfolded in
labyrinths. The scribbled words on the walls teach front of my eyes today took place in my adopted
us how to deal with country, among my adopted people. "We didn't
the changes to live with less stress and achieve know" is an
much more success at work and personal life. unpardonable excuse. "We didn't care" is more like
(Spencer Johnson, M.D) it. Without hesitation, America spends billions and
205
PROVAS DA AFA 1998 – 2010
wastes human life in a country that is not Sometimes consumers prefer to buy an illegal copy
interested in democracy. Yet we quibble about the of a video, CD-ROM, cassette or software package
cost of Katrina, a cost that will affect everyone in because it costs less. In Russia, for example,
our own backyard for years to come." copies of Microsoft's Office 2000 program sell for
(Lucie L. Liebman - Staten Island, New York) just 1% of the list price.
Frequently, however, buyers don't realize they are
18) The letter was written by someone who buying a fake instead of the genuine article.
a) subsisted a terrible hurricane even in her own (Taken from Skyline 5 - Macmillan)
country.
b) resisted the Holocaust though she had been in 21) The sentence “... the criminals’ gains are other
an unfamiliar country. people’s losses" means
c) didn't die despite the tragedy. a) criminals gain more than everyone.
d) continued living in New Orleans otherwise she b) criminals gain, whereas others lose.
had passed away. c) people lose more than expected by the
criminals.
19) The Present Tense of the underlined verb in d) people expect to lose less than the criminals
the first line is gain.
a) lead. c) lid. 22) Read the statements below and classify them
b) led. d) leaden. as (T) true or (F) false, respectively.
( ) More than ninety percent of the worldwide
20) We realize that Lucie feels extremely sorry trade is made of fake products.
about _______because ________. ( ) Two famous brands in China are responsible for
a) the whirlpool / it destroyed part of her life a loss of more than a hundred million dollars per
b) America / it isn't democratic year.
c) the hurricane / she lost almost everything ( ) More than two thirds of Yamaha's bikes sold
d) katrina / it destroyed her country across the globe are fake products.
( ) The brand Nike has a great profit caused by
counterfeit goods.
Read the text and answer questions 21 to 23 Mark the correct alternative.
according to it. a) F / F / T / F c) F / T / T / F
b) T / F / T / T d) T / T / F / F
Fakes – a world of copycats
Every year, criminals make millions of dollars 23) According to the last paragraph, consumers
selling fake perfumes, clothes, medicines and I- have always intended to buy counterfeit goods.
computer software. Counterfeit goods account for II- are sometimes deceived by the fakes.
about 7% of total trade across the globe. And the III- occasionally prefer pirate products because
criminals' gains are other people's losses. they are cheaper.
Take, for example, governments which are unable IV- see the illegal copies as a solution to their
to collect revenue from indirect taxes and customs growing economical problems.
duty on legitimate sales. The only correct sentences are
Over 30% of sales in mainland China are a) I and III. c) III and IV.
estimated to be counterfeit. In India, fake products b) II and III. d) II and IV.
account for 10% of the revenue for the entire
health sector. Five out of six Yamaha bikes sold Read the fragment and answer questions 24 and
worldwide are not the real thing. Nike, the brand 25.
which tells you to "just do it", loses $70 million Image, image, image...
annually to the menace of fake brands and pirate Here's the coolest ... wildest ... hippest ...
products. Identical fakes cost Gillette $20 million a funkiest ...object! The image-conscious person
year, and Proctor and Gamble loses a staggering cannot do without it!
$150 million on a twelve-month average in China This style is ideal for fashion and consumer
alone. objects (cell phones, backpacks, watches, and so
on). It's the most
206
PROVAS DA AFA 1998 – 2010
common form of advertising for many products. 28) The text affirms that "the generation gap is
But don't forget all those other types of message. lessening". It means it has
Be smart: don't let the advertisers fool you! a) increased. c) stabilized.
(Taken from Consumers Portfolio - Macmillan) b) decreased. d) not changed.

24) There's one option which DOESN'T match the Read the paragraph of an article from Newsweek,
context, mark it. Nov/14, 2005, whose title is "Rethinking Arafat"
a) You may be persuaded by the advertisers. and answer question 29 according to it.
b) You need to be perceptive to avoid being
influenced by the ads. "It might also take years for Palestinians to assess
c) You should pay attention to the message behind Arafat's legacy. Given the lingering suspicion that
the advertisements. he was poisoned (the cause of death is listed as
d) According to ads, the image-conscious person undetermined), many Palestinians aren't in the
doesn't have to live without their products. mood to start sorting through his record. 'As long
as this matter is not resolved, you won't hear
25) According to "Image, image, image ...", to sell people questioning his leadership', says Hafes
fashion and consumer objects people announce Barghouti, editor of the Palestinian daily AI-
them as the most... Jadidah, just bemoaning the consequences."
Mark the option that DOESN'T suit the text.
a) stylish c) different 29) According to the writer,
b) natural d) modern a) Arafat's legacy will certainly take time to be
assessed.
Read the paragraph and answer questions 26 to 28 b) Arafat must have been poisoned.
according to it. c) Arafat's leadership won't be questioned, unless
the matter of his death is resolved.
d) many Palestinians want to clarify the suspicion
concerning Arafat's death as soon as possible.
Read the headline of the article referred above and
answer question 30.
"If the Palestinian leader was the real problem,
why haven't things improved in the year since his
death?"

30) The author


a) wonders if Arafat was really a problem to his
26) The sentence in italics, "when a common country.
...thirty" has the function of b) assures things will be better after the leader's
a) establishing contrast. death.
b) emphasizing a concept. c) thinks improvements need time.
c) explaining something. d) says things have changed after the leader died.
d) showing a condition.

27) What's the main idea of the text?


a) Never believe older people.
AFA 2007/2008 – Inglês
b) The excessive amount of births after war caused
Read the text below and answer questions 01 to 04.
the generation gap.
c) What is known as generation gap was spread by
The Moon
the young people in the sixties and seventies.
d) The United States and Canada have a lot of
The Moon is the Earth’s only natural satellite. It is
problems concerning people coming from different
relatively large for a moon, with a diameter of
generations.
about 5,470 kilometers just over a quarter of the
Earth. The Moon takes the same time to rotate on
207
PROVAS DA AFA 1998 – 2010
its axis as it takes to orbit the Earth (27.3), and so a) The Moon, which is the Earth’s only natural
the same side (the near side) always faces us. satellite, is relatively large for a moon. (lines 1, 2)
However, the amount of the surface we can see – b) A diameter of about 5,470 kilometers, that is,
the phase of the Moon – depends on how much of just over a quarter of the Earth. (lines 2, 3)
the near side is in sunlight. The Moon is dry and c) The amount of the surface we can see whose
barren, with no atmosphere or water. It consists phase of the Moon depends on how much of the
mainly of solid rock, although its core may contain near side is sunlight. (lines 6 – 8)
molten rock or iron. The surface is dusty, with d) Maria occur mainly on the near side who has a
highlands covered in craters caused by meteorite thinner crust than the far side. (lines 14 – 16)
impacts, and lowlands in which large craters have
been filled by solidified lava to form dark areas
called maria or “seas”. Maria occur mainly on the Lovers' Moon
near side, which has a thinner crust than the far The fabled Lovers' Moon
side. Many of the craters are rimmed by mountain illuminates the night.
ranges that form the crater walls and can be Shining upon a couple with
thousands of meters high. its magic light.
Visual Encyclopedia They treasure just one thought
two hearts so crystal clear.
1) Mark the option that is contextually and To hold in their arms one that is so dear.
grammatically correct according to the use of One that makes life worth living
adverbs. just by ________ near.
a) Even being a satellite, the Moon is natural and
considerably wide. They promise that forever
b) Considering the size of the of the Moon, together they ________ as they bathe
otherwise it is only 1/4 of the Earth. in the magic that others do not see.
c) It takes almost the same time to the Moon So if you feel that you ________ lonely
rotates its own axis and the orbit of the Earth. and hope to find love soon.
d) As the near side ever faces us we can totally Look toward the heavens and make
watch the face of the Moon. a wish on the Lovers' Moon.
From Internet, Quacmoto 1/4/00
2) The pronoun it in line 9 refers to the
a) atmosphere or water.
b) solid rock. 5) Complete the brackets with the right tense of the
c) sunlight. verb to be and mark the correct alternative.
d) moon. a) being / will be / are
b) to be / to be / ‘re going to be
3) Choose the alternative in which the sentence is c) been / are / will be
an example of If Clause according to the context of d) been / are going to be / are
The Moon.
a) The dusty surface with highlands would be 6) In the 1st verse of the poem the author
covered in craters if there were no meteorite a) imagines the Moon as a shelter to the ones who
impacts. are in love.
b) Unless lowlands with large craters weren’t filled b) insists on the idea that the Moon belongs to
by solidified lava they were dark areas. those ones who believe in love.
c) We can only see the amount of surface if part of c) considers the brightness of Lover’s Moon is
the near side is in sunlight. different from the real one.
d) If many of the craters haven’t been rimmed by d) sees a celestial light over all couples.
mountain ranges they would form the craters walls.
7) The author also
4) Mark the statement from the text which is a) promises to solve the lovers’ problem of people
rewritten as a correct example of Relative or Non- who are alone.
Relative Clause.
208
PROVAS DA AFA 1998 – 2010
b) suggests the lonely ones to look at the sky if independente do que diz o nosso coração.
they want to find out a lover. IV - nossos segredos estão todos escondidos
c) tries to help lonely people to split up with somente nos sonhos.
someone they wish to. São corretas somente as assertivas
d) wants to teach people how to get involved with a) I, II e III. b) II e IV.
someone special. c) I e II. d) III e IV.

8) Mark the right option considering the two texts 10) As palavras referentes a unidades de medidas
about Moon. usadas no texto são
a) Both texts explain the influence of Moon in our a) altura e profundidade.
lives. b) distância e altura.
b) They try to display the importance of this c) peso e profundidade.
symbol. d) distância e peso.
c) They sum each other the meaning of this
heavenly body veraciously. 11) Mark the option which contains an indirect
d) One shows the moon in a celestial way and the form to complete the
other in a figurative one. prophet’s idea in the following gap.
The Prophet __________ in silence the secrets of
Read the following text to answer questions 09 to the days and the nights.
12. a) said to the man that his heart has known
On Self-Knowledge b) said to people’s hearts know
And a man said, “Speak to us of Self-Knowledge.” c) told him: your heart knows
And the Prophet answered, saying: “Your hearts d) told them that their hearts knew
know in silence the secrets of the days and the
nights. But your ears thirst for the sound of your 12) Considering only the boldfaced sentences
heart’s knowledge. You would know in words explain the meaning of the articles in the text,
that which you have always known in thought. respectively. The author
You would touch with your fingers the naked body a) persuades us to understand that there is much
of your dreams. And it is well you should. The more than just one truth / thinks there’s just one
hidden well-spring of your soul must need rise and truth.
run murmuring to the sea; and the treasure of your b) guesses truths can’t be different / is right about
infinite depths would be revealed to your eyes. But the truth he knows.
let, there be no scales to weigh your unknown c) knows partially the truth / is in doubt about it.
treasure; and seek not the depths of your d) believes in that truth / is right about just one
knowledge with staff or sounding line. For self is truth.
a sea boundless and measureless. Say not, I have
found the truth, but rather, I have found a truth. A throw-away world
Say not, I have found the path of the soul. Say
rather, I have met the soul walking upon my path. A lot of people feel that pollution has
For the soul walks upon all paths. The soul walks become one of the biggest problems in the world
not upon a line, neither does it grow like a reed. today. But when we talk about pollution, what do
The soul unfolds itself, like a lotus of countless we really mean? Well, when you pollute
petals.” something, you make it dirty or dangerous for
Adapted from The PROPHET, by Kahlil Gibran other people or animals. If you put engine oil in
water, you will pollute it; nobody will be able to
9) O Profeta tenta explicar a pergunta ao homem drink it or wash in it. All over the world, there are
dizendo que people polluting the land, the sea and the air.
I - é desejo das pessoas compreender o que o The main reason for pollution is waste –
íntimo já conhece. something which is no longer needed. Waste can
II - na verdade já possuímos o conhecimento de be many things. It can be yesterday’s newspaper,
nós mesmos. an old car, your dirty bath water, or smoke from a
III - as nossas verdades inexistem dentro de nós factory chimney. Some waste is dangerous
209
PROVAS DA AFA 1998 – 2010
because it contains poisons. This kind of waste is d) took advantage of a chaotic situation.
called toxic waste, and it is the problem of toxic
waste which is worrying many people – and Read the text and answer the following questions
governments – today. 17 to 19 according to it.
All living things, especially people, make Get Ready
waste. There are more than six billion people in the
world. They all need to eat, dress and travel about. You will go directly to the labor market
Most of them need to heat their homes after high school, or you will prepare yourself to
as well. They buy things, they use them and they the university. No matter what you do, there is one
throw their old things away. Nowadays, we live in very important thing to remember: you have to
a ‘throw-away’ world. get ready for what is coming.
(Adapted from Recycling – Factfiles, Oxford.) And you have to do it right, otherwise
people will not hire you. They will only pay you a
13) Mark the option that justifies the tittle of the salary when they recognize that your work has
text above. value. If you want a higher salary, you have to
a) We make much waste from different kinds. provide more value.
b) People must pollute the land, the sea and the air. Consider your experience and your
c) Actually, toxic waste worries the governments. knowledge today. What can you do today at work?
d) Consumers buy more than necessary. What are your skills? What are the things that you
can do that meet the needs of other people?
14) Choose the alternative which DOESN’T Taken from Challenge, Richmond
express an example of waste.
a) Useless objects. b) Tainted water. 17) After graduating from high school it is highly
c) Ashen smoke. d) Glassy sea. important to
a) be prepared to deal with the labor market or to
15) Mark the alternative that represents the same go to the university.
idea as the first sentence of the text. b) remember in detail what you have just learnt.
a) The majority of people believe pollution won’t c) achieve what you want quickly.
be a problem to worry about. d) make yourself essential questions to get a raise.
b) Many persons are concerned about pollution.
c) Everybody thinks pollution is becoming a 18) Considering the ideas expressed by the text,
serious problem. mark the INCORRECT statement.
d) The great deal of pollution became a world a) People need to deserve their salary.
problem for anybody. b) Workers cannot wait for a better salary if they
don’t work in a satisfactory way.
c) We don’t have to work more than necessary if
the salary is good.
d) Employees must have a higher salary if they
improve the quality of work they offer.

19) The fragment “meet the needs of other people”


taken from the text means
a) to face the chief and co-workers.
b) the skills a worker has.
c) to correspond to people’s expectations.
d) the necessity we have to be part of a group.
16) According to the cartoon the businessman
a) took a decision to benefit the company.
b) created a strategy to achieve a goal shared by
everyone.
c) decided to change the situation of sales.

210
PROVAS DA AFA 1998 – 2010
a) Taking simple actions we may cause wider
impacts than we imagine. (headline 1)
b) An easy beginning can be to reduce as much as
possible paper waste. (headline 2)
c) Use electrical equipment that needs little energy
as well as only harsh chemicals. (headline 3)
d) Do not pollute water sources; use local farms
Lucy Van Pelt works hard at being bossy, crabby properly. (headline 4)
and selfish. She is loud and yells a lot. Her smiles
and motives are rarely pure. She's a know-it-all 23) Mark the alternative that has the best title to
who dispenses advice whether you want it or not – the group of sentences above.
and for Charlie Brown, there's a charge. (...) a) What to do in case of necessary loss.
The absence of logic in her arguments holds a kind b) Some ways of saving energy.
of shining lunacy. When it comes to compliments, c) How to use the resources in a satisfactory way.
Lucy only likes receiving them. If she's paying one d) Using the garbage smartly.
– or even smiling – she's probably up to something
devious. Read the text bellow and answer questions 24 and
25 according to it.
20) According to the paragraph above, mark the Child Labor
correct option about Lucy Van Pelt. “An estimated 246 million children are engaged
a) If you see her smiling it means she must be in child labor worldwide. Of those, approximately
thinking of something pure. three-quarters (171 million) work in hazardous
b) Lucy might be the right person if you want to situations or conditions, such as working with
give some advice. pesticides in agriculture or working with
c) There will be some logic in her arguments, dangerous machinery.
unless she is bossy. They are everywhere but they are invisible,
d) Whether you want to express your admiration working in homes, shops and plantations.
for her, she will like it. Millions of girls who work as unpaid domestic
servants are vulnerable to exploitation and abuse.
21)Choose the alternative that has the best Millions of others work under horrific
sequence of antonyms to the boldfaced words from circumstances: they are trafficked (12 million),
the text respectively. forced into several forms of slavery (57 million),
a) servile / even-tempered / altruistic into prostitution and pornography(18 million), into
b) obedient / moody / generous participating in armed conflict 0,3 million) or other
c) submissive / unstable / unfriendly illicit activities (0,6 million). However, the vast
d) despotic / bad-tempered/ confident majority of child laborers – 70 per cent or more –
work in agriculture.
Read the following headlines and answer questions The Brazilian Household Survey ( Pesquisa
22 and 23. Nacional por Amostra de Domicílios – PNAD)
1. Many simple actions have wider impacts than which was published in October 2002 confirmed
we realize. that 2.9 million children between the ages of 5 and
2. Minimizing paper waste is an easy way to start. 15 still worked in Brazil. In 1992 there were about
3. Use energy-efficient appliances, fewer harsh 8 million laboring children and in 1995 that
chemicals. number dropped to about 4.5 million.
4. Keep water sources clean; make good use of Challenge, Richmond Publishing
local farms. 24) Child labor in agriculture is considered
www.nationalgeographic.com hazardous because (of)
a) children receive little money.
22) Mark the alternative which has a sentence with b) pesticides and dangerous machinery.
a DIFFERENT meaning from the headlines 1 to 4, c) the vast majority of child laborers work in it and
respectively. forms slaves.

211
PROVAS DA AFA 1998 – 2010
d) most dangerous machinery and horrific d) ajudar todos do grupo a se conhecerem e se
circumstances exploit and abuse them. ajustarem em pouco tempo é a melhor tática.

25) The sentence, “They are everywhere but are 28 ) The fragment from the text, “As you adjust to
invisible”, means that these children in fact the routine of daily life, that initial sense of
a) didn’t exist but nowadays their number is huge. adventure will wear off.” means that
b) receive no payment and are forced into several a) people’s attitude get weakened ending the
forms of slavery. feeling of adventure.
c) are hidden by people all over the world but b) all the initial sense of adventure makes people
protected by PNAD. feel they can be stronger.
d) are seldom seen by us. c) maintaining adventure sense to their routine is
the best way they wear off or not excitement and
Culture Shock happiness.
“Adjusting to a new culture and way of life d) at this point to help or not people to wear off is
is both exciting and challenging – you will the same to withdraw from them.
experience some highs and lows during your time
abroad. During these times of difficulty, just Read the text and answer questions 29 to 30.
remind yourself that these cultural differences are “Before I came to America I had dreams of life
the reason we all love to travel – to experience the here. I thought about tall Anglos, big buildings,
unknown! Remember it’s just a phase! and houses with lawns. I was surprised when I
As you adjust to the routine of daily life, arrived to see so many kinds of people –Black
that initial sense of adventure will wear off. During people, Asians. I found people from Korea and
this phase you may start to miss your friends and Cambodia and Mexico. In California I found not
families. During this time, you may find it hard just America, I found the world.”
to keep a positive attitude. You may be tempted to Olsen – Voices from the Language Classroom by
withdraw from people around you or even become M. Bailey and Nunan, 1996.
hostile. At this point it is helpful to know that for
most people this phase doesn’t last too long.” 29) The author affirms in the text that he
Challenge, Richmond Publishing a) didn’t guess he could meet several cultures
there.
26) One of the statements from the text shows b) thought he would surely find there not only the
different moments lived by people who are abroad. same way of culture but also many kinds of
Mark it, according to the text. people.
a) You may be tempted to withdraw from people c) only there he could be in contact with a variety
around you... of cultures and thoughts.
b) During this time you may start to miss your d) found in America the right way to make several
friends and families. and
c) You will experience highs and lows during your different friends.
time abroad.
d) At this point it is helpful to know that for most 30) The boldfaced words in the text mean that it is
people this phase doesn’t last too long. a(an)
a) information from some peoples and their
27) De acordo com o texto, é correto afirmar que cultures.
a) durante a fase de ajuste às diferentes rotinas b) reported one and that’s why all the verbs are in
todos podem se tornar hostis. the past.
b) se as pessoas não souberem se ajustar às c) tourist presentation from different places with
diferenças, tal the meaning of inviting people to visit them.
atitude pode resultar num afastamento dos demais d) attractive way to improve such relevant and
membros do grupo comprometendo seu beautiful countries.
relacionamento com o mesmo.
c) o período de ajuste é longo, logo qualquer
atitude mais estressante deve ser evitada. AFA 2008/2009 – Inglês
212
PROVAS DA AFA 1998 – 2010
c) shall develop the capacity of producing
Biotechnology microorganisms.
d) is able to diagnose, predict and sabotage
Biotechnology is one of the new diseases.
professional courses of studies offered by
universities. Biotechnology uses chemical and 3) The only one matter that is NOT mentioned in
biological knowledge and the knowledge of new the text is
technologies in the areas of healthcare, food, a) drinks. c) different professions.
chemistry, and the environment. Biotechnology b) some subjects. d) illnesses.
graduates are multidisciplinary professionals. They
study biology, chemistry, physics, statistics, and Upside Down
information technology.
In the area of microbiology, these Who’s to say
professionals study fungi, What’s impossible
bacteria, viruses, and protozoa and the diseases Well they forgot
that they cause in plants, animals, and human This world keeps spinning
beings. They research the methods to use such And with each new day
microorganisms in the production of foods and I can feel a change in everything
beverages, such as dairy products, beer, and wine. And as the surface breaks reflections fade
The biotechnologist specializing in But in some ways they remain the same
immunology uses the microorganisms in the And as my mind begins to spread its wings
production of vaccines and kits for diagnosis. In There’s no stopping curiosity
the food and pharmaceutical industries, they (Jack Johnson)
control microbial growth, safety, and hygiene at
the workplace. They work in research for the 4) Mark the right definition for the title of the
development of new pharmaceutical drugs. lyrics above.
They also work in the environment area, to a) To cause something to change completely and in
evaluate and prevent water and soil contamination. a bad way.
(Challenge – Richmond) b) To be friendly with someone, especially because
they can help you.
1) The text shows that c) When you cause something to move in a circle
I. the one who studies Biotechnology is able to round a fixed point.
recognize different studies of health, cooking and d) Having the part which is usually at the top
chemistry among others. turned to be at the bottom.
II. the study of this subject is a new area the
universities are offering. 5) In the lyrics the author affirms that
III. technology involves multiple studies in a) people have forgotten their past.
biotechnology area. b) although this world spins things do not change
IV. the environment is actually a new version of at all.
the biotechnology study. c) the surface of the earth reflects some lofty
ideals.
The correct options considering the statements are d) everything is moving and he’s not interested in
only knowing it.
a) I and II. b) III and IV.
c) I, II and III. d) I, II and IV. Read the text below to answer questions 06 to 08.

2) The one who graduates in biotechnology The Car Washer Who Became An Executive
a) can acquire the knowledge to several
professions and also carry out researches. Robert L. Johnson is the CEO (Chief
b) will be able to prescribe and it’s quite likely that Executive Officer) of BET. Black Entertainment
he/she can produce forms of organic stuff. Television, a cable TV channel in the United
States, BET specializes in producing programs for
213
PROVAS DA AFA 1998 – 2010
the African-American community.
Read what this successful executive says about his Read the cartoon and answer questions 09 and 10
first job. according to it.
“I ________ in Freeport, Illinois. My first
job was at the local carwash. I ________ sixteen
years old. I ________ every day, all summer, for a
dollar an hour. I worked with ten other.
guys. All of us ________from different racial,
religious and economic backgrounds. We had to
clean cars in teams, and we quickly learned to
work together.
I learned the better way to become
indispensable: you know how to do all aspects of
your job. At the carwash, ‘all aspects’ included
vacuuming the interior, scrubbing whitewalls
and polishing chrome until it shined. Sixteen years
later, when I started my own business, I again had
to know how to do every job in the company –
advertising, marketing, producing and negotiation
contracts. 9) When Charlie Brown was asked by Lucy he
Working at the carwash taught me that a) mocked himself.
there is a direct connection between work and a b) clamoured for Lucy.
feeling of self-esteem. Young people who refuse c) threated the idea.
jobs that they consider inferior or low-paying d) repelled her speech.
are only hurting themselves. As long as you do
your best, every job is a learning experience and a 10) The cartoons also show that Charlie Brown
step to a better job.” a) didn’t hear her advice.
(Adapted from Reader’s Digest, January, 1999) b) was speaking in a hypothetical way of having
the very same life.
6) Mark the option that completes the gaps c) argued with Lucy and went away.
respectively. d) not even replied.
a) woke up / were / came back / came
b) was born / had / traveled / got Questions 11 to 14 are related to the following
c) grew up / was / worked / were text.
d) lived / got / tried / lived Dealing with Sensitive Materials on the Internet
With the emergency of user-friendly online
7) Mark the option which ISN’T applied correctly systems, the World Wide Web and its introduction
in the text. into the classroom, more and more children are
a) the better (line 13) c) themselves (line 23) taking advantage of the power of the internet.
b) of your (line 14) d) a better (line 24) However, it remains largely an adult forum, and so
it carries with it adult subject matter.
8) Another way to write the sentence “BET Does it raise the question of what
specializes in producing programs for the African- __________ when adult topics and a child’s naive
American community” (lines 3 and 4), is a) the explorations meet? The debate has raised not only
African-American community is specialized in questions of obscenity, harassment, free speech,
producing programs by BET. and censorship, but also of government control of
b) programs are specialized for the African- the Internet, and its very nature as a
American community by BET. communications resource.
c) producing programs is a specialization for the Whatever the outcome of this war is, it will
African-American community in BET. set a precedent for how society and government
d) BET is specialized in producing The African- deal with the exchange of information in the
American community.
214
PROVAS DA AFA 1998 – 2010
future. Is the internet a free forum for discussion or 14) Comparing the pieces of content the internet
is it a broadcasting service and therefore subject provides, it may be said that
to the same restrictions as television, print, or a) there are more childish articles than grown up
radio? ones.
Are to internet communications on the right b) there isn’t any solution to control our kids in the
privacy internet without limiting ourselves.
covered by the, or can e-mail messages be legally c) children have to surf the internet less than adult.
observed…? Are web pages free speech or are d) the adult’s content provides much more useful
some subjects taboo on the internet because a child information.
may stumble upon
them? And who is responsible for internet content Warning: Flying is bad for your health
communications medium where traditional Flying is the safest way to travel…or is it?
publishing scenarios no longer apply and content Some doctors think the airplane is a dangerous
can be posted anonymously? place, especially for the old or the unhealthy.
For people who wish to control or limit the Although the airplane is pressurized, there
use of internet, the issue of children and is less oxygen than on the ground. So anyone who
pornography has been valuable tool for gaining has had a heart attack should not try for at least
public support. The key is to find solution to two weeks after the attack. After an operation, you
protect our children while at the same time, avoid should stay on the ground for at least ten days.
setting up a climate of control that will limit our Sitting on a plane for many hours –
rights as adults. especially in economy class where there isn’t very
(Adapted from Framework Level 3 – Richmond) much leg room – gives everyone aches and pain,
so you should get some exercise, especially
11) Complete the gap (line 6) with the right verbal on long flights.
tense. Flying also causes dehydration. If you
a) does happen c) do happen drink or eat too much, you’ll wake up feeling sick.
b) happens d) happen Everyone needs to drink more in the air, but you
shouldn’t drink alcohol because it makes you even
12) The underlined question (lines 16 and 17) has more thirsty.
been mixed up. Put the words in the right order The most common problem is jet lag. You
according to the context. should change to your new time zone as soon as
a) Are the internet to communications covered on possible, and you shouldn’t sleep if it’s still
by the right privacy? daylight.
b) Are the right by privacy to communications on Crowded airports, long lines, and delays
the internet covered? cause stress and high blood pressure. So, be
c) Are the internet covered on privacy by the right careful! Flying is the safest way to travel, but is it
to communications? the healthiest?
d) Are communications on the internet covered by (Move Up, Heinemann )
the right to privacy?
15) According to the text it’s correct to affirm that
13) Mark the correct question to the answer below a) unhealthy people shouldn’t travel by planes.
extracted from the text. b) even being safe flying might bring some
“The debate” (line 7) disorders on one’s body.
a)Which subject has explored children? c) people who prefer planes mustn’t have health
b)Which topic protected children from the problems.
argument of the text? d) flying can cause heart attacks.
c)What matter is trying to control the use of
internet by children? 16) Aches and pain (line 10)
d)What has raised questions of obscenity, a) are caused due to economy classes.
harassment, etc? b) should change your health.
c) might be results of long hours without stretching
the legs.
215
PROVAS DA AFA 1998 – 2010
d) show passengers they have to exercise before Eryn: You also get bad pollution from burning
sitting on a plane. down the rainforest. We
should give money to poor people in Africa and
17) From the text it’s also possible to get places.
a) pieces of advice before taking a flight.
b) rules of flying a safe plane. Chelsea: We should spread out the people evenly.
c) zones of dangerous in a route. We could say,
d) healthier ways of flying. “Put your hands up all those who want to live in
Africa”. And then we could spread out the food.
18) In the statement, There’s enough to go around.
“…we usually say a few words to people in Eryn: We use up far more of the earth than people
stores…” (lines 2 and 3) the underlined words may in Africa so it’s a good idea for the whole world to
be understood as discuss the environment.
a) many words could be spoken by people in (Move up – Heinemann)
stores.
b) some words can be said by us. 20) Both Chelsea and Eryn think that we should
c) lots of words are used to say people about us. a) not to burn a lot of fuel.
d) a small number of words are said by people in b) let poor people spread out the food.
stores. c) burn down rain forests.
d) clean rivers and farmlands.
19) The sentence, “In the United States it’s just
hard to make real friends as it is anywhere else” 21) Chelsea and Eryn want to show us that in the
(lines 5 and 6), means future
a) making friends for them is something they a) the population of the whole world will get
aren’t able to do. bigger and face tough
b) although difficult, real friends is something we situations.
have to fight for finding in America. b) rainforests will be burnt and life can get better
c) only in the United States people can’t find easily in the planet.
real friends. c) fumes and the ozone layer might spread out.
d) Americans show to the rest of the world how d) bikes and cars could be more expensive in order
easy relationship is in their country. to reduce
pollution.
Read the following opinions from two children,
Chelsea and Eryn (both aged 8) and then answer The Reluctant Learner
questions 20 and 21 related to them. My friend Tom is one of those six-to-midnight,
enthusiastic,determined, and well-mentioned
Is there future for us? studiers. At six o’clock he approaches his desk,
and carefully organizes everything in preparation
Chelsea: The biggest problem with the for the study period to follow. Having everything
environment is the ozone layer there’s a hole, and in place, he next carefully adjusts each item again,
it’s getting bigger. It’s made by cars and airplanes giving him time to think up the first excuse; he
– things which give off fumes. recalls that in the morning he did not have quite
enough time to read all the items of interest in the
Eryn: The ozone layer’s like a piece of paper newspaper. He also realizes distractions
covering a rock. It’s supposed to protect us. I’m completely out of the way before setting down to
scared the hole will get bigger and move around the task at hand.
the world and people will get cancer. (Skyline 5 – Macmillan)

Chelsea: We could get tandems, and longer bikes, 22) The sentence, “…he recalls that in the morning
so children could ride on the back. Cars should be he did not have quite enough time to read all the
very, very expensive. items of interest in the newspaper” (lines
6, 7 and 8) shows us how Tom
216
PROVAS DA AFA 1998 – 2010
a) returns home in order to read the newspaper
later.
b) complains about his needs of having more time
to read.
c) remembers his lack of time in doing things.
d) assumes how organized he is every time he
reads the news.

23 ) “He also realizes that if he is going to study it


is better to have such distractions completely out
of the way before setting down to the task
at hand.” (lines 8, 9 and 10) The underlined word
can be replaced by
a) understands c) hopes
b) learns d) thinks

Read the following paragraph and then answer


questions 24 and 25 according to it.

“The Kremlin hoping a young strongman can


preserve its brutal victory in Chechnya.”
(Newsweek, September, 2006)

24) The underlined verbs is a (an)


a) regular one and means permission. 1) The correct words to fill the gaps(lines 07 and
b) modal giving an idea of ability. 26) are.
c) defective verb which expresses possibility. a) that/may b) who/might c)
d) irregular form followed by an infinitive. which/could d) that/Should

25) Mark the alternative that completes the blanks 2) One of causes of the high number of suicides in
of the statement below. the Far North may be the.
The word its is a (an) ___________ pronoun and a) micorscopic electric impulses caused by
refers to brainwaves activities attributed to dark glasses.
____________. b) absence of light resulting from a long season
a) personal / young strong man associated to geomagnetism.
b) reflexive / Chechnya c) Northern Lights that increase the morale among
c) possessive adjective / The Kremlin hoping the Alaskans.
d) objective / victory d) link between Aurora Borealis and serious health
risks, such as backaches, discovered by Dr Anita.

AFA 2009/2010 – Inglês 3) The Best Word to describe Professor Hallinan’s


reaction to Dr Bush’s proposition is
Read the below to answer question 01 to 05 a) doubt b) surprise
c) belief d) confidence

4) Mark the option which contains the correct


Question Tag of the following sentence.

“ She hás not yet demonstrated a link between


increased brainwaves and suicidal tendencies...”
a) hasn’t b)hás she?
c) does she? d) doen’t she?
217
PROVAS DA AFA 1998 – 2010
8) When the interviewer said, “Don’t call us” (line
5) Considering the Reported Speech, Dr. Bush 25) he
Said that a) suggested don’t call us.
a) among the states, suicide levels are risen sharply b) told to not call him
to six times. c) advised not to call them
b) suicide levels rose sharply in recent years. d) asked didn’t call us.
c) in recent years, suicide levels had risen sharply
to six times.
d) suicide among 15 to 24-year-olds rises in recent
ten years.

Read this dialogue between melanine and Sam and


the choose the correct alternaties according to it.

9) According to the text above you’ve Just read,


who in fact, performed the job was
a) Everybody b) Nobody
c) Anybody d) Somebody.

10) The pronoun “ it ” (line 07) refers to


a) the story b) someone
c) the job d) Nobody

6) “...he’d be in touch.” (line 24) means that He 11) Mark the alternative that completes the gaps
a) had already bee in touch in the past. (lines 04 and 08) correctly.
b) has already been in touch a) asked/asked b) told/said
c) will be in touch in the future c) Said/tell d) tell/told to
d) was going to be in touch.
12) The contraction ‘d (line 04) means
7) The Direct Speech of the two sentences “...I’d a) had b) would c) did d) do
missed the train.” And “...I was sorry...” ( lines 16
and 17) is. 13) At the airport Nick
a) “I missed the train” and “I am sorry”. a) met his Americam girlfriend
b) “ I miss the train” and “I was sorry”. b) planned to go back to New York
c) “I have been missing the train” and “I was c) answered some questions
sorry”. d) talked to the policemen.
d) “I would miss the train” and “I have been
sorry”. 14) The Word ‘enough’ (line 07) can be replaced
in the text above by
218
PROVAS DA AFA 1998 – 2010
a) inadequate b) enormous
c) efficient d) sufficient

Rosana Fischer works at an autdoor activities


centre on the West coast of Scotland.She teaches
mountain climbing, scuba diving and hang-gliding.
She’s talking to some Young people who’ve Just
arrived at the centre:

I - ‘You cant’t do any of the activities unless


you’re with na instructor.
II – ‘We won’t let you start na activity If you
don’t have the correct equipmente.
III - ‘You cant’t go scuba diving unless you’ve
done the training course.
IV –‘Remember you can’t leave the centre unless
you say where you’re going.’
Englisch Grammar in Steps – Richmond 17) Mark the correct pronouns to fill in the blanks
(line 02 and 10)
15) Mark the option which shows another way to
rewrite the conditional sentences above, correctly. a) us/it b) them/they
I - You can do none of the activities If you dont’t c) you/him d) me/ours
be with na instructor.
II –We Will not let you start an activity unless you 18) The Passive Voice of the sentence ‘He is now
haver the correct equipment. testing the theory...’(line 15) is
III – You cannot going scuba diving If you haven’t a) Now the theory was testing by him.
done the training course. b) The theory was now tested.
IV – Remember you can’t leave the centre If you c)The theory is being tested now
don’t say where you go. d) He tested the theory.
a) I, II na III b) II nad IV c) I , II and IV d)
III and IV 19) We can replace the pronoun ‘who’ (line 13) by
a) that b) whose c) which d) whom
16) According to the text, Rosana Fishe is
a) explaining the importance of sports. No time to relax
b) inviting the readers to practice her activities
c) teaching who have always attended her classes Even when we relax we do everything more
d) establisching conditions. quickly.Ten years ago whe people went to art
galeries they spend tem seconds looking at each
Picture.Today they spend Just three seconds!
Oxford, New English File

20) According to the paragraph


a) in the past people dindn’t appreciate arts.
b) nowadays people spend less time visiting art
galleries
c) only tem years ago people liked to vist art
galleries
d) we only relax visiting interesting places.

219
PROVAS DA AFA 1998 – 2010
AFA 1997/1998 – Matemática 10) Seja P(3,1) o ponto médio do segmento AB,
onde A é intersecção da reta (t) com a reta (r) 3x -
1) Em um grupo de n cadetes da Aeronáutica, 17 y = 0 e B, a intersecção de (t) com a reta (s) x + 5y
nadam, 19 jogam basquetebol, 21 jogam voleibol, = 0. O coeficiente angular de (t) é
5 nadam e jogam basquetebol, 2 nadam e jogam a) negativo.
voleibol, 5 jogam basquetebol e voleibol e 2 fazem b) par positivo.
os três esportes. Qual o valor de n, sabendo-se que c) 5, pois (t) é perpendicular à (s).
todos os cadetes desse grupo praticam pelo menos d) nulo, isto é, a reta é do tipo y = k, k = constante.
um desses esportes?
a) 31 b) 37 c) 47 e) 51 11) A reta (s), simétrica de (r) x - y + 1 = 0 em
relação à reta (t) 2x + y + 4 = 0,
2) Entrevistando 100 oficiais da AFA, descobriu- a) passa pela origem.
se que 20 deles pilotam a aeronave TUCANO, 40 b) forma um ângulo de 60O com (r).
1
pilotam o helicóptero ESQUILO e 50 não são c) tem - como coeficiente angular.
5
pilotos. Dos oficiais entrevistados, quantos pilotam
o TUCANO e o ESQUILO? d) é paralela à reta de equação 7y - x + 7 = 0.
a) 5 b) 10 c) 15 d) 20
12) Inscreve-se um quadrilátero convexo ABCD

3) Quanto devemos adicionar a cada um dos em uma circunferência tal que ABC = xO. Então,
números k + 3, k, k - 2 para que, nesta ordem,  
formem uma Progressão Geométrica? A C B + BD C ,
em graus, é o
a) 6 – k b) 6 + k c) 1 - 6k d) 1 + 6k a) suplementar de x. b) suplementar de 2x.
c) complementar de x. d) complementar de 2x.
4) As raízes da equação x3 - 7x2 + 14x - 8 = 0
formam uma Progressão Geométrica de razão 13) Seja f: [1,  ) → [-3,  ) a função definida por
a) 2 b) 3 c) 4 d) 5
f(x) = 3x2 - 6x. Se g: [-3,  ) → [1,  ) é a função
5) Se A, B e C são as raízes da equação inversa de f, então [g(6) - g(3)]2 é
1 1 1 a) 5 b) 2 6 c) 5 – 2 6 d) -5 + 2 6
5x3 - 7x + 12 = 0, então + + é
AB BC AC
a) –2 b) –1 c) 0 d) 1 14) O lugar geométrico dos pontos do plano
cartesiano que, juntamente com os pontos A(-3,5)
6) A equação 7x4 - 5x3 + (R - 6)x2 + (3S - 2)x + T e B(3,5), determina triângulos com perímetro 2p =
- 9 = 0 tem uma raiz tripla em x = 0. O Produto 16 cm é uma
R S T é
a) elipse. b) parábola.
a) –18 b) –14 c) 16 d) 36 c) hipérbole. d) circunferência.
7) Lançando-se 4 dados, sucessivamente, o número 15) Seja ABC um triângulo retângulo em A,
de maneiras de se obter soma 7 é circunscrito por uma circunferência de raio r, e
a) 20 b) 24 c) 72 d) 216 
A B C = x. A razão entre a área do triângulo e o
8) O número de anagramas da palavra quadrado da metade do valor da hipotenusa é
ALAMEDA que não apresenta as 4 vogais juntas sen2 x cos 2 x cos 2x
é a) sen 2x b) c) d)
2 2 2
a) 96 b) 744 c) 816 d) 840
16) O valor da expressão
9) A quantidade de números naturais de 4 cos 35O (sen 25O + cos 55O ) +
algarismos distintos, formados por 1, 2, 3, 4, 5 e 6, o
tg 31 + tg 14
o
O O O
que contém o algarismo 3 ou o algarismo 4 é + sen 35 (cos 25 - sen 55 ) + o o
1 − tg 31  tg 14
a) 196 b) 286 c) 336 d) 446
é

220
PROVAS DA AFA 1998 – 2010
2 −3 3+ 2 2+ 3 2− 3 23) Uma aeronave decola, iniciando seu vôo sob
a) b) c) d) um ângulo de 30O, em relação ao solo, mantendo-
2 2 2 3
se sob tal inclinação nos primeiros 500 metros.
17) A área da intersecção das regiões do plano Em seguida, diminui em 15O o seu ângulo de
cartesiano limitada por x2 + (y - 4)2  25 e inclinação, mantendo-se assim por 1 quilômetro.
x  Logo após, nivela-se até iniciar a aterrissagem.
y  4 + 1 é Qual é, aproximadamente, a altura dessa aeronave,
3 
9 17  25  31 em metros, em relação ao solo, durante o seu vôo
a) b) c) d) nivelado?
2 2 2 2
a) 400 b) 500 c) 600 d) 700
−y
a
18) Seja −y
= x, com a  R, a > 0 e a  1. 24) O conjunto-solução, em R, da equação
1+ a (cos x)(sen 2x) = (sen x)(1 + cos 2x), é
Determinando-se y em função de x, o domínio da a)  b) R.
função assim definida é c) {x  R l x = 2k  /2, k  Z}.
a) {x  R / x  0}. b) {x  R / x  1}. d) {x  R l x = 2k  /3, k  Z}.
c) {x  R / x < 1}. d) {x  R / 0 < x < 1}.
25) No triângulo retângulo ABC, os catetos AB e
19) Corta-se um pedaço de arame de comprimento AC medem, respectivamente, 2 + 2 e 2. Seja D
98 cm em duas partes. Com uma, faz-se um um ponto de AB, tal que AD = AC . Se  e  são,
quadrado, com a outra, um retângulo com base e  
altura na razão de 3 para 2. Se a soma das áreas respectivamente, as medidas de AD C e A B C,
compreendidas pelas duas figuras for mínima, o então tg( + ) é
comprimento, em cm, do arame destinado à a) 2 - 1 b) 2 + 2 c) 2 2 - 1 d) 2 2 + 1
construção do quadrado será
a) 36 b) 48 c) 50 d) 54 26) Seja P o produto dos fatores (sen nO + cos nO ),
onde n = 45, 46, 47, ..., 149, 150. Pode-se afirmar
20) Seja f uma função real que satisfaz as que
seguintes propriedades: a) P = 0 b) P = 290 c) 1  P < 8 d) 8  P  290
I) f(0) = 1;
II) 0 < f(1) < 1; e 27) A função real f(x) = sen2x + cos x tem valor
III) f(x + y) = f(x)f(y)  x, y  R. máximo em
Então, a expressão f(0) + f(1) + f(2) + f(3) +...+ a) x = (2k + 1), k  Z
f(9) é b) x = (k  1/6), k  Z
equivalente a c) x = (2k + 1/2), k  Z
9 10
[ f (1)] − 1 [ f (1)] −1 d) x = (2k  1/3), k  Z
a) b)
f (1) − 1 f (1) − 1
9 10    
c)
[ f (1)] − f (1)
d)
[ f (1)] − f (1) 28) O valor de sen  + +...+
n
+... , n  N, é
2 4 2 
f (1) − 1 f (1) − 1
1
a) – 1 b) 0 c) d) 1
21) Se log10 x  (log 2 4  log 4 6  log 6 8 ) - 1 , então 2

a) 0 < x  102 b) 102 < x  104 1


c) 104 < x  106 d) 106 < x  108 29) O conjunto-solução da inequação  sen x
4
2
22) O conjunto-solução da inequação cos x < , para 0  x  , é
(0,5)x(x - 2) < (0,25)x - 1,5 é 2
   5
a) {x  R / x <1}. a) {x  R /  x }. b) {x  R /  x }.
b) {x  R / x >3}. 12 6 12 3
5 5  5
c) {x  R / 1 < x <3}. c) {x  R /  x }. d) {x  R /  x }.
d) {x  R / x < 1 ou x > 3}. 12 6 12 12

221
PROVAS DA AFA 1998 – 2010
30) 2
Um círculo com área 100 cm possui uma d) Se duas retas concorrentes de um plano são,
corda de 16 cm. Qual a área, em cm2, do maior respectivamente, paralelas a duas retas de outro
círculo tangente a essa corda e a esse círculo em plano, então estes planos são paralelos.
pontos distintos?
a) 36 b) 49 c) 64 d) 81 37) A relação entre o raio da esfera inscrita, e o da
esfera circunscrita a um tetraedro regular é
31) O pentágono ABCDE está inscrito em uma a) 1/3 b) 3/4 c) 1/4 d) 2/3

circunferência de centro O. Se o ângulo A O B 38) Seja uma pirâmide de base quadrada com

O
mede 40 , então, a soma dos ângulos BC D e arestas de mesma medida. O arc cos do ângulo
 entre as faces laterais que se interceptam numa
A E D, em graus, é aresta é
a) 144 b) 180 c) 200 d) 214 a) -2/3 b) -1/3 c) 1/3 d) 2/3

32) Dois vértices de um triângulo eqüilátero 39) A área total da pirâmide regular de apótema
pertencem a dois lados de um quadrado cuja área é A2, onde A1 e 2p são, respectivamente, apótema e
1 m2. Se o terceiro vértice do triângulo coincide perímetro de sua base, é
com um dos vértices do quadrado, então, a área do p
a) p(A1 + A2) b) (A1 + A2)
triângulo, em m2, é 2
A2
a) 2 3 - 1 b) 2 3 + 1 c) -3 + 2 3 d) 3 + 2 3 c) 2p(A1 + A2) d) p(A1 + )
2

33) Seja ABCD um quadrado, ABE um triângulo 40) A razão entre os volumes de dois cones
eqüilátero e E um ponto interior ao quadrado. O eqüiláteros de alturas h e 2h é

ângulo A E D mede, em graus, a) 1/2 b) 1/4 c) 1/6 d) 1/8
a) 55 b) 60 c) 75 d) 90

34) Seja o triângulo eqüilátero DEF, inscrito no AFA 1998/1999


triângulo isósceles ABC, com AB = AC e DE
 
paralelo a BC. Tomando-se AD E = , C E F =  e 1) Se a seqüência de inteiros positivos (2, x, y) é
 uma Progressão Geométrica e (x+1, y, 11) uma
D F B =  pode-se afirmar que Progressão Aritmética, então, o valor de x + y é
a)  +  = 2 b)  +  = 2 a) 11. b) 12. c) 13. d) 14.
c) 2 +  = 3 d)  + 2 = 3
2) A soma das raízes da equação log2 (x2 - 6x) = 4
35) A intersecção de 3 superfícies esféricas é
distintas pode ser, somente, ou a) 4. b) 5. c) 6. d) 7.
a) 1 ponto, ou vazia, ou 1 circunferência.
b) 1 ponto, ou vazia, ou 2 circunferências. 3) A representação trigonométrica do conjugado
c) 1 segmento de reta, ou vazia, ou 1 do número complexo z = (1 + 3 i)5, sendo i a
circunferência. unidade imaginária e k  Z, é
d) 2 pontos, ou 1 ponto, ou vazia, ou 1 a) 32cos(/3 + 2k) - 32isen(/3 + 2k).
circunferência. b) 32cos(5/4 + 10k) - 32isen(5/4 + 10k).
c) 32cos(5/6 + 10k) - 32isen(5/6 + 10k).
36) Qual das afirmações abaixo é verdadeira? d) 32cos(5/3 + 10k) - 32isen(5/3 + 10k).
a) Por uma reta dada pode-se conduzir um plano
paralelo a um plano dado. 4) A equação polinomial de menor grau com raízes
b) Se uma reta é paralela a dois planos, então esses
1 e i, onde i = − 1 , é
planos são paralelos.
a) x3 - x2 + x - 1 = 0. b) x3 + x2 - x - 1 = 0.
c) Por um ponto qualquer é possível traçar uma
c) x3 - x2 - x + 1 = 0. d) x3 + x2 + x + 1 = 0.
reta que intercepta duas retas reversas dadas.

222
PROVAS DA AFA 1998 – 2010
5) O conjunto-solução da inequação 12) Em uma reunião social, cada participante
cumprimenta todos os outros uma única vez. Se
1 + 2x − 3 x 2 <5é
houve um total de 36 cumprimentos, o número de
 1 + 19  participantes da reunião é
a) x R 
19
 x . a) 7. b) 8. c) 9. d) 10.

 3 3  
 1 − 19 1 + 19 
b) x  R x . 13) Uma bola é solta de uma altura de 128 metros
 3 3  em relação ao solo, e, ao atingir o mesmo, ela sobe
 − 1 − 19 − 1 + 19  a metade da altura anterior. Esse movimento se
c) x R x

. repete até atingir o solo pela décima vez. Nesse

 3 3 
 momento, quanto a bola terá percorrido, em
 1 − 19 1 + 19 
d) x  R x  ou x 

.
metros?

 3 3 
 a) 255,50 b) 383,00 c) 383,50 d) 383,63

6) Sendo P(x) = x3 - x2 + x + a divisível por (x - 14) A solução da equação 3 + 315 , x0,5 = 48 x é


1), a média geométrica de suas raízes complexas é a) 3-1. b) 3-1/2. c) 31/2. d) 3.
a) 1. b) i . c) - i . d) i.
15) No desenvolvimento de (x + 2)nx3, o
7) Sendo a unidade imaginária uma das raízes da coeficiente de xn+1 é
n(n + 1) n(n − 1)
equação x4 - 3x2 - 4 = 0, pode-se afirmar que esta a) . b) .
equação 2 4
a) não tem raízes reais. b) tem duas raízes c) 2n(n - 1). d) 4n(n - 1).
racionais.
c) possui duas raízes irracionais. 16) A probabilidade de observarmos um número
d) possui uma raiz de multiplicidade 2. na face superior de um dado viciado é diretamente
proporcional a esse número. Ao lançarmos esse
8) Dado P(x) = x3 - 3x2 - 4x + m - 1, o valor de m, dado, a probabilidade de ocorrer um número par é
1 11 4 13
para o qual P(x) é divisível por (x - 2), é a) . b) . c) . d) .
2 21 7 21
a) 1. b) 7. c) 13. d) 17.
17) Quatro pontos não-coplanares determinam,
9) Se a, b, c, d, e são as raízes do polinômio
exatamente, quantos planos?
P(x) = 2x5 - 6x4 + 3x3 + x2 + 7x + 5, então, o valor a) 1 b) 2 c) 3 d) 4
1 1 1 1 1
de + + + + é 1 1
a b c d e 18) Se x + = 2, então, x3+ é igual a
x x3
7 2 3 1 a) 1. b) 2. c) 6. d) 8.
a) - . b) - . c) . d) .
5 5 7 3
19) O gráfico que melhor representa a função
10) Os valores reais de x, para os quais a parte real
do número complexo z =
x − 2i
é negativa,
f(x) =
1
2
(x− x ) é
y
x+ i a) c)
pertencem ao conjunto (intervalo) y
a)   . b) 0 . c) ( −11
, ). d) − 2, 2 . ( ) x

x
11) O valor de m que satisfaz a expressão
m
 m
 3 k  k  = 1024 é b)
y
d)
y
k =0
a) 2. b) 3. c) 4. d) 5.
x x

223
PROVAS DA AFA 1998 – 2010
2
(x − 4) ( y − 3 )2
a) + = 1. y
9 16
20) Seja D = 12 , ,3,4,5 e f: D → R, a função 9
definida por f(x) = (x - 2)(x - 4). Então, pode-se 2 2
( x − 5) ( y + 1)
b) + = 1.
afirmar que f 9 16
a) é bijetora.
b) é somente injetora. ( x + 1)2 ( y − 5)2
c) é somente sobrejetora. c) + = 1.
16 9 1
d) possui conjunto imagem com 3 elementos.
-1 2 x
( x + 1)2 ( y − 5)2
  d) + = 1.
21) O valor de - log 2 log 2 2 é 9 16
 
a) 1. b) 2. c) 3. d) 4. 28) Os pontos A(-5,2) e B(1,6) são extremos de
um dos diâmetros da circunferência de equação
22) Seja f uma função real do primeiro grau com a) x2 + y2 - 2y - 25 = 0.
f(0) = 1 + f(1) e f(-1) = 2 - f(0). Então, o valor de b) x2 + y2 + 4x - 8y + 7 = 0.
f(3) é c) x2 + y2 - 4x + 4y - 57 = 0.
a) -3. b) -2,5. c) -2. d) -1,5. d) x2 + y2 + 8x - 14y + 39 = 0.

23) O valor da expressão cos 15O + sen 105O é 29) A distância entre o ponto de interseção das
6+ 2 6− 2 x = t − 2
a) . b) . retas r: 2x - 3y + 4 = 0 e s:  , t R ea
4 4
y = 2t + 1
6+ 2 6− 2 1 1
c) . d) . reta q: y = x+ é
2 2 2 8
3 7 3 5
x 0 1 a) 4 5 . b) . c) . d)
20 10
24) O determinante 0 1 x é 5 7
1 0 x .
4
a) positivo para x  R.
b) negativo para x  R 0  x  1 . 30) O eixo das ordenadas, a reta r: y = 2x -1 e s,
c) positivo para x  R x  −1 ou x  1 . que é perpendicular a r e passa pela origem,
determinam um polígono cujo valor da área é
d) negativo para x  R x  −1 . 1 2 5 2 5
a) . b) . c) . d) .
5 5 5 5
25) Se os elementos da matriz A3x4 são definidos
por aij = 2i - j, então, o elemento b23 da matriz B = 31) Na figura abaixo o perímetro do triângulo
2-1A.At é equilátero ABC é 72 cm, M é o ponto médio de
a) 1. b) 7. c) 10. d) 13. AB e CE = 16 cm. Então, a medida do segmento
CN, em cm, é um sétimo de
26) O valor da excentricidade da cônica a) 48.
2 2
(x − 5) ( y − 2) A
− =1 é
4 9 b) 49.
13 5 M
a) 2 b) c) d) 3 N
2 2 c) 50.
B E
C
27) A equação reduzida da cônica, representada no d) 51.
gráfico abaixo, é

32) Na figura abaixo, o lado do quadrado é 1 cm.


Então, a área da região hachurada, em cm2, é

224
PROVAS DA AFA 1998 – 2010
 1
a) − .
4 2 38) Qual o volume, em cm3, da esfera inscrita em
um cone reto, cuja altura e diâmetro da base são,
 1 respectivamente, 16 cm e 24 cm.?
b) − .
2 2 500
a) 27 b) b)  c) 288 d) 686
3
 1
c) − .
4 4 39) Qual deve ser a medida da altura de um prisma
reto, cuja base é um triângulo equilátero de lado a,
 1 para que seu volume tenha valor a3?
d) − .
2 4 a 3 3a 3 a 3 4a 3
a) b) c) d)
4 4 3 3
33) A área do quadrado menor, da figura abaixo, 40) O apótema de uma pirâmide regular, com base
vale hexagonal, é 9 3 cm. Se a sua área lateral é o
10
triplo da área de sua base, então, o seu volume, em
a) 2 . 2 cm3, é
2
3 323 81 35
b) 2 . 10 10 a) . b) .
4 4
c) 81 3 . d) 324 2 .
c) 5 . 2
2

10
d) 8 . AFA 1999/2000 – Matemática
1. A imagem da função real f definida por f(x)
34) Considere um triângulo equilátero, um 2+x
quadrado e um hexágono regular, todos com o = é
mesmo perímetro. Sejam AT, AQ e AH as áreas do 2−x
triângulo, do quadrado e do hexágono, a) R – {1} b) R – {2}
respectivamente. Então, pode-se afirmar que c) R – {-1} d) R – {-2}
a) AT < AQ < AH . b) AT = AQ = AH .
c) AT < AQ e AQ > AH . d) AT < AQ e AQ = AH . 2. Dadas f e g, duas funções reais definidas por f(x)
= x3 – x e g(x) = sen x, pode-se afirmar que a
35) De 2h 45mim a 4h 35mim, o ponteiro das expressão de (f o g)(x) é
horas de um relógio percorre, em radianos, a) sen2x cos x b) – sen (x3 – x)
11  c) – sen x cos2 x d) sen x3 – sen x
a) . b) .
36 3
5 7 3. O domínio da função real f(x) = log(– x2 + 6x +
b) . d) . 16) + log(x2 – 6x + 8) é
18 24
a) {xR | – 2  x  2 ou 4 < x  8}
36) Seja um triângulo com dois de seus lados b) {xR | – 2 < x < 2 ou 4 < x < 8}
medindo 2 m e 5 m e área igual a 3 m2. Se o c) {xR | x < – 2 ou 2 < x < 4 ou x > 8}
ângulo entre esses dois lados do triângulo triplicar, d) {xR | x < – 2 ou 2 < x < 4 ou x > 4}
a área do mesmo será aumentada, em quantos m2?
36 42 12 14 4. A soma das raízes da equação 32 – x + 31 + x = 28
a) b) c) d)
25 25 5 5 é
a) 1 b) 2 c) 3 d) 4
37) O valor real que satisfaz a equação
  | x + y |= a
arc sen x + arc sen 2x = , para x pertencente ao 5. O sistema  é indeterminado
2
intervalo (0,1), é x − by = −a
1 5 1 2
quando
a) . b) . c) . d) .
5 5 2 2
225
PROVAS DA AFA 1998 – 2010
a) ab = -1 b) ab-1 = -1 a) 2 (cos 2 /4 + isen /4)
c) a + b = -1 d) a – b = -1 b) 2 (cos 3 2 /4 + isen 3/4)
6. Se os números reais x e y satisfazem c) 2 (cos 5 2 /4 + isen 5/4)
2 d) 2 (cos 7 2 /4 + isen 7/4)
−2 y
− 81 = 0 , então, dado
log = 0e 3
x+y − 3−8y 3 2 xy 12. Se a divisão do polinômio P(x) = ax20 + bx11 –
−1 2x9 por Q(x) = 4x2 – 4 tiver resto R(x) = –1, com
i = −1 , x y é a, bR, então
a) 0 b) i c) 2i d) 3i 1
a) ba = b) b a = 2
2
7. O produto das raízes da equação
a 1
c) = − d) log b a = 0
2x 8x 0 b 3
log 2 x log 2 x 2
0 = 0, com x *+ , é
1 2 3 13. O valor de sen (arc cos 1/2 + arc sen 1/3) é
2 2 −1 2 6 +1
a) 1/2 b) 3/4 c) 4/3 d) 3/2 a) b)
2 6
8. A expressão 2 3 −1 − 2 6 +1
c) d)
 a    b2 
 3 6
   +    = 0 , com a,
(log a ) log  2  (log c ) log
 c 
  b     14. Os valores de mR, para os quais a
b,c  *+ , é verdadeira quando equação 2 (sen x – cos x) = m2 – 2 admite
soluções, são
a) b = ac ou a = c b) c2 = ab ou a = b
2

c) a = bc2 ou b = c d) ac-1 = b2 ou a = b a) – 1  m  1 b) – 2  m  2
c) 0  m  2 d) − 2  m  2
9. Se b = 2 − x + x +12 , então o número de soluções
2

inteiras que satisfaz a inequação 2
15. A inequação 2senx    , com x[0, 2] e 
5 3 3
log b    log b   é
7 4 log 2
= , tem como solução os valores de x
a) 4 b) 5 c) 6 d) 7 log 2 − log 3
pertencentes a
10. Seja z o conjugado do número complexo z a) [0, /3]  [2/3, 2]
1 i b) [0, /2]  [3/2, 2]
= + . A seqüência de todos os valores de
2 2 c) [0, /6]  [5/6, 2]
nN, tal que (z )− n seja um imaginário puro, é d) [0, 4/3]  [5/6, 2]
uma progressão
a) aritmética com primeiro termo igual a 2 e razão 5
16. Se a + b = , então (1 + tg a)(1 + tg b) é
8. 4
b) geométrica com primeiro termo igual a 2 e a) 0 b) 1 c) 2 d) 3
razão 2.
c) aritmética com primeiro termo igual a 2 e razão 17. Se (sen x, sen 2x, cos x) é uma progressão
4. geométrica estritamente crescente, com 0 < x < 2?,
d) geométrica com primeiro termo igual a 2 e então o valor de x é
razão 1. a) /12 b) /10 c) /8 d) /6

11. Considere o polinômio P(z) = z2 – 2z + iw, 18. Se a soma dos 6 primeiros termos de uma
wC. Se P(3 + 2i) = 1 + 10i, onde i = − 1 , progressão aritmética é 21 e o sétimo termo é o
então uma forma trigonométrica de w é
226
PROVAS DA AFA 1998 – 2010
triplo da soma do terceiro com o quarto termo, 9 13 10 13
então o primeiro termo dessa progressão é a) b)
13 13
a) –7 b) –8 c) –9 d) –10
11 13 12 13
c) d)
19. Seja (x, y, z, w) uma progressão aritmética 13 13
crescente cuja soma é 10 e (a, b, c, d) uma
progressão geométrica com a + b = 1 e c + d = 9. 26. A área do polígono que tem como vértices os
Se ambas têm a mesma razão, então o produto yw extremos dos eixos maior e menor da elipse 4x2 +
é y2 – 24x – 6y + 41 = 0, é
a) –8 b) –2 c) 7 d) 9 a) 1 b) 2 c) 3 d) 4

20. Usando-se 5 dos algarismos 1, 2, 3, 4, 5, 6 e 7, 27. A excentricidade da elipse que tem centro na
sem repeti-los, a quantidade de números pares que origem, focos em um dos eixos coordenados e que
se pode formar é passa pelos pontos A(3,2) e B(1,4) é
a) 1080 b) 2160 c) 2520 d) 5040 2 3 2 3
a) b) c) d)
3 3 2 2
21. Se, no desenvolvimento do binômio (x + y)m +
5
, ordenado segundo as potências decrescentes de 28. Se P(1, y) pertencente ao primeiro quadrante, é
x, o quociente entre os termos que ocupam as o único ponto de intersecção da curva
2
posições (m + 3) e (m + 1) é y 2 x − 2 , então o : x2 + y2 + 2x – 2y – 6 = 0 com a reta r, então a
3 equação reduzida de r é
valor de m é a) y = – x b) y = – x + 4
a) par. b) primo. c) ímpar. d) múltiplo de 3. c) y = – 2x + 7 d) y = – 2x + 1

22. Os coeficientes do quinto, sexto e sétimo 29. Os pontos P(a, b) e Q(1, -1) são intersecção das
termos do desenvolvimento de (1 + x)n estão em circunferências  e , com centros
progressão aritmética. Se n  13, então o valor de C(-2, y) e C(b, a + 1), respectivamente. Sendo
2n + 1 é C  C  perpendicular a PQ que, por sua vez, é
a) 7 b) 13 c) 15 d) 27
paralelo ao eixo das ordenadas, a equação geral de
23. Uma urna contém 1 bola preta e 9 brancas. ?é
Uma segunda urna contém x bolas pretas e as a) x2 + y2 – 8x – 4y + 2 = 0
restantes brancas, num total de 10 bolas. Em um b) x2 + y2+ 4x – 4y – 10 = 0
primeiro experimento, retira-se ao acaso uma bola c) x2+ y2 – 10x – 2y + 6 = 0
de cada urna. Em um segundo experimento, todas d) x2 + y2 – 10x – 4y + 4 = 0
as bolas são reunidas em uma única urna, e duas
são retiradas, ao acaso, uma seguida à outra, sem 30. O valor de x2, na figura abaixo, é
reposição. O menor valor de x, tal que a
probabilidade de se obterem duas bolas pretas seja
estritamente maior no segundo experimento, é
a) 1 b) 2 c) 3 d) 4

24. O parâmetro da parábola que passa pelo ponto


P(6,2) e cujo vértice V(3,0) é o seu ponto de
tangência com o eixo das abcissas, é
a) 9/5 b) 9/4 c) 3 d) 9/2

25. No plano cartesiano, a distância da origem à


reta que passa pelos pontos A(0,4) e B(6,0) é

227
PROVAS DA AFA 1998 – 2010
a2
a 4
a2 AS = x. Nessas condições, a área da figura
a) b 2 − b) − sombreada pode ser expressa por
4 2 4
b
2 4
b b b4
c) − d) b 2 −
4 a2 4a 2

31. Seja P um ponto interior a um triângulo


equilátero de lado k. Qual o valor de k, sabendo-se
que a soma das distâncias de P a cada um dos
lados do triângulo é 2?
2 3 4 3
a) b) 3 c) d) 2 3
3 3
a) 5x se x[0,5] e x2 – 10x + 50 se x[5, 10]
32. Uma corda de comprimento a define em uma b) x2 se x[0,5] e x2 – 10x + 50 se x[5, 10]
circunferência de raio 2a um arco , 0   < /2. c) 5x se x[0,5] e –x2 + 20x – 50 se x[5, 10]
Nessa mesma circunferência, o arco 2? é definido d) x2 se x[0,5] e –x2 + 20x – 50 se x[5, 10]
por uma corda de comprimento
a 11 a 13 36. Se as dimensões de um paralelepípedo reto
a) b) retangular são as raízes de 24x3–26x2+9x–1=0,
4 3 então sua diagonal é
a 15 a 15 a) 9/24 b) 7/12 c) 61 / 12 d) 73 / 24
c) d)
4 2
37. Seja um tronco de cone reto com altura h e
33. Na figura, O e M são centros das bases de raio R e r (R > r). Retira-se desse sólido
semicircunferências. O perímetro do triângulo um cone reto invertido com base coincidente com
DBC, quando AO = r = 2AM, é a base menor do tronco e altura h. Se o volume do
sólido resultante é igual ao volume do sólido
retirado, então
a) R2 + Rr – r2= 0 b) R2 + Rr – 2r2 = 0
c) 2R2 – Rr – r2 = 0 d) 2R2 + Rr – 2r2 = 0

38. A razão entre os volumes das esferas inscrita e


circunscrita em um cone equilátero é
a) 1/16 b) 1/8 c) 1/4 d) 1/2

39. A distância entre as arestas reversas em um


r (3 2 + 5 ) r( 2 + 3 5 ) tetraedro regular de aresta a e apótema g é
a) b)
2 2 4g 2 − a 2 4g 2 − a 2
a) b)
r ( 2 + 3 10 ) r (3 2 + 10 ) 2 4
c) d)
2 2
g 2 − 4a 2 g 2 − 4a 2
c) d)
34. No quadrilátero ABCD, AB = AD = 2BC = 2 4
2CD e B̂  D̂ = 90 o . O valor do ângulo interno Â
40. Na figura a seguir, AD = 2 e CB = 5. Se tg  =
é
a) arc cos 1/5 b) arc cos 2/5 4/5, então cotg  é
c) arc sen 3/5 d) arc sen 4/5

35. Na figura abaixo, AC = BC, h = AB = 10 e SP


é perpendicular a AB . O ponto S percorre AB e
228
PROVAS DA AFA 1998 – 2010
A

a) ab 1− a 
3 b

b) 2ab(1 −a2)


c) 2ab 1− a
3
C B
d) 2ab 1− a2 x
a) 15/17 b) 13/17 c) 17/20 d) 19/20
5. O acesso ao mezanino de uma construção
deve ser feito por uma rampa plana, com 2m de
AFA 2000/2001 – Matemática comprimento. O ângulo  que essa rampa faz com
o piso inferior (conforme figura) para que nela
1. Os valores de , 0   < 2, que satisfazem sejam construídos 8 degraus, cada um com 21,6
a desigualdade − x2 + 1/2 < sen , para todo cm de altura, é, aproximadamente, igual a
x real, pertencem ao intervalo
a) 15O
a) 
0
 b) 30O 2m
b)  c) 45O
0
6 
c) 5
d) 60O

6
d)
 5 6. Na figura abaixo, a circunferência de
 
6 6 centro O é trigonométrica, o arco AM tem medida
, 0 <  < /2, e OMP é um triângulo retângulo
em M. Esse triângulo tem por perímetro
2. Os valores de x que satisfazem a equação
x(x cotg  − cos ) = –x + sen , 0 <  < /2, são
y
a) sen  e –tg 
b) sen  e cos  M

c) tg  e –cotg 
d) sec  e –cossec  
O A P x
3. Simplificando a expressão
(cos sec x )2 − 2 ,
(cos sec x )2
para cossec x  0, obtemos
a) cos x
1 + sen  + cos 
b) cos2 x a)
cos 
c) sen2 x
1 + sen  + cos 
d) cos 2x b)
sen 
1 + 2 sen  + cos 
  c)
4. Sejam sen =a, 0 <  < , e CB um cos 
3 2
1 + sen 2 + cos 
segmento de medida x, conforme a figura abaixo. d)
sen 
O valor de x é
7. Conforme a figura abaixo, s e t são,
respectivamente, retas secante e tangente à
circunferência de centro O. Se T é um ponto da
229
PROVAS DA AFA 1998 – 2010
circunferência comum às retas tangente e secante, a) 1 y
então o ângulo , formado por t e s, é 8

a) 10O b) 0,5

b) 20O O s
c) 0,25
O
c) 30 −2 –x x 2 x
80O 
d) 40O t d) 0,125
T

8. O gráfico que melhor representa a função 10. A quantidade de pares de retas reversas que
y = sen x + cos x, com 0 x < 2, é contêm as arestas de um cubo é
a) 12 b) 24 c) 36 d) 48
y
a) 2
11. Sejam r e s retas paralelas. A medida do
ângulo , na figura abaixo, é
1
r
O
50 –y
y
0  2 x
40O 
y
2 s

b) a) 115O b) 125O c) 135O d) 145O


1
12. A equação reduzida da hipérbole, cujos
focos são os extremos do eixo menor da elipse de
0  2 x equação 16x2 + 25y2 = 625, e cuja excentri-cidade
é igual ao inverso da excentricidade da elipse dada,
y é
2 a) 16y2 – 9x2 = 144 b) 9y2 – 16x2 = 144
c) 9x2 – 16y2 = 144 d) 16x2 – 9y2 = 144
c)
1 13. O volume, em cm3, do octaedro regular
inscrito numa esfera com volume 36 cm3 é
a) 18 b) 36 c) 54 d) 72
0  2 x
14. A soma dos quadrados das raízes da
equação x3 – 2x2 – 4x + 1 = 0 é
y
2 a) 10 b) 11 c) 12 d) 14

15. Na figura abaixo, F1 e F2 são focos da


d) 1 x2 y2
elipse + = 1. O ponto C, de coordenadas
25 9
 3
0  2 x  0,  , pertence ao segmento MN . Os
 2
9. O retângulo, com base no eixo das
segmentos AC, CB e MN são, respectivamen-te,
abcissas, está inscrito numa parábola, conforme
figura abaixo. O valor de x que faz esse retângulo paralelos aos segmentos F1P, PF2 e F1F2 . A área da
ter perímetro máximo é figura sombreada, em unidades de área, é

230
PROVAS DA AFA 1998 – 2010
ângulo formado pelos ponteiros de um relógio às
y
a) 3 P 9h25min. O valor do ângulo  = CB̂E é

M C N
b) 6 a) 120O D
E

c) 9 F1 A B F2 x
O
A
b) 119,45O  B
d) 12 

c) 126,25O C
2 2
16. A circunferência x + y = 5 possui duas
retas tangentes t1 e t2 que são paralelas à reta
r: y = –2x + 3. As equações gerais das retas t1 e t2, d) 132,50O
respectivamente, são
a) 2x + y – 5 = 0 e 2x + y + 5 = 0 22. O termo independente de x no desenvolvi-
b) 2x + y – 15 = 0 e 2x + y + 15 = 0  
7
1
c) 2x + y – 5 5 = 0 e 2x + y + 5 5 = 0 mento de  x 4 +  é

 x3 
4 5 4 5
d) 2x + y – = 0 e 2x + y + =0 a) 4 b) 10 c) 21 d) 35
5 5

23. Colocam-se em ordem crescente todos os


x y
17. A reta − = 1, a > 0, intercepta os eixos números com 5 algarismos distintos, sem
a a
repetição, formados com 2, 4, 5, 7 e 8. A posição
coordenados x e y nos pontos P e Q,
do número 72584 é
respectivamente. A equação geral da
a) 76a b) 78a c) 80a d) 82a
circunferência tangente ao eixo x no ponto P e
tangente ao eixo y no ponto Q é
24. Seja S o espaço amostral de um
a) x2 + y2 – 2ax + 2ay + a2 = 0
experimento aleatório e A um evento de S. A
b) x2 + y2 + 2ax – 2ay + a2 = 0
probabilidade de ocorrer o evento A é dada por
c) x2 + y2 + 2ax + 2ay + a2 = 0 n − 10
d) x2 + y2 – 2ax – 2ay + a2 = 0 P ( A) = . O número máximo de elementos de
4

2 2 a) 10 b) 11 c) 14 d) 15
18. O valor de cotg (arc sen )é
3
2 2 3 2 25. Sejam a e b números naturais diferentes de
a) b) 2 2 c) d)
2 4 4 zero.
) Se f é uma função tal que
19. A reta s: y = –x + 4 intercepta a f(a + b) = f(a) + f(b), então f(ab) = af(b)
circunferência C: x2 + y2 + 2x – 4y – 4 = 0 nos ) Se log (a + b) = log a + log b, então
pontos P e Q. Se O é o centro de C, então a área do 1 1
triângulo OPQ, em unidades de área, é + =1
a b
a) 4 b) 5 c) 4,5 d) 5,5
) Se para todo x real a função
a b
, então f   = f  
A soma de todos os valores reais que 1
20.
log x f(x–1) =
satisfazem a equação x 4 = 16x, x > 0, é f(x) b a
Considerando (V) verdadeiro e (F) falso, as
assertivas acima são, respectivamente
17 33 65 129 a) V, V, V b) F, V, V
a) b) d) e)
4 4 4 4 c) V, F, F d) V, V, F

21. Na figura, O é o centro da circunferência 26. O sistema


de raio r, AD = DE = EB = r e  é o menor

231
PROVAS DA AFA 1998 – 2010
x + y + az = 1  cos  − sen  
 34. Considere T( ) =   matriz
x + 2y + z = 2  sen  cos  
2x + 5y − 3z = b quadrada definida para todo  real. Sendo cof

é indeterminado para (T()) e det (T()), respectivamente, a matriz
a) a  6 e b = 5 b) a = 6 e b = 5 cofatora e o determinante da matriz T(), é correto
c) a = 6 e b  5 d) a  6 e b  5 afirmar que
a) T(–) = –T()
27. Sejam A uma matriz quadrada de ordem 3, b) cof T() = T(–)
det A = d, det(2A  At ) = 4k, onde At é a matriz c) T(–) = (T())–1
transposta de A, e d é a ordem da matriz quadrada d) det(T(2)) = 4 det(T())
B. Se det B = 2 e det 3B = 162, então o valor de k
+dé 35. Se f e g são funções de IR em IR definidas
a) 4 b) 8 c) 32 d) 36 3x − 2
por f(3x+2) = e g(x–3) = 5x – 2, então
5
28. A soma dos treze primeiros termos da f(g(x)) é
progressão geométrica (2i, –2, ...), onde i = − 1 , é x−4
a)
a) 0 b) 2i c) –2i d) 2i – 2 5
5x + 9
b)
29. A diferença entre os quadrados de dois 5
números naturais é 27. Um dos possíveis valores c) 5x + 13
do quadrado da soma desses dois números é 5x + 11
d)
a) 529 b) 625 c) 729 d) 841 5

30. Se x  IR e 7 5x = 243, então 7 –3x é igual a 36. A figura abaixo representa um quadrado de
a) 1/3 b) 1/9 c) 1/27 d) 1/81 8 cm de lado. A área, em cm2 , da figura hachurada
é
31. Se a soma dos n primeiros termos de uma 3
progressão aritmética (PA) é dada pela fórmula
2
3n2 + n a) 23,02 6
Sn = , então a soma do quarto com o sexto
2 b) 24,01
termo dessa PA é c) 25,04 60O
a) 25 b) 28 c) 31 d) 34 d) 26,10

32. Seja An,p o número de arranjos simples de n


elementos distintos, tomados p a p. A equação An,3
= 6n tem como solução 37. Os números inteiros do domínio da função
a) uma raiz nula. real f(x) = (5 + 2x )  (2 − 3 x ) são as raízes da
b) uma raiz positiva. equação g(x) = 0. Uma expressão analítica da
c) duas raízes positivas. função g(x) é
d) uma raiz positiva e outra negativa. a) x2 + x2 +2x b) x3 + x2 – 2x
c) x3 – 3x2 + 2x d) x3 + 3x2 + 2x
33. Seja P(x) um polinômio de grau 4 com
coeficientes reais. Na divisão de P(x) por x–2, 38. No intervalo [–1, 100], o número de
obtém-se um quociente Q(x) e resto igual a 26. Na soluções inteiras da inequação 3x – 8 > 32–x é
divisão de P(x) por x2 + x –1, obtém-se um a) 97 b) 98 c) 99 d) 100
quociente H(x) e resto 8x – 5. Se Q(0) = 13 e Q(1)
= 26, então H(2) + H(3) é igual a 39. Na figura abaixo existem n triângulos
a) 0 b) 16 c) –47 d) –28 retângulos onde ABC é o primeiro, ACD o
segundo e APN é o n-ésimo triângulo. A medida
do segmento HN é

232
PROVAS DA AFA 1998 – 2010
a n 03 - Os pontos A, B e C são afixos das raízes
a) cúbicas do número complexo z. Se n é o menor
n D ...
a P natural não nulo para o qual zn é um real positivo,
a então n é igual a
a n +1 Im
b) C N
n +1
a H a) 8
B A
a n −1
b) 6
c) c) 4 Re
n −1 B a A
30
d) 2 O

a n +1 OC = 2
d)
n C

40. Considere um triângulo retângulo de


04 - A cada ano que passa, o valor de uma
catetos b e c, hipotenusa a e altura relativa à
máquina diminui 10% em relação ao do valor do
hipotenusa h, h  1. A alternativa correta é ano anterior. Se V for o valor da máquina no ano
a) log a + log b + log c = log h da compra, após 10 anos será
b) log a – log b – log c = log h a) (0,9)10 V b) (0,5)9 V
c) logh (b2 – h2) + log (c2 – h2) = 4
h c) (0,1)9 V d) (0,1)10 V
d) logh (b2 – h2) – log
h (c2 – h2) = 4
05 - Sendo
π π π π π π π π
S= + + + + + + .... + n + n + ... , o
2 3 4 9 8 27 2 3
AFA 2001/2002 – Matemática valor do cos (S – x) é igual a
a) – sen x b) sen x c) – cos x d) cos x
01 - Assinale a alternativa que contém a
afirmação correta. 06 - Se o polinômio P(x ) = x m − 2bn x m −n + bm é
a) x, y, x e y  , (x + y )2 = x+y divisível por x + b, sendo n < m, n  , m 
x y N* e b  0, então, ocorrerá necessariamente
b) x, y, x e y  IN*, se é inteiro, então é
y x a) m par e n ímpar.
inteiro b) m ímpar e n par.
x+y c) m ímpar e n ímpar.
c) x, y, x e y  IN, é um número racional
1+ x d) m par e n par.
x+y
d) x, y, x e y IN, é um número racional
1 + x2 07 - As equações 1) y3 − y2 + ay − b = 0 , onde a e b 
2) y2 − 3y + 2 = 0
02 - Considere no campo complexo uma curva IR, apresentam, respectivamente, as soluções:
2 S1 = , ,  e S 2 = ,  sendo  <  < . É correto
tal que Im    k , onde z é um complexo não nulo.
z
afirmar que
Se k = 2, tem-se sua representação gráfica dada
a) a – b  0 b) 2 = a
pelo
c)  –  = 0 d)  +  = a + b
a) círculo de raio 1 e tangente ao eixo real.
4
1 08 - A palavra que não muda o seu sentido, quer
b) círculo de raio e tangente ao eixo imaginário.
2 se leia da esquerda para a direita ou da direita para
c) conjunto de pontos do plano complexo exterior a esquerda, é chamada palíndromo (Ex., ovo, asa,
1  1  acaiaca, serres, etc.). Considerando-se as 23 letras
ao círculo de raio e centro  − , 0
2  2  do nosso alfabeto, quantos anagramas de 6 letras
d) círculo de raio 1
e tangente ao eixo real. com características de um palíndromo, pode-se
2 formar?
a) 236 b) 233 c) 323 d) 623

233
PROVAS DA AFA 1998 – 2010
6
6
09 - Sendo p(x) =   p x 6 −p . 2p , a soma das Assinale a opção verdadeira.
p=0 a) A figura I representa um sistema de três
raízes de p(x) é um número do intervalo equações com uma única solução.
a) ] –13,0 [ b) ] 11,15 [ b) A figura III representa um sistema de três
c) ] 60,70 [ d) ] –3,3 [ equações cujo conjunto solução é vazio.
c) A figura II representa um sistema de três
10 - Numa demonstração de paraquedismo, equações com uma infinidade de soluções.
durante a queda livre, participam 10 paraquedistas. d) As figuras I e III representam um sistema de
Em um certo momento, 7 deles devem dar as mãos três equações com soluções iguais.
e formar um círculo. De quantas formas distintas e)
eles poderão ser escolhidos e dispostos nesse x = sen2 t

círculo? 15 - As equações paramétricas 
y = cos 2 t
a) 120 b) 720 c) 86400 d) 151200
representam
11 - Na Academia da Força Aérea, existem 8 a) um segmento de reta de extremos (0, 1) e (1, 0)
professores de matemática e 6 de física. Para b) uma elipse de eixo maior igual a 1
2
participar de um congresso no Rio de Janeiro,
c) uma hipérbole de eixo real horizontal
deverá ser formada uma comissão de 4
d) uma circunferência de centro (0, 0) e raio igual
professores. A probabilidade de participarem dessa
a1
comissão 3 professores de matemática e 1 de física
é de
16 - As diagonais de um losango estão contidas
a) 3 b) 48 c) 21 d) 4 nas retas (r) (3m – 1)x + (m –2)y = 0 e (t)
1001 143 286 13
x + (m + 1)y + m + 2 = 0. É correto afirmar que os
12 - As matrizes A, B e C são do tipo m x 3, n x possíveis valores de m
p e 4 x r, respectivamente. Se a matriz transposta a) têm soma igual a 2
de (ABC) é do tipo 5 x 4, então b) têm produto igual a 3
a) m = p b) mp = nr c) n + p = m + r d) r = n c) pertencem ao intervalo ]–3, 3]
d) têm sinais opostos.
 a b
13 - É dada a matriz A= , onde a e b são
 − b a 17 - A equação y = 3 + 4 − ( x − 1) 2 representa:
 0 1  a   5 
números reais. Se   .   =   , então o
 2 3   b   25  a) elipse de eixo maior igual a 2
determinante de A vale 1
b) parábola de vértice V (1, 3) e parâmetro p=
2
a) 2a 2 b) − 2a 2 c) zero d) 2a + 2b
c) hipérbole de eixo real vertical e centro C (1, 3)
d) semicircunferência de centro C (1, 3) e raio r =
14 - O conjunto de soluções de uma única
2
equação linear a1x + a 2 y + a 3 z = b é representado
por um plano no sistema de coordenadas 18 - Dada a equação ax 2 + by 2 = c , onde a, b e c
retangulares xyz (quando a1, a2, a3 não são todos
são reais NÃO nulos, é correto afirmar que,
iguais a zero). Analise as figuras a seguir.
necessariamente, sua representação gráfica é uma
(I) Três planos se (II) Três planos se (III) Três planos a) circunferência, se a = b
cortando numa reta cortando num sem interseção
ponto b) hipérbole, se a = – b e c = b
c) elipse de centro na origem, se a  b e c = 1
d) circunferência, se a = b e c > 0

19 - “O Brasil tem um encontro marcado com o


caos. No dia 1o de junho começa o plano de
racionamento de energia.”

234
PROVAS DA AFA 1998 – 2010
“O modelo energético brasileiro é baseado quase   
( ) f: − 2 , 2  → B definida por y = sen x é
que exclusivamente em hidrelétricas, que  
produzem 97% da energia consumida no país. Sem inversível, se, e somente se, B = [0, 1].
chuva, entra em colapso”. A alternativa que corresponde à seqüência correta
Revista Veja – 16/05/01 é
No gráfico abaixo, tem-se o nível da água a) V F V F
armazenada em uma barragem ao longo dos b) F F V F
últimos anos, que foi construída para represar água c) V F V V
a fim de mover as turbinas de uma usina d) F V F V
hidrelétrica.
22 - Sejam as funções g e f definidas por g: IR
nível (m)

2, se x  2
→ IR tal que e f: IR → IR tal
o nível máximo
120 g(x) = 
− 1, se x  2

80
que f (x ) = x − 2 . Sobre a composta (gof)(x), é
correto afirmar que
30
a) se x  1, então (gof)(x) = –1
10
o nível mínimo para gerar energia b) se x  0, então (gof)(x) = 2
c) se x  –1, então (gof)(x) = –1
0 1989 1995 2000 tempo d) se x  1 e x > 0, então (gof)(x) = –1

Analise as alternativas e marque a opção correta. 23 - Um veículo de transporte de passageiro tem


a) O nível da água permaneceu constante num seu valor comercial depreciado linearmente, isto é,
período de 8 anos. seu valor comercial sofre desvalorização constante
b) O nível de 80 metros foi atingido exatamente por ano. Veja a figura seguinte.
duas vezes até o ano 2000. valor (R$)
c) Após o ano de 2000, o nível da água da
barragem foi insuficiente para gerar energia.
d) No período de 1995 a 2000, o nível da água só
diminuiu.
0 20 tempo (anos)
20 - Considere a função f: IR → IR definida por
x 2 + x + 2, se x  0
 Esse veículo foi vendido pelo seu primeiro dono,
f ( x ) = 1, se 0  x  2 . Então, pode-se afirmar que
− x + 2, se x  2 após 5 anos de uso, por R$ 24.000,00. Sabendo-se

que o valor comercial do veículo atinge seu valor
o conjunto imagem dessa função é mínimo após 20 anos de uso, e que esse valor
a) {y  IR / y < 0} mínimo corresponde a 20% do valor que tinha
b) {y  IR / y < 0 ou y = 1 ou y > 2} quando era novo, então esse valor mínimo é, em
c) {y  IR / y < 0 ou y = 1 ou y  7 } reais,
4 a) menor que 4500
7
d) {y  IR / y = 1 ou y } b) maior que 4500 e menor que 7000
4
c) múltiplo de 7500
d) um número que NÃO divide 12000
21 - Analise e classifique as sentenças como V
(verdadeiras) ou F (falsas).
24 - Uma malharia familiar fabrica camisetas a
( ) f:  →  definida por f(x) = cos x é par. um custo de R$ 2,00 cada uma e tem uma
( ) f:  →  definida por f(x) = sen x é despesa fixa semanal de R$ 50,00. Se são
sobrejetora. vendidas x camisetas por semana, ao preço de
( ) f: [0, ] → [–1, 1] definida por y = cos x é  22 x 
 −  reais a unidade, então, o número de
inversível.  3 30 

235
PROVAS DA AFA 1998 – 2010
camisetas que deve ser vendido por semana para se 29 - Um aro circular de arame tem 5 cm de raio.
obter o maior lucro possível é Esse aro é cortado e o arame é estendido ao longo
a) 60 de uma polia circular de raio 24 cm. O valor do
b) 65 seno do ângulo central (agudo), que o arco
c) 80 formado pelo arame determina na polia é
d) 90 6− 2
a)
4
25 - O domínio da função real expressa pela lei b) 6+ 2

f (x) = x (x + 1) −1 − ( x − 1) −1 é x , tal que c) 6+ 2


  4
a) x < –1 ou 0  x < 1 6+ 2
b) –1 < x  0 ou x > 1 d)
2
c) x < –1 ou 0 < x < 1
d) –1 < x < 0 ou x > 1 30 - Em uma apresentação da esquadrilha da
fumaça, dois pilotos fizeram manobras em
26 - A curva abaixo representa o gráfico da momentos diferentes deixando rastros de fumaça,
função f definida por f (x ) = log a x . Se B e C têm conforme mostra a figura abaixo.
coordenadas respectivamente iguais a (2, 0) e (8, altura
0), e se a área do trapézio BCDE é igual a 6, então, f2
pode-se dizer que a área do triângulo ABE é 5
4
y D
E
3
2 f1
1
A B C 0 x 0 3 3 9 3 15 9 21 distância
8 4 8 2 8 4 8
a) um número irracional.
b) um número primo. As funções f1 e f2 que correspondem às manobras
c) um número quadrado perfeito. executadas pelos pilotos são
d) uma dízima periódica. 4  4 
a) f1(x ) = 2 − sen x  e f2 (x ) = 4 − sen x 
3  3 
27 - Sejam f e g funções definidas por 4  4 
b) f1(x ) = 2 + sen x  e f2 (x ) = 4 − sen x 
2
f ( x ) = x − 4x + 3 e g( x ) = log x +1 x . O domínio de 3  3 
 2   4 
(gof)(x) é o conjunto dos números reais x, tais que c) f1( x ) = 4 + sen − x  e f2 ( x ) = 2 − sen + x 
2 3  2 3 
a) 0 < x < 1 ou x > 3
2   4 
b) x < 1 ou x > 3 d) f1(x ) = 1 + sen x  e f2 ( x ) = 1 − 3 sen − x 
3  2 3 
c) 1 < x < 3 e x  0
d) x > 3
31 - Analise as alternativas seguintes e
28 - Todo número real positivo pode ser classifique-as como verdadeiras (V) ou falsas (F).
descrito na forma 10x. Tendo em vista que 2 = I- O período e o conjunto-imagem da função
100,30, então o expoente x, tal que 5 = 10x f: IR → IR definida por f (x) = 1 . sen x . cos x são,
4
vale, aproximadamente,  1 1
a) 0,15 respectivamente, 2 e − ,
4 4 

b) 0,33
II - A função y = 2 arc cos 4x tem por
c) 0,50
domínio o conjunto de todos os valores de x
d) 0,70  1
pertencentes a 0, 4 
 

236
PROVAS DA AFA 1998 – 2010
III - Para todo x     ,
− 2 , 2 
2
o valor de (tg x +
  Quantos metros de comprimento deverá ter o muro
1) . (sen2 x –1) é –1 que o proprietário do terreno II construirá para
A opção que corresponde à classificação acima é fechar o lado que faz frente com a rua B?
a) F – V – F a) 28 b) 29 c) 32 d) 35
b) V – V – F
c) F – F – V 36 - Na figura abaixo, os pontos A, B e C
d) V – F – V pertencem à circunferência de centro O e raio r. Se
 = 140 e  = 50, então, a área do triângulo BOC
32 - Ao saltar do avião que sobrevoa o ponto A é
(veja figura), um paraquedista cai e toca o solo no r 3
ponto V. Um observador que está em R contacta a a) 
2
equipe de resgate localizada em O. A distância, em 2 B
r 2
km, entre o ponto em que o paraquedista tocou o b)
3
solo e a equipe de resgate é igual a r 2 
a) 1,15 c) O
V O 1km A 9 C
b) 1,25 2 A
r 3
c) 1,35 d)
4
d) 1,75 
3 km

37 - Na figura abaixo, os triângulos ABC e
CDE são equiláteros. Se a razão entre as áreas
R
desses triângulos é 9 e o perímetro do menor é 12,
4
então, a área do quadrilátero ABDE é
B
33 - Uma dasraízes da equação (I) a) 2+ 3
3 2
4x − 12 x − x + m = 0 (m  IR) é a solução da b) 9 3 D

equação (II) tg πx = 1 no intervalo [0,]. Então, c) 11 − 3


12
d) 19 3
pode-se afirmar que o produto das raízes da A C E
equação (I) vale
a) − 1 b) − 1 c) − 2 d) − 3 38 - Considere as proposições a seguir:
3 2 5 4
I- Se dois planos são paralelos, então toda
34 - O conjunto dos valores reais de x que reta que é paralela a um deles é paralela ou está
tornam verdadeira a desigualdade cos2(x – )   é contida no outro.
a) {x  IR x  −  ou x   } II - Se uma reta é paralela a um plano, então é
paralela a todas as retas do plano.
b) {x  IR −   x   }
III - Se uma reta possui dois pontos distintos
c) IR
num plano, então ela está contida no plano.
d)
IV - Se dois planos são secantes, toda reta de
um, sempre intercepta o outro plano.
35 - No desenho abaixo, estão representados os
Pode-se afirmar que as proposições verdadeiras
terrenos I, II e III.
são
a) I e IV b) II e III c) I e III d) II e IV
RUA C

24 m 15 m 39 - A área total do sólido gerado pela rotação


RUA A

do polígono ABCDE em torno do eixo y, que


I II
III contém o lado AE, é, em m2, igual a
20 m
RUA B

237
PROVAS DA AFA 1998 – 2010
Dados:
reajustes no seu salário: 40% em janeiro e 30% em
y
a) 144 junho. Se, em julho de 2002, o aumento daquela
b) 150 D C AE = 2m prestação foi de 130%, que porcentagem de seu
c) 168 AB = 6m salário a pessoa passou a gastar?
d) 170 BC = 6m a) 29,7% c) 34,5%
E b) 32,7% d) 36,9%
CD = 3m
A B
03 - Dado o número complexo z tal que
z + 2 z − 9 = 3i , é correto afirmar que
 7 7 
40 - A figura seguinte representa uma pirâmide a) z = 3 10 b) z = 3 2  cos + i sen 
 4 4 
regular de base quadrada, onde M é o ponto médio
1+ i
de DE e CM pertence ao plano da base. Se c) z = 9 − 3i e) z −1 =
3
DE = 100 m , AB = 10 m , AC = 12 m e AM = 28 m , então,
1
o volume (em m3) de uma esfera cujo raio é da 04 - Analise as alternativas e marque a correta.
5
altura dessa pirâmide é igual a a) Dado o complexo z = m + mi, onde m * e i
é a unidade imaginária, pode-se dizer que o afixo
de ( z )2 é, em relação à origem, simétrico do afixo
(–2m2,0).
a) 4500 b) No plano de Argand-Gauss os complexos z, tais
b) 3375
B que |z – 1|  1, são representados pelos pontos do
E
c) 2200 círculo de centro (0, 1) e raio unitário.
d) 1125 M A
C
c) Se n  IN e i é a unidade imaginária, então (in+1
D + in)8 é um número real maior do que zero.
d) Se z = a + bi (a *, b   e i é a unidade
imaginária) é um complexo, então z − z é sempre
um número complexo imaginário puro.
AFA 2002/2003 – Matemática
05 - Uma P.A. cujo primeiro termo é zero e uma
01 - Hotel Fazenda B P.G. cujo primeiro termo é 1 possuem a mesma
Chalés com acomodação para até 10 pessoas. razão. O nono termo da P.G. é igual ao quadrado
Diária do Chalé: 80 reais do nono termo da P.A.. Então
Refeição opcional (14 reais por dia por pessoa) a) uma das razões comum é –2.
b) a razão comum é –1.
c) a razão comum é 1.
O Sr. Souza, esposa e filhos optaram pelo passeio d) não existem as duas progressões.
acima anunciado e, aproveitando as férias
escolares, passaram 5 dias hospedados no Hotel 06 - Considere uma P.G. onde o 1o termo é a, a
Fazenda B fazendo todas as refeições, gastando ao > 1, a razão é q, q > 1, e o produto dos seus
todo 1100 reais, dos quais 280 reais cobriram termos é c. Se loga b = 4, logq b = 2 e logc b
despesas com telefone, frigobar e lazer. = 0,01, então a soma dos termos da P.G. é
É correto afirmar que
a 41 − a a 40 − a a 41 − 1 40
−1
a) a família levou 6 filhos. a) 2
b) 2
c) 2
d) a 2
a −1 a −1 a −1 a −1
b) as despesas com refeições totalizaram 400 reais.
c) no chalé sobraram 4 acomodações.
d) se não tivessem ocorrido as despesas extras com 07 - Analise as proposições abaixo,
frigobar, telefone e lazer, eles poderiam ter ficado classificando-as em V (verdadeiro) ou F (falso):
mais 1 dia e teriam economizado ainda 120 reais. ( ) Se p( x ) = 2 x 3 − (p − 1) x + 4 e m( x ) = qx 3 + 2 + q
são polinômios idênticos, então p2 + q2 = 5
02 - Em julho de 2001, uma pessoa gastava ( ) Dividindo-se A ( x ) = x 3 + x 2 + x + 1 por B(x),
27,3% do seu salário com o pagamento da obtém-se o quociente C(x) = 1 + x e resto R(x) =
prestação da casa própria. Em 2002, houve dois C(x). Pode-se afirmar que B(x) é tal que B(0) = 0
238
PROVAS DA AFA 1998 – 2010
( ) Se f, g e h são polinômios de grau m, n e q (m, 12 - Em um balcão de supermercado, foram
n, q são naturais e m > n > q), então o grau de (f + esquecidas 2 sacolas. Uma continha 3 latas de
g) . h é dado por m + q atum, 2 latas de ervilha e 5 de sardinha; a outra, x
A seqüência correta é latas de atum, 3 latas de ervilha e 3 de sardinha.
a) F – V – V c) V – F – V Escolhe-se ao acaso uma sacola e retira-se uma
b) V – V – F d) V – V – V lata. Qual é o menor valor de x para que a
probabilidade de tratar-se de uma lata de atum
08 - Marque a alternativa correta. seja, no mínimo, 50%?
a) Se a unidade real é raiz de multiplicidade k da a) 13 c) 15
equação P(x) = 0, então P(x) é divisível por (x – b) 14 d) 16
1)m, com 0  m  k e m inteiro
b) A equação de coeficientes reais a0 + a1x + a2x2 + 13 - Sejam m e n números reais com m  n e as
a3x3 + a4x4 = 0, pode ter duas raízes NÃO reais 2 1 −1 1
matrizes A= , B=  . Para que a matriz
conjugadas se a0 = a1 = a3 = 0, a2 > 0 e a4 < 0 3 5  0 1
c) Se P(x) = 0 tem 1, 2 e 3 como raízes, e se P(x) é mA + nB seja NÃO inversível é necessário que
um polinômio não nulo de grau m, então m > 3 a) m e n sejam positivos.
d) Considerando i a unidade imaginária, se a b) m e n sejam negativos.
equação x2 + bx + c = 0 ,{ b, c }  C ,admite  + c) n + 7m = 0
i (   e   *) como raiz, necessariamente d) n2 = 7m2
admitirá também a raiz  – i
14 - O valor do determinante de uma matriz de
09 - Seja a > 1 e e a base dos logaritmos ordem n é 21. Se dividirmos a segunda linha desta
neperianos, o valor real de m para o qual a matriz por 7 e multiplicarmos a matriz por 3, o
equação x3 – 9x2 + (loge am + 8)x – loge am = 0 valor do novo determinante será
tenha raízes em progressão aritmética, é dado por a) 3n c) 3n
b) 3n+1 d) 3n + 3
a) m = loge a – 8 c) m = 15
loge a

b) m = loge a – 9 d) 9
m = − loge a
15 - A condição que deve ser satisfeita pelos
8 termos independentes a, b e c ( a, b e c  *) para
x + 2y − z = a

10 - Marque V para verdadeiro F para falso e, a que seja compatível o sistema  y + 2z = b é
seguir, assinale a opção correspondente. x + 3 y + z = c

( ) Sendo A um conjunto com x elementos e estabelecida por
B um conjunto com y elementos, o número de a) c – a + b = 0 c) c + a – b = 0
funções f: A → B é xy b) a + b + c = 0 d) a + b – c = 0
( ) Uma urna contém n bolas numeradas (de
1 a n). Se s bolas são retiradas sucessivamente e 16 - As quantidades dos produtos que Elaine,
com reposição, o número de seqüências de Pedro e Carla compraram num mercado estão
resultados possíveis é ns esquematizadas na tabela que segue
( ) Com n algarismos distintos, entre eles o produto A produto B produto C
zero, pode-se escrever n4 números distintos de 4 Elaine 1 2 3
algarismos. Pedro 3 6 2
a) F – V – V c) V – F – F Carla 2 4 1
b) V – F – V d) F – V – F Sabendo-se que Pedro gastou R$ 21,00 e Carla R$
13,00, pode-se concluir, necessariamente, que
11 - No desenvolvimento de (xr + x–r)n, a) Elaine gastou R$ 10,00.
ordenado pelas potências decrescentes de x, sendo b) o preço do produto C é R$ 3,00.
r > 0 e n natural, o coeficiente do 5o termo que é c) o preço do produto A é R$ 1,00.
independente de x é igual a d) o preço do produto B é R$ 3,00.
a) 252 c) 10
b) 70 d) 8 17 - Dadas as retas de equações
r : y = ax + b
239
PROVAS DA AFA 1998 – 2010
r1: y = a1x + b1 22 - Analise as proposições abaixo
determine a relação entre a, a1, b e b1 que está classificando-as em V (verdadeiro) ou F (falso),
correta. considerando funções reais.
a) Se a = a1 e b  b1 tem-se r // r1 ( ) O domínio e a imagem da função g definida
b) Se a = a1 e b = b1 tem-se r  r1 por g( x ) = 9 − x2 são, respectivamente, −3,3 e
c) Se a  a1 tem-se r = r1 0,+ 
d) Se a  a1 e b  b1 tem-se r  r1 ( ) Se f(x) = x2 e g(x) = f(x + m) – f(x) então g(2)
é igual a m(4 + m)
18 - Na figura abaixo, as retas r e s são ( ) Se h( x ) =
1
, então h–1(x) = h(x)
paralelas. Se P(x,y)  s, então x + y é igual a x
y A seqüência correta é
3
a) F – V – V c) V – F – V
b) F – V – F d) V – V – F

0 45 23 - Analise o gráfico abaixo das funções f e g e


x
marque a opção correta.
r y
a) 3 s c) − 6 f

b) − 3 d) 6
0 t x
19 - Considere as afirmativas abaixo: g
I) as retas r : x + y = 1 e s :  x = 2t + 1 são k
2 −3 y = 3t
perpendiculares.
II) a equação 4 x = y 2 representa uma parábola a) O gráfico da função h(x) = g(x) – f(x) é uma
reta ascendente.
com eixo de simetria horizontal.
b) O conjunto imagem da função s(x) = f(g(x)) é 
x2 y2
III) −
3

9
=1 representa uma hipérbole. c) f(x) . g(x)  0  x  t
É(são) correta(s) a(s) afirmativa(s) d) g(f(x)) = g(x)  x 
a) I, II e III. c) III somente.
b) I somente. d) II somente. 24 - Considere a função f: → tal que
 x − 1, se x  1
f(x) =  e assinale a alternativa
20 - A circunferência de equação 1 − x, se x  1
x 2 + y 2 − 8 x + 8 y + 16 = 0 e centro C é tangente ao eixo verdadeira.
das abscissas no ponto A e é tangente ao eixo das a) f é sobrejetora.
ordenadas no ponto B. A área do triângulo ABC b) f é par.
vale c) f não é par nem ímpar.
a) 4 c) 12 d) Se f é definida de  em  + , f é bijetora.
b) 8 d) 16
25 - Na figura abaixo, tem-se o gráfico da
21 - Sobre o triângulo PF1F2 onde P(2, 2) e F1 e função real f em que f(x) representa o preço, pago
x2 y2
em reais, de x quilogramas de um determinado
F2 são focos da elipse + = 1, é correto afirmar produto. (Considere f(x)  )
9 25
que f(x)
a) é isósceles.
b) é obtusângulo. 36
c) tem área igual a 16
30
d) tem perímetro igual a 2 2 +8
0 60 x

240
PROVAS DA AFA 1998 – 2010
De acordo com o gráfico, é INCORRETO afirmar 30 - "Na semana passada, a Secretaria
que Municipal de Saúde do Rio de Janeiro anunciou
a) o preço pago por 30 quilogramas do produto foi que 5000 bombeiros participarão da campanha de
R$ 18,00. combate à epidemia de dengue na cidade. É mais
b) com R$ 110,00, foi possível comprar 55 uma tentativa de deter o ritmo alucinante de
quilogramas do produto. crescimento da doença."
c) com R$ 36,00, foi possível comprar 72 Veja. 13 de março de 2002
quilogramas do produto. Suponha uma cidade com 128.000 habitantes e
d) com R$ 32,00, compra-se tanto 53,333... que, em determinada ocasião, fosse constatado que
quilogramas, quanto 64 quilogramas do produto. 8000 habitantes estavam com dengue. Num estudo
realizado, constatou-se que a taxa de aumento de
26 - Observe o gráfico da função f abaixo. pessoas contaminadas era de 50% ao mês. Com
y base nisso, pode-se afirmar que, caso não tomasse
nenhuma providência,
Dados: log 2 = 0,3 e log 3 = 0,48
a) toda população seria contaminada em dois
1 45 meses.
0 x b) em três meses, apenas 18.000 pessoas seriam
contaminadas.
c) 40.500 pessoas seriam contaminadas em quatro
meses.
Sabendo que f é definida por d) dez mil pessoas seriam contaminadas

ax 2 + bx + c, se x  1 exatamente na metade de um mês.
f(x) =  analise as alternativas e

px + k, se x  1
marque a opção correta. 31 - As duas polias da figura giram
a) ac < 0 c) p = –1 simultaneamente em torno de seus respectivos
centros O e O’, por estarem ligadas por uma
b) pk  0 d) ab > 0
correia inextensível.
27 - O conjunto {x  / f(x) < 0}, onde f: 3R
R
→ é definida por f(x) = ax2 + 2a2x + a3, com
O O’
a  IR *− , é
a) ]–; –a[ b) ]– ; –a[]–a; + [
c) ]– ; a[  ]a; + [ d) ]–  + [
Quantos graus deve girar a menor polia para que a
28 - Analise os itens abaixo classificando-os em maior dê uma volta completa?
V (verdadeiro) ou F (falso). a) 1080 b) 120º c) 720 d) 2160
( ) Em , o conjunto solução da inequação 8 .
(0,5)x – 1  0 é dado por [4, + [ 32 - Simplificando a expressão
( ) A função real y = e 1− x é crescente  x   3
sen
  3
− x  + cos(4  − x ) + tg

− x  , obtém-se uma nova
(considere e a base dos logaritmos neperianos)  2   2 
( ) Se f(x) = 2x, então f(a) . f(b) é sempre igual a expressão E. O conjunto domínio, o
f(a + b), onde a e b são reais quaisquer conjunto-imagem e o período da função f(x) = E
A seqüência correta é são, respectivamente,
a) F – F – V c) F – V – V a) {x  / x  k, k  IN}, , 
b) V – V – F d) V – F – F b) , [–1, 1], 2
c) {x  / x   + k, k  Z}, , 
29 - O conjunto-solução da equação 2
2
log x − 2 ( x + 2 ) = 2 é d) {x  / x  k, k  }, [–1, 1], 2
a)  c) {x  / 2 < x < 3}
b) {x  / x > 3} d) {x  / x > 2 e x  3}

241
PROVAS DA AFA 1998 – 2010
33 - Considere a função real definida por das figuras hachuradas está para a área do
y=
cos 2 x
e as seguintes afirmações: triângulo RST na razão
1 + sen 2 x R
I- A função é decrescente em todo seu
m n
domínio
II - O gráfico da função apresenta assíntotas S
T

nos arcos  + k , k  Z
p

2 q
 
III - A função é negativa em  0,  1 1 3
 4  a) b) c) 1 d)
  3 2 2
IV - A função admite inversa em  0, 
 2 
38 - Um poliedro platônico, cujas faces são
São verdadeiras somente as afirmações contidas
triangulares, tem 30 arestas. Determine o número
nos itens
de arestas que concorrem em cada vértice.
a) I e II b) II e III c) III e IV d) d) I e IV
a) 3 c) 4
b) 5 d) 6
3
34 - Dado que sen x + cos x = , tem-se que
3 39 - Seja P uma pirâmide cujo vértice é o centro
  de uma das faces de um cubo de aresta a e cuja
cos x −  vale
 4 base é a face oposta. Então, a área lateral dessa
− 1+ 3 2 6 6 pirâmide é igual a
a) b) − c) d)
2 3 3 6 a2 5
a) a2 5 b) 2a 2 3 c) a2 3 d)
4
35 - ABC é um triângulo retângulo em A e CX
é bissetriz do ângulo BCA, onde X é ponto do lado 40 - Na figura seguinte, tem-se uma esfera de
AB . A medida CX é 4 cm e a de BC , 24 cm. Sendo maior raio contida num cone reto e tangente ao
assim, a medida do lado AC , em centímetros, é plano da base do mesmo. Sabe-se que o raio da
igual a base e a altura desse cone são, respectivamente,
a) 3 c) 5 iguais a 6 cm e 8 cm. A metade do volume da
b) 4 d) 6 região do cone exterior à esfera é, em cm3, igual a
A
36 - Na figura, o triângulo AEC é equilátero e
ABCD é um quadrado de lado 2 cm. A distância
BE, em cm, vale r D

E
C
B
A B
a) 66 c) 30
b) 48 d) 18

D C AFA 2003/2004 – Matemática


a) 2 3 c) 3+ 2
b) 6 −1 d) 6− 2 1) Analise as proposições abaixo, classificando-as
em VERDADEIRA(S) ou FALSAS(S).
37 - Na figura, RST é um triângulo retângulo I – Se x  IR, então x 2 = x para x  0 ou x2 =
em S. Os arcos RnSpT, RmS e SqT são – x se x < 0.
semicircunferências cujos diâmetros são,
respectivamente, RT, SR e ST. A soma das áreas

242
PROVAS DA AFA 1998 – 2010
II – Se a e b são números reais, a > 0, b > 0, p > 1 1º) perde-se a quantia x apostada;
2º) recebe-se a quantia 2x.
a + bp 2 a
e > p, então
a+b b > p.
Uma pessoa jogou 21 vezes da seguinte maneira:
III – Se um mesmo serviço pode ser feito pelo na 1ª vez, apostou 1 centavo; na 2ª vez, apostou 2
operário A em 8 horas e por B em 12 horas, centavos; na 3ª vez, apostou 4 centavos e assim por
quando operam separadamente, então durante 3 diante, apostando em cada vez o dobro do que
horas, trabalhando juntos, executam uma parte havia apostado na vez anterior. Nas 20 primeiras
correspondente a 62,5% do serviço. vezes, ela perdeu. Na 21ª vez, ela ganhou.
Comparando a quantia total T perdida e a quantia
Tem-se a seqüência correta: Q recebida, tem-se que Q é igual a
a) V, F, V b) V, F, F T
c) F, F, V d) V, V, V a) b) 2T c) 2(T + 1) d) T + 1
2
2) No conjunto universo S dado por
5) Sendo P(x) = x + 3x3 + 5x5 + 7x7 + 9x9 + … +
S = {(x, y)  IR x IR | 0  x  1 e 0  y  1}, é 999x999, o resto da divisão de P(x) por (x – 1) é
definido o subconjunto M = {(x, y)  IR x IR | 0  a) 249.500 b) 250.000
1
x  1 e 0  y  . c) 250.500 d) 251.000
2
M 6) A equação x3 + mx2 + 2x + n = 0, onde m e n
Pode-se afirmar que CS é igual a são números reais e i2 = –1, admite 1 + i como
1 raiz. Então m + n é igual a
a) {(x, y)IR x IR | 0 < x < 1 e < y < 1} a) – 2 b) 0 c) 1 d) 2
2
1 1
b) {(x, y)IR x IR | 0 < x  e  y  1} 7) Se você vai comprar algo que custa cinqüenta e
2 2
cinco centavos, em uma máquina automática, e
1 1
c) {(x, y)IR x IR | <x1e0y } dispõe de oito moedas de cinco centavos do
2 2 mesmo modelo e cinco de dez centavos também do
1 mesmo modelo, então, existem n seqüências
d) {(x, y)IR x IR | 0  x  1 e < y  1}
2 possíveis de introduzir as moedas, totalizando
cinqüenta e cinco centavos. O valor de n é
3) Analise as sentenças abaixo, classificando-as a) 133 b) 127 c) 24 d) 4
em VERDADEIRAS(S) ou FALSAS(S),
considerando i = − 1 . A seguir, assinale a 8) Sabendo-se que no desenvolvimento de (1 +
alternativa que apresenta a seqüência correta. x)26 os coeficientes dos termos de ordem (2r + 1) e
I – A representação geométrica dos números (r + 3) são iguais, pode-se afirmar que r é igual a
complexos z tais que |z – (1 – i)|  2 é um círculo a) 8 ou 4 b) 8 ou 2
de centro C(1, –1) e raio 2. c) 4 ou 2 d) 2 ou 1
1+ i
II – A forma trigonométrica de z = é z = 9) Em uma urna contendo 12 bolas amarelas, 15
i bolas brancas e 18 bolas pretas, a probabilidade de
 7 7  retirar três bolas de cores diferentes é
2  cos + i sen  . 1
 4 4  a) 38% b) 22,8% c) 11,4% d)
III – Se z = cos  + i sen , então z.z = – i2,   376
 IR.
10) Se A = (aij)2x3 e B = (bij)3x4, a expressão para
a) V, V, V b) V, V, F
encontrar o elemento c23, onde AB = (cij), é igual a
c) F, F, V d) V, F, V
a) a21b31 + a22b32 + a23b33
b) a31b11 + a32b21 + a33b31
4) Num certo jogo de azar, apostando-se uma
c) a21b13 + a22b23 + a23b33
quantia x, tem-se uma das duas possibilidades
d) a23b32
seguintes:
243
PROVAS DA AFA 1998 – 2010
11) O determinante associado a matriz 14) Na figura abaixo tem-se a representação
1 0 0  gráfica da função real f (x) = 2 sen
x
para x[a,
3 sen a 1  é igual ao menor valor da 2
 g].
4 1 2 sen a 
função y = x2 – 2x + 1. então, o maior valor de a
no intervalo [0, 2] é
 5 3 7
a) b) c) d)
6 6 4 4

12) Analise as proposições abaixo, classificando-as


em VERDADEIRAS(S) ou FALSAS(S).
 x+y=0

I – O sistema linear  x + z = 0 é indeterminado É correto afirmar que o baricentro do triângulo
 y + mz = 0 DEF é o ponto

para m = –1 e uma de suas soluções é a terna   1   2  1  2
a)  ,  b)  ,  c)  ,  d)  , 
ordenada (–1, 1, 1).  2 3  2 3  3  3
(m + 1) x + 7 y = 10
II – Para que o sistema  seja 15) A equação (x + y)(x – y) = 1 representa
 4x + (m − 2) y = 0
impossível deve-se ter m = –5, somente. a) uma hipérbole com excentricidade e = 2
III – Na equação matricial b) duas retas perpendiculares entre si
x − 1 y + 2  1 − 1  3 0 c) uma elipse com centro na origem
 z =
x + y + z  0 1  − 2 5
. a soma x + d) uma hipérbole cuja distância focal é igual a 2

y + z é igual a 3. 16) Com relação ao conjunto de pontos P(x,y)
eqüidistantes da reta y = 3 e da origem do sistema
Tem-se a seqüência correta: cartesiano ortogonal, é INCORRETO afirmar que
a) V, V, F b) F, V, F c) V, F, V d) F, F, V é uma curva
a) representada por x2 – 6y – 9 = 0
13) Os pontos A (0,0) e B (3,0) são vértices b) cujas coordenadas do vértice tem soma igual a
consecutivos de um paralelogramo ABCD situado 1,5
no primeiro quadrante. O lado AD é perpendicular c) que representa uma função par
a reta y = –2x e o ponto D pertence a d) cujo parâmetro é igual a 3
circunferência de centro na origem e raio 5 .
Então, a diagonal AC mede 17) Considere as funções reais
(fog) (x) = 4x2 – 6x – 1 se x  1
4x + x se x < 1
e g(x) = 2x –3
Com base nessas funções classifique as afirmativas
abaixo em VERDADEIRA(S) ou FALSA(S).
I) f (x) é par.
II) f (x) admite inversa em todo seu domínio.
III) f (x) é crescente em {x  IR | x < –1 ou x  –
1}
IV) se x < – 6 então f (x) > – 3
a) 38 b) 37 c) 34 d) 26
A seqüência correta é
a) V, V, F, V b) F, F, V, F
c) F, F, V, V d) F, V, V, F

244
PROVAS DA AFA 1998 – 2010
18) Se a função f: IR → IR definida por f (x) = ax II) Se f (x) = x + 2 x + sen ,   [0,2], é
2

– 1,  IR*, for crescente e f(f(4)) = 32, então pode-  5


se afirmar que a mesma positiva  x  IR, então <  <
6 6
a) é positiva para x < 0 III) O gráfico de f (x) = sen (arc sen x) é uma reta
1
b) é negativa para x <
3 A seqüência correta é
c) é nula para x = 3 a) V, V, F b) F, V, F
2 c) F, V, V d) V, F, V
d) admite o valor – quando x = –1
3
23) Seja f: D → IR, definida por f (x) =
19) Seja f (x) = ax + bx + c (a  0) uma função
2 cos x 1 + sen x
+ . O gráfico que MELHOR
real definida para todo número real. Sabendo-se 1 + sen x cos x
que existem dois números x1 e x2, distintos, tais representa um período completo da função f é
que f (x1).f (x2) < 0, pode-se afirmar que a)
a) f passa necessariamente por um máximo.
b) f passa necessariamente por um mínimo.
c) x1.x2 é necessariamente negativo.
d) b2 – 4ac > 0

20) O gráfico abaixo expressa a variação de log y


em função de log x, onde log é o logaritmo na base
decimal.

b)

A relação correta entre x e y é igual a


a) y = 2 + 2x b) y = 3 + x
2
5
c) y = 100x2 d) y = +x c)
2

21) Todos os valores reais de x para os quais existe


f (x) = x 4 x −1 − x são tais que
1
a) x > 1 b) 0 < x  ou x  1
2
1 1
c) 0 < x < d) 0 < x < ou x > 1
2 2

22) Analise os itens abaixo classificando-os como


VERDADEIRO(S) ou FALSO(S).
1
I) Se sen x + cos x = , então sen 2x = –0,666...
3

245
PROVAS DA AFA 1998 – 2010
d)
27) Assinale a única alternativa FALSA
a) Se um plano  é perpendicular a um plano ,
então existem infinitas retas contidas em  e
perpendiculares a .
b) Se  e  são planos perpendiculares entre si e 
é um plano perpendicular à reta comum a  e ,
então pode-se afirmar que as retas r, r =    e s,
s =   , são perpendiculares entre si.
c) Se duas retas r e s são reversas, então não
existem dois planos  e , perpendiculares entre si,
tais que r   e s  .
d) Duas retas do espaço, paralelas a uma terceira,
são paralelas entre si.
24) Um passageiro em um avião voando a 10,5 km
de altura avista duas cidades à esquerda da
28) Uma pirâmide regular de 6 faces laterais tem
aeronave. Os ângulos de depressão em relação às
sua base inscrita num círculo de raio R. sabendo-se
cidades são 30º e 75º conforme a figura abaixo. A
que suas arestas laterais tem comprimento L, então
distância, em km, entre os prédios A e B situados
o volume dessa pirâmide é
nessas cidades é igual a
R2 2
a) R 3(L − R ) b)
2 2 2
L − R2
2
2
R R2
c) 2(L2 − R 2 ) d) 3(L2 − R 2 )
3 2

29) Uma esfera de 10cm de raio e um cone reto de


10 cm de raio da base e altura 20 cm, estão
a) 21( 3 –1) b)
21
( 3 –1) situados sobre um plano . A distância x, de um
2 plano  paralelo ao plano , tal que as áreas das
21 secções obtidas pela intercessão do plano  com os
c) 3 d) 3–1
2 sólidos, esfera e cone, sejam iguais, é, em cm,
igual a
25) Um trapézio  tem por bases 80m e 60m e por a) 1 b) 2 c) 4 d) 6
altura 24m. a 6m da maior base, traça-se uma
paralela situada entre as duas bases do trapézio , 30) Assinale a alternativa que preenche
determinando, assim, dois outros trapézios  e . O corretamente a lacuna abaixo.
O volume do sólido gerado pela rotação de 360º da
módulo da diferença entre as áreas dos trapézios 
região hachurada da figura em torno do eixo é de
e  é, em m2, igual a
______ cm3.
a) 700 b) 750 c) 820 d) 950
a) 230
26) Seja PQ tangente à circunferência de centro O
224
e raio r. Se C Q = r, pode-se afirmar que b)
3
P Q + P C é igual a c) 374
608
d)
a) r + 3 3
b) 2r + r 3
c) r 3
d) r + r 3

246
PROVAS DA AFA 1998 – 2010
AFA 2004/2005 – Matemática b) Toda equação polinomial de grau n admite, no
máximo, n raízes reais.
1) Considere um subconjunto A contido em IR* e c) Toda equação polinomial de grau n admite
constituído por y elementos dos quais exatamente n raízes complexas.
a) 13 são múltiplos de 4 d) Se  e  são números reais positivos e a
b) 7 são múltiplos de 10 equação x3 – x2 + x –  = 0 admite duas raízes
c) 5 são múltiplos de 20 e simétricas, então todas as suas raízes são reais.
d) 9 são números ímpares.
7) Três crianças A, B e C vão dividir entre si 450
É correto dizer que y é um número balas da seguinte maneira: A recebe uma bala; B,
a) par menor que 19 c) primo maior que 21 duas e C, três. Repetindo-se o processo, A recebe
b) ímpar entre 10 e 20 d) múltiplo de 12 quatro balas; B, cinco e C, seis e, novamente, A
recebe sete e assim por diante, até que não haja
mais balas para continuar o processo. A criança
2) Seja: A = {x  IN* / 24
= n e n  IN}
x seguinte, então, receberá as balas restantes.
3x + 4
Seja: B = {x  IN+ / – 1  0}
2x + 9 Com base nessas informações, é correto afirmar
É correto afirmar que que
a) B – A = {0} c) A  B a) o número de balas restantes foi 29 e quem
b) A  B tem 8 elementos d) A  B = B recebeu foi a criança B.
b) as crianças A e C, juntas, receberam 300 balas.
3) Considere P1, P2, P3, P4, ..., Pn os n primeiros c) a criança B recebeu 10 balas a mais que a
números naturais primos consecutivos com n  5. criança A.
Se x = P1 . P22 . P33 . P44 . ... . Pnn e y = P1.P2.P3.P4 . ... . d) o maior número de balas que uma criança
Pn, então o número total de divisores positivos de recebe antes da conclusão do processo é 15.
x
é dado por 8) Uma prova consta de 3 partes, cada uma com 5
y
a) (n + 1)! b) n! c) n! + 1 d) (n – 1)! questões. Cada questão, independentemente da
parte a que pertença, vale 1 ponto, sendo o critério
 de correção “certo ou errado”. O número de
4) Considere i2 = –1 e   [0, 2],  e 
3
maneiras diferentes de se alcançar 10 pontos nessa
2 2
Se z = tg  + i, então a soma dos valores de  prova, se devem ser resolvidas pelo menos 3
para os quais |z| = 2 é igual a questões de cada parte e 10 questões no total, é
a) 2 b) 3 c) 4 d) 5 igual a
a) 75 b) 150 c) 1500 d) 1600
5) Considere o número complexo z tal que
z + z = 2 − i , onde i = − 1 e identifique entre as
9) Dentro de uma caixa há nove etiquetas. Cada
etiqueta recebe um número de 01 a 09, sem repetir
opções abaixo, as que são corretas. nenhum. Retira-se três delas, uma a uma, sem
(01) O afixo de z é ponto do 1o quadrante. reposição. A probabilidade de que os três números
1002
 3 correspondentes às etiquetas retiradas sejam, nesta
z − é real positivo.
(02)  4 ordem, ÍMPAR – PAR – ÍMPAR ou PAR –
(04) O menor inteiro positivo n para o qual ÍMPAR – PAR é de
1 5
 1
n
c) 20 d) 5
z +  é real negativo pertence ao intervalo ]2, 5[ a) 28 b) 18 81 36
 4
A soma das opções corretas é igual a
a) 6 b) 5 c) 3 d) 2 10) Analise as afirmativas abaixo e classifique-as
em (V) verdadeiras ou (F) falsas.
6) Escolha a opção INCORRETA. ( ) No desenvolvimento de (2x + k )7 , k  IN*, o
a) O polinômio P(x) = x5 – 12x4 + 3 x2 – 1 tem coeficiente numérico do termo em x4 é quatro
pelo menos uma raiz real.

247
PROVAS DA AFA 1998 – 2010
3
vezes o coeficiente numérico do termo em x . a) A e B são disjuntos se m = − 3 2
Então k vale 1 b) A  B   se m  3 2
4
c) A é subconjunto de B se |m|  3 2
( ) Sejam m e p números inteiros positivos, tais d) A e B nunca terão apenas um ponto comum.
 m − 1   m − 1  m 
que m −1 p . Então,   +   +   é igual a
 p − 2   p − 1  p  15) Analise as proposições abaixo, classificando-as
 m + 1 em (V) verdadeiras ou (F) falsas.
 
 p  ( ) Considere a circunferência  e a hipérbole 2y2
n n  n  n – x2 = 8 tendo mesmo centro. Se  passa pelos
( ) Se   +   +   + ... +   = 1023 , o valor de n é
1   2   3  n focos da hipérbole, uma de suas equações é x2 + y2
igual a 10 = 12
A seqüência correta é ( ) Numa hipérbole equilátera, uma das assíntotas
a) V, V, V c) V, F, F 2
b) F, F, V d) F, V, V tem coeficiente angular igual a
2
( ) A excentricidade da elipse x2 + 4y2 = 4 é igual

11) Considere   [0,2[ e   + k , k  IN e ln a 3
2
2
x, o logaritmo neperiano de x (x > 0). Calcule o
ln x
ln x sen( − ) Tem-se a seqüência
valor do determinante D = sec 
sen  cos(− )
a) V, F, V c) F, V, F
b) F, F, V d) V, V, F
É correto afirmar que o valor de D 16) Seja f a função real cujo gráfico se apresenta a
a) depende do ângulo  seguir:
b) nunca será nulo.
c) será positivo xIR+*
d) será negativo se 0  x  1

2x − 4 y + 10z = 6
12) Considere o sistema  em que m
3 x − 6 y + mz = n
e n são números reais e x, y e z são incógnitas.
Para que este sistema seja possível e
indeterminado deve-se ter
a) m = 15 e n = 9 c) m  15 e n  9 Analisando o gráfico, é INCORRETO afirmar
b) m = 15 e n qualquer d) m  15 e n = 9 que
a) f(f(1)) = f(0,5)
13) Considere duas circunferências de mesmo raio, b) f(x) + 1  0,  x  IR
sendo x2 + y2 – 4x – 8y + 4 = 0 a equação da c) f(0)  f(x),  x  IR
primeira e C2(4,2), o centro da segunda. Se a reta s 5
d) se g(x) = f(x) – 1, então g( −2) = f  
contém uma corda comum a ambas 2
circunferências, é FALSO que s
a) é perpendicular à bissetriz dos quadrantes pares. 17) Observe os gráficos abaixo, das funções f e g,
b) tem declividade positiva. definidas no intervalo [0,1]
c) admite equação na forma segmentária.
d) tem coeficiente linear nulo.

14) Dados os conjuntos A e B, tais que


A = {(x, y)  IR2 / x2 + y2  9} e
B = {(x, y)  IR2 / x – y  m, m  IR}

É correto afirmar que

248
PROVAS DA AFA 1998 – 2010
Com base nos gráficos, assinale a alternativa
FALSA. 1, se 0  x  2
20) Considere a função f(x) = 
a) g(f(0,4))  g(f(x)),  x  [0,1] − 2, se − 2  x  0
b) g(f(0,05))  g(f(0,1)) A função g(x) = |f(x)| –1 terá o seguinte gráfico:
c) g(g(x)) = x,  x  [0,3; 0,8]
d) g(f(0,6))  g(f(1)) a) c)

18) Dada a função real f definida por f(x) = x 2 ,


considere a função real g definida por g(x) =
f(x+m) + k, sendo m e k  R.

É INCORRETO afirmar que


a) o gráfico da função g em relação ao gráfico da
função f é deslocado k unidades para cima, se k > b) d)
0, e m unidades para a direita, se m < 0
b) a equação do eixo de simetria da parábola que
representa g é dada por x = m
c) se m = 0 e k = 1, então o conjunto imagem de g
é dado por Im = {y  IR / y  1}
d) se m = –2 e k = –3, então as coordenadas do
vértice da parábola que representa g são (– m, k)
21) Os valores de x que satisfazem a equação
19) Considere as funções f, g e h, todas de domínio
IxI + 1 + IxI = 2
a, b e contra-domínio c, d , representadas através
têm produto igual a
dos gráficos abaixo.
a) − 81 b) − 27 c) − 9
d) − 3
256 64 16 4

22) A soma dos números inteiros que satisfazem a


sentença 3  |2x – 3|  6 é um número
a) ímpar. c) divisível por 3
b) primo. d) que é divisor de 7

23) Uma casa que custa R$ 50.000,00 à vista pode


ser comprada conforme um dos dois
financiamentos abaixo:
I – 50% de entrada e o restante, ao final de 2
meses, com juros compostos de 5% ao mês.
II – R$ 20.000,00 de entrada e uma parcela de
R$ 36.300,00, ao final de x meses com juros
compostos de 10% ao mês.

De acordo com a situação acima, é FALSO


Com base nos gráficos, é correto afirmar que afirmar que
a) f é uma sobrejeção, g não é uma injeção, h é a) o financiamento I é mais vantajoso que o
uma sobrejeção. financiamento II
b) f é uma sobrejeção, g é uma injeção, h não é b) o valor pago a prazo no financiamento II
uma sobrejeção. corresponde a 72,6% do preço da casa à vista.
c) f é uma injeção, g não é uma sobrejeção, h é c) o valor dos juros do financiamento I,
uma bijeção. corresponde a 5,2% do valor da casa à vista.
d) f é uma bijeção, g não é uma injeção, h não é d) quem optar pelo financiamento II pagará a
uma sobrejeção. parcela de R$ 36.300,00 ao final de dois meses.
249
PROVAS DA AFA 1998 – 2010
e GF paralelos a AB e AC, respectivamente,
24) O domínio da função real definida por f(x) conforme a figura abaixo. O perímetro do triângulo
= x1 + loga x − a2x é GEF é um número que, escrito na forma de fração
irredutível, tem a soma do numerador com o
a) a 2  x  a− 2 se 0 < a < 1 denominador igual a
b) 0 < x  a− 2 ou x  a 2 se 0 < a < 1
c) a 2  x  a− 2 se a  1
d) x < a − 2 ou x  a 2 se a  1

25) Sabendo que o gráfico abaixo é da função y =


a + sen bx, pode-se afirmar que a + b é um número

a) 43 b) 40 c) 38 d) 35

29) Um recipiente no formato de uma superfície de


um cone circular reto, conforme figura, tem a sua
superfície lateral desenvolvida em um semicírculo
de área igual a 18 cm2. Se tal recipiente, em seu
interior, armazena um líquido até os 2 de sua
3
altura, pode-se dizer que o volume do líquido
armazenado, em cm3, é igual a

a) par. c) divisor de 18 2 3
b) primo. d) múltiplo de 7 a)
3
b) 2 3

26) Sabendo que 0  x  , analise as proposições 8 3
2 c)
3
e classifique-as como verdadeiras (V) ou falsas
(F). d) 8 3

( ) Se  + x = 2 , então, tg x = –tg 

( ) Se +x = , então, sec x = cossec 
2 30) A diagonal de um paralelepípedo reto
( ) Sendo sen   − x  = 3 , então cos ( − x ) =3 retângulo mede 3 35 cm e suas dimensões são
2  5 5 
proporcionais a 1, 3 e 5. A fração irredutível

( ) A função f(x) = sen  x −   + 2 é idêntica a
 2 que representa a razão entre a área total do
função g(x) = 2 – cos x paralelepípedo e seu volume é tal que
a)  e  são dois números primos.
Tem-se a seqüência b)  +  = 100
a) V, V, V, V. c) F, V, F, F. c)  −  = 11
b) V, F, F, F. d) V, V, F, V. d)  –  = –1
27) Considere m a raiz da equação
cos2x + 3sen 2 x – senx – 3 = 0 no intervalo 0, 2 .
AFA 2005/2006 – Matemática
O número cotg m – sec 2m é
3
a) 0 b) –1 c) 1 d) − 1)Considere o número complexo Z= 1 − i 3
e
3
2 2
calcule zn. No conjunto formado pelos quatro
28) Considere o triângulo ABC, de lados AB = 15 , menores valores naturais de n para os quais zn é
AC = 10 , BC = 12 e seu baricentro G. Traçam-se GE um número real,
250
PROVAS DA AFA 1998 – 2010
a) existem números que estão em progressão d) todas são verdadeiras.
aritmética de razão igual a 4
b) há elementos cuja soma é igual a 30 5) O conjunto solução S de P(x) = 0, possui 3
c) existe um único número ímpar. elementos. Sabendo-se que P(x) = x6 – mx4 + 16x3,
d) existe apenas um elemento que é número primo. onde m  IR, assinale a alternativa
INCORRETA.
2) Analise as afirmativas abaixo referentes aos a) O número m é múltiplo de 3.
3 i b) Os elementos de S formam uma progressão
números complexos z = + e w = 1 – i. aritmética.
2 2
c) S é constituído só de números pares.
d) R(x), resto da divisão de P(x) por (x – 1), é um
01- |z|.w10 é um número imaginário puro.
polinômio de grau zero.
1 1
02- O afixo de w-1 é o ponto  , 
2 2
6) Com base no conhecimento sobre análise
04- A forma trigonométrica de
combinatória, é correto afirmar que.
11 11
z = cos + isen 01 - existem 2160 possibilidades de 8 pessoas
6 6 ocuparem um veículo com 3 lugares voltados para
08 - As raízes quartas de w são vértices de um trás e 5 lugares voltados para frente, sendo que 2
quadrado inscrito numa circunferência de centro na das pessoas preferem bancos voltados para trás, 3
origem e raio r = 4 2 delas preferem bancos voltados para frente e as
demais não têm preferência.
Somando-se os números associados às afirmativas 04 - com os algarismos 0, 1, 2, 3, 4 e 5, pode-se
verdadeiras obtém-se um total t, tal que formar 525 números ímpares com 4 algarismos e
a) t  [1, 4] c) t  [9, 12] que não tenham zeros consecutivos.
b) t  [5, 8] d) t  [13, 15] 08 - podem ser formados 330 paralelogramos a
partir de 7 retas paralelas entre si, interceptadas
3) São dadas uma progressão aritmética e uma por outras 4 retas paralelas entre si.
progressão geométrica alternante com primeiro A soma das alternativas corretas é
termo igual a 1. Multiplicando-se os termos a) 05 c) 12
correspondentes das duas seqüências obtém-se a b) 09 d) 13
seqüência (–1, 1, 3, ...). A soma dos 5 primeiros
termos desta seqüência é 7) Os três primeiros coeficientes do
a) 61 c) 103 n
 2 1 
b) 97 d) 111 desenvolvimento de x +  segundo as
 2x 
4) Analise as proposições abaixo classificando-as potências decrescentes de x estão em progressão
em (V) verdadeira(s) ou (F) falsa(s). aritmética. O valor de n é um número.
a) primo.
( ) O resto da divisão de P(x) = 5x2n – 4x 22n+1 – 2 b) quadrado perfeito.
(n  IN) por x + 1 varia de acordo com o valor de c) cubo perfeito.
n d) maior que 9 e menor que 15
( ) Se P(x) + xP(3 – x) = x2 + 1, então P(3) = 13
8) Numa caixa existem 6 canetas pretas, 4 azuis e
3 2
( ) Se 1 + i é raiz de P(x) = x + bx + cx + d, sendo 3 vermelhas. Se 3 canetas são retiradas ao acaso, e
sem reposição, a probabilidade de que pelo menos
{b, c, d}  IR, então uma das raízes tem forma
duas tenham cores distintas é.
 3 3 
trigonométrica igual a 2  cos + isen  .
 4 4 
Tem-se que
a) todas são falsas.
b) apenas duas são falsas.
c) apenas uma é falsa.

251
PROVAS DA AFA 1998 – 2010
( ) Se m = 0, as retas r, s e t determinam um
9) Analise as sentenças abaixo: triângulo retângulo.
I) Seja a matriz A = (aij) 3x3 definida por ( ) As retas r e s poderão ser retas suportes de lados
opostos de um paralelogramo se m = –1,5

A seqüência correta é
O elemento da terceira linha e segunda coluna da a) F – V – F – F c) V – F – F – V
matriz transposta de A é 8. b) V – V – V – F d) F – V – V – V
II) Seja a matriz B = A – At (At é transposta de A),
onde A é uma matriz quadrada de ordem n. Então, 13) Um cursinho tem representado na figura
a diagonal principal de B é nula. abaixo o seu logotipo que é contornado por um
 1 sen  triângulo equilátero ABC, cujo baricentro é o
III) A matriz A =   é inversível se
 sen 1  ponto P
 
 0, 3  .
 3 
 
  + k , k  Z. No interior desse3 triângulo há o quadrado DEFG
2
inscrito na circunferência 1 e, ao mesmo tempo,
circunscrito à circunferência 2. Considerando os
dados acima, classifique as alternativas abaixo em
IV) Se a matriz é simétrica, (V) verdadeira(s) ou (F) falsa(s).
então o produto dos elementos de sua diagonal
principal é igual a 36.

É(são) FALSA(S) apenas


a) I e III. c) IV.
b) II e IV. d) I e II.

2 3
10) Sendo então. ( ) A equação geral de 1 é x2 + y2 - 3 = 0.
a) x = 3y c) y = –3x
b) x = –27y d) y = 27x ( ) A coroa circular sombreada na figura pode ser
representada pelo conjunto de pontos Q (x, y), tais
11) (x, y, z) são as soluções do sistema

que .
Se x, y e z formam, nesta ordem, uma progressão
aritmética, então a razão dessa progressão
( ) A reta suporte que contém o segmento BC
aritmética é igual a.
pode ser representada por y = − 3x + 3 .
A seqüência correta é
a) V - V - V c) F - V - V
b) V - F - V d) V - V – F
12) Considerando no plano cartesiano ortogonal as 14) Considere o sistema cartesiano ortogonal e as
x = 2 v + 3 opções abaixo. Marque a FALSA.
retas r, s e t, tais que (r)  , (s) mx + y +
 y = 3v − 2
m = 0 e (t) x = 0, analise as proposições abaixo, a) A medida de um dos eixos da elipse de equação
classificando- as em (V) verdadeira(s) ou (F) x2 + 4y2 = 1 é a quarta parte da medida do outro.
falsa(s).
( )  m  IR / r = s
( )  m  IR / s ⊥ t
252
PROVAS DA AFA 1998 – 2010
b) As retas de equação y = mx representam as 16) Com relação às funções reais f, g e h, cujos
x 2 y2 gráficos estão representados abaixo, assinale a
assíntotas da curva − =1 se, e somente se, ms
16 25 alternativa INCORRETA.
5
= .
4
c) As circunferências x2 + y2 – 2x = 0 e x2 + y2 +
4x = 0 são tangentes exteriormente.
d) A equação x – y2 = 0 representa uma parábola
cuja reta diretriz não tem coeficiente angular
definido.

15) Sabe-se que 100 g de soja seca contém 39 g de


proteínas e que 100 g de lentilha seca contém 26 g
de proteínas. Homens de estatura média, vivendo
em clima moderado, necessitam de 65 g de
proteínas em sua alimentação diária. Suponha que
um homem queira nutrir-se com esses 65 g de a) Se x é tal que 3  x  5, então f(x)  g(x)  h(x)
proteínas alimentando-se de soja e/ou lentilha. Seja b) Se x é tal que –1  x  1 então g(x) h(x)  f(x)
x a quantidade diária de soja e y a quantidade 2
1
diária de lentilha, x e y positivos e medidos em c) Se x é tal que  x  3, então g(x)  f(x)  h(x)
2
porções de 100 g.
d) Se x é tal que - 5  x  4, então f(x).g(x).h(x)
2
É INCORRETO afirmar que 0
a) a relação estabelecida entre x e y é 3x + 2y = 5
b) se um homem deseja adquirir pelo menos 65 g 17) Dada a função real f definida
de x
proteínas, tem-se que y  –1,5x + 2,5 f (x) = , se D = [a, b] é o domínio
c) o esboço do gráfico que melhor representa o x −4 + 8−x
consumo mínimo de soja e/ou lentilha que um de f e Im = [c, d] é o conjunto imagem de f, então,
homem precisa é pode-se dizer que
a) se Im – D = [m, n), então m – n = –2
b) se D – Im = ]p, q], então p + q = 10
c) c + d = 2
d) ab = 36

18) Dadas as funções reais f e g definidas por f(x)


x
= x 2 − 5x + 6 e g ( x ) = ,sabendo-se que
x
existe (gof)(x), pode-se afirmar que o domínio de
d) o esboço do gráfico que representa as possíveis gof é
combinações de tais alimentos para fornecer pelo a) IR – ]2, 3[ c) IR – {2, 3}
menos a quantidade de proteínas requerida é b) IR – [2, 3] d) IR* – [2, 3]

19) Dois irmãos, Pedro e Paulo, sem nenhuma


renda, ganharam uma bolsa de estudos por 1 ano,
sendo que cada um receberá x reais por mês.
Fizeram, então, uma previsão de despesas e Pedro
concluiu que pode economizar mensalmente 2
7
do valor de sua bolsa. Já Paulo, que gastará por
mês R$ 300,00 a mais que Pedro, acumulará uma
dívida de R$ 1680,00 ao fim do ano. Pedro, então,
propõe ao irmão ajudá-lo todo mês com metade do
253
PROVAS DA AFA 1998 – 2010
que economizaria mensalmente. Baseado nisso, é
correto afirmar que.
a) o valor de x não chega a R$ 500,00 por mês.
b) Paulo gasta por mês exatamente 120% do valor
de sua bolsa.
c) contando apenas com a ajuda de Pedro, Paulo
não conseguirá pagar todas as suas despesas.
d) Pedro pretende guardar, ao final dos 12 meses,
R$ 980,00, mesmo ajudando o irmão.

20) Considere no sistema cartesiano ortogonal o


triângulo de vértices A(0, 3), B(0, –2) e C(3, 0).
Neste triângulo ABC estão inscritos diversos 24) Considere as funções reais f e g definidas por
retângulos com base no eixo das ordenadas. Em f(x) = log3 x e g(x) = f(x + 1) Sabendo-se que
relação ao retângulo de maior área, é existem f – 1 e g – 1, é correto afirmar que o
INCORRETO afirmar que o mesmo possui conjunto solução da equação g – 1(x) + f – 1(x) = 2
a) altura e base proporcionais a 3 e 5 é.
b) perímetro representado por um número inteiro. a) {1} c) {log3 2 – 1}
c) área maior que 4 b)  d) {1 – log3 2}
d) área correspondente a 50% da área do triângulo
ABC. 25) Num certo dia, a temperatura ambiente era de
40 ºC. A água que fervia em uma panela, cinco
21) Seja f : IR→B a função definida por f(x) = minutos depois de apagado o fogo tinha a
1 temperatura de 70 ºC. Pela lei de resfriamento de
− a x − 1 (a  IR e a > 1). Analise as afirmativas
2 Newton, a diferença de temperatura D entre um
abaixo, classificando-as em (V) verdadeira(s) ou objeto e o meio que o contém é dada por D(t) = D0
(F) falsa(s). . e– t, em que D0 é a diferença de temperatura no
( ) f(p + q) = f(p) – f(q), p,q  IR instante t = 0 e D(t) a diferença num instante t
( ) f é crescente x  IR qualquer. Sabendo-se que a temperatura de
ebulição da água é de 100 ºC, ℓn 2 = 0,7 e ℓn 5 =
( ) Se x  ] – , 0[, então y  ] - 3 ,-1[
2 1,6, pode-se dizer que a água atingirá a
( ) Se B = ] – , –1[, então f é bijetora. temperatura de 46 ºC.
A seqüência correta é a) 10 minutos após o fogo ter sido apagado.
a) F - F - V - V c) V - F - F - F b) entre 18 e 20 minutos após o fogo ter sido
b) F - V - F – V d) F - V - V – V apagado.
c) exatamente 30 minutos após o fogo ter sido
22) Assinale a alternativa INCORRETA. apagado.
a) O conjunto solução da inequação (2- 3 ) x > -1 é d) aproximadamente 16 minutos após apagado o
IR. fogo.
b) O número real que satisfaz a sentença
2 26) Identifique as alternativas FALSAS,
 3 x − 2  = 52− x é divisor de 1024 assinalando, a seguir, a alternativa que corresponde
 
à soma dos números a elas associados.
c) A função exponencial definida por f(x) = – (a –
4)x é decrescente se 4 < a < 5
d) Se y = 10 é um número entre 10.000 e 100.000, 01- A função para qualquerque
então x está entre 4 e 6 seja x pertencente ao seu domínio, tem imagem 2.
02- sen x + cos x  1 para todo x  [0, ]
5
23) Assinale a alternativa correta. 04- Se sec x = , então cossec  x −   = − 5
4  2 4
 x −1
08 - O domínio da função f(x) = arc sen  6  é o
 

254
PROVAS DA AFA 1998 – 2010
intervalo] –7, 7] quantidade de água restante no reservatório após
16 - O período da função f(x) = |(sen x)(cos x)| é  4 minutos é, em
3
litros.
a) 26 c) 15
b) 23 d) 07

27) Um balão sobrevoa certa cidade a uma altura


de 750 m em relação ao solo, na horizontal. Deste
balão avistam-se pontos luminosos A, B e C, 30) O produto da maior diagonal pela menor
conforme a figura abaixo. O valor da tg  é igual diagonal de um prisma hexagonal regular de área
a. lateral igual a 144 cm2 e volume igual a 144 3
cm3 é.

AFA 2006/2007 – Matemática


1) Uma pessoa caminha, ininterruptamente, a partir
de um marco inicial, com velocidade constante, em
uma pista circular. Ela chega à marca dos 1500 m
quando são exatamente 5 horas. Se às 5 horas e 25
minutos ela atinge a marca dos 4000 m, é
INCORRETO afirmar que
a) a velocidade média da pessoa é 100 m/min.
b) a pessoa começou a caminhar às 4 horas e 15
minutos.
28) Considere as afirmativas abaixo: c) para caminhar 2500 m essa pessoa gastou 25
I - Se  e  são planos interceptando-se na reta r e minutos.
a reta s é paralela a  e a , então s também é d) se a pessoa deu 4 voltas completas em 1 hora e
paralela a r. 20 minutos, então a pista tem 2 km de
II - Se uma reta intercepta um plano , existe um comprimento.
plano  paralelo a  que não é interceptado pela
reta. 2) Apliquei meu capital da seguinte maneira: 30%
III - Se dois planos são paralelos, toda reta contida em caderneta de poupança, 40% em letras de
em um deles é paralela ao outro plano. câmbio e o restante em ações. Na 1a aplicação,
IV - Dois planos perpendiculares a um terceiro lucrei 20%; na 2a, lucrei 30% e na 3a perdi 25%.
plano são sempre paralelos entre si. Se o resultado final corresponde a um lucro de x%
V - Se três retas têm um ponto comum, elas são sobre o capital aplicado, então x é igual a.
coplanares. O número de afirmativas verdadeiras é a) 7,5 c) 15
a) 1 c) 3 b) 10,5 d) 17
b) 2 d) 4
3) Seja z um número complexo não nulo e i a
29) Um reservatório de forma cilíndrica (cilindro unidade imaginária (i2 = -1), z  1 i. O conjunto de
circular reto) de altura 30 cm e raio da base 10 cm Z+i
todos os valores de z, para os quais é um
está cheio de água. São feitos, simultaneamente, 1 + iz
dois furos no reservatório: um no fundo e outro a número real, representa um(a)
10 cm de altura do fundo. Cada um desses furos a) elipse. c) hipérbole.
permite uma vazão de 1 litro por minuto. A b) circunferência. d) círculo.

255
PROVAS DA AFA 1998 – 2010
14
4) Sabe-se que o isótopo do carbono, C , tem uma
meia vida de 5760 anos, isto é, o número N de 7) O termo em x8 no desenvolvimento de (x – 2)4 .
N (x + 1)5 é
átomos de C14 na substância é reduzido a após a) –3x8 c) 72x8
2
um espaço de tempo de 5760 anos. Essa substância b) –32x 8
d) 80x8
radioativa se degrada segundo a sequência N = N0 .
2– t, t  {0, 1, 2, ...} em que N0 representa o 8) Numa pesquisa realizada com um grupo de 55
número de átomos de C14 na substância no instante mulheres e 45 homens quanto à preferência de uma
t = 0 e t é o tempo medido em unidades de 5760 (única) modalidade esportiva, obtiveram-se os
anos. Com base nas informações acima, pode-se resultados registrados na seguinte tabela:
dizer que.
a) o número de átomos quando t = 1 era 5760
b) após 11520 anos haverá a quarta parte do
número inicial de átomos.
c) o número de átomos será igual a um terço de N0 Escolhidos ao acaso, uma pessoa X do grupo todo
quando decorridos 1920 anos. pesquisado; um homem H do grupo de homens
d) quando t = 5760 haverá metade do número pesquisados e uma mulher M do grupo de
inicial de átomos. mulheres pesquisadas, é FALSO afirmar que a
probabilidade de
5) Classifique em (V) verdadeiro ou (F) falso cada a) a pessoa X ser homem ou preferir vôlei é 4
5
item a seguir. b) a pessoa X ser homem e preferir vôlei é 10%
( ) O número  de raízes complexas de B(x) = 0 c) o homem H preferir natação é igual à
sendo B(x) = x2n + 1+ ax2n + b onde a e b são probabilidade de a mulher M também preferir
números reais e n é número natural, é  = 2n + 1 natação.
( ) Se A(x) = xn + 4x + 2, onde n  IN, n > 1, então d) a pessoa X preferir natação é 0,6
A(x) = 0 não admite raízes racionais.
( ) Se o polinômio D(x) de grau 3 admite as raízes 9) Assinale a alternativa INCORRETA.
,  e , então, o polinômio Q(x) = [D(x)]2
admitirá o mesmo conjunto solução.
( ) Se P(x) = x2n + 1 + 4xn + k, onde n  IN e k 
a) Se C = então C2 é matriz nula.
IR, então, P(x) = 0 terá pelo menos uma raiz real.
Tem-se a seqüência correta
a) V – F – F – V c) V – V – F – V
b) F – V – V – F d) V – V – V – V

6) Assinale a alternativa correta. b) Se A = então A2 = A


a) Pode-se codificar quinhentos pacientes, por uma
palavra de duas letras quando as letras são c) A matriz M = (mij)3x3 tal que mij = [i(j + 1)],
escolhidas de um alfabeto de 25 letras. sendo i  {1, 2, 3} e j  {1, 2, 3}, é uma matriz
b) Nas calculadoras, os algarismos são simétrica.
freqüentemente representados, iluminando-se d) Dada uma matriz quadrada T não-nula, a
algumas das sete barras reunidas na forma padrão operação T – Tt , em que Tt é a matriz transposta
8 . O número de diferentes símbolos que podem de T, tem como resultado uma matriz anti-
ser expressos pelas sete barras simétrica.
igual a 7! (fatorial de 7)
c) Entre 10 machos e 7 fêmeas de gatos
experimentais, foi escolhida uma amostra de dois
machos e duas fêmeas. O número de maneiras que
isto pode ser feito é igual 945
d) O número de anagramas da palavra
ASTRONAUTA igual a 10! (fatorial de 10)

256
PROVAS DA AFA 1998 – 2010

10) Dados e det A = – 4 , o ( ) A reta (t) é perpendicular à reta que


passa pelos pontos A e C.

A seqüência correta é
valor de x em A =
a) F – V – V – F c) V – F – F – V
b) V – V – F – F d) V – F – V – V

13) Seja  uma circunferência inscrita em um


triângulo retângulo AOB cujos catetos estão sobre
os eixos cartesianos e medem 3 cm e 4 cm,
conforme a figura abaixo.
11) Seja o sistema de equações S= em
que a e b são números reais. É correto afirmar que.

a) se a = 0, existe b tal que S é impossível.

b) se b é tal que  0, o sistema terá uma


única solução, qualquer que seja o valor de a.
c) se b = 1 e a = 1, o sistema tem mais de uma
solução. É INCORRETO afirmar que
d) se a =0, o sistema possui somente a solução a) o ponto de  mais próximo da origem tem a
trivial. soma coordenadas igual a 2 − 2
b) a área da região sombreada é menor que 3 cm2
12) No plano cartesiano, a figura abaixo representa c) a região sombreada é definida por
duas circunferências concêntricas 1 e 2, cujo
centro é o ponto C. Sabe-se que 1 é contorno de
um círculo representado pela equação (x – 1)2 + (y
+ 2)2  4 e que AB , que mede 8 cm, é corda da
circunferência maior 2. Considerando também d) o conjunto de pontos do plano cartesiano
que AB é tangente a 1, classifique em (V) eqüidistantes de A e B é representado por
verdadeira ou (F) falsa, cada proposição a seguir. 8x – 6y – 7 = 0

14) Classifique em VERDADEIRO ou FALSO


cada item a seguir.
2 - A parábola cuja equação é x2 – 4y = 0 tem
diretriz representada pela reta y + 1 = 0 e foco
coincidente com o baricentro do triângulo ABC,
onde A é a origem do sistema cartesiano, B (2, 3) e
C (-2, 0)
3 - O conjunto de pontos representados pela
equação x2 – y2 + x + y = 0 é uma hipérbole
( ) 1 é tangente ao eixo das abscissas. equilátera que NÃO tem centro na origem do
( ) A soma das coordenadas de A e B é um número sistema cartesiano.
maior que 5. 8 - Na elipse 16x2 + 64y2 = 1 a medida do eixo
( ) A região sombreada é representada por vertical é 50% da medida do eixo horizontal.
16 - Existem apenas 4 números inteiros entre os
valores de k, para os quais o vértice da parábola y 2
257
PROVAS DA AFA 1998 – 2010
= 4x + 1 é ponto exterior à circunferência x + y –
2 2 Tem-se, então, a seqüência correta.
2x + 4y + k = 0
A soma dos itens VERDADEIROS é um número a) V – F – V – F c) F – F – V – V
do intervalo b) F – V – F – V d) V – V – F – F
a) [2, 10[ c) [16, 22[
b) [10, 16[ d) [22, 30[ 17) A função f definida por f(x) =

15) No gráfico abaixo estão representadas as


funções reais f e g sendo A = f  g.
a) não admite inversa porque não é injetora.
b) admite inversa e uma das sentenças que define a
mesma é y = − 1 − − x − 3 se x  -3
c) não admite inversa porque existem valores de x
com várias imagens.
d) admite inversa f –1 tal que f –1 (–5) = –2

18)Analise as alternativas abaixo e marque a


FALSA.
É FALSO afirmar sobre as mesmas funções que a) Se a função f: IR→IR é tal que f(x) = ax + b,
f(3) = 0 e f () > 0, então f é crescente em todo o
a) (fog)(x)  0  g(x)  –2 seu domínio.
b) Seja f: IR→IR tal que f(x) = x2 – 3x + 2 e A um
−1
b) se s(x) = então o domínio subconjunto do domínio de f. Se f é crescente em
[f ( x )]100 .[g( x )]101 A e f (x) > 0 em A, então A = [1, 2].
de s é dado por IR *− - {2} c) Se o gráfico da função quadrática f definida por
f(x) = x2 + kx + m é o da figura abaixo, então k –
c) se h: IR→B tal que h(x) = f(x).g(x), então h será
m=–2
bijetora se B = [–2, + [
f −1 ( x )
d) o gráfico da função j definida por j(x) = −1
g (x)
possui pontos no 4º quadrante. d) Se na função f: IR→IR tal que f(x) = ax2 + bx +
b2
(a  0) c = ,então, necessariamente, o gráfico
16) No gráfico abaixo está representada a função 4a
real f: A→B. Classifique em (V) verdadeira ou (F) da função f é tangente ao eixo das abscissas.
falsa cada proposição a seguir sobre a função f.
19) As funções reais f e g são tais que f(x) = |x| – 2
e g(x) = f(2x) + f(|x|). A melhor representação
gráfica de g é.

( ) No conjunto A existem apenas 15 números


inteiros.
( ) Se B = [– 4 , 4], então f é sobrejetora, mas não é
injetora.
( ) A composta (fofofo ... f)(4) = f(4) ou f(– 4)
( ) f é função par.

258
PROVAS DA AFA 1998 – 2010
20) Sobre a função real definida por

que
a) f(x)  7  x  2 ou x  –2
b) tem valor máximo igual a 1
c) f(x) > 0, x IR
d) se –1 < x < 1, então 0 < y  1

21) De acordo com Richter (1935), a energia E a) y = loga (2x) e sua inversa, sendo a > 1
(medida em joules) liberada por um terremoto de b) y = ax e sua inversa, sendo a > 0
magnitude M, obedece à equação. c) y = loga (x + 1) e sua inversa, sendo a > 1
M = 0,67 log E – 3,25 d) y = –loga x e sua inversa, sendo 0 < a < 1
Baseando-se nisso, é FALSO afirmar que (adotar
log 2 = 0,3) 24) Classifique em (V) verdadeira ou (F) falsa
a) se a energia de 2,0 . 1012 joules equivale à de cada afirmativa abaixo.
uma bomba atômica como a lançada sobre
Hiroshima, então, o valor da magnitude de um I - O domínio da função real f definida por f(x) =
terremoto cuja energia liberada equivale a 2000 arc cos 1 é o conjunto {x  IR | x  0 ou x  2}
x −1
bombas atômicas como a lançada sobre Hiroshima,
II - No intervalo [0, 2] o gráfico da função real
é um número do intervalo ] 7; 7,3 ]
y = –2sen3 x corta o eixo x um número ímpar de
b) o acréscimo de 0,67 unidades na magnitude de
vezes.
um terremoto na escala Richter corresponde a um
terremoto cerca de 10 vezes mais intenso em
III - A função real f:A→[0, 1] tal que f(x) =
termos de energia liberada.
c) a energia de 2,0 . 1012 joules (equivalente à de
uma bomba atômica como a lançada sobre sen2(2x) admite inversa, se A =
Hiroshima) corresponde à ocorrência de um
terremoto de magnitude superior a 5 pontos na Conclui-se que são verdadeiras
escala Richter. a) I, II e III c) apenas II e III
d) o crescimento na magnitude de terremotos na b) apenas I e III d) apenas I e II
escala Richter, acarreta um aumento exponencial
da energia liberada. 25) Analise as proposições seguintes e classifique-
as em (V) verdadeiras ou (F) falsas.
22) Dada a função real f tal que f(x) = ( ) Se o ponteiro dos minutos de um relógio mede
10 cm, então a distância que sua extremidade
percorre em 30 minutos é de aproximadamente
onde e = 2,71... é a base de 31,4 cm.
logaritmos neperianos, é correto afirmar que o ( ) O domínio da função real f definida por f(x)
conjunto D, domínio de f é igual a. sec x + cossec x é o conjunto D = {x  IR | x 
a) {x IR / x > 1 e x  2} k  , com k  Z}
b) {x  IR* / –2 < x < 2} 2

c) {x  IR+* / x  1} ( ) A equação cos x . tg x – cos x = 0 possui 4


d) {x  IR / x < –2 ou x > 2} raízes no intervalo [0, 2]
( ) O período e a imagem da função trigonométrica
23) As funções que melhor descrevem as curvas f definida por f(x) = 2cos2x – 2sen2x, são
abaixo são. respectivamente iguais a 2 e [–2, 2]

A seqüência correta é
a) V – V – F – F c) F – V – F – V
259
PROVAS DA AFA 1998 – 2010
b) F – F – V – V d) V – V – V – F superfície de uma esfera, a medida do raio da
esfera, em dm, é
26) Considere {a, b, c, d}  IR e as funções reais f
e g tais que f (x) = a + b.cos(cx + d) e g(x) = a +
b.tg(cx + d). Sabendo-se que a, b, c e d formam,
nessa ordem, uma P.G. cuja soma dos termos é

20
e o primeiro termo 1 , é correto afirmar que 30) Considere um triângulo retângulo inscrito em
9 9
a) o período da função é 2 uma circunferência de raio R, tal que a projeção de
um dos catetos sobre a hipotenusa mede, em cm,
R
(m  1) m. Considere a esfera gerada pela
m
b) a função g está definida para x =
rotação desta circunferência em torno de um de
seus diâmetros. O volume da parte desta esfera,
c) o conjunto imagem da função f é
que não pertence ao sólido gerado pela rotação do
triângulo em torno da hipotenusa, em cm3, é dado
por.
d) a função g é crescente para x 

27) Um triângulo retângulo está circunscrito a um


círculo de raio 15 m e inscrito em um círculo de
raio 37,5 m. A área desse triângulo, em m2, mede
a) 350 c) 1050
b) 750 d) 1350
AFA 2007/2008 – Matemática
28) Um cubo tem quatro vértices nos pontos
médios das arestas laterais de uma pirâmide 1) Analise as alternativas abaixo e marque a
quadrangular regular, e os outros quatro na base da correta.
pirâmide, como mostra a figura abaixo. a) Se B = {m ∈ N| m2 < 40}, então o número de
elementos do conjunto B é 6
b) Se ∝ = , então ∝ ∈ [(R – Q) ∩
1 1
+
2 −1 2 +1
(R – Z )]
c) Se c = a + b e b é divisor de a, então c é
múltiplo de a, necessariamente.
d) Se A = ]1,5[ e B = ]-3,3[, então B – A = ] –3,1[

2) Um fabricante de camisetas que pretendia


vender seu estoque no prazo de 4 meses, mantendo
o preço de cada camiseta, obteve o seguinte
resultado:
A razão entre os volumes do cubo e da pirâmide é . no primeiro mês, vendeu 10% de seu estoque;
. no segundo, 20% do restante das mercadorias; e
. no terceiro, 50% do que sobrou.
Ao ver que sobraram 3.600 camisetas, no quarto
mês, o fabricante reduziu o preço de cada uma em
1
33 %, conseguindo assim liquidar todo seu
3
29) Num cone reto, a medida do raio da base, da estoque e recebendo R$ 21.600,00 pelas vendas
altura, e da geratriz estão, nessa ordem, em deste mês. É correto afirmar que o fabricante
progressão aritmética de razão igual a 1. Sabendo- a) arrecadaria a mesma importância total, durante
se que a soma destas medidas é 12 dm e que a área os 4 meses, se cada camiseta fosse vendida por x
total da superfície deste cone é igual à área da reais, x ∈ [7, 8]
260
PROVAS DA AFA 1998 – 2010
a) 0,  1 
b) tinha um estoque que superava 834 dúzias de 1
b)  2 ,1 c) [1,2] d) [2,3[
camisetas.  2  
c) no terceiro mês, vendeu uma quantidade de
camisetas 200% a mais que no segundo mês. 6) Considere   3,14 ei = − 1 emarque a alternativa
d) no primeiro mês, recebeu mais de R$ 9.000,00 correta.
a) Se S(x) = x2(x – a) + bx – c, onde a, b e c são
3) Considere no Plano de Argand-Gauss os números reais positivos, admite duas raízes
números complexos z1 = –x – 2i, z2 = –2i, z3 = –2 1
+ 3i e z4 = x + yi, onde x e y são números reais simétricas, então, log a log co log b
c
quaisquer e i2 = –1 b) O polinômio P(x) ao ser dividido por (x – 1)
Sobre o conjunto desses números complexos que deixa resto 6 e ao ser dividido por (x + 3) deixa
atendem simultaneamente às condições resto –2. Se P(x) dividido por Q(x) = x2 + 2x – 3
I) Re (Z1 .Z 2 ) ≤ Im (Z1 .Z 2 ) deixa resto R(x), então R(0) = 2P(–3)
II) |z3 + z4| ≤ 2 c) Se os números complexos 2π, 2i e i – 5 são
raízes do polinômio A(x) de coeficientes reais e
termo independente nulo, então, o grau de A(x) é,
é correto afirmar que
necessariamente, um número par maior do que 4
a) representa uma região plana cuja área é menor
d) Se no polinômio B(x) = x4 + ax3 + bx2 + cx +
que 6 unidades de área.
16 os coeficientes a, b e c são números reais, então
b) possui vários elementos que são números
as possíveis raízes racionais de B(x) estão entre os
imaginários puros.
divisores de 16, necessariamente.
c) possui vários elementos que são números reais.
d) seu elemento z de menor módulo possível
7) Sabendo-se que x0 = – i, x1 = 3 e x2 =
possui afixo que pertence à reta (r) 3x+2y = 0 20
1 
 + 3  são raízes de P(x) = x6 – 3x5 + x4 – 4x3
4) Um cão e um gato, ambos parados, observam-se 2 2 
 
a uma distância de 35 m. No mesmo instante, em + 3x2 – ax + 3, onde i é a unidade imaginária e a é
que o cão inicia uma perseguição ao gato, este número real, marque a alternativa FALSA.
parte em fuga. a) O número a também é raiz de P(x)
O cão percorre 2 m no primeiro segundo, 4 m no b) A soma das raízes reais de P(x) é um número
seguinte, 6 m no terceiro segundo e, assim, par.
sucessivamente. O gato, apavorado, percorre 3 m c) O produto das raízes imaginárias de P(x) é
no primeiro segundo, 4 m no seguinte, 5 m no diferente de a
terceiro segundo e, assim, sucessivamente. d) P(x) é divisível por x2 + x + 1
Considerando que os dois animais se deslocam
sempre sem interrupção em seu movimento e 8) Uma pessoa fará uma viagem e em cada uma de
numa trajetória retilínea de mesmo sentido, suas duas malas colocou um cadeado contendo um
assinale a alternativa INCORRETA. segredo formado por cinco dígitos. Cada dígito é
a) Até o quinto segundo, o cão terá percorrido uma escolhido dentre os algarismos 0, 1, 2, 3, 4, 5, 6, 7,
distância igual àquela que o separa do gato naquele 8 e 9.
instante. Na primeira mala, o segredo do cadeado começa e
b) Ao final dos três primeiros segundos, o cão termina com dígito par e os demais são dígitos
ainda está 35 m distante do gato. consecutivos em ordem crescente. Na segunda
c) Em dez segundos, o cão alcançará o gato. mala, o segredo do cadeado termina em dígito
d) No oitavo segundo, o gato percorre 14 metros. ímpar e apenas o 1o e o 2o dígitos são iguais entre
si.
5) Sejam as seqüências de números reais (–3, x, y, Dessa maneira, se ela esquecer
...) que é uma progressão aritmética de razão r, e a) o segredo do cadeado da primeira mala, deverá
(x, y, 24, ...) que é uma progressão geométrica de fazer no máximo (52 x 83) tentativas para abri-lo.
r b) o segredo do cadeado da segunda mala, o
razão q. O valor de pertence ao intervalo.
q número máximo de tentativas para abri-lo será de
1.890
261
PROVAS DA AFA 1998 – 2010
c) apenas os três dígitos consecutivos em ordem 1
determinante de B é igual a , então o
crescente do cadeado da primeira mala, ela 2
conseguirá abri-lo com, no máximo, 8 tentativas. determinante da matriz Y é igual a 2n–2
d) apenas os dois primeiros dígitos do cadeado da 1 0  1 0
III) Seja a matriz A =   então Na =   .n
segunda mala, deverá tentar no máximo 10 vezes 1 1 n 1
para abri-lo. , n ∈ N*
É correto afirmar que são verdadeiras
9) Uma pessoa deve escolher (não importando a a) todas as proposições. b) apenas II e III.
ordem) sete, dentre dez cartões numerados de 1 a c) apenas I e II. d) apenas I e III.
10, cada um deles contendo uma pergunta
diferente. Se nessa escolha houver, pelo menos 12) Um suspeito de assaltar dois caixas de um
três, dos cinco primeiros cartões, ela terá n formas supermercado foi intimado a prestar depoimento e
de escolha. Sendo assim, pode-se afirmar que n é fez a seguinte declaração:
um número “No primeiro caixa foram roubados dois pacotes
a) quadrado perfeito. c) ímpar. de notas de 20 reais, cinco pacotes de notas de 50
b) múltiplo de 11 d) primo. reais e um pacote de notas de 100 reais, totalizando
100 mil reais. No segundo caixa, foram roubados
10) Analise as proposições seguintes. um pacote de notas de 20 reais e três pacotes de
(02) Se 1(1!) + 2(2!) + 3(3!) +... + n(n!) = (n +1)! – notas de 100 reais, num total de 50 mil reais. Os
1, com n ∈ {1, 2, 3, 4, ...}, então, o valor de pacotes de notas de mesmo valor tinham a mesma
1(1!) + 2(2!) + ... + 10(10!) + 1 quantidade de notas.
igual a 18
8!(1 + 2 + 3 + 4... + 10) Cada pacote de notas de 100 reais tinha igual valor
P
 m  de cada pacote de notas de 50 reais.”
(04) O valor de   m − 1 é p2 Diante do depoimento do suspeito, pode-se
m =1
(08) Uma caixa ( I ) contém 6 garrafas com rótulo concluir que
e duasgarrafas sem rótulo; outra caixa ( II ) contém a) ele pode ter falado a verdade.
4 garrafascom rótulo e uma sem rótulo. Uma caixa b) ele falou, necessariamente, a verdade.
é selecionada aleatoriamente e dela uma garrafa é c) havia, necessariamente, 940 notas em cada
retirada. A probabilidade dessa garrafa retirada ser pacote de notas de 20 reais.
sem rótulo é de 22,5% d) ele mentiu, necessariamente.
(16) Dois dígitos distintos são selecionados
aleatoriamente dentre os dígitos de 1 a 9. Se a 13) A circunferência (λ) x2 + y2 – 2x – 2y + k = 0
soma entre eles é par, a probabilidade de ambos passa pelo ponto A(0, 1). Sabendo-se que o ponto
5 P de (λ) mais próximo da origem coincide com o
serem ímpares é baricentro do triângulo MNQ, onde M(0, k), N(2k,
8
A soma das proposições verdadeiras é igual a 0) e Q(xQ, yQ) é correto afirmar que a área do
a) 14 b) 24 c) 26 d) 30 triângulo MNQ é um número do intervalo
a) 1,  b)  ,2 c) 2,  d)  ,3
3 3 5 5
11)Analise cada proposição a seguir classificando-  2 2   2 2 
a como VERDADEIRA ou FALSA.
I) Sejam as matrizes A = (aij)3xn e B = (bjk) nx4 14) - Classifique em (V) verdadeira ou (F) falsa
(n ≥ 1) então a matriz C = A.B é tal que o cada afirmativa abaixo sobre o ponto P(x, y) no
plano cartesiano.
elemento
4
( ) Se o ponto P pertence simultaneamente às
c21 =  a 2 j .b j1 bissetrizes dos quadrantes ímpares e dos
j=1 quadrantes pares, então o ponto simétrico de P em
II) A e B são matrizes inversíveis de ordem n. Se relação à reta y = k (k ∈ IR*) tem a soma das
AYB = 2Bt, onde Bt é a transposta de B, o coordenadas igual a 2k
1
determinante da inversa de A é igual a e o
4

262
PROVAS DA AFA 1998 – 2010
( ) Sendo {x, y} ⊂ Z, então existem apenas dois b) Se a função s: D → R é tal que s(x) = f  x +  ,
3

pontos P(x, y) que atendem às condições  2


3
 x0 então s(0) = -
 2 2
 y − 3y  x c) O domínio da função r: E → R tal que r(x) =
( ) Os pontos P(x, y) tais que a sua distância ao f(x) – 3 é o intervalo real [–6, 6]
eixo das abscissas é igual à metade da distância de d) A função r: E → R tal que r(x) = f(x) – 3 NÃO
P ao ponto Q(0, 6) formam uma hipérbole de possui raízes em R.
excentricidade igual a 2
Sobre as afirmativas tem-se
17) Considere todo x ∈ R que torne possível e
a) apenas uma falsa. c) todas falsas.
b) apenas duas falsas. d) todas verdadeiras. verdadeira a igualdade log [f(x2 –1)] = log
x 4 − 2 x 2 + 1 , onde f é função real de A em B e
15) Considere as curvas, dadas pelas equações marque a alternativa correta.
(I) 16x2 + 4y2 + 128x – 24y + 228 = 0 a) O conjunto imagem de f é Im = + R+ - {1}
(II) y = 7 – | x | b) f é uma função injetora.
(III) y2 – 6y – x + 5 = 0 c) Se B = R+ - {1} , então existe a inversa de f
Analise cada afirmação a seguir, classificando-a d) f tem domínio A { x ∈ R| x| >1}
em VERDADEIRA ou FALSA.
(01) O gráfico de (I) é representado por uma 18) As funções f: R → R do 1º grau e g: R → [b,
elipse, de (II) por duas retas e de (III) por uma +∞[ do 2º grau estão representadas no gráfico
parábola. abaixo.
(02) O centro de (I) é um ponto de (II) e coincide
com o vértice de (III)
(04) A soma das coordenadas do foco de (III) é um
número menor que –1

(08) A excentricidade de (I) é igual a cos
6
A soma dos itens verdadeiros é um número do
intervalo
a) [1, 3] b) [4, 7] c) [8, 11] d) [12, 15]

16) Na figura abaixo, está representado o gráfico


da função real
f: [–a, a] → R, onde f(0) = 0

Com base nas informações acima, é correto


afirmar que
a) o menor valor de b que torna a função g
sobrejetora é um número inteiro.
b) (gogof –1)    0
5
2
Analise as alternativas abaixo e marque a c)
f ( x ) > 0 ⇔ {x ∈ R | x < 1 ou x > 4}
2

INCORRETA. g(x )
a) O conjunto imagem da função h: A → B, d) f(x) – g(x) ≤ 0 ⇔ {x ∈ R | x ≤ 0 ou x ≥ 6}
definida por
é Im = 0, 
2 9
h (x) = f (x) + 19 - Considere as funções reais
3  2
f: R*+ → R tal que f(x) = x – 2
x
g : R →R*+ tal que g(x) =  
1
2
263
PROVAS DA AFA 1998 – 2010
h : R*+ →R tal que h(x) = -log2 x e marque a
alternativa correta.
a) O domínio da função k definida por k (x) =
g( x )
é o conjunto dos números reais positivos.
h(x)
f ( x ).h −1 ( x )
b) A função j definida por j(x) = se
(gof )(x )
anula em dois pontos distintos. Para se conhecer o rendimento para base de
c) A função m definida por m(x) = –1 + (gof)(x) cálculo, deve-se subtrair do rendimento bruto todas
não possui raiz. as deduções a que se tem direito. Esse rendimento
d) Se g(h(a)) = 8 e h(g(2b)) = log3 9 , então (a – b) para base de cálculo é multiplicado
é um número primo. pela alíquota correspondente. Em seguida, subtrai-
se a parcela a deduzir correspondente, de acordo
20) “A arrecadação da CPMF, devido à ampliação com a tabela acima, obtendo-se assim o valor do
de sua abrangência, e ao aumento da alíquota, imposto de renda a ser pago.
cresceu mais de 140% nos últimos anos (em Um trabalhador, cujo rendimento bruto foi de R$
bilhões de reais por ano)” 50.000,00 teve direito às seguintes deduções: R$
Revista veja – 14/03/2007 4.400,00 com o total de gastos em educação, R$
5.000,00 com o total pago à Previdência e R$
1.500,00 por dependente.
Nessas condições, sabendo-se que o valor do
imposto pago
por esse trabalhador, no ano de 2007, foi de R$
3.515,00, o número de dependentes considerado
foi
a) 2 b) 3 c) 4 d) 6

22) Sabendo-se que b é um número real tal que b >


1 e que a função real f: R→ B é tal que f(x) = 2 – b
–|x|
, analise as alternativas abaixo e marque a
FALSA.
a) A função f admite valor mínimo.
1
b) x ≤ –1 ⇔ 2 – ≤ f (x) < 2
b
c) A função f é par.
d) Se B = [0, 2[ então f é sobrejetora.
Supondo que o crescimento da arrecadação
representado no gráfico acima é linear do ano de 23) Sabendo-se que a função real f: D → B
2005 ao ano de 2007 e que y% representa o definida por
aumento da arrecadação do ano de 2005 ao ano de x
2006, é correto afirmar que y é um número do f(x) = é inversível e que D e B são conjuntos
1− x
intervalo os mais amplos possíveis, é FALSO afirmar que
a) [8, 9 b) [9, 10[ c) [10, 11[ d) [11, 12[ a) f é crescente para todo x tal que x < 1 ou x > 1
b) a equação da assíntota horizontal de f é y = –1
21) Considere a tabela para cálculo do imposto de c) se g é tal que g(x) = | f –1(x) |, então não existe x
renda a ser pago à Receita Federal no ano de 2007 real tal que g(x) = 1
– ano base 2006
d) f-1 (0) + f-1  −   0
1
(valores arredondados para facilitar os cálculos).  2

24) No cubo da figura abaixo, considere P o ponto


de encontro das diagonais da face ABCD e Q o

264
PROVAS DA AFA 1998 – 2010
ponto de encontro das diagonais da face EFGH e θ II) Suponha também que uma seqüência de saltos
é medida do ângulo Q ÊP . uniformes de uma lebre, possa ser representada
como no esquema da Figura 2.

Transportando as situações acima para um plano


cartesiano, considere
Analise as proposições seguintes. . o eixo das abscissas coincidindo com o nível da
(01) 2θ é um ângulo maior que 90º água gelada para o pingüim;
(02) θ é um ângulo do intervalo [45º, 60º] . o eixo das abscissas coincidindo com o solo para
(04) tg 2θ = –2tg θ a lebre;
1 . a altura do salto do pingüim e da lebre indicada
(08) sen 2θ = tg2θ
2 no eixo das ordenadas.
 3  Tendo por base as situações apresentadas, nas
(16) cossec  −   = tg 60º
 2  figuras 1 e 2 e ainda a teoria dos gráficos das
funções trigonométricas, pode-se relacionar aos
O número que representa a soma das proposições saltos um tipo de gráfico dessas funções. Assim
verdadeiras é múltiplo de sendo, as funções P e L estabelecem os saltos
a) 2 b) 3 c) 5 d) 7 do Pingüim e da Lebre, respectivamente.
A opção que contém funções que podem
25) Considerando as definições e propriedades das representar a situação descrita, sabendo-se que a
funções trigonométricas, marque a alternativa função P está restrita a um único período, é

a) P(x) = - tg  x −  e L(x) = 2|senx|
correta.
a) A função f definida por f(x) = | sen 2x - cos 2x|  2
possui período e imagem, respectivamente, iguais 
aπe][2,0 b) P(x) = cotg  x +  e L(x) = 2sen|x|
 2
b) Se f e g são funções tais que f (x) = tg (x) = |x| , c) P(x) = tg(x) e L(x) = | x 2 sen | 2
sabendo-se que existe a função j definida por j(x)= d) P(x) = –2tg(x) e L(x) = | x 2 sen |
(fog) (x) então j é periódica.
 
c) No intervalo de  ,  a função h definida por 27) Considere um triângulo MNP, eqüilátero,
4 2 inscrito numa circunferência de centro O e raio r.
h(x) = |cos 2x| é decrescente. Seja RS uma corda que intercepta os lados MN e
d) O domínio da função g definida por g(x) = 3arc MP do triângulo nos pontos T e V, pontos médios
3x − 1
é D =  ,1 dos respectivos lados. Se RT ≡ VS = 1cm , então o
1
sen
2 3 
valor da área do quadrilátero NPVT, em cm2, é
dado por um número do
26) Considere as situações a seguir. intervalo
I) Suponha que a passagem de um pingüim, da
(DADOS: 3 = 1,73 e 5 = 2,23 )
água para a superfície de uma geleira, possa ser
representada como no esquema da Figura 1. a) [1,3[ b) [3,5[ c) [5,7[ d) [7,9[

28) Um triângulo ABC é não isósceles. Sejam M,


N e P, respectivamente, os pontos médios dos
lados AB,BC e AC desse triângulo, de forma que
AN = 3 cm e BP = 6 cm. Se a área do triângulo
ABC mede 3 15 cm2, então o comprimento da
outra mediana, CM , em cm, é igual a
265
PROVAS DA AFA 1998 – 2010
a) 2 b) 3 c) 3 6 d) 6 15 cumprir o tempo que falta, ou seja, o segundo
expediente. Hoje, excepcionalmente, quando
29) Considere um hexaedro regular S onde A, B e cheguei, o relógio de ponto registrou um horário
C são pontos médios de três de suas arestas tal que o tempo transcorrido do dia era igual aos
4
concorrentes no mesmo vértice. Seja α um plano do tempo restante do dia e eu fui, então,
11
que secciona S nos pontos A, B e C separando-o
alertado que estava atrasado. Acertei meu relógio
em dois sólidos S1 e S2 de volumes V1 e V2,
pelo relógio de ponto e, para compensar meu
respectivamente, onde V1 < V2
3
Marque (V) verdadeiro ou (F) falso em cada atraso, pretendo cumprir os de minha jornada
4
afirmativa. e sair para almoçar reduzindo o tempo de meu
( ) S2 ainda poderia ser dividido em 47 sólidos de 1
volume igual a V1 intervalo de almoço em Imediatamente
( ) A área total de S1 é 6 (3 + 3 ) da área total de S
5
retornarei para o trabalho e sairei no meu horário
( ) Se em cada três arestas concorrentes de S forem habitual.”
retirados os sólidos com volumes iguais ao do Considerando que o relógio de ponto estivesse
sólido S1, então, o volume do sólido restante seria certo e em perfeito funcionamento, é correto
aproximadamente igual a 83,33% do volume de S afirmar que, nesse dia, Gabriel, com sua pretensão
Tem-se a seqüência correta em a) sairá para o almoço antes de 12 horas e 23
a) V – F – V c) F – F – V minutos.
b) F – V – F d) V – V – F b) retornará após o intervalo de almoço,
exatamente, às 13 horas e 50 minutos.
2 x + y  16 c) cumprirá sua jornada diária na íntegra e ainda

30) Seja S a região do plano dada por  x−y2 sobrarão dois minutos.
 x−20 1
 d) ficará devendo de sua jornada diária.
60
O volume do sólido gerado pela rotação de 360° de
S em torno da reta x + 1 = 0 é, em unidade de
3) Considerando todos os números complexos z =
volume, igual a
a)208π b) 235π c) 252π d) 316π x + yi, onde x ∈ R, y ∈ R e i = − 1 , tais que |z -
2
| z − − 1 |≤ Sobre esses números complexos z,
1+ i
AFA 2008/2009 – Matemática é correto afirmar que .
a) nenhum deles é imaginário puro.
1) Sr. Osvaldo possui certa quantia com a qual b) existe algum número real positivo
deseja adquirir um eletrodoméstico. Caso a loja c) são todos imaginários
ofereça um desconto de 40%, ainda lhe faltarão d) apenas um é número real.
1000 reais. Se o Sr. Osvaldo aplicar sua quantia a
juros (simples) de 50% ao mês, ajunta, em três 4) Considere as proposições abaixo.
meses, o montante correspondente ao valor do I) A soma dos infinitivos termos da seqüência cujo
eletrodoméstico sem o desconto. Assim, o valor do n 3
termo geral é , n  N* , converge para .
eletrodoméstico e da quantia que o Sr. Osvaldo 3 n 4
2k 
II) Se ak = cos 
possui somam, em reais,
 ; k ∈ N , o valor de
*
a) 4000 b) 5000 c) 7000 d) 8000  3 
a1+a2+...+a97 é zero.
2) Perguntaram a Gabriel qual era seu horário de III) Se (3,a,b) formam um progressão geométrica
trabalho e ele respondeu: de razão q e (a, b, 45), uma progressão aritmética
“Habitualmente começo às 6 horas da manhã
de razão r, com a, b ∈ N, então
r
=6
minha jornada de trabalho que é de 8 horas diárias, q
dividida em dois expedientes.Cumpro no primeiro Pode-se afirmar que, entre as proposições.
3 a) apenas uma é falsa b) apenas duas são falsas
expediente dessa jornada, tenho um intervalo de
4 c) todas são falsas d) todas são verdadeiras.
almoço de 1 hora e 45 minutos e retorno para
266
PROVAS DA AFA 1998 – 2010
5) João Victor e Samuel são dois atletas que a) o número n é uma das raízes
competem numa mesma maratona. Num b) nenhuma delas é número imaginário
determinado momento, João Victor encontra-se no c) todas são números inteiros
ponto M, enquanto Samuel encontra-se no ponto d) uma delas é um número par.
N, 5 m à sua frente. A partir desse momento, um
observador passa a acompanhá-los registrando as 7) As senhas de acesso a um determinado arquivo
distâncias percorridas em cada intervalo de tempo de um microcomputador de uma empresa deverão
de 1 segundo, conforme tabelas abaixo. ser formadas apenas por 6 dígitos pares, não nulos.
Sr. José, um dos funcionários dessa empresa, que
João Victor utiliza esse microcomputador, deverá criar sua
Intervalo Distância (m) única senha.
1º 1 Assim, é INCORRETO afirmar que o Sr. José
2 a) poderá escolher sua senha dentre as 212
2º 3 possibilidades de formá-las.
4 b) terá 4 opções de escolha, se sua senha possuir
3º 9 todos os dígitos iguais.
8 c) poderá escolher dentre 120 possibilidades, se
∶ ∶ decidir optar por uma senha com somente 4 dígitos
iguais.
Samuel d) terá 480 opções de escolha, se preferir uma
Intervalo Distância (m) senha com apenas 3 dígitos iguais.
1º 1
n
8) Com relação ao binômio  x 2 +  é correto
2 2
2º 3  x
4 afirmar que
3º 1,0 a) se o 5o termo do desenvolvimento desse
∶ ∶ binômio, segundo as potências decrescentes de x, é
560x2, então n é igual a 7
Sabe-se que os números da tabela acima que b) se n é ímpar, seu desenvolvimento possui um
representam as distâncias percorridas por João número ímpar de termos.
Victor formam uma progressão geométrica, c) possui termo independente de x, ∀ n ∈ N*
enquanto os números da tabela acima que d) a soma de seus coeficientes binomiais é igual a
representam as distâncias percorridas por Samuel 64 quando esse binômio possui seis termos.
formam uma progressão aritmética.
Com base nessas informações, é INCORRETO 9) No lançamento de um dado viciado, a face 6
afirmar que ao final do ocorre com o dobro da probabilidade da face 1, e
a) 5º segundo, João Victor já terá atingido o ponto as outras faces ocorrem com a probabilidade
N esperada em um dado não viciado de 6 faces
b) 5º segundo, Samuel percorreu uma distância numeradas de 1 a 6.
igual à que os separava nos pontos M e N Dessa forma, a probabilidade de ocorrer a face 1
c) 6º segundo, João Victor terá alcançado Samuel. nesse dado viciado é
d) 8º segundo, João Victor estará mais de 8 metros 1 2 1 2
a) b) c) d)
à frente de Samuel. 3 3 9 9

6) O polinômio P1(x) = mx3 – 2nx2 – mx + n2, 10) Um trailer de sanduíches anunciou para a
onde {m, n} ⊂ R é unitário e não é divisível por segunda-feira, a seguinte promoção.
P2(x) = x.
“Saboreie: 1 X-banana, 1porção de batatas fritas, 1
Sabe-se que P1(x) = 0 admite duas raízes
refrigerante em lata e pague apenas y reais.”
simétricas.
Sobre as raízes do P1(x) = 0 é INCORRETO
Como o movimento da noite de segunda-feira
afirmar que
estava fraco, o proprietário resolveu manter os
267
PROVAS DA AFA 1998 – 2010
preços individuais de cada componente da oferta  x = 2−t
(s)  onde K, t ∈ R, pode-se afirmar
para quaisquer combinações de pedidos dos  y = −1 + (1 − k ) t
produtos citados. que
Assim, as famílias A, B e C pagaram juntas 56 a) poderão ser paralelas coincidentes para algum
reais pelos produtos consumidos, conforme o valor de k
quadro abaixo: b) nunca serão perpendiculares entre si.
Quantidad X- Porção de Refrigerna c) se forem paralelas, não terão equação na forma
e/família banana fritas te em lata reduzida.
A 5 4 4 d) sempre poderão ser representadas na forma
B 3 0 2 segmentária.
C 1 2 2
Sabendo-se que a família A gastou 3 reais a mais 13) Os vértices de um triângulo ABC são os
que o dobro do valor gasto pela família B e que a centros das circunferências:
família C gastou 3 reais a menos que a família B, é (λ1) x2 + y2 + 2x – 4y – 1 = 0
INCORRETO afirmar que. (λ2) 4x2 + 4y2 + 12x – 8y – 15 = 0
a) 6 refrigerantes em lata custam tanto quanto 10 (λ3) (x – 7)2 + (y + 3)2 = 8
porções de batatas fritas. O tetraedro cuja base é o triângulo ABC e cuja
b) a família B gastou o equivalente a 30% das altura, em metros, é igual à média aritmética dos
despesas das famílias A e C juntas. quadrados dos raios das circunferências acima,
c) o preço y da promoção sugerida não ultrapassa também em metros, possui volume, em m3, igual a
R$ 7,50 21 21 49 49
a) b) c) d)
d) a família B poderia ter optado por pedir duas 2 4 2 4
promoções e sua despesa seria a mesma.
14) Suponha um terreno retangular com medidas
11) Analise as proposições e classifique-as em de 18 m de largura por 30 m de comprimento,
verdadeiro (V) ou falso (F). como na figura abaixo:
( ) Seja A uma matriz quadrada de ordem 2 em que
det (3A) = 36
Se dividirmos a 1a linha de A por 2 e
multiplicarmos a 2a coluna de A por 4, o valor de
det A será 8
( ) Sejam M e N matrizes quadradas de ordem 3 e Um jardineiro deseja construir nesse terreno um
N = aM, a ∈ R* jardim elíptico que tenha os dois eixos com o
maior comprimento possível. Ele escolhe dois
3 pontos fixos P e Q, onde fixará a corda que vai
Sabendo-se que detM , det (Nt) = 96 e que Nt é auxiliar no traçado.
2
a transposta de N, então a vale 12. Nesse jardim, o jardineiro pretende deixar para o
bc a a2 1 a2 a3 plantio de rosas uma região limitada por uma
( ) Se A = ac b b 2 e B = 1 b 2 b 3 , então A = hipérbole que possui:
. eixo real com extremidades em P e Q;
ab c c2 1 c2 c3
5
. excentricidade =
B 4
( ) Sejam A e B duas matrizes quadradas de ordem Considerando o ponto A coincidente com a origem
n do plano cartesiano e a elipse tangente aos eixos
É correto afirmar que (A + B) (A – B) = A2 – B2, coordenados, no primeiro quadrante, julgue as
quaisquer que sejam A e B. afirmativas abaixo.
Marque a seqüência correta. (01) O centro da elipse estará a uma distância de
a) V – F – V – F b) F – V – F – V 3 34 m do ponto A
c) V – V – F – F d) V – F – V - V (02) Para fazer o traçado da elipse o jardineiro
precisará de menos de 24 m de corda.
12) Sobre as retas ( r ) (1 – K) x + 10y + 3k = 0 e (04) O número que representa a medida do eixo
real da hipérbole, em metros, é múltiplo de 5
268
PROVAS DA AFA 1998 – 2010
(08) Um dos focos dessa hipérbole estará sobre um Hotel Araújos: possui 40 quartos disponíveis,
dos eixos coordenados. todos indivíduos, sem direito a cama extra. A
A soma dos itens verdadeiros pertence ao intervalo diária de cada quarto é dada por uma taxa fixa de
a) [1, 5[ b) [5, 7[ c) [7, 11[ d) [11, 15] R$ 200,00 mais R$ 10,00 por quarto não ocupado.

15) Se f: R →R é uma função afim crescente de Hotel Fabianos: possui 50 quartos disponíveis,
todos indivíduos, sem direito a cama extra. O valor
raiz r < 0, g : R → R é uma função linear
da diária de cada quarto é 0, 6 de 6 décimos d 125
decrescente e h : A → R é uma função definida por milésimos de R$ 6000,00
1
h (x) = , então, o conjunto A,
− f ( x )]20 .[g(− x )]7 A viagem citada foi programada para x cadetes (x
mais amplo possível, é dado por. ≤40) e, no período em que eles estiverem
a) ] r, 0 [ b) ] - ∞, 0 [ - { r} hospedados, os hotéis só receberão como hóspedes
esses x cadetes.
c) ]r, + ∞[ - {0} d) ] - ∞,0 [ Com base nisso, marque a alternativa
INCORRETA.
16) Considere as funções reais f : R → R dada por a) Se forem viajar menos de 30 cadetes, então é
f(x) = x + a, g : R→ R dada por g(x) = x – a, h : mais vantajoso para os cadetes optarem pelo Hotel
R→ R dada por h(x) = – x – a Fabiano’s.
Sabendo-se que a < 0, é INCORRETO afirmar que b) O faturamento diário do Hotel Araújo’s será de
a) h(x) ≤ f(x) < g(x) ⇔x ≥ –a R$ 8000,00 se, e somente se, o número de cadetes
b) se a < x < – a, então f(x) < h(x) < g(x) que forem à viagem for 20.
c) ∄ x ∈ R | g(x) ≤ f(x) c) Se 15 cadetes forem viajar, então o valor da
d) se x < a, então f(x) < g(x) < h(x) diária do Hotel Fabiano’s é 0, 6 do valor da diária
do Hotel Araújo’s.
17) Considere que g : R → B, definida por g(x) = d) O maior faturamento diário que o Hotel
– bx2 + cx – a é função par e possui como gráfico o Araújo’s poderá ter, caso os cadetes optem por
esboço abaixo. hospedarem nele, acontecerá quando 10 quartos
não forem ocupados.

19) Considere as funções reais f: R →R*+ tal que


f(x) = ax, g: R →R*+ tal que g (x) = bx, h: R →R*+
tal que h (x) = cx.
Marque a alternativa INCORRETA. Sabendo-se que 0 < a <1< b < c, marque a
a) A função t : R → R dada por t(x) = g(x) + a é alternativa INCORRETA.
positiva ∀x ∈ R a) h (x) < g (x) < f (x) , ∀x ∈] – 1,0 [
f (x) − 2
b) Se B = [– a, +∞[ , então a função g é b) Se x ∈] - ∞, logoa 2 [, então 0
h(x) − 1
sobrejetora.
c) b < c < a c) A função real t: A → B dada por t(x) = (fof-1)
d) A função h: R → R dada por h(x) = – g(x) – a (x) é crescente.
possui um zero real duplo. d) A função real s: M →D definida por s(x) = | -
g(x) +| é positiva ∀x ∈ M
18) Alguns cadetes da AFA decidiram programar
uma viagem de férias à cidade de Natal para
janeiro de 2009. Fizeram pesquisa de preços das 20) Se a função real f é definida por f(x) = log3
diárias de alguns hotéis e verificaram que as duas (3x+4) – log3 (2x-1), então o conjunto de valores
melhores propostas seriam as dos hotéis Araújo’s e de x para os quais f (x) <1 é
a) x  R |  x   b) x  R | x  
Fabiano’s, que foram as seguintes: 1 7 1
 2 3  2

269
PROVAS DA AFA 1998 – 2010
PC  AE  CG  El  GL  IN  NA  LP  a
c) x  R | x  oux   d) x  R | x  
1 7 7 IV -
 2 3  3

21) Um estudo sobre a concentração de um


candidato em provas de memorização indicou que,
com o tempo decorrido, sua capacidade de reação
diminui.
A capacidade de reação (E), E > 0, e o tempo
decorrido (t), medido em horas, podem ser
2t + 1
expressos pela relação E =
1
t−
3
A área da região sombreada da figura, em função
Sendo assim, é INCORRETO afirmar que
de a, é
a) a concentração tende a ser máxima por volta de
20 minutos do início da prova. a) 6a2 – 4a2 2 b) 6a2 + 4a2 2
b) a capacidade de reação nunca é menor que 2 c) 12a2 + 8a2 2 d) 12a2 - 8a2 2
c) a cada intervalo de 1h de prova há uma queda de
33, 3 % na capacidade de reação. 25) Ultimamente, vários adereços têm sido
d) se a capacidade de reação é 24, então o tempo t utilizados em bailes e em festas noturnas. Em
decorrido é maior que 24 minutos. alguns casos, “lá pelas tantas horas”, são
distribuídos óculos coloridos, colares, chapéus e
plumas. É um dos momentos de maior
22) Considere a função real f: A →[1,3] definida
descontração na festa.
x  Em geral, acima da pista de dança, é colocado um
sen − 
por f (x) = 1 2 6 objeto luminoso, chamado “sputinik”.
−2 1
Considere um “sputinik” construído do seguinte
Sabendo-se que a função f é inversível, é correto modo:
afirmar que um possível intervalo para o conjunto 1o) toma-se um cubo de aresta 3pcm
Aé 2o) em cada encontro de três arestas, retira-se um
 7 4 7 
a)  ,  b)  ,  tetraedro cuja base é um triângulo equilátero de
3 3   3 3 lado p 2 cm e
 4 10  7 10
c)  3 , 3  d)  ,  3o) no sólido restante, são acopladas pirâmides
   3 3  triangulares de altura 3p 3 cm e pirâmides
octogonais de altura 3pcm ;
23) Em relação à função real f definida por ambos os tipos de pirâmides são retas e possuem
f(x) = |1 – 8sen2(2x)cos2(2x)| - 2 é INCORRETO bases coincidentes com as faces desse sólido.
afirmar que Se o volume desse “sputinik” é xp3 cm3 , então x é
a) Im (f) = [- 2 ,- 1] um número do
b) tem seu valor mínimo como imagem de algum intervalo
 
x ,  a) [ 73, 78 [ c) [ 83, 88 [
8 4 b) [ 78, 83 [ d) [ 88, 103 ]

c) seu período é igual a
8
 3
d) é estritamente crescente em  ,  AFA 2009/2010 – Matemática
16 16 

24) Considere num mesmo plano os pontos da 1) Uma pequena fábrica de cintos paga a seus
figura abaixo, forma que: funcionários o salário, conforme tabela abaixo.
I - AW  CW  EW  GW  IW  LW  NW  PW
II - BW  DW  FW  HW  JW  MW  OW  QW
^ ^ ^ ^
III - A WB  B W C   P W Q  Q W A

270
PROVAS DA AFA 1998 – 2010
CARGO SALÁRIOS Nº DE Sabe-se que x = 0 ou x = 2 são raízes de P e que o
(em reais) FUNCIONÁRIOS resto da divisão de P(x) por [(x-2).(x-1).x] é R(x)
COSTUREIRO 1000 10 As raízes de R(x) são números
(A) a) inteiros pares b)inteiros ímpares
SECRETÁRIOS 1500 4 c) fracionários opostos d) irracionais opostos.
(A)
CONSULTOR 2000 3 5) Numa sala de aula, estão presentes 5 alunos e 6
GERENTE X 1 alunas.Para uma determinada atividade, o
professor deverá escolher um grupo formado por 3
Certo mês, houve um aumento de 10% sobre os dessas alunas e 3 dos alunos.Em seguida, os
salários da tabela acima para todos os cargos. escolhidos serão dispostos em círculo de tal forma
Sabendo-se que a nova média salarial passou a ser que alunos do mesmo sexo não fiquem lado a
de 1650 reais, o novo salário do gerente é, em lado.Isso poderá ocorrer de n maneiras distintas.
reais, igual a O número n é igual a
a) 5500 b) 5000 a) 24 000 b) 2 400
c) 3300 d) 3000 c) 400 d) 200

2) Sejam z = x + yi (x ∈ IR*, y ∈ IR* e i a unidade 6) Três estudantes A, B e C estão em uma


imaginária), z o conjugado de z e λ o lugar competição de natação. Os estudantes A e B têm a
geométrico dos pontos P(x, y) do plano cartesiano mesma probabilidade de vencer e cada um tem o
para os quais z. z = 2x + 3. Se A e B são os pontos dobro da probabilidade de vencer que o estudante
de interseção de λ com o eixo Oy e se A’ é o ponto C. Admitindo-se que não haja empate na
competição, é FALSO afirmar que a probabilidade
de interseção de λ com o eixo Oy que possui a
de
menor abscissa, então a área do triângulo A’AB é,
a) A ou B vence é igual a 0,8
em unidades de área, igual a
b) A vencer é igual a 0,4
a) 2 3 b) 2 2 c) C vencer é maior que 0,2
c) 3 d) 2 d) B ou C vencer é igual 0,6

3) Sejam as funções f: IN →IR e g : IN →IR 7) Seja o sistema S de equações nas incógnitas x, y


definidas por f(x) = x e g(x) = 2-x e z e parâmetro real m
2  x + 2y − z = 0
Considere os números A e B, tais que 
S=  x − my − 3z = 0
A = f(1) + f(2) + ... + f(50) e  x + 3 y + mz = m

B = 1 + g(1) + g(2) + ...+ g(n) + ...
Analise as proposições a seguir e assinale a
Se o produto de A por B tende para o número α,
INCORRETA.
então α é.
a) Se m = -3, então S é impossível.
a) ímpar múltiplo de 9
b) par divisor de 10 000 b) S é determinado se, e somente se, m ≠ 0.
c) par múltiplo de 15 c) Se S é homogêneo, então x + y + z é sempre um
d) ímpar múltiplo de 25 número múltiplo de 3.
d) S admite solução para todo m ≠ -3
4) Observe a função polinomial P esboçada no
gráfico abaixo. 8) Para a fabricação de três modelos de aviões, a
Embraer precisa de alguns equipamentos,
conforme a tabela abaixo.

Modelo/Equipamentos A B C
Poltornas 20 30 60
Extintores 6 10 15

271
PROVAS DA AFA 1998 – 2010
Para o ano de 2009, a Embraer recebeu 11)Analise o gráfico abaixo da função real g : IR
encomendas dos três modelos, conforme a tabela → IR
abaixo.

ANO DE PRIMEIRO SEGUNDO


2009\ SEMESTRE SEMESTRE
MODELO
A 20 50% a mais que
no 1º semestre
B y 25
C 10 20% a menos que
no 1º semestre
Se h é uma função real tal que h(x) = g(x) + 2,
Sabe-se que a quantidade necessária de poltronas então, marque a alternativa verdadeira.
para a fabricação dos três modelos de aviões no a) (h o h o h o ...oh) (0) = 4
ano de 2009 é 3280, então a soma dos algarismos b) (h o h o h) (3) > (h o h o h o h) (2)
de y é igual a.    1 
a) 5 b) 6 c) 7 d) 8 c) Se y = h  h h    então
   2 
y ∈ ]2,3[
  

9) Pedro e Maria com seus filhos Gabriel e João


   3 
foram a uma clínica médica para uma revisão de d) Se x = h  h h   
   2 
então x ∈ ]1,2[
saúde.Fazia parte da avaliação aferir o peso de   

cada um.A balança da clínica era muito antiga e


tinha um defeito, só indicava pesos maiores que 12) Considere a reta r simétrica da reta (s) 2x+y-2
60kg. Para resolver a pesagem, procedeu-se da = 0 em relação à reta (t) x – 3y -2 = 0
seguinte maneira: Com base nisso, marque a alternativa verdadeira.
Pesou-se a) Se - 10 < y < 0 então r ⋂ t = ∅
3
• Peso, Maria e Gabriel, totalizando 150kg
• Pedro, Gabriel e João, totalizando 117 kg b) ∃, P(x, y) ∈ r tal que x < 0 e y > 0
• Maria, Gabriel e João, totalizando 97 kg
• Pedro,Maria, Gabriel e João, totalizando c) Na reta r, se x > 8
então y < - 2
172kg 7 7

d) ∄ P(x , y) ∈ r tal que x > 0 e y < - 10


3
Com base nessas informações, é correto afirmar
que
a) com essa balança é possível pesar Gabriel e 13) Na figura abaixo, tem-se representado as
João juntos. funções f, g e h que indicam os valores pagos,
b) a diferença entre os pesos de Pedro e Maria é o respectivamente, às locadoras de automóveis α, β e
peso de João. γ para x quilômetros rodados por dia. Uma pessoa
c) Pedro é mais pesado que Maria e João juntos. pretende alugar um carro e analisa as três opções.
d) não é possível pesar Maria sozinha nessa
balança.

10) Considere as circunferência dadas pela


equação x2 + y2 = 12 (b ∈ IN*)
b
A circunferência que circunscreve um quadrado de
área igual a 1250 é tal que b pertence ao intervalo
a) 0, 301  b)  1 1 
 30 , 28  c)  1 1 
 28 , 26  d)  1 1 
 26 , 24 
     

272
PROVAS DA AFA 1998 – 2010
Após a análise, essa pessoa conclui que optar pela a) IR - ]1,5[ b) IR – [1,5] c) IR – [1,3]
locadora α ao invés das outras duas locadoras, é d) IR - ]1,3[
mais vantajoso quando x ∈ ]m, + ∞ [, m ∈ IR 17) Sejam as funções reais dadas por f(x) = 22x+1 e

O menor valor possível para m é g(x) = 3x +1 Se b ∈ IR tal que f  1  = 2g(b) e p =


2
a) 60 b) 70 c) 80 d) 90 log3b, então sobre p é correto afirmar que
a) não está definido
14) Sobre a função real f: D →IR dada por f(x) = 1 b) é positivo e menor que 1
+ log2(x2), é INCORRETO afirmar que é c) é negativo e menor que 1
a) par b) sobrejetora x ∈ D d) é positivo e maior que 1
c) crescente se x ∈ [1,+∞[ d) injetora x ∈ D 18) Sobre a função real f definida por f(x) = -1 +
|6(senx)(consx)|, é INCORRETO afirmar que
15) Seja a função real f definida por f(x) = cos (4x) a) Im(f) = [-1,2]
    3 
– sen  − 6x  Marque a alternativa que possui a b) é decrescente para todo x ∈
 2  4 , 4 
 
melhor representação, no ciclo trigonométrico, de c) possui 8 raízes no intervalo [0,2π]
todas as raízes da função f
d) tem período igual ao período da função real g
dada por g(x) = 2f(x)

19) A Revista Época publicou uma reportagem em


fevereiro de 2009 a respeito do impacto da crise
financeira mundial no crescimento da economia.

Desaceleração recorde

Em 2009, a economia mundial deverá ter o menor


crescimento desde a 2ª Guerra Mundial – em % ao
ano.
O gráfico abaixo indica o percentual de
crescimento da economia mundial de alguns anos,
no período de 1980 e 2009.

16) Considere o esboço dos gráficos das funções


reais f, g e h, tais que f é do 2º grau e g e h são do
1º grau. Sabe-se que V é o vértice da parábola.

Fonte:Revista Época – 02/02/2009/ nº 559 – pág.


85. (Adaptado)

O conjunto de todos os valores de x para os quais Sabendo-se que o ano de 2009 o percentual foi
h(x) > g(x) > f(x) é estimado, analise o gráfico e marque a alternativa
FALSA.
273
PROVAS DA AFA 1998 – 2010
a) Houve um aumento superior a 42% do
percentual de crescimento do ano de 1995 para o
ano 2000.
b) A queda de crescimento do ano de 2005 para o
percentual estimado no ano de 2009 é menor que
90%
c) O aumento do percentual de crescimento do ano
de 1985 em relação ao ano de 1980 é
aproximadamente 95% do percentual de
crescimento do ano de 1980.
d) A taxa de crescimento do ano de 2000 em
relação ao ano de 1985 é a mesma que a taxa de
crescimento do ano de 1990 em relação ao ano de
1980.

20) Considere uma chapa de aço circular de


espessura desprezível e raio 15 cm. Recortando-se,
dessa chapa, dois setores circulares de ângulo
2
rad cada, e juntando-se em cada um desses
3
setores os lados de mesma medida, sem perda de
material, obtêm-se dois objetos em forma de cone.
Unindo-se as bases desses cones, obtém-se um
objeto A. Dentro desse objeto A foram inseridas
esferas de ferro cuja área da superfície, de cada
uma, é 9π cm2. Sabendo-se que foram inseridas a
maior quantidade possível dessas esferas dentro do
objeto A, o espaço vago dentro desse objeto, é tal
que, volume é, em cm3 igual a Dado: 2 = 1,41
a) 2 π b) π c)  d) 
2 4

274
PROVAS DA AFA 1998 – 2010

GABARITOS – FÍSICA
97/98 98/99 99/00 00/01 01/02 02/03 03/04 04/05 05/06 06/07 07/08 08/09 09/10
1. D C D A D C D C B B A B C
2. D C A C B C D D C B B C C
3. A B D B C D D D D C C D Nula
4. B C B C A B B D A A D C A
5. B D C D C C A C D C B D A
6. B B D C B A C D C D D A B
7. A A C A C A A C B A A D C
8. A D A D A D D B B D B A D
9. D C C C C D C A D A Nula C B
10. A D D B C D A B A C A B B
11. D D A B C D B D A C B B A
12. D C C A D D B C B A B C A
13. B C A A D C C B B C D C B
14. B A D C A A C A D A Nula B B
15. C B A D B C A A B D C B D
16. D C D C A C B A C B D D A
17. B D C B D B B B B D C D C
18. B B D C B C D A A B D B D
19. C B C A A A A C C D A C A
20. B C C A C C C C B B C A D
21. C A C C C B A D D C D A
22. A D C A C B D B A B C D
23. D D C C D B B C D A A A
24. A A B C A B A A C A A Nula
25. B C B A A A C A A B A C
26. C B C B B D C C C B D
27. A B A B B D A B D D B
28. C B D C A A A A A A A
29. D B D D D Nula D D C D Nula
30. C A B A B B B B A C D
31. D B D D D B
32. B D D D B A
33. C C C D A B
34. B D C D A D
35. A B D D D D
36. C C A D D B
37. B B C D B C
38. A C D B D A
39. B C B A C C
40. B C C B B A

275
PROVAS DA AFA 1998 – 2010

GABARITOS – PORTUGUÊS
97/98 98/99 99/00 00/01 01/02 02/03 03/04 04/05 05/06 06/07 07/08 08/09 09/10
1. A D B A D D D D A B D D D
2. A D B C B C B B D D B C A
3. D A C C D B B A D B B C B
4. D C A A C C C C C D A B D
5. B D B D A A C B C D D B A
6. B B A C C D A D C C C B A
7. C C D A A A C D C A C A D
8. D D A A D C C A D C D C A
9. D C C C A B B D C C A D B
10. B C A B A C D D A B D D B
11. B C B D B A B B B B A A D
12. B A C A C D D B A D A C C
13. A D A D B C B B B D D A B
14. C B B D A D A B B D D D C
15. C B A B C D D A B A C B D
16. D C A A A A B D B D B Nula C
17. C A D D C B C A C C C D B
18. C D B B D B A C A D C B D
19. C C A D D C D C C B D B A
20. D D C B B A C B C A B D C
21. D D D D C D B D A A C D
22. D C C C B B B A D A B C
23. C A D B B C D B A C A B
24. B A C A D B C B C B A D
25. D B D C B A B D A D B B
26. B C C A A B A D D C A
27. D C B B D Nula A B D C C
28. D C D B C A C B C B D
29. C B B B A D C B D A A
30. A A D D B A A A C B B
31. C B C A D A
32. B C C B C C
33. D D D D D D
34. D A A D A C
35. D A C D C B
36. D B D A A C
37. D D D D D B
38. D A A A C D
39. C A D D B D
40. C A C C B A

276
PROVAS DA AFA 1998 – 2010

GABARITOS – INGLÊS
97/98 98/99 99/00 00/01 01/02 02/03 03/04 04/05 05/06 06/07 07/08 08/09 09/10
1. A C B B B C A C C A A A B
2. B C C D D D B C B C D A B
3. B B A C A A B C B D C C A
4. A A/C * A C C B B B A C A D B
4. C C B A A A C A C B A B C
6. B B D A C A A B C D C C C
7. D C B D C D C B B B B A A
8. B B B A A B A B A B D Nula C
9. D A D D C D D B D C C D B
10. A C B A C D B D A A C D C
11. A D C B D B A B C D D B A
12. C B A B C A D C B A D D A
13. D B B A D D A A C D A D C
14. A B C A B C C C D B D B D
15. A C B D B C B A D B B B B
16. B C B B A C C A B D D C D
17. A A A C A D A B B C A A A
18. A B D A B A B C A C C B C
19. D D D C A C D B B A C B A
20. A B B D B B A A C B D Nula B
21. C C B C D B A D A B A A
22. B B A D B C A D A C C C
23. C A D C A D C B C B C Nula
24. C A A B D B D D D D B B
25. A D D D B A D B D B D Nula
26. B C A B C C D B A C C
27. D C A B C C A C B C B
28. B C C C C C C C A B A
29. B A C B A A B B A C A
30. C D D D D C B B C A B
31. D C C C B A
32. D D C C D C
33. B B B C D C
34. B D C B B D
35. C C A D A B
36. C D D A B B
37. C D B B B B
38. A B C A D C
39. B D D B A A
40. B B A C B A

277
PROVAS DA AFA 1998 – 2010

GABARITOS – MATEMÁTICA
97/98 98/99 99/00 00/01 01/02 02/03 03/04 04/05 05/06 06/07 07/08 08/09 09/10
1. C B C D D C A D D B B C A
2. B C D A D C D A B B A Nula C
3. A D B D C B A B D B D D D
4. A A A D A C C C A B D A A
5. C B C D A A B B B D C C B
6. D A C A D A A D A C A A C
7. A B A A B D A C C A C C Nula
8. B C A A B A B C A A C A B
9. C A C B A C B B C C B C D
10. A D C B C D C D C C C B Nula
11. D D D C B B D D C B B Nula C
12. A C A A A B C A D D A B Nula
13. C C B B A C D C B C B D A
14. A D B C B B D B A D A C D
15. A C C B A D A A D C C B A
16. C C C A D B A B B A D B B
17. C D A A D A B D A B Nula A A
18. D B C C B C B B B B B B B
19. B C C C C D D D C B D D D
20. B D A C C B C A C A B D Nula
21. A C A C A B NULA A A C C C
22. C B C D D B A C C C Nula B
23. B C C A B D C C B D Nula D
24. B B/C* B C C C A A D A D A
25. D D D D A B B A D A Nula B
26. A B D B C D D D A A A
27. D D B D A B C C B D Nula
28. B B B B D A D B B C C
29. D C D C C A C C C D A
30. C A D C B C B D D D A
31. C A C B C A
32. C A D B B A
33. C B D B D D
34. A A D C D D
35. D A D B C A
36. D B C C D D
37. A B A D D C
38. B C B B C B
39. A D A B B A
40. D D C C A C

278

Você também pode gostar